ЭлСктричСство Ρ„ΠΈΠ·ΠΈΠΊΠ° Ρ„ΠΎΡ€ΠΌΡƒΠ»Ρ‹: ЭлСктричСский Ρ‚ΠΎΠΊ – Π€ΠΈΠ·ΠΈΠΊΠ° – ВСория, тСсты, Ρ„ΠΎΡ€ΠΌΡƒΠ»Ρ‹ ΠΈ Π·Π°Π΄Π°Ρ‡ΠΈ

Π‘ΠΎΠ΄Π΅Ρ€ΠΆΠ°Π½ΠΈΠ΅

ЭлСктричСский Ρ‚ΠΎΠΊ – Π€ΠΈΠ·ΠΈΠΊΠ° – ВСория, тСсты, Ρ„ΠΎΡ€ΠΌΡƒΠ»Ρ‹ ΠΈ Π·Π°Π΄Π°Ρ‡ΠΈ

ОглавлСниС:

Β 

ΠžΡΠ½ΠΎΠ²Π½Ρ‹Π΅ тСорСтичСскиС свСдСния

ЭлСктричСский Ρ‚ΠΎΠΊ. Π‘ΠΈΠ»Π° Ρ‚ΠΎΠΊΠ°. Π‘ΠΎΠΏΡ€ΠΎΡ‚ΠΈΠ²Π»Π΅Π½ΠΈΠ΅

К оглавлСнию…

Π’ ΠΏΡ€ΠΎΠ²ΠΎΠ΄Π½ΠΈΠΊΠ°Ρ… ΠΏΡ€ΠΈ ΠΎΠΏΡ€Π΅Π΄Π΅Π»Π΅Π½Π½Ρ‹Ρ… условиях ΠΌΠΎΠΆΠ΅Ρ‚ Π²ΠΎΠ·Π½ΠΈΠΊΠ½ΡƒΡ‚ΡŒ Π½Π΅ΠΏΡ€Π΅Ρ€Ρ‹Π²Π½ΠΎΠ΅ упорядочСнноС Π΄Π²ΠΈΠΆΠ΅Π½ΠΈΠ΅ свободных носитСлСй элСктричСского заряда. Π’Π°ΠΊΠΎΠ΅ Π΄Π²ΠΈΠΆΠ΅Π½ΠΈΠ΅ называСтся элСктричСским Ρ‚ΠΎΠΊΠΎΠΌ. Π—Π° Π½Π°ΠΏΡ€Π°Π²Π»Π΅Π½ΠΈΠ΅ элСктричСского Ρ‚ΠΎΠΊΠ° принято Π½Π°ΠΏΡ€Π°Π²Π»Π΅Π½ΠΈΠ΅ двиТСния ΠΏΠΎΠ»ΠΎΠΆΠΈΡ‚Π΅Π»ΡŒΠ½Ρ‹Ρ… свободных зарядов, хотя Π² Π±ΠΎΠ»ΡŒΡˆΠΈΠ½ΡΡ‚Π²Π΅ случаС двиТутся элСктроны – ΠΎΡ‚Ρ€ΠΈΡ†Π°Ρ‚Π΅Π»ΡŒΠ½ΠΎ заряТСнныС частицы.

ΠšΠΎΠ»ΠΈΡ‡Π΅ΡΡ‚Π²Π΅Π½Π½ΠΎΠΉ ΠΌΠ΅Ρ€ΠΎΠΉ элСктричСского Ρ‚ΠΎΠΊΠ° слуТит сила Ρ‚ΠΎΠΊΠ° I – скалярная физичСская Π²Π΅Π»ΠΈΡ‡ΠΈΠ½Π°, равная ΠΎΡ‚Π½ΠΎΡˆΠ΅Π½ΠΈΡŽ заряда q, пСрСносимого Ρ‡Π΅Ρ€Π΅Π· ΠΏΠΎΠΏΠ΅Ρ€Π΅Ρ‡Π½ΠΎΠ΅ сСчСниС ΠΏΡ€ΠΎΠ²ΠΎΠ΄Π½ΠΈΠΊΠ° Π·Π° ΠΈΠ½Ρ‚Π΅Ρ€Π²Π°Π» Π²Ρ€Π΅ΠΌΠ΅Π½ΠΈ t, ΠΊ этому ΠΈΠ½Ρ‚Π΅Ρ€Π²Π°Π»Ρƒ Π²Ρ€Π΅ΠΌΠ΅Π½ΠΈ:

Если Ρ‚ΠΎΠΊ Π½Π΅ постоянный, Ρ‚ΠΎ для нахоТдСния количСства ΠΏΡ€ΠΎΡˆΠ΅Π΄ΡˆΠ΅Π³ΠΎ Ρ‡Π΅Ρ€Π΅Π· ΠΏΡ€ΠΎΠ²ΠΎΠ΄Π½ΠΈΠΊ заряда Ρ€Π°ΡΡΡ‡ΠΈΡ‚Ρ‹Π²Π°ΡŽΡ‚ ΠΏΠ»ΠΎΡ‰Π°Π΄ΡŒ Ρ„ΠΈΠ³ΡƒΡ€Ρ‹ ΠΏΠΎΠ΄ Π³Ρ€Π°Ρ„ΠΈΠΊΠΎΠΌ зависимости силы Ρ‚ΠΎΠΊΠ° ΠΎΡ‚ Π²Ρ€Π΅ΠΌΠ΅Π½ΠΈ.

Если сила Ρ‚ΠΎΠΊΠ° ΠΈ Π΅Π³ΠΎ Π½Π°ΠΏΡ€Π°Π²Π»Π΅Π½ΠΈΠ΅ Π½Π΅ ΠΈΠ·ΠΌΠ΅Π½ΡΡŽΡ‚ΡΡ со Π²Ρ€Π΅ΠΌΠ΅Π½Π΅ΠΌ, Ρ‚ΠΎ Ρ‚Π°ΠΊΠΎΠΉ Ρ‚ΠΎΠΊ называСтся

постоянным. Π‘ΠΈΠ»Π° Ρ‚ΠΎΠΊΠ° измСряСтся Π°ΠΌΠΏΠ΅Ρ€ΠΌΠ΅Ρ‚Ρ€ΠΎΠΌ, ΠΊΠΎΡ‚ΠΎΡ€Ρ‹ΠΉ Π²ΠΊΠ»ΡŽΡ‡Π°Π΅Ρ‚ΡΡ Π² Ρ†Π΅ΠΏΡŒ ΠΏΠΎΡΠ»Π΅Π΄ΠΎΠ²Π°Ρ‚Π΅Π»ΡŒΠ½ΠΎ. Π’ ΠœΠ΅ΠΆΠ΄ΡƒΠ½Π°Ρ€ΠΎΠ΄Π½ΠΎΠΉ систСмС Π΅Π΄ΠΈΠ½ΠΈΡ† БИ сила Ρ‚ΠΎΠΊΠ° измСряСтся Π² Π°ΠΌΠΏΠ΅Ρ€Π°Ρ… [А]. 1 А = 1 Кл/с.

БрСдняя сила Ρ‚ΠΎΠΊΠ° находится ΠΊΠ°ΠΊ ΠΎΡ‚Π½ΠΎΡˆΠ΅Π½ΠΈΠ΅ всСго заряда ΠΊΠΎ всСму Π²Ρ€Π΅ΠΌΠ΅Π½ΠΈ (Ρ‚.Π΅. ΠΏΠΎ Ρ‚ΠΎΠΌΡƒ ΠΆΠ΅ ΠΏΡ€ΠΈΠ½Ρ†ΠΈΠΏΡƒ, Ρ‡Ρ‚ΠΎ ΠΈ срСдняя ΡΠΊΠΎΡ€ΠΎΡΡ‚ΡŒ ΠΈΠ»ΠΈ любая другая срСдняя Π²Π΅Π»ΠΈΡ‡ΠΈΠ½Π° Π² Ρ„ΠΈΠ·ΠΈΠΊΠ΅):

Если ΠΆΠ΅ Ρ‚ΠΎΠΊ Ρ€Π°Π²Π½ΠΎΠΌΠ΅Ρ€Π½ΠΎ мСняСтся с Ρ‚Π΅Ρ‡Π΅Π½ΠΈΠ΅ΠΌ Π²Ρ€Π΅ΠΌΠ΅Π½ΠΈ ΠΎΡ‚ значСния I1 Π΄ΠΎ значСния I2, Ρ‚ΠΎ ΠΌΠΎΠΆΠ½ΠΎ Π·Π½Π°Ρ‡Π΅Π½ΠΈΠ΅ срСднСго Ρ‚ΠΎΠΊΠ° ΠΌΠΎΠΆΠ½ΠΎ Π½Π°ΠΉΡ‚ΠΈ ΠΊΠ°ΠΊ срСднСарифмСтичСскоС ΠΊΡ€Π°ΠΉΠ½ΠΈΡ… Π·Π½Π°Ρ‡Π΅Π½ΠΈΠΉ:

ΠŸΠ»ΠΎΡ‚Π½ΠΎΡΡ‚ΡŒ Ρ‚ΠΎΠΊΠ° – сила Ρ‚ΠΎΠΊΠ°, приходящаяся Π½Π° Π΅Π΄ΠΈΠ½ΠΈΡ†Ρƒ ΠΏΠΎΠΏΠ΅Ρ€Π΅Ρ‡Π½ΠΎΠ³ΠΎ сСчСния ΠΏΡ€ΠΎΠ²ΠΎΠ΄Π½ΠΈΠΊΠ°, рассчитываСтся ΠΏΠΎ Ρ„ΠΎΡ€ΠΌΡƒΠ»Π΅:

ΠŸΡ€ΠΈ ΠΏΡ€ΠΎΡ…ΠΎΠΆΠ΄Π΅Π½ΠΈΠΈ Ρ‚ΠΎΠΊΠ° ΠΏΠΎ ΠΏΡ€ΠΎΠ²ΠΎΠ΄Π½ΠΈΠΊΡƒ Ρ‚ΠΎΠΊ испытываСт сопротивлСниС со стороны ΠΏΡ€ΠΎΠ²ΠΎΠ΄Π½ΠΈΠΊΠ°. ΠŸΡ€ΠΈΡ‡ΠΈΠ½Π° сопротивлСния – взаимодСйствиС зарядов с Π°Ρ‚ΠΎΠΌΠ°ΠΌΠΈ вСщСства ΠΏΡ€ΠΎΠ²ΠΎΠ΄Π½ΠΈΠΊΠ° ΠΈ ΠΌΠ΅ΠΆΠ΄Ρƒ собой. Π•Π΄ΠΈΠ½ΠΈΡ†Π° измСрСния сопротивлСния 1 Ом. Π‘ΠΎΠΏΡ€ΠΎΡ‚ΠΈΠ²Π»Π΅Π½ΠΈΠ΅ ΠΏΡ€ΠΎΠ²ΠΎΠ΄Π½ΠΈΠΊΠ° R опрСдСляСтся ΠΏΠΎ Ρ„ΠΎΡ€ΠΌΡƒΠ»Π΅:

Π³Π΄Π΅: l – Π΄Π»ΠΈΠ½Π° ΠΏΡ€ΠΎΠ²ΠΎΠ΄Π½ΠΈΠΊΠ°, S – ΠΏΠ»ΠΎΡ‰Π°Π΄ΡŒ Π΅Π³ΠΎ ΠΏΠΎΠΏΠ΅Ρ€Π΅Ρ‡Π½ΠΎΠ³ΠΎ сСчСния, ρ – ΡƒΠ΄Π΅Π»ΡŒΠ½ΠΎΠ΅ сопротивлСниС ΠΌΠ°Ρ‚Π΅Ρ€ΠΈΠ°Π»Π° ΠΏΡ€ΠΎΠ²ΠΎΠ΄Π½ΠΈΠΊΠ° (Π±ΡƒΠ΄ΡŒΡ‚Π΅ Π²Π½ΠΈΠΌΠ°Ρ‚Π΅Π»ΡŒΠ½Ρ‹ ΠΈ Π½Π΅ ΠΏΠ΅Ρ€Π΅ΠΏΡƒΡ‚Π°ΠΉΡ‚Π΅ послСднюю Π²Π΅Π»ΠΈΡ‡ΠΈΠ½Ρƒ с ΠΏΠ»ΠΎΡ‚Π½ΠΎΡΡ‚ΡŒΡŽ вСщСства), ΠΊΠΎΡ‚ΠΎΡ€ΠΎΠ΅ Ρ…Π°Ρ€Π°ΠΊΡ‚Π΅Ρ€ΠΈΠ·ΡƒΠ΅Ρ‚ ΡΠΏΠΎΡΠΎΠ±Π½ΠΎΡΡ‚ΡŒ ΠΌΠ°Ρ‚Π΅Ρ€ΠΈΠ°Π»Π° ΠΏΡ€ΠΎΠ²ΠΎΠ΄Π½ΠΈΠΊΠ° ΠΏΡ€ΠΎΡ‚ΠΈΠ²ΠΎΠ΄Π΅ΠΉΡΡ‚Π²ΠΎΠ²Π°Ρ‚ΡŒ ΠΏΡ€ΠΎΡ…ΠΎΠΆΠ΄Π΅Π½ΠΈΡŽ Ρ‚ΠΎΠΊΠ°. Π’ΠΎ Π΅ΡΡ‚ΡŒ это такая ΠΆΠ΅ характСристика вСщСства, ΠΊΠ°ΠΊ ΠΈ ΠΌΠ½ΠΎΠ³ΠΈΠ΅ Π΄Ρ€ΡƒΠ³ΠΈΠ΅: ΡƒΠ΄Π΅Π»ΡŒΠ½Π°Ρ Ρ‚Π΅ΠΏΠ»ΠΎΠ΅ΠΌΠΊΠΎΡΡ‚ΡŒ, ΠΏΠ»ΠΎΡ‚Π½ΠΎΡΡ‚ΡŒ, Ρ‚Π΅ΠΌΠΏΠ΅Ρ€Π°Ρ‚ΡƒΡ€Π° плавлСния ΠΈ Ρ‚.Π΄. Π•Π΄ΠΈΠ½ΠΈΡ†Π° измСрСния ΡƒΠ΄Π΅Π»ΡŒΠ½ΠΎΠ³ΠΎ сопротивлСния 1 Ом·м. УдСльноС сопротивлСниС вСщСства – табличная Π²Π΅Π»ΠΈΡ‡ΠΈΠ½Π°.

Π‘ΠΎΠΏΡ€ΠΎΡ‚ΠΈΠ²Π»Π΅Π½ΠΈΠ΅ ΠΏΡ€ΠΎΠ²ΠΎΠ΄Π½ΠΈΠΊΠ° зависит ΠΈ ΠΎΡ‚ Π΅Π³ΠΎ Ρ‚Π΅ΠΌΠΏΠ΅Ρ€Π°Ρ‚ΡƒΡ€Ρ‹:

Π³Π΄Π΅: R0 – сопротивлСниС ΠΏΡ€ΠΎΠ²ΠΎΠ΄Π½ΠΈΠΊΠ° ΠΏΡ€ΠΈ 0Β°Π‘, t – Ρ‚Π΅ΠΌΠΏΠ΅Ρ€Π°Ρ‚ΡƒΡ€Π°, выраТСнная Π² градусах ЦСльсия, α – Ρ‚Π΅ΠΌΠΏΠ΅Ρ€Π°Ρ‚ΡƒΡ€Π½Ρ‹ΠΉ коэффициСнт сопротивлСния. Он Ρ€Π°Π²Π΅Π½ ΠΎΡ‚Π½ΠΎΡΠΈΡ‚Π΅Π»ΡŒΠ½ΠΎΠΌΡƒ измСнСнию сопротивлСния, ΠΏΡ€ΠΈ ΡƒΠ²Π΅Π»ΠΈΡ‡Π΅Π½ΠΈΠΈ Ρ‚Π΅ΠΌΠΏΠ΅Ρ€Π°Ρ‚ΡƒΡ€Ρ‹ Π½Π° 1Β°Π‘.

Для ΠΌΠ΅Ρ‚Π°Π»Π»ΠΎΠ² ΠΎΠ½ всСгда большС нуля, для элСктролитов Π½Π°ΠΎΠ±ΠΎΡ€ΠΎΡ‚, всСгда мСньшС нуля.

Π”ΠΈΠΎΠ΄ Π² Ρ†Π΅ΠΏΠΈ постоянного Ρ‚ΠΎΠΊΠ°

Π”ΠΈΠΎΠ΄ – это Π½Π΅Π»ΠΈΠ½Π΅ΠΉΠ½Ρ‹ΠΉ элСмСнт Ρ†Π΅ΠΏΠΈ, сопротивлСниС ΠΊΠΎΡ‚ΠΎΡ€ΠΎΠ³ΠΎ зависит ΠΎΡ‚ направлСния протСкания Ρ‚ΠΎΠΊΠ°. ΠžΠ±ΠΎΠ·Π½Π°Ρ‡Π°Π΅Ρ‚ΡΡ Π΄ΠΈΠΎΠ΄ ΡΠ»Π΅Π΄ΡƒΡŽΡ‰ΠΈΠΌ ΠΎΠ±Ρ€Π°Π·ΠΎΠΌ:

Π‘Ρ‚Ρ€Π΅Π»ΠΊΠ° Π² схСматичСском ΠΎΠ±ΠΎΠ·Π½Π°Ρ‡Π΅Π½ΠΈΠΈ Π΄ΠΈΠΎΠ΄Π° ΠΏΠΎΠΊΠ°Π·Ρ‹Π²Π°Π΅Ρ‚, Π² ΠΊΠ°ΠΊΠΎΠΌ Π½Π°ΠΏΡ€Π°Π²Π»Π΅Π½ΠΈΠΈ ΠΎΠ½ пропускаСт Ρ‚ΠΎΠΊ. Π’ этом случаС Π΅Π³ΠΎ сопротивлСниС Ρ€Π°Π²Π½ΠΎ Π½ΡƒΠ»ΡŽ, ΠΈ Π΄ΠΈΠΎΠ΄ ΠΌΠΎΠΆΠ½ΠΎ Π·Π°ΠΌΠ΅Π½ΠΈΡ‚ΡŒ просто Π½Π° ΠΏΡ€ΠΎΠ²ΠΎΠ΄Π½ΠΈΠΊ с Π½ΡƒΠ»Π΅Π²Ρ‹ΠΌ сопротивлСниСм. Если Ρ‚ΠΎΠΊ Ρ‚Π΅Ρ‡Π΅Ρ‚ Ρ‡Π΅Ρ€Π΅Π· Π΄ΠΈΠΎΠ΄ Π² ΠΏΡ€ΠΎΡ‚ΠΈΠ²ΠΎΠΏΠΎΠ»ΠΎΠΆΠ½ΠΎΠΌ Π½Π°ΠΏΡ€Π°Π²Π»Π΅Π½ΠΈΠΈ, Ρ‚ΠΎ Π΄ΠΈΠΎΠ΄ ΠΎΠ±Π»Π°Π΄Π°Π΅Ρ‚ бСсконСчно большим сопротивлСниСм, Ρ‚ΠΎ Π΅ΡΡ‚ΡŒ Π½Π΅ пропускаСт Ρ‚ΠΎΠΊ совсСм, ΠΈ являСтся Ρ€Π°Π·Ρ€Ρ‹Π²ΠΎΠΌ Π² Ρ†Π΅ΠΏΠΈ. Π’ΠΎΠ³Π΄Π° участок Ρ†Π΅ΠΏΠΈ с Π΄ΠΈΠΎΠ΄ΠΎΠΌ ΠΌΠΎΠΆΠ½ΠΎ просто Π²Ρ‹Ρ‡Π΅Ρ€ΠΊΠ½ΡƒΡ‚ΡŒ, Ρ‚Π°ΠΊ ΠΊΠ°ΠΊ Ρ‚ΠΎΠΊ ΠΏΠΎ Π½Π΅ΠΌΡƒ Π½Π΅ ΠΈΠ΄Π΅Ρ‚.

Β 

Π—Π°ΠΊΠΎΠ½ Ома. ΠŸΠΎΡΠ»Π΅Π΄ΠΎΠ²Π°Ρ‚Π΅Π»ΡŒΠ½ΠΎΠ΅ ΠΈ ΠΏΠ°Ρ€Π°Π»Π»Π΅Π»ΡŒΠ½ΠΎΠ΅ соСдинСниС ΠΏΡ€ΠΎΠ²ΠΎΠ΄Π½ΠΈΠΊΠΎΠ²

К оглавлСнию…

НСмСцкий Ρ„ΠΈΠ·ΠΈΠΊ Π“.Ом Π² 1826 Π³ΠΎΠ΄Ρƒ ΡΠΊΡΠΏΠ΅Ρ€ΠΈΠΌΠ΅Π½Ρ‚Π°Π»ΡŒΠ½ΠΎ установил, Ρ‡Ρ‚ΠΎ сила Ρ‚ΠΎΠΊΠ° I, Ρ‚Π΅ΠΊΡƒΡ‰Π΅Π³ΠΎ ΠΏΠΎ ΠΎΠ΄Π½ΠΎΡ€ΠΎΠ΄Π½ΠΎΠΌΡƒ мСталличСскому ΠΏΡ€ΠΎΠ²ΠΎΠ΄Π½ΠΈΠΊΡƒ (Ρ‚ΠΎ Π΅ΡΡ‚ΡŒ ΠΏΡ€ΠΎΠ²ΠΎΠ΄Π½ΠΈΠΊΡƒ, Π² ΠΊΠΎΡ‚ΠΎΡ€ΠΎΠΌ Π½Π΅ Π΄Π΅ΠΉΡΡ‚Π²ΡƒΡŽΡ‚ сторонниС силы) сопротивлСниСм R, ΠΏΡ€ΠΎΠΏΠΎΡ€Ρ†ΠΈΠΎΠ½Π°Π»ΡŒΠ½Π° Π½Π°ΠΏΡ€ΡΠΆΠ΅Π½ΠΈΡŽ U Π½Π° ΠΊΠΎΠ½Ρ†Π°Ρ… ΠΏΡ€ΠΎΠ²ΠΎΠ΄Π½ΠΈΠΊΠ°:

Π’Π΅Π»ΠΈΡ‡ΠΈΠ½Ρƒ R принято Π½Π°Π·Ρ‹Π²Π°Ρ‚ΡŒ элСктричСским сопротивлСниСм. ΠŸΡ€ΠΎΠ²ΠΎΠ΄Π½ΠΈΠΊ, ΠΎΠ±Π»Π°Π΄Π°ΡŽΡ‰ΠΈΠΉ элСктричСским сопротивлСниСм, называСтся

рСзистором. Π­Ρ‚ΠΎ ΡΠΎΠΎΡ‚Π½ΠΎΡˆΠ΅Π½ΠΈΠ΅ Π²Ρ‹Ρ€Π°ΠΆΠ°Π΅Ρ‚ Π·Π°ΠΊΠΎΠ½ Ома для ΠΎΠ΄Π½ΠΎΡ€ΠΎΠ΄Π½ΠΎΠ³ΠΎ участка Ρ†Π΅ΠΏΠΈ: сила Ρ‚ΠΎΠΊΠ° Π² ΠΏΡ€ΠΎΠ²ΠΎΠ΄Π½ΠΈΠΊΠ΅ прямо ΠΏΡ€ΠΎΠΏΠΎΡ€Ρ†ΠΈΠΎΠ½Π°Π»ΡŒΠ½Π° ΠΏΡ€ΠΈΠ»ΠΎΠΆΠ΅Π½Π½ΠΎΠΌΡƒ Π½Π°ΠΏΡ€ΡΠΆΠ΅Π½ΠΈΡŽ ΠΈ ΠΎΠ±Ρ€Π°Ρ‚Π½ΠΎ ΠΏΡ€ΠΎΠΏΠΎΡ€Ρ†ΠΈΠΎΠ½Π°Π»ΡŒΠ½Π° ΡΠΎΠΏΡ€ΠΎΡ‚ΠΈΠ²Π»Π΅Π½ΠΈΡŽ ΠΏΡ€ΠΎΠ²ΠΎΠ΄Π½ΠΈΠΊΠ°.

ΠŸΡ€ΠΎΠ²ΠΎΠ΄Π½ΠΈΠΊΠΈ, ΠΏΠΎΠ΄Ρ‡ΠΈΠ½ΡΡŽΡ‰ΠΈΠ΅ΡΡ Π·Π°ΠΊΠΎΠ½Ρƒ Ома, Π½Π°Π·Ρ‹Π²Π°ΡŽΡ‚ΡΡ Π»ΠΈΠ½Π΅ΠΉΠ½Ρ‹ΠΌΠΈ. ГрафичСская Π·Π°Π²ΠΈΡΠΈΠΌΠΎΡΡ‚ΡŒ силы Ρ‚ΠΎΠΊΠ° I ΠΎΡ‚ напряТСния U (Ρ‚Π°ΠΊΠΈΠ΅ Π³Ρ€Π°Ρ„ΠΈΠΊΠΈ Π½Π°Π·Ρ‹Π²Π°ΡŽΡ‚ΡΡ Π²ΠΎΠ»ΡŒΡ‚-Π°ΠΌΠΏΠ΅Ρ€Π½Ρ‹ΠΌΠΈ характСристиками, сокращСнно ВАΠ₯) изобраТаСтся прямой Π»ΠΈΠ½ΠΈΠ΅ΠΉ, проходящСй Ρ‡Π΅Ρ€Π΅Π· Π½Π°Ρ‡Π°Π»ΠΎ ΠΊΠΎΠΎΡ€Π΄ΠΈΠ½Π°Ρ‚. Π‘Π»Π΅Π΄ΡƒΠ΅Ρ‚ ΠΎΡ‚ΠΌΠ΅Ρ‚ΠΈΡ‚ΡŒ, Ρ‡Ρ‚ΠΎ сущСствуСт ΠΌΠ½ΠΎΠ³ΠΎ ΠΌΠ°Ρ‚Π΅Ρ€ΠΈΠ°Π»ΠΎΠ² ΠΈ устройств, Π½Π΅ ΠΏΠΎΠ΄Ρ‡ΠΈΠ½ΡΡŽΡ‰ΠΈΡ…ΡΡ Π·Π°ΠΊΠΎΠ½Ρƒ Ома, Π½Π°ΠΏΡ€ΠΈΠΌΠ΅Ρ€, ΠΏΠΎΠ»ΡƒΠΏΡ€ΠΎΠ²ΠΎΠ΄Π½ΠΈΠΊΠΎΠ²Ρ‹ΠΉ Π΄ΠΈΠΎΠ΄ ΠΈΠ»ΠΈ газоразрядная Π»Π°ΠΌΠΏΠ°. Π”Π°ΠΆΠ΅ Ρƒ мСталличСских ΠΏΡ€ΠΎΠ²ΠΎΠ΄Π½ΠΈΠΊΠΎΠ² ΠΏΡ€ΠΈ достаточно Π±ΠΎΠ»ΡŒΡˆΠΈΡ… Ρ‚ΠΎΠΊΠ°Ρ… Π½Π°Π±Π»ΡŽΠ΄Π°Π΅Ρ‚ΡΡ ΠΎΡ‚ΠΊΠ»ΠΎΠ½Π΅Π½ΠΈΠ΅ ΠΎΡ‚ Π»ΠΈΠ½Π΅ΠΉΠ½ΠΎΠ³ΠΎ Π·Π°ΠΊΠΎΠ½Π° Ома, Ρ‚Π°ΠΊ ΠΊΠ°ΠΊ элСктричСскоС сопротивлСниС мСталличСских ΠΏΡ€ΠΎΠ²ΠΎΠ΄Π½ΠΈΠΊΠΎΠ² растСт с ростом Ρ‚Π΅ΠΌΠΏΠ΅Ρ€Π°Ρ‚ΡƒΡ€Ρ‹.

ΠŸΡ€ΠΎΠ²ΠΎΠ΄Π½ΠΈΠΊΠΈ Π² элСктричСских цСпях ΠΌΠΎΠΆΠ½ΠΎ ΡΠΎΠ΅Π΄ΠΈΠ½ΡΡ‚ΡŒ двумя способами: ΠΏΠΎΡΠ»Π΅Π΄ΠΎΠ²Π°Ρ‚Π΅Π»ΡŒΠ½ΠΎ ΠΈ ΠΏΠ°Ρ€Π°Π»Π»Π΅Π»ΡŒΠ½ΠΎ. Π£ ΠΊΠ°ΠΆΠ΄ΠΎΠ³ΠΎ способа Π΅ΡΡ‚ΡŒ свои закономСрности.

1. ЗакономСрности ΠΏΠΎΡΠ»Π΅Π΄ΠΎΠ²Π°Ρ‚Π΅Π»ΡŒΠ½ΠΎΠ³ΠΎ соСдинСния:

Π€ΠΎΡ€ΠΌΡƒΠ»Π° для ΠΎΠ±Ρ‰Π΅Π³ΠΎ сопротивлСния ΠΏΠΎΡΠ»Π΅Π΄ΠΎΠ²Π°Ρ‚Π΅Π»ΡŒΠ½ΠΎ соСдинСнных рСзисторов справСдлива для любого числа ΠΏΡ€ΠΎΠ²ΠΎΠ΄Π½ΠΈΠΊΠΎΠ². Если ΠΆΠ΅ Π² Ρ†Π΅ΠΏΡŒ ΠΏΠΎΡΠ»Π΅Π΄ΠΎΠ²Π°Ρ‚Π΅Π»ΡŒΠ½ΠΎ Π²ΠΊΠ»ΡŽΡ‡Π΅Π½ΠΎ n ΠΎΠ΄ΠΈΠ½Π°ΠΊΠΎΠ²Ρ‹Ρ… сопротивлСний R, Ρ‚ΠΎ ΠΎΠ±Ρ‰Π΅Π΅ сопротивлСниС R

0 находится ΠΏΠΎ Ρ„ΠΎΡ€ΠΌΡƒΠ»Π΅:

2. ЗакономСрности ΠΏΠ°Ρ€Π°Π»Π»Π΅Π»ΡŒΠ½ΠΎΠ³ΠΎ соСдинСния:

Π€ΠΎΡ€ΠΌΡƒΠ»Π° для ΠΎΠ±Ρ‰Π΅Π³ΠΎ сопротивлСния ΠΏΠ°Ρ€Π°Π»Π»Π΅Π»ΡŒΠ½ΠΎ соСдинСнных рСзисторов справСдлива для любого числа ΠΏΡ€ΠΎΠ²ΠΎΠ΄Π½ΠΈΠΊΠΎΠ². Если ΠΆΠ΅ Π² Ρ†Π΅ΠΏΡŒ ΠΏΠ°Ρ€Π°Π»Π»Π΅Π»ΡŒΠ½ΠΎ Π²ΠΊΠ»ΡŽΡ‡Π΅Π½ΠΎ n ΠΎΠ΄ΠΈΠ½Π°ΠΊΠΎΠ²Ρ‹Ρ… сопротивлСний R, Ρ‚ΠΎ ΠΎΠ±Ρ‰Π΅Π΅ сопротивлСниС R0 находится ΠΏΠΎ Ρ„ΠΎΡ€ΠΌΡƒΠ»Π΅:

Π­Π»Π΅ΠΊΡ‚Ρ€ΠΎΠΈΠ·ΠΌΠ΅Ρ€ΠΈΡ‚Π΅Π»ΡŒΠ½Ρ‹Π΅ ΠΏΡ€ΠΈΠ±ΠΎΡ€Ρ‹

Для измСрСния напряТСний ΠΈ Ρ‚ΠΎΠΊΠΎΠ² Π² элСктричСских цСпях постоянного Ρ‚ΠΎΠΊΠ° ΠΈΡΠΏΠΎΠ»ΡŒΠ·ΡƒΡŽΡ‚ΡΡ ΡΠΏΠ΅Ρ†ΠΈΠ°Π»ΡŒΠ½Ρ‹Π΅ ΠΏΡ€ΠΈΠ±ΠΎΡ€Ρ‹ – Π²ΠΎΠ»ΡŒΡ‚ΠΌΠ΅Ρ‚Ρ€Ρ‹ ΠΈ Π°ΠΌΠΏΠ΅Ρ€ΠΌΠ΅Ρ‚Ρ€Ρ‹.

Π’ΠΎΠ»ΡŒΡ‚ΠΌΠ΅Ρ‚Ρ€ ΠΏΡ€Π΅Π΄Π½Π°Π·Π½Π°Ρ‡Π΅Π½ для измСрСния разности ΠΏΠΎΡ‚Π΅Π½Ρ†ΠΈΠ°Π»ΠΎΠ², ΠΏΡ€ΠΈΠ»ΠΎΠΆΠ΅Π½Π½ΠΎΠΉ ΠΊ Π΅Π³ΠΎ ΠΊΠ»Π΅ΠΌΠΌΠ°ΠΌ.

Он ΠΏΠΎΠ΄ΠΊΠ»ΡŽΡ‡Π°Π΅Ρ‚ΡΡ ΠΏΠ°Ρ€Π°Π»Π»Π΅Π»ΡŒΠ½ΠΎ участку Ρ†Π΅ΠΏΠΈ, Π½Π° ΠΊΠΎΡ‚ΠΎΡ€ΠΎΠΌ производится ΠΈΠ·ΠΌΠ΅Ρ€Π΅Π½ΠΈΠ΅ разности ΠΏΠΎΡ‚Π΅Π½Ρ†ΠΈΠ°Π»ΠΎΠ². Π›ΡŽΠ±ΠΎΠΉ Π²ΠΎΠ»ΡŒΡ‚ΠΌΠ΅Ρ‚Ρ€ ΠΎΠ±Π»Π°Π΄Π°Π΅Ρ‚ Π½Π΅ΠΊΠΎΡ‚ΠΎΡ€Ρ‹ΠΌ Π²Π½ΡƒΡ‚Ρ€Π΅Π½Π½ΠΈΠΌ сопротивлСниСм RB. Для Ρ‚ΠΎΠ³ΠΎ Ρ‡Ρ‚ΠΎΠ±Ρ‹ Π²ΠΎΠ»ΡŒΡ‚ΠΌΠ΅Ρ‚Ρ€ Π½Π΅ вносил Π·Π°ΠΌΠ΅Ρ‚Π½ΠΎΠ³ΠΎ пСрСраспрСдСлСния Ρ‚ΠΎΠΊΠΎΠ² ΠΏΡ€ΠΈ ΠΏΠΎΠ΄ΠΊΠ»ΡŽΡ‡Π΅Π½ΠΈΠΈ ΠΊ измСряСмой Ρ†Π΅ΠΏΠΈ, Π΅Π³ΠΎ Π²Π½ΡƒΡ‚Ρ€Π΅Π½Π½Π΅Π΅ сопротивлСниС Π΄ΠΎΠ»ΠΆΠ½ΠΎ Π±Ρ‹Ρ‚ΡŒ Π²Π΅Π»ΠΈΠΊΠΎ ΠΏΠΎ ΡΡ€Π°Π²Π½Π΅Π½ΠΈΡŽ с сопротивлСниСм Ρ‚ΠΎΠ³ΠΎ участка Ρ†Π΅ΠΏΠΈ, ΠΊ ΠΊΠΎΡ‚ΠΎΡ€ΠΎΠΌΡƒ ΠΎΠ½ ΠΏΠΎΠ΄ΠΊΠ»ΡŽΡ‡Π΅Π½.

АмпСрмСтр ΠΏΡ€Π΅Π΄Π½Π°Π·Π½Π°Ρ‡Π΅Π½ для измСрСния силы Ρ‚ΠΎΠΊΠ° Π² Ρ†Π΅ΠΏΠΈ. АмпСрмСтр Π²ΠΊΠ»ΡŽΡ‡Π°Π΅Ρ‚ΡΡ ΠΏΠΎΡΠ»Π΅Π΄ΠΎΠ²Π°Ρ‚Π΅Π»ΡŒΠ½ΠΎ Π² Ρ€Π°Π·Ρ€Ρ‹Π² элСктричСской Ρ†Π΅ΠΏΠΈ, Ρ‡Ρ‚ΠΎΠ±Ρ‹ Ρ‡Π΅Ρ€Π΅Π· Π½Π΅Π³ΠΎ ΠΏΡ€ΠΎΡ…ΠΎΠ΄ΠΈΠ» вСсь измСряСмый Ρ‚ΠΎΠΊ. АмпСрмСтр Ρ‚Π°ΠΊΠΆΠ΅ ΠΎΠ±Π»Π°Π΄Π°Π΅Ρ‚ Π½Π΅ΠΊΠΎΡ‚ΠΎΡ€Ρ‹ΠΌ Π²Π½ΡƒΡ‚Ρ€Π΅Π½Π½ΠΈΠΌ сопротивлСниСм

RA. Π’ ΠΎΡ‚Π»ΠΈΡ‡ΠΈΠ΅ ΠΎΡ‚ Π²ΠΎΠ»ΡŒΡ‚ΠΌΠ΅Ρ‚Ρ€Π°, Π²Π½ΡƒΡ‚Ρ€Π΅Π½Π½Π΅Π΅ сопротивлСниС Π°ΠΌΠΏΠ΅Ρ€ΠΌΠ΅Ρ‚Ρ€Π° Π΄ΠΎΠ»ΠΆΠ½ΠΎ Π±Ρ‹Ρ‚ΡŒ достаточно ΠΌΠ°Π»Ρ‹ΠΌ ΠΏΠΎ ΡΡ€Π°Π²Π½Π΅Π½ΠΈΡŽ с ΠΏΠΎΠ»Π½Ρ‹ΠΌ сопротивлСниСм всСй Ρ†Π΅ΠΏΠΈ.

Β 

Π­Π”Π‘. Π—Π°ΠΊΠΎΠ½ Ома для ΠΏΠΎΠ»Π½ΠΎΠΉ Ρ†Π΅ΠΏΠΈ

К оглавлСнию…

Для сущСствования постоянного Ρ‚ΠΎΠΊΠ° Π½Π΅ΠΎΠ±Ρ…ΠΎΠ΄ΠΈΠΌΠΎ Π½Π°Π»ΠΈΡ‡ΠΈΠ΅ Π² элСктричСской Π·Π°ΠΌΠΊΠ½ΡƒΡ‚ΠΎΠΉ Ρ†Π΅ΠΏΠΈ устройства, способного ΡΠΎΠ·Π΄Π°Π²Π°Ρ‚ΡŒ ΠΈ ΠΏΠΎΠ΄Π΄Π΅Ρ€ΠΆΠΈΠ²Π°Ρ‚ΡŒ разности ΠΏΠΎΡ‚Π΅Π½Ρ†ΠΈΠ°Π»ΠΎΠ² Π½Π° участках Ρ†Π΅ΠΏΠΈ Π·Π° счСт Ρ€Π°Π±ΠΎΡ‚Ρ‹ сил нСэлСктростатичСского происхоТдСния. Π’Π°ΠΊΠΈΠ΅ устройства Π½Π°Π·Ρ‹Π²Π°ΡŽΡ‚ΡΡ источниками постоянного Ρ‚ΠΎΠΊΠ°. Π‘ΠΈΠ»Ρ‹ нСэлСктростатичСского происхоТдСния, Π΄Π΅ΠΉΡΡ‚Π²ΡƒΡŽΡ‰ΠΈΠ΅ Π½Π° свободныС носитСли заряда со стороны источников Ρ‚ΠΎΠΊΠ°, Π½Π°Π·Ρ‹Π²Π°ΡŽΡ‚ΡΡ

сторонними силами.

ΠŸΡ€ΠΈΡ€ΠΎΠ΄Π° сторонних сил ΠΌΠΎΠΆΠ΅Ρ‚ Π±Ρ‹Ρ‚ΡŒ Ρ€Π°Π·Π»ΠΈΡ‡Π½ΠΎΠΉ. Π’ Π³Π°Π»ΡŒΠ²Π°Π½ΠΈΡ‡Π΅ΡΠΊΠΈΡ… элСмСнтах ΠΈΠ»ΠΈ аккумуляторах ΠΎΠ½ΠΈ Π²ΠΎΠ·Π½ΠΈΠΊΠ°ΡŽΡ‚ Π² Ρ€Π΅Π·ΡƒΠ»ΡŒΡ‚Π°Ρ‚Π΅ элСктрохимичСских процСссов, Π² Π³Π΅Π½Π΅Ρ€Π°Ρ‚ΠΎΡ€Π°Ρ… постоянного Ρ‚ΠΎΠΊΠ° сторонниС силы Π²ΠΎΠ·Π½ΠΈΠΊΠ°ΡŽΡ‚ ΠΏΡ€ΠΈ Π΄Π²ΠΈΠΆΠ΅Π½ΠΈΠΈ ΠΏΡ€ΠΎΠ²ΠΎΠ΄Π½ΠΈΠΊΠΎΠ² Π² ΠΌΠ°Π³Π½ΠΈΡ‚Π½ΠΎΠΌ ΠΏΠΎΠ»Π΅. Под дСйствиСм сторонних сил элСктричСскиС заряды двиТутся Π²Π½ΡƒΡ‚Ρ€ΠΈ источника Ρ‚ΠΎΠΊΠ° ΠΏΡ€ΠΎΡ‚ΠΈΠ² сил элСктростатичСского поля, благодаря Ρ‡Π΅ΠΌΡƒ Π² Π·Π°ΠΌΠΊΠ½ΡƒΡ‚ΠΎΠΉ Ρ†Π΅ΠΏΠΈ ΠΌΠΎΠΆΠ΅Ρ‚ ΠΏΠΎΠ΄Π΄Π΅Ρ€ΠΆΠΈΠ²Π°Ρ‚ΡŒΡΡ постоянный элСктричСский Ρ‚ΠΎΠΊ.

ΠŸΡ€ΠΈ ΠΏΠ΅Ρ€Π΅ΠΌΠ΅Ρ‰Π΅Π½ΠΈΠΈ элСктричСских зарядов ΠΏΠΎ Ρ†Π΅ΠΏΠΈ постоянного Ρ‚ΠΎΠΊΠ° сторонниС силы, Π΄Π΅ΠΉΡΡ‚Π²ΡƒΡŽΡ‰ΠΈΠ΅ Π²Π½ΡƒΡ‚Ρ€ΠΈ источников Ρ‚ΠΎΠΊΠ°, ΡΠΎΠ²Π΅Ρ€ΡˆΠ°ΡŽΡ‚ Ρ€Π°Π±ΠΎΡ‚Ρƒ. ЀизичСская Π²Π΅Π»ΠΈΡ‡ΠΈΠ½Π°, равная ΠΎΡ‚Π½ΠΎΡˆΠ΅Π½ΠΈΡŽ Ρ€Π°Π±ΠΎΡ‚Ρ‹ Aст сторонних сил ΠΏΡ€ΠΈ ΠΏΠ΅Ρ€Π΅ΠΌΠ΅Ρ‰Π΅Π½ΠΈΠΈ заряда

q ΠΎΡ‚ ΠΎΡ‚Ρ€ΠΈΡ†Π°Ρ‚Π΅Π»ΡŒΠ½ΠΎΠ³ΠΎ полюса источника Ρ‚ΠΎΠΊΠ° ΠΊ ΠΏΠΎΠ»ΠΎΠΆΠΈΡ‚Π΅Π»ΡŒΠ½ΠΎΠΌΡƒ ΠΊ Π²Π΅Π»ΠΈΡ‡ΠΈΠ½Π΅ этого заряда, называСтся элСктродвиТущСй силой источника (Π­Π”Π‘):

Π’Π°ΠΊΠΈΠΌ ΠΎΠ±Ρ€Π°Π·ΠΎΠΌ, Π­Π”Π‘ опрСдСляСтся Ρ€Π°Π±ΠΎΡ‚ΠΎΠΉ, ΡΠΎΠ²Π΅Ρ€ΡˆΠ°Π΅ΠΌΠΎΠΉ сторонними силами ΠΏΡ€ΠΈ ΠΏΠ΅Ρ€Π΅ΠΌΠ΅Ρ‰Π΅Π½ΠΈΠΈ Π΅Π΄ΠΈΠ½ΠΈΡ‡Π½ΠΎΠ³ΠΎ ΠΏΠΎΠ»ΠΎΠΆΠΈΡ‚Π΅Π»ΡŒΠ½ΠΎΠ³ΠΎ заряда. ЭлСктродвиТущая сила, ΠΊΠ°ΠΊ ΠΈ Ρ€Π°Π·Π½ΠΎΡΡ‚ΡŒ ΠΏΠΎΡ‚Π΅Π½Ρ†ΠΈΠ°Π»ΠΎΠ², измСряСтся Π²Β Π²ΠΎΠ»ΡŒΡ‚Π°Ρ… (Π’).

Π—Π°ΠΊΠΎΠ½ Ома для ΠΏΠΎΠ»Π½ΠΎΠΉ (Π·Π°ΠΌΠΊΠ½ΡƒΡ‚ΠΎΠΉ) Ρ†Π΅ΠΏΠΈ: сила Ρ‚ΠΎΠΊΠ° Π² Π·Π°ΠΌΠΊΠ½ΡƒΡ‚ΠΎΠΉ Ρ†Π΅ΠΏΠΈ Ρ€Π°Π²Π½Π° элСктродвиТущСй силС источника, Π΄Π΅Π»Π΅Π½Π½ΠΎΠΉ Π½Π° ΠΎΠ±Ρ‰Π΅Π΅ (Π²Π½ΡƒΡ‚Ρ€Π΅Π½Π½Π΅Π΅ + внСшнСС) сопротивлСниС Ρ†Π΅ΠΏΠΈ:

Π‘ΠΎΠΏΡ€ΠΎΡ‚ΠΈΠ²Π»Π΅Π½ΠΈΠ΅ r – Π²Π½ΡƒΡ‚Ρ€Π΅Π½Π½Π΅Π΅ (собствСнноС) сопротивлСниС источника Ρ‚ΠΎΠΊΠ° (зависит ΠΎΡ‚ Π²Π½ΡƒΡ‚Ρ€Π΅Π½Π½Π΅Π³ΠΎ строСния источника). Π‘ΠΎΠΏΡ€ΠΎΡ‚ΠΈΠ²Π»Π΅Π½ΠΈΠ΅

R – сопротивлСниС Π½Π°Π³Ρ€ΡƒΠ·ΠΊΠΈ (внСшнСС сопротивлСниС Ρ†Π΅ΠΏΠΈ).

ПадСниС напряТСния Π²ΠΎ внСшнСй Ρ†Π΅ΠΏΠΈ ΠΏΡ€ΠΈ этом Ρ€Π°Π²Π½ΠΎ (Π΅Π³ΠΎ Π΅Ρ‰Π΅ Π½Π°Π·Ρ‹Π²Π°ΡŽΡ‚ напряТСниСм Π½Π° ΠΊΠ»Π΅ΠΌΠΌΠ°Ρ… источника):

Π’Π°ΠΆΠ½ΠΎ ΠΏΠΎΠ½ΡΡ‚ΡŒ ΠΈ Π·Π°ΠΏΠΎΠΌΠ½ΠΈΡ‚ΡŒ: Π­Π”Π‘ ΠΈ Π²Π½ΡƒΡ‚Ρ€Π΅Π½Π½Π΅Π΅ сопротивлСниС источника Ρ‚ΠΎΠΊΠ° Π½Π΅ ΠΌΠ΅Π½ΡΡŽΡ‚ΡΡ, ΠΏΡ€ΠΈ ΠΏΠΎΠ΄ΠΊΠ»ΡŽΡ‡Π΅Π½ΠΈΠΈ Ρ€Π°Π·Π½Ρ‹Ρ… Π½Π°Π³Ρ€ΡƒΠ·ΠΎΠΊ.

Если сопротивлСниС Π½Π°Π³Ρ€ΡƒΠ·ΠΊΠΈ Ρ€Π°Π²Π½ΠΎ Π½ΡƒΠ»ΡŽ (источник замыкаСтся сам Π½Π° сСбя) ΠΈΠ»ΠΈ ΠΌΠ½ΠΎΠ³ΠΎ мСньшС сопротивлСния источника, Ρ‚ΠΎ Ρ‚ΠΎΠ³Π΄Π° Π² Ρ†Π΅ΠΏΠΈ ΠΏΠΎΡ‚Π΅Ρ‡Π΅Ρ‚ Ρ‚ΠΎΠΊ ΠΊΠΎΡ€ΠΎΡ‚ΠΊΠΎΠ³ΠΎ замыкания:

Π‘ΠΈΠ»Π° Ρ‚ΠΎΠΊΠ° ΠΊΠΎΡ€ΠΎΡ‚ΠΊΠΎΠ³ΠΎ замыкания – максимальная сила Ρ‚ΠΎΠΊΠ°, ΠΊΠΎΡ‚ΠΎΡ€ΡƒΡŽ ΠΌΠΎΠΆΠ½ΠΎ ΠΏΠΎΠ»ΡƒΡ‡ΠΈΡ‚ΡŒ ΠΎΡ‚ Π΄Π°Π½Π½ΠΎΠ³ΠΎ источника с элСктродвиТущСй силой Ρ и Π²Π½ΡƒΡ‚Ρ€Π΅Π½Π½ΠΈΠΌ сопротивлСниСм r. Π£ источников с ΠΌΠ°Π»Ρ‹ΠΌ Π²Π½ΡƒΡ‚Ρ€Π΅Π½Π½ΠΈΠΌ сопротивлСниСм Ρ‚ΠΎΠΊ ΠΊΠΎΡ€ΠΎΡ‚ΠΊΠΎΠ³ΠΎ замыкания ΠΌΠΎΠΆΠ΅Ρ‚ Π±Ρ‹Ρ‚ΡŒ ΠΎΡ‡Π΅Π½ΡŒ Π²Π΅Π»ΠΈΠΊ, ΠΈ Π²Ρ‹Π·Ρ‹Π²Π°Ρ‚ΡŒ Ρ€Π°Π·Ρ€ΡƒΡˆΠ΅Π½ΠΈΠ΅ элСктричСской Ρ†Π΅ΠΏΠΈ ΠΈΠ»ΠΈ источника. НапримСр, Ρƒ свинцовых аккумуляторов, ΠΈΡΠΏΠΎΠ»ΡŒΠ·ΡƒΠ΅ΠΌΡ‹Ρ… Π² автомобилях, сила Ρ‚ΠΎΠΊΠ° ΠΊΠΎΡ€ΠΎΡ‚ΠΊΠΎΠ³ΠΎ замыкания ΠΌΠΎΠΆΠ΅Ρ‚ ΡΠΎΡΡ‚Π°Π²Π»ΡΡ‚ΡŒ нСсколько сотСн Π°ΠΌΠΏΠ΅Ρ€. ОсобСнно опасны ΠΊΠΎΡ€ΠΎΡ‚ΠΊΠΈΠ΅ замыкания Π² ΠΎΡΠ²Π΅Ρ‚ΠΈΡ‚Π΅Π»ΡŒΠ½Ρ‹Ρ… сСтях, ΠΏΠΈΡ‚Π°Π΅ΠΌΡ‹Ρ… ΠΎΡ‚ подстанций (тысячи Π°ΠΌΠΏΠ΅Ρ€). Π§Ρ‚ΠΎΠ±Ρ‹ ΠΈΠ·Π±Π΅ΠΆΠ°Ρ‚ΡŒ Ρ€Π°Π·Ρ€ΡƒΡˆΠΈΡ‚Π΅Π»ΡŒΠ½ΠΎΠ³ΠΎ дСйствия Ρ‚Π°ΠΊΠΈΡ… Π±ΠΎΠ»ΡŒΡˆΠΈΡ… Ρ‚ΠΎΠΊΠΎΠ², Π² Ρ†Π΅ΠΏΡŒ Π²ΠΊΠ»ΡŽΡ‡Π°ΡŽΡ‚ΡΡ ΠΏΡ€Π΅Π΄ΠΎΡ…Ρ€Π°Π½ΠΈΡ‚Π΅Π»ΠΈ ΠΈΠ»ΠΈ ΡΠΏΠ΅Ρ†ΠΈΠ°Π»ΡŒΠ½Ρ‹Π΅ Π°Π²Ρ‚ΠΎΠΌΠ°Ρ‚Ρ‹ Π·Π°Ρ‰ΠΈΡ‚Ρ‹ сСтСй.

НСсколько источников Π­Π”Π‘ Π² Ρ†Π΅ΠΏΠΈ

Если Π² Ρ†Π΅ΠΏΠΈ присутствуСт нСсколько Π­Π”Π‘ ΠΏΠΎΠ΄ΠΊΠ»ΡŽΡ‡Π΅Π½Π½Ρ‹Ρ… ΠΏΠΎΡΠ»Π΅Π΄ΠΎΠ²Π°Ρ‚Π΅Π»ΡŒΠ½ΠΎ, Ρ‚ΠΎ:

1. ΠŸΡ€ΠΈ ΠΏΡ€Π°Π²ΠΈΠ»ΡŒΠ½ΠΎΠΌ (ΠΏΠΎΠ»ΠΎΠΆΠΈΡ‚Π΅Π»ΡŒΠ½Ρ‹ΠΉ полюс ΠΎΠ΄Π½ΠΎΠ³ΠΎ источника присоСдиняСтся ΠΊ ΠΎΡ‚Ρ€ΠΈΡ†Π°Ρ‚Π΅Π»ΡŒΠ½ΠΎΠΌΡƒ Π΄Ρ€ΡƒΠ³ΠΎΠ³ΠΎ) ΠΏΠΎΠ΄ΠΊΠ»ΡŽΡ‡Π΅Π½ΠΈΠΈ источников ΠΎΠ±Ρ‰Π΅Π΅ Π­Π”Π‘ всСх источников ΠΈ ΠΈΡ… Π²Π½ΡƒΡ‚Ρ€Π΅Π½Π½Π΅Π΅ сопротивлСниС ΠΌΠΎΠΆΠ΅Ρ‚ Π±Ρ‹Ρ‚ΡŒ Π½Π°ΠΉΠ΄Π΅Π½ΠΎ ΠΏΠΎ Ρ„ΠΎΡ€ΠΌΡƒΠ»Π°ΠΌ:

НапримСр, Ρ‚Π°ΠΊΠΎΠ΅ ΠΏΠΎΠ΄ΠΊΠ»ΡŽΡ‡Π΅Π½ΠΈΠ΅ источников осущСствляСтся Π² ΠΏΡƒΠ»ΡŒΡ‚Π°Ρ… дистанционного управлСния, Ρ„ΠΎΡ‚ΠΎΠ°ΠΏΠΏΠ°Ρ€Π°Ρ‚Π°Ρ… ΠΈ Π΄Ρ€ΡƒΠ³ΠΈΡ… Π±Ρ‹Ρ‚ΠΎΠ²Ρ‹Ρ… ΠΏΡ€ΠΈΠ±ΠΎΡ€Π°Ρ…, Ρ€Π°Π±ΠΎΡ‚Π°ΡŽΡ‰ΠΈΡ… ΠΎΡ‚ Π½Π΅ΡΠΊΠΎΠ»ΡŒΠΊΠΈΡ… Π±Π°Ρ‚Π°Ρ€Π΅Π΅ΠΊ.

2. ΠŸΡ€ΠΈ Π½Π΅ΠΏΡ€Π°Π²ΠΈΠ»ΡŒΠ½ΠΎΠΌ (источники ΡΠΎΠ΅Π΄ΠΈΠ½ΡΡŽΡ‚ΡΡ ΠΎΠ΄ΠΈΠ½Π°ΠΊΠΎΠ²Ρ‹ΠΌΠΈ полюсами) ΠΏΠΎΠ΄ΠΊΠ»ΡŽΡ‡Π΅Π½ΠΈΠΈ источников ΠΈΡ… ΠΎΠ±Ρ‰Π΅Π΅ Π­Π”Π‘ ΠΈ сопротивлСниС рассчитываСтся ΠΏΠΎ Ρ„ΠΎΡ€ΠΌΡƒΠ»Π°ΠΌ:

Π’ ΠΎΠ±ΠΎΠΈΡ… случаях ΠΎΠ±Ρ‰Π΅Π΅ сопротивлСниС источников увСличиваСтся.

ΠŸΡ€ΠΈ ΠΏΠ°Ρ€Π°Π»Π»Π΅Π»ΡŒΠ½ΠΎΠΌ ΠΏΠΎΠ΄ΠΊΠ»ΡŽΡ‡Π΅Π½ΠΈΠΈ ΠΈΠΌΠ΅Π΅Ρ‚ смысл ΡΠΎΠ΅Π΄ΠΈΠ½ΡΡ‚ΡŒ источники Ρ‚ΠΎΠ»ΡŒΠΊΠΎ c ΠΎΠ΄ΠΈΠ½Π°ΠΊΠΎΠ²ΠΎΠΉ Π­Π”Π‘, ΠΈΠ½Π°Ρ‡Π΅ источники Π±ΡƒΠ΄ΡƒΡ‚ Ρ€Π°Π·Ρ€ΡΠΆΠ°Ρ‚ΡŒΡΡ Π΄Ρ€ΡƒΠ³ Π½Π° Π΄Ρ€ΡƒΠ³Π°. Π’Π°ΠΊΠΈΠΌ ΠΎΠ±Ρ€Π°Π·ΠΎΠΌ суммарноС Π­Π”Π‘ Π±ΡƒΠ΄Π΅Ρ‚ Ρ‚Π°ΠΊΠΈΠΌ ΠΆΠ΅, ΠΊΠ°ΠΊ ΠΈ Π­Π”Π‘ ΠΊΠ°ΠΆΠ΄ΠΎΠ³ΠΎ источника, Ρ‚ΠΎ Π΅ΡΡ‚ΡŒ ΠΏΡ€ΠΈ ΠΏΠ°Ρ€Π°Π»Π»Π΅Π»ΡŒΠ½ΠΎΠΌ соСдинСнии ΠΌΡ‹ Π½Π΅ ΠΏΠΎΠ»ΡƒΡ‡ΠΈΠΌ Π±Π°Ρ‚Π°Ρ€Π΅ΡŽ с большим Π­Π”Π‘. ΠŸΡ€ΠΈ этом ΡƒΠΌΠ΅Π½ΡŒΡˆΠ°Π΅Ρ‚ΡΡ Π²Π½ΡƒΡ‚Ρ€Π΅Π½Π½Π΅Π΅ сопротивлСниС Π±Π°Ρ‚Π°Ρ€Π΅ΠΈ источников, Ρ‡Ρ‚ΠΎ позволяСт ΠΏΠΎΠ»ΡƒΡ‡Π°Ρ‚ΡŒ Π±ΠΎΠ»ΡŒΡˆΡƒΡŽ силу Ρ‚ΠΎΠΊΠ° ΠΈ ΠΌΠΎΡ‰Π½ΠΎΡΡ‚ΡŒ Π² Ρ†Π΅ΠΏΠΈ:

Π’ этом ΠΈ состоит смысл ΠΏΠ°Ρ€Π°Π»Π»Π΅Π»ΡŒΠ½ΠΎΠ³ΠΎ соСдинСния источников. Π’ любом случаС ΠΏΡ€ΠΈ Ρ€Π΅ΡˆΠ΅Π½ΠΈΠΈ Π·Π°Π΄Π°Ρ‡ сначала Π½Π°Π΄ΠΎ Π½Π°ΠΉΡ‚ΠΈ ΡΡƒΠΌΠΌΠ°Ρ€Π½ΡƒΡŽ Π­Π”Π‘ ΠΈ ΠΏΠΎΠ»Π½ΠΎΠ΅ Π²Π½ΡƒΡ‚Ρ€Π΅Π½Π½Π΅Π΅ сопротивлСниС ΠΏΠΎΠ»ΡƒΡ‡ΠΈΠ²ΡˆΠ΅Π³ΠΎΡΡ источника, Π° Π·Π°Ρ‚Π΅ΠΌ Π·Π°ΠΏΠΈΡΠ°Ρ‚ΡŒ Π·Π°ΠΊΠΎΠ½ Ома для ΠΏΠΎΠ»Π½ΠΎΠΉ Ρ†Π΅ΠΏΠΈ.

Β 

Π Π°Π±ΠΎΡ‚Π° ΠΈ ΠΌΠΎΡ‰Π½ΠΎΡΡ‚ΡŒ Ρ‚ΠΎΠΊΠ°. Π—Π°ΠΊΠΎΠ½ ДТоуля-Π›Π΅Π½Ρ†Π°

К оглавлСнию. ..

Π Π°Π±ΠΎΡ‚Π° A элСктричСского Ρ‚ΠΎΠΊΠ° I, ΠΏΡ€ΠΎΡ‚Π΅ΠΊΠ°ΡŽΡ‰Π΅Π³ΠΎ ΠΏΠΎ Π½Π΅ΠΏΠΎΠ΄Π²ΠΈΠΆΠ½ΠΎΠΌΡƒ ΠΏΡ€ΠΎΠ²ΠΎΠ΄Π½ΠΈΠΊΡƒ с сопротивлСниСм R, прСобразуСтся Π² Ρ‚Π΅ΠΏΠ»ΠΎΡ‚ΡƒΒ Q, Π²Ρ‹Π΄Π΅Π»ΡΡŽΡ‰Π΅Π΅ΡΡ Π½Π° ΠΏΡ€ΠΎΠ²ΠΎΠ΄Π½ΠΈΠΊΠ΅. Π­Ρ‚Ρƒ Ρ€Π°Π±ΠΎΡ‚Ρƒ ΠΌΠΎΠΆΠ½ΠΎ Ρ€Π°ΡΡΡ‡ΠΈΡ‚Π°Ρ‚ΡŒ ΠΏΠΎ ΠΎΠ΄Π½ΠΎΠΉ ΠΈΠ· Ρ„ΠΎΡ€ΠΌΡƒΠ» (с ΡƒΡ‡Π΅Ρ‚ΠΎΠΌ Π·Π°ΠΊΠΎΠ½Π° Ома всС ΠΎΠ½ΠΈ ΡΠ»Π΅Π΄ΡƒΡŽΡ‚ Π΄Ρ€ΡƒΠ³ ΠΈΠ· Π΄Ρ€ΡƒΠ³Π°):

Π—Π°ΠΊΠΎΠ½ прСобразования Ρ€Π°Π±ΠΎΡ‚Ρ‹ Ρ‚ΠΎΠΊΠ° Π² Ρ‚Π΅ΠΏΠ»ΠΎ Π±Ρ‹Π» ΡΠΊΡΠΏΠ΅Ρ€ΠΈΠΌΠ΅Π½Ρ‚Π°Π»ΡŒΠ½ΠΎ установлСн нСзависимо Π΄Ρ€ΡƒΠ³ ΠΎΡ‚ Π΄Ρ€ΡƒΠ³Π° Π”ΠΆ.Π”ΠΆΠΎΡƒΠ»Π΅ΠΌ ΠΈ Π­.Π›Π΅Π½Ρ†Π΅ΠΌ ΠΈ носит Π½Π°Π·Π²Π°Π½ΠΈΠ΅ Π·Π°ΠΊΠΎΠ½Π° ДТоуля–ЛСнца. ΠœΠΎΡ‰Π½ΠΎΡΡ‚ΡŒ элСктричСского Ρ‚ΠΎΠΊΠ° Ρ€Π°Π²Π½Π° ΠΎΡ‚Π½ΠΎΡˆΠ΅Π½ΠΈΡŽ Ρ€Π°Π±ΠΎΡ‚Ρ‹ Ρ‚ΠΎΠΊΠ° A ΠΊ ΠΈΠ½Ρ‚Π΅Ρ€Π²Π°Π»Ρƒ Π²Ρ€Π΅ΠΌΠ΅Π½ΠΈ Ξ”t, Π·Π° ΠΊΠΎΡ‚ΠΎΡ€ΠΎΠ΅ эта Ρ€Π°Π±ΠΎΡ‚Π° Π±Ρ‹Π»Π° ΡΠΎΠ²Π΅Ρ€ΡˆΠ΅Π½Π°, поэтому ΠΎΠ½Π° ΠΌΠΎΠΆΠ΅Ρ‚ Π±Ρ‹Ρ‚ΡŒ рассчитана ΠΏΠΎ ΡΠ»Π΅Π΄ΡƒΡŽΡ‰ΠΈΠΌ Ρ„ΠΎΡ€ΠΌΡƒΠ»Π°ΠΌ:

Π Π°Π±ΠΎΡ‚Π° элСктричСского Ρ‚ΠΎΠΊΠ° Π² БИ, ΠΊΠ°ΠΊ ΠΎΠ±Ρ‹Ρ‡Π½ΠΎ, выраТаСтся Π² дТоулях (Π”ΠΆ), ΠΌΠΎΡ‰Π½ΠΎΡΡ‚ΡŒ – Π² Π²Π°Ρ‚Ρ‚Π°Ρ… (Π’Ρ‚).

Β 

ЭнСргобаланс Π·Π°ΠΌΠΊΠ½ΡƒΡ‚ΠΎΠΉ Ρ†Π΅ΠΏΠΈ

К оглавлСнию…

Рассмотрим Ρ‚Π΅ΠΏΠ΅Ρ€ΡŒ ΠΏΠΎΠ»Π½ΡƒΡŽ Ρ†Π΅ΠΏΡŒ постоянного Ρ‚ΠΎΠΊΠ°, ΡΠΎΡΡ‚ΠΎΡΡ‰ΡƒΡŽ ΠΈΠ· источника с элСктродвиТущСй силой Ρ и Π²Π½ΡƒΡ‚Ρ€Π΅Π½Π½ΠΈΠΌ сопротивлСниСм r ΠΈ внСшнСго ΠΎΠ΄Π½ΠΎΡ€ΠΎΠ΄Π½ΠΎΠ³ΠΎ участка с сопротивлСниСм R. Π’ этом случаС полСзная ΠΌΠΎΡ‰Π½ΠΎΡΡ‚ΡŒ ΠΈΠ»ΠΈ ΠΌΠΎΡ‰Π½ΠΎΡΡ‚ΡŒ, выдСляСмая Π²ΠΎ внСшнСй Ρ†Π΅ΠΏΠΈ:

Максимально возмоТная полСзная ΠΌΠΎΡ‰Π½ΠΎΡΡ‚ΡŒ источника достигаСтся, Ссли R = r ΠΈ Ρ€Π°Π²Π½Π°:

Если ΠΏΡ€ΠΈ ΠΏΠΎΠ΄ΠΊΠ»ΡŽΡ‡Π΅Π½ΠΈΠΈ ΠΊ ΠΎΠ΄Π½ΠΎΠΌΡƒ ΠΈ Ρ‚ΠΎΠΌΡƒ ΠΆΠ΅ источнику Ρ‚ΠΎΠΊΠ° Ρ€Π°Π·Π½Ρ‹Ρ… сопротивлСний R1 ΠΈ R2 Π½Π° Π½ΠΈΡ… Π²Ρ‹Π΄Π΅Π»ΡΡŽΡ‚ΡΡ Ρ€Π°Π²Π½Ρ‹Π΅ мощности Ρ‚ΠΎ Π²Π½ΡƒΡ‚Ρ€Π΅Π½Π½Π΅Π΅ сопротивлСниС этого источника Ρ‚ΠΎΠΊΠ° ΠΌΠΎΠΆΠ΅Ρ‚ Π±Ρ‹Ρ‚ΡŒ Π½Π°ΠΉΠ΄Π΅Π½ΠΎ ΠΏΠΎ Ρ„ΠΎΡ€ΠΌΡƒΠ»Π΅:

ΠœΠΎΡ‰Π½ΠΎΡΡ‚ΡŒ ΠΏΠΎΡ‚Π΅Ρ€ΡŒ ΠΈΠ»ΠΈ ΠΌΠΎΡ‰Π½ΠΎΡΡ‚ΡŒ Π²Π½ΡƒΡ‚Ρ€ΠΈ источника Ρ‚ΠΎΠΊΠ°:

Полная ΠΌΠΎΡ‰Π½ΠΎΡΡ‚ΡŒ, развиваСмая источником Ρ‚ΠΎΠΊΠ°:

ΠšΠŸΠ” источника Ρ‚ΠΎΠΊΠ°:

Β 

Π­Π»Π΅ΠΊΡ‚Ρ€ΠΎΠ»ΠΈΠ·

К оглавлСнию…

Π­Π»Π΅ΠΊΡ‚Ρ€ΠΎΠ»ΠΈΡ‚Π°ΠΌΠΈ принято Π½Π°Π·Ρ‹Π²Π°Ρ‚ΡŒ проводящиС срСды, Π² ΠΊΠΎΡ‚ΠΎΡ€Ρ‹Ρ… ΠΏΡ€ΠΎΡ‚Π΅ΠΊΠ°Π½ΠΈΠ΅ элСктричСского Ρ‚ΠΎΠΊΠ° сопровоТдаСтся пСрСносом вСщСства. НоситСлями свободных зарядов Π² элСктролитах ΡΠ²Π»ΡΡŽΡ‚ΡΡ ΠΏΠΎΠ»ΠΎΠΆΠΈΡ‚Π΅Π»ΡŒΠ½ΠΎ ΠΈ ΠΎΡ‚Ρ€ΠΈΡ†Π°Ρ‚Π΅Π»ΡŒΠ½ΠΎ заряТСнныС ΠΈΠΎΠ½Ρ‹. К элСктролитам относятся ΠΌΠ½ΠΎΠ³ΠΈΠ΅ соСдинСния ΠΌΠ΅Ρ‚Π°Π»Π»ΠΎΠ² с ΠΌΠ΅Ρ‚Π°Π»Π»ΠΎΠΈΠ΄Π°ΠΌΠΈ Π² расплавлСнном состоянии, Π° Ρ‚Π°ΠΊΠΆΠ΅ Π½Π΅ΠΊΠΎΡ‚ΠΎΡ€Ρ‹Π΅ Ρ‚Π²Π΅Ρ€Π΄Ρ‹Π΅ вСщСства. Однако основными прСдставитСлями элСктролитов, ΡˆΠΈΡ€ΠΎΠΊΠΎ ΠΈΡΠΏΠΎΠ»ΡŒΠ·ΡƒΠ΅ΠΌΡ‹ΠΌΠΈ Π² Ρ‚Π΅Ρ…Π½ΠΈΠΊΠ΅, ΡΠ²Π»ΡΡŽΡ‚ΡΡ Π²ΠΎΠ΄Π½Ρ‹Π΅ растворы нСорганичСских кислот, солСй ΠΈ оснований.

ΠŸΡ€ΠΎΡ…ΠΎΠΆΠ΄Π΅Π½ΠΈΠ΅ элСктричСского Ρ‚ΠΎΠΊΠ° Ρ‡Π΅Ρ€Π΅Π· элСктролит сопровоТдаСтся Π²Ρ‹Π΄Π΅Π»Π΅Π½ΠΈΠ΅ΠΌ вСщСства Π½Π° элСктродах. Π­Ρ‚ΠΎ явлСниС ΠΏΠΎΠ»ΡƒΡ‡ΠΈΠ»ΠΎ Π½Π°Π·Π²Π°Π½ΠΈΠ΅ элСктролиза.

ЭлСктричСский Ρ‚ΠΎΠΊ Π² элСктролитах прСдставляСт собой ΠΏΠ΅Ρ€Π΅ΠΌΠ΅Ρ‰Π΅Π½ΠΈΠ΅ ΠΈΠΎΠ½ΠΎΠ² ΠΎΠ±ΠΎΠΈΡ… Π·Π½Π°ΠΊΠΎΠ² Π² ΠΏΡ€ΠΎΡ‚ΠΈΠ²ΠΎΠΏΠΎΠ»ΠΎΠΆΠ½Ρ‹Ρ… направлСниях. ΠŸΠΎΠ»ΠΎΠΆΠΈΡ‚Π΅Π»ΡŒΠ½Ρ‹Π΅ ΠΈΠΎΠ½Ρ‹ двиТутся ΠΊ ΠΎΡ‚Ρ€ΠΈΡ†Π°Ρ‚Π΅Π»ΡŒΠ½ΠΎΠΌΡƒ элСктроду (ΠΊΠ°Ρ‚ΠΎΠ΄Ρƒ), ΠΎΡ‚Ρ€ΠΈΡ†Π°Ρ‚Π΅Π»ΡŒΠ½Ρ‹Π΅ ΠΈΠΎΠ½Ρ‹ – ΠΊ ΠΏΠΎΠ»ΠΎΠΆΠΈΡ‚Π΅Π»ΡŒΠ½ΠΎΠΌΡƒ элСктроду (Π°Π½ΠΎΠ΄Ρƒ). Π˜ΠΎΠ½Ρ‹ ΠΎΠ±ΠΎΠΈΡ… Π·Π½Π°ΠΊΠΎΠ² ΠΏΠΎΡΠ²Π»ΡΡŽΡ‚ΡΡ Π² Π²ΠΎΠ΄Π½Ρ‹Ρ… растворах солСй, кислот ΠΈ Ρ‰Π΅Π»ΠΎΡ‡Π΅ΠΉ Π² Ρ€Π΅Π·ΡƒΠ»ΡŒΡ‚Π°Ρ‚Π΅ расщСплСния части Π½Π΅ΠΉΡ‚Ρ€Π°Π»ΡŒΠ½Ρ‹Ρ… ΠΌΠΎΠ»Π΅ΠΊΡƒΠ». Π­Ρ‚ΠΎ явлСниС называСтся элСктролитичСской диссоциациСй.

Π—Π°ΠΊΠΎΠ½ элСктролиза Π±Ρ‹Π» ΡΠΊΡΠΏΠ΅Ρ€ΠΈΠΌΠ΅Π½Ρ‚Π°Π»ΡŒΠ½ΠΎ установлСн английским Ρ„ΠΈΠ·ΠΈΠΊΠΎΠΌ М.Π€Π°Ρ€Π°Π΄Π΅Π΅ΠΌ Π² 1833 Π³ΠΎΠ΄Ρƒ. Π—Π°ΠΊΠΎΠ½ ЀарадСя опрСдСляСт количСства ΠΏΠ΅Ρ€Π²ΠΈΡ‡Π½Ρ‹Ρ… ΠΏΡ€ΠΎΠ΄ΡƒΠΊΡ‚ΠΎΠ², Π²Ρ‹Π΄Π΅Π»ΡΡŽΡ‰ΠΈΡ…ΡΡ Π½Π° элСктродах ΠΏΡ€ΠΈ элСктролизС. Π˜Ρ‚Π°ΠΊ, масса m вСщСства, Π²Ρ‹Π΄Π΅Π»ΠΈΠ²ΡˆΠ΅Π³ΠΎΡΡ Π½Π° элСктродС, прямо ΠΏΡ€ΠΎΠΏΠΎΡ€Ρ†ΠΈΠΎΠ½Π°Π»ΡŒΠ½Π° заряду Q, ΠΏΡ€ΠΎΡˆΠ΅Π΄ΡˆΠ΅ΠΌΡƒ Ρ‡Π΅Ρ€Π΅Π· элСктролит:

Π’Π΅Π»ΠΈΡ‡ΠΈΠ½Ρƒ k Π½Π°Π·Ρ‹Π²Π°ΡŽΡ‚ элСктрохимичСским эквивалСнтом. Он ΠΌΠΎΠΆΠ΅Ρ‚ Π±Ρ‹Ρ‚ΡŒ рассчитан ΠΏΠΎ Ρ„ΠΎΡ€ΠΌΡƒΠ»Π΅:

Π³Π΄Π΅: n – Π²Π°Π»Π΅Π½Ρ‚Π½ΠΎΡΡ‚ΡŒ вСщСства, NA – постоянная Авогадро, M – молярная масса вСщСства, Π΅ – элСмСнтарный заряд. Иногда Ρ‚Π°ΠΊΠΆΠ΅ вводят ΡΠ»Π΅Π΄ΡƒΡŽΡ‰Π΅Π΅ ΠΎΠ±ΠΎΠ·Π½Π°Ρ‡Π΅Π½ΠΈΠ΅ для постоянной ЀарадСя:

Β 

ЭлСктричСский Ρ‚ΠΎΠΊ Π² Π³Π°Π·Π°Ρ… ΠΈ Π² Π²Π°ΠΊΡƒΡƒΠΌΠ΅

К оглавлСнию…

ЭлСктричСский Ρ‚ΠΎΠΊ Π² Π³Π°Π·Π°Ρ…

Π’ ΠΎΠ±Ρ‹Ρ‡Π½Ρ‹Ρ… условиях Π³Π°Π·Ρ‹ Π½Π΅ проводят элСктричСский Ρ‚ΠΎΠΊ. Π­Ρ‚ΠΎ ΠΎΠ±ΡŠΡΡΠ½ΡΠ΅Ρ‚ΡΡ элСктричСской Π½Π΅ΠΉΡ‚Ρ€Π°Π»ΡŒΠ½ΠΎΡΡ‚ΡŒΡŽ ΠΌΠΎΠ»Π΅ΠΊΡƒΠ» Π³Π°Π·ΠΎΠ² ΠΈ, ΡΠ»Π΅Π΄ΠΎΠ²Π°Ρ‚Π΅Π»ΡŒΠ½ΠΎ, отсутствиСм носитСлСй элСктричСских зарядов. Для Ρ‚ΠΎΠ³ΠΎ Ρ‡Ρ‚ΠΎΠ±Ρ‹ Π³Π°Π· стал ΠΏΡ€ΠΎΠ²ΠΎΠ΄Π½ΠΈΠΊΠΎΠΌ, ΠΎΡ‚ ΠΌΠΎΠ»Π΅ΠΊΡƒΠ» Π½Π΅ΠΎΠ±Ρ…ΠΎΠ΄ΠΈΠΌΠΎ ΠΎΡ‚ΠΎΡ€Π²Π°Ρ‚ΡŒ ΠΎΠ΄ΠΈΠ½ ΠΈΠ»ΠΈ нСсколько элСктронов. Π’ΠΎΠ³Π΄Π° появятся свободныС носитСля зарядов – элСктроны ΠΈ ΠΏΠΎΠ»ΠΎΠΆΠΈΡ‚Π΅Π»ΡŒΠ½Ρ‹Π΅ ΠΈΠΎΠ½Ρ‹. Π­Ρ‚ΠΎΡ‚ процСсс называСтся ΠΈΠΎΠ½ΠΈΠ·Π°Ρ†ΠΈΠ΅ΠΉ Π³Π°Π·ΠΎΠ².

Π˜ΠΎΠ½ΠΈΠ·ΠΈΡ€ΠΎΠ²Π°Ρ‚ΡŒ ΠΌΠΎΠ»Π΅ΠΊΡƒΠ»Ρ‹ Π³Π°Π·Π° ΠΌΠΎΠΆΠ½ΠΎ внСшним воздСйствиСм – ΠΈΠΎΠ½ΠΈΠ·Π°Ρ‚ΠΎΡ€ΠΎΠΌ. Π˜ΠΎΠ½ΠΈΠ·Π°Ρ‚ΠΎΡ€Π°ΠΌΠΈ ΠΌΠΎΠΆΠ΅Ρ‚ Π±Ρ‹Ρ‚ΡŒ: ΠΏΠΎΡ‚ΠΎΠΊ свСта, рСнтгСновскиС Π»ΡƒΡ‡ΠΈ, ΠΏΠΎΡ‚ΠΎΠΊ элСктронов ΠΈΠ»ΠΈ Ξ±-частиц. ΠœΠΎΠ»Π΅ΠΊΡƒΠ»Ρ‹ Π³Π°Π·Π° Ρ‚Π°ΠΊΠΆΠ΅ ΠΈΠΎΠ½ΠΈΠ·ΠΈΡ€ΡƒΡŽΡ‚ΡΡ ΠΏΡ€ΠΈ высокой Ρ‚Π΅ΠΌΠΏΠ΅Ρ€Π°Ρ‚ΡƒΡ€Π΅. Π˜ΠΎΠ½ΠΈΠ·Π°Ρ†ΠΈΡ ΠΏΡ€ΠΈΠ²ΠΎΠ΄ΠΈΡ‚ ΠΊ возникновСнию Π² Π³Π°Π·Π°Ρ… свободных носитСлСй зарядов – элСктронов, ΠΏΠΎΠ»ΠΎΠΆΠΈΡ‚Π΅Π»ΡŒΠ½Ρ‹Ρ… ΠΈΠΎΠ½ΠΎΠ², ΠΎΡ‚Ρ€ΠΈΡ†Π°Ρ‚Π΅Π»ΡŒΠ½Ρ‹Ρ… ΠΈΠΎΠ½ΠΎΠ² (элСктрон, объСдинившийся с Π½Π΅ΠΉΡ‚Ρ€Π°Π»ΡŒΠ½ΠΎΠΉ ΠΌΠΎΠ»Π΅ΠΊΡƒΠ»ΠΎΠΉ).

Если ΡΠΎΠ·Π΄Π°Ρ‚ΡŒ Π² пространствС, занятом ΠΈΠΎΠ½ΠΈΠ·ΠΈΡ€ΠΎΠ²Π°Π½Π½Ρ‹ΠΌ Π³Π°Π·ΠΎΠΌ, элСктричСскоС ΠΏΠΎΠ»Π΅, Ρ‚ΠΎ носитСли элСктричСских зарядов ΠΏΡ€ΠΈΠ΄ΡƒΡ‚ Π² упорядочСнноС Π΄Π²ΠΈΠΆΠ΅Π½ΠΈΠ΅ – Ρ‚Π°ΠΊ Π²ΠΎΠ·Π½ΠΈΠΊΠ°Π΅Ρ‚ элСктричСский Ρ‚ΠΎΠΊ Π² Π³Π°Π·Π°Ρ…. Если ΠΈΠΎΠ½ΠΈΠ·Π°Ρ‚ΠΎΡ€ пСрСстаСт Π΄Π΅ΠΉΡΡ‚Π²ΠΎΠ²Π°Ρ‚ΡŒ, Ρ‚ΠΎ Π³Π°Π· снова становится Π½Π΅ΠΉΡ‚Ρ€Π°Π»ΡŒΠ½Ρ‹ΠΌ, Ρ‚Π°ΠΊ ΠΊΠ°ΠΊ Π² Π½Π΅ΠΌ происходит рСкомбинация – ΠΎΠ±Ρ€Π°Π·ΠΎΠ²Π°Π½ΠΈΠ΅ Π½Π΅ΠΉΡ‚Ρ€Π°Π»ΡŒΠ½Ρ‹Ρ… Π°Ρ‚ΠΎΠΌΠΎΠ² ΠΈΠΎΠ½Π°ΠΌΠΈ ΠΈ элСктронами.

ЭлСктричСский Ρ‚ΠΎΠΊ Π² Π²Π°ΠΊΡƒΡƒΠΌΠ΅

Π’Π°ΠΊΡƒΡƒΠΌΠΎΠΌ называСтся такая ΡΡ‚Π΅ΠΏΠ΅Π½ΡŒ разрСТСния Π³Π°Π·Π°, ΠΏΡ€ΠΈ ΠΊΠΎΡ‚ΠΎΡ€ΠΎΠΌ ΠΌΠΎΠΆΠ½ΠΎ ΠΏΡ€Π΅Π½Π΅Π±Ρ€Π΅Ρ‡ΡŒ соударСниСм ΠΌΠ΅ΠΆΠ΄Ρƒ Π΅Π³ΠΎ ΠΌΠΎΠ»Π΅ΠΊΡƒΠ»Π°ΠΌΠΈ ΠΈ ΡΡ‡ΠΈΡ‚Π°Ρ‚ΡŒ, Ρ‡Ρ‚ΠΎ срСдняя Π΄Π»ΠΈΠ½Π° свободного ΠΏΡ€ΠΎΠ±Π΅Π³Π° ΠΏΡ€Π΅Π²Ρ‹ΡˆΠ°Π΅Ρ‚ Π»ΠΈΠ½Π΅ΠΉΠ½Ρ‹Π΅ Ρ€Π°Π·ΠΌΠ΅Ρ€Ρ‹ сосуда, Π² ΠΊΠΎΡ‚ΠΎΡ€ΠΎΠΌ Π³Π°Π· находится.

ЭлСктричСским Ρ‚ΠΎΠΊΠΎΠΌ Π² Π²Π°ΠΊΡƒΡƒΠΌΠ΅ Π½Π°Π·Ρ‹Π²Π°ΡŽΡ‚ ΠΏΡ€ΠΎΠ²ΠΎΠ΄ΠΈΠΌΠΎΡΡ‚ΡŒ мСТэлСктродного ΠΏΡ€ΠΎΠΌΠ΅ΠΆΡƒΡ‚ΠΊΠ° Π² состоянии Π²Π°ΠΊΡƒΡƒΠΌΠ°. ΠœΠΎΠ»Π΅ΠΊΡƒΠ» Π³Π°Π·Π° ΠΏΡ€ΠΈ этом ΡΡ‚ΠΎΠ»ΡŒ ΠΌΠ°Π»ΠΎ, Ρ‡Ρ‚ΠΎ процСссы ΠΈΡ… ΠΈΠΎΠ½ΠΈΠ·Π°Ρ†ΠΈΠΈ Π½Π΅ ΠΌΠΎΠ³ΡƒΡ‚ ΠΎΠ±Π΅ΡΠΏΠ΅Ρ‡ΠΈΡ‚ΡŒ Ρ‚Π°ΠΊΠΎΠ³ΠΎ числа элСктронов ΠΈ ΠΈΠΎΠ½ΠΎΠ², ΠΊΠΎΡ‚ΠΎΡ€Ρ‹Π΅ Π½Π΅ΠΎΠ±Ρ…ΠΎΠ΄ΠΈΠΌΡ‹ для ΠΈΠΎΠ½ΠΈΠ·Π°Ρ†ΠΈΠΈ. ΠŸΡ€ΠΎΠ²ΠΎΠ΄ΠΈΠΌΠΎΡΡ‚ΡŒ мСТэлСктродного ΠΏΡ€ΠΎΠΌΠ΅ΠΆΡƒΡ‚ΠΊΠ° Π² Π²Π°ΠΊΡƒΡƒΠΌΠ΅ ΠΌΠΎΠΆΠ΅Ρ‚ Π±Ρ‹Ρ‚ΡŒ обСспСчСна лишь с ΠΏΠΎΠΌΠΎΡ‰ΡŒΡŽ заряТСнных частиц, Π²ΠΎΠ·Π½ΠΈΠΊΡˆΠΈΡ… Π·Π° счСт эмиссионных явлСний Π½Π° элСктродах.

Π€ΠΎΡ€ΠΌΡƒΠ»Ρ‹ для Ρ€Π΅ΡˆΠ΅Π½ΠΈΡ Π·Π°Π΄Π°Ρ‡ ΠΏΠΎ Ρ„ΠΈΠ·ΠΈΠΊΠ΅. ЭлСктричСство – Π€ΠΈΠ·ΠΈΠΊΠ°

Π­Π›Π•ΠšΠ’Π Π˜Π§Π•Π‘Π’Π’Πž
НаимСнованиС ΠΏΠ°Ρ€Π°ΠΌΠ΅Ρ‚Ρ€Π°Π€ΠΎΡ€ΠΌΡƒΠ»Π°ΠžΠ±ΠΎΠ·Π½Π°Ρ‡Π΅Π½ΠΈΡ
Π—Π°ΠΊΠΎΠ½ ΠšΡƒΠ»ΠΎΠ½Π°Q1 ΠΈ Q2 ― Ρ‚ΠΎΡ‡Π΅Ρ‡Π½Ρ‹Π΅ заряды, Ξ΅0 = 8,85βˆ™10βˆ’12 Π€/ΠΌ ― элСктричСская постоянная, Ξ΅ ― диэлСктричСская ΠΏΡ€ΠΎΠ½ΠΈΡ†Π°Π΅ΠΌΠΎΡΡ‚ΡŒ срСды, r ― расстояниС ΠΌΠ΅ΠΆΠ΄Ρƒ зарядами
Π•ΠΌΠΊΠΎΡΡ‚ΡŒ плоского кондСнсатораΡ ― диэлСктричСская ΠΏΡ€ΠΎΠ½ΠΈΡ†Π°Π΅ΠΌΠΎΡΡ‚ΡŒ срСды ΠΌΠ΅ΠΆΠ΄Ρƒ пластинами, Ξ΅0 = 8,85βˆ™10βˆ’12 Π€/ΠΌ ― элСктричСская постоянная, S ― ΠΏΠ»ΠΎΡ‰Π°Π΄ΡŒ пластины, d ― расстояниС ΠΌΠ΅ΠΆΠ΄Ρƒ пластинами
Π•ΠΌΠΊΠΎΡΡ‚ΡŒ сфСричСского кондСнсатораΡ ― диэлСктричСская ΠΏΡ€ΠΎΠ½ΠΈΡ†Π°Π΅ΠΌΠΎΡΡ‚ΡŒ срСды ΠΌΠ΅ΠΆΠ΄Ρƒ сфСрами, Ξ΅0 = 8,85βˆ™10βˆ’12 Π€/ΠΌ ― элСктричСская постоянная, R1 ΠΈ R2 ― радиусы Π²Π½ΡƒΡ‚Ρ€Π΅Π½Π½Π΅ΠΉ ΠΈ внСшнСй сфСр соотвСтствСнно
ΠŸΠΎΡ‚Π΅Π½Ρ†ΠΈΠ°Π» элСктричСского поля, созданного Ρ‚ΠΎΡ‡Π΅Ρ‡Π½Ρ‹ΠΌ зарядомq ― заряд сфСры, R ― радиус сфСры, Ξ΅ ― диэлСктричСская ΠΏΡ€ΠΎΠ½ΠΈΡ†Π°Π΅ΠΌΠΎΡΡ‚ΡŒ срСды, Ξ΅0 = 8,85βˆ™10βˆ’12 Π€/ΠΌ ― элСктричСская постоянная, r ― расстояниС ΠΎΡ‚ Ρ†Π΅Π½Ρ‚Ρ€Π° сфСры
ΠŸΠΎΡ‚Π΅Π½Ρ†ΠΈΠ°Π» элСктричСского поля, созданного мСталличСской сфСрой Π½Π° расстоянии r ΠΎΡ‚ Ρ†Π΅Π½Ρ‚Ρ€Π° сфСры: Π²Π½ΡƒΡ‚Ρ€ΠΈ сфСры ΠΈ Π½Π° повСрхности (r ≀ R)Β Π²Π½Π΅ сфСры (r > R)q ― заряд сфСры, R ― радиус сфСры, Ξ΅ ― диэлСктричСская ΠΏΡ€ΠΎΠ½ΠΈΡ†Π°Π΅ΠΌΠΎΡΡ‚ΡŒ срСды, Ξ΅0 = 8,85βˆ™10βˆ’12 Π€/ΠΌ ― элСктричСская постоянная, r ― расстояниС ΠΎΡ‚ Ρ†Π΅Π½Ρ‚Ρ€Π° сфСры
Π’Π΅ΠΎΡ€Π΅ΠΌΠ° Гаусса-ΠžΡΡ‚Ρ€ΠΎΠ³Ρ€Π°Π΄ΡΠΊΠΎΠ³ΠΎS ― ΠΏΠ»ΠΎΡ‰Π°Π΄ΡŒ гауссовой повСрхности, Π•n ― Π½ΠΎΡ€ΠΌΠ°Π»ΡŒΠ½Π°Ρ ΠΊ повСрхности ΡΠΎΡΡ‚Π°Π²Π»ΡΡŽΡ‰Π°Ρ Π²Π΅ΠΊΡ‚ΠΎΡ€Π° напряТСнности элСктростатичСского поля, Q ― заряд, ΠΎΡ…Π²Π°Ρ‡Π΅Π½Π½Ρ‹ΠΉ ΠΏΠΎΠ²Π΅Ρ€Ρ…Π½ΠΎΡΡ‚ΡŒΡŽ интСгрирования, Ξ΅ ― диэлСктричСская ΠΏΡ€ΠΎΠ½ΠΈΡ†Π°Π΅ΠΌΠΎΡΡ‚ΡŒ срСды, Ξ΅0 = 8,85βˆ™10βˆ’12 Π€/ΠΌ ― элСктричСская постоянная
ΠΠ°ΠΏΡ€ΡΠΆΠ΅Π½Π½ΠΎΡΡ‚ΡŒ поля, создаваСмого зарядом бСсконСчной пластиныσ ― повСрхностная ΠΏΠ»ΠΎΡ‚Π½ΠΎΡΡ‚ΡŒ заряда, Ξ΅ ― диэлСктричСская ΠΏΡ€ΠΎΠ½ΠΈΡ†Π°Π΅ΠΌΠΎΡΡ‚ΡŒ срСды, Ξ΅0 = 8,85βˆ™10βˆ’12 Π€/ΠΌ ― элСктричСская постоянная, r ― расстояниС ΠΎΡ‚ пластины
ΠΠ°ΠΏΡ€ΡΠΆΠ΅Π½Π½ΠΎΡΡ‚ΡŒ элСктричСского поля, создаваСмого мСталличСской заряТСнной сфСрой: Π²Π½ΡƒΡ‚Ρ€ΠΈ сфСры (r < R) Π½Π° повСрхности сфСры (r = R) Π²Π½Π΅ сфСры (r > R)Ο„ ― линСйная ΠΏΠ»ΠΎΡ‚Π½ΠΎΡΡ‚ΡŒ заряда; Ξ΅ ― диэлСктричСская ΠΏΡ€ΠΎΠ½ΠΈΡ†Π°Π΅ΠΌΠΎΡΡ‚ΡŒ срСды ΠΌΠ΅ΠΆΠ΄Ρƒ пластинами, Ξ΅0 = 8,85βˆ™10βˆ’12 Π€/ΠΌ ― элСктричСская постоянная, r ― расстояниС ΠΎΡ‚ оси Π½ΠΈΡ‚ΠΈ
ЭнСргия кондСнсатораБ ― Π΅ΠΌΠΊΠΎΡΡ‚ΡŒ кондСнсатора; U ― напряТСниС Π½Π° пластинах
Π‘ΠΎΠΏΡ€ΠΎΡ‚ΠΈΠ²Π»Π΅Π½ΠΈΠ΅ проводаρ0 ― ΡƒΠ΄Π΅Π»ΡŒΠ½ΠΎΠ΅ сопротивлСниС ΠΌΠ°Ρ‚Π΅Ρ€ΠΈΠ°Π»Π° ΠΏΡ€ΠΎΠ²ΠΎΠ΄Π°, S ― ΠΏΠ»ΠΎΡ‰Π°Π΄ΡŒ сСчСния ΠΏΡ€ΠΎΠ²ΠΎΠ΄Π°; для ΠΌΠ΅Π΄ΠΈ ρ0 = 0,0175βˆ™10βˆ’6 ΠžΠΌβˆ™ΠΌ; для алюминия ρ0 = 0,028βˆ™10βˆ’6 ΠžΠΌβˆ™ΠΌ; для Π²ΠΎΠ»ΡŒΡ„Ρ€Π°ΠΌΠ° ρ0 = 0,055βˆ™10βˆ’6 ΠžΠΌβˆ™ΠΌ; для ΠΆΠ΅Π»Π΅Π·Π° ρ0 = 0,1βˆ™10βˆ’6 ΠžΠΌβˆ™ΠΌ
Π Π°Π±ΠΎΡ‚Π°, ΡΠΎΠ²Π΅Ρ€ΡˆΠ°Π΅ΠΌΠ°Ρ элСктричСским ΠΏΠΎΠ»Π΅ΠΌ ΠΏΡ€ΠΈ ΠΏΠ΅Ρ€Π΅ΠΌΠ΅Ρ‰Π΅Π½ΠΈΠΈ Ρ‚ΠΎΡ‡Π΅Ρ‡Π½ΠΎΠ³ΠΎ заряда q ΠΈΠ· Ρ‚ΠΎΡ‡ΠΊΠΈ 1 поля Π² Ρ‚ΠΎΡ‡ΠΊΡƒ 2Β  Β  Β  Β  Β  Β  Β  Β  Β  Β  Β  Β  Β  Β  Β  Β  Β  Β  Β  Β  Β  Β  Β  Β  Β  Β  Β  Β  Β  Β  Β  Β  Β  Β  Β  Β  Β  Β  Β  Β  Β  Β  Β  Β  Β  Β  Β  Β  Β  Β  Β  Β  Β  Β  Β  Β  Β  Β  Β  Β  Β  Β  Β  Β  Β  Β  Β  Β  Β  Β  Β  Β  Β  Β  Β  Β  Β  Β  Β  Β  Β  Β  Β  Β  Β  Β  Β  Β  Β  Β  Β  Β  Β  Β  Β  Β  Β  Β  Β  Β  Β  Β  Β  Β  Β  Β  Β  Β  Β  Β  Β  Β  Β  Β  Β  Β  Β  Β  Β  Β  Β  Β  Β  Β  Β  Β  Β  Β  Β  Β  Β  Β  Β  Β  Β  Β  Β  Β  Β  Β  Β  Β  Β  Β  Β  Β  Β  Β  Β  Β  Β  Β  Β  Β Ο†1 ΠΈ Ο†2 ― ΠΏΠΎΡ‚Π΅Π½Ρ†ΠΈΠ°Π»Ρ‹ Ρ‚ΠΎΡ‡Π΅ΠΊ 1 ΠΈ 2 соотвСтствСнно
ΠŸΠ΅Ρ€ΠΈΠΎΠ΄ ΠΊΠΎΠ»Π΅Π±Π°Π½ΠΈΠΉ ΠΊΠΎΠ»Π΅Π±Π°Ρ‚Π΅Π»ΡŒΠ½ΠΎΠ³ΠΎ ΠΊΠΎΠ½Ρ‚ΡƒΡ€Π°L ― ΠΈΠ½Π΄ΡƒΠΊΡ‚ΠΈΠ²Π½ΠΎΡΡ‚ΡŒ ΠΊΠ°Ρ‚ΡƒΡˆΠΊΠΈ, C ― Π΅ΠΌΠΊΠΎΡΡ‚ΡŒ кондСнсатора
Π˜Π½Π΄ΡƒΠΊΡ†ΠΈΡ ΠΌΠ°Π³Π½ΠΈΡ‚Π½ΠΎΠ³ΠΎ поля, создаваСмого бСсконСчно Π΄Π»ΠΈΠ½Π½Ρ‹ΠΌ прямым ΠΏΡ€ΠΎΠ²ΠΎΠ΄Π½ΠΈΠΊΠΎΠΌ с Ρ‚ΠΎΠΊΠΎΠΌ ΠΠ°ΠΏΡ€ΡΠΆΠ΅Π½Π½ΠΎΡΡ‚ΡŒ ΠΌΠ°Π³Π½ΠΈΡ‚Π½ΠΎΠ³ΠΎ поляμ ― магнитная ΠΏΡ€ΠΎΠ½ΠΈΡ†Π°Π΅ΠΌΠΎΡΡ‚ΡŒ срСды, ΞΌ0 = 4Ο€βˆ™10βˆ’7 Π“Π½/ΠΌ ― магнитная постоянная, I ― сила Ρ‚ΠΎΠΊΠ° Π² ΠΏΡ€ΠΎΠ²ΠΎΠ΄Π½ΠΈΠΊΠ΅, a ― расстояниС Π΄ΠΎ ΠΏΡ€ΠΎΠ²ΠΎΠ΄Π½ΠΈΠΊΠ°
Π˜Π½Π΄ΡƒΠΊΡ†ΠΈΡ ΠΌΠ°Π³Π½ΠΈΡ‚Π½ΠΎΠ³ΠΎ поля Π² Ρ†Π΅Π½Ρ‚Ρ€Π΅ ΠΊΡ€ΡƒΠ³ΠΎΠ²ΠΎΠ³ΠΎ ΠΏΡ€ΠΎΠ²ΠΎΠ΄Π½ΠΈΠΊΠ° с Ρ‚ΠΎΠΊΠΎΠΌ ΠΠ°ΠΏΡ€ΡΠΆΠ΅Π½Π½ΠΎΡΡ‚ΡŒ ΠΌΠ°Π³Π½ΠΈΡ‚Π½ΠΎΠ³ΠΎ поляμ ― магнитная ΠΏΡ€ΠΎΠ½ΠΈΡ†Π°Π΅ΠΌΠΎΡΡ‚ΡŒ срСды, ΞΌ0 = 4Ο€βˆ™10βˆ’7 Π“Π½/ΠΌ ― магнитная постоянная, I ― сила Ρ‚ΠΎΠΊΠ° Π² ΠΏΡ€ΠΎΠ²ΠΎΠ΄Π½ΠΈΠΊΠ΅, R ― радиус ΠΏΡ€ΠΎΠ²ΠΎΠ΄Π½ΠΈΠΊΠ°
Π˜Π½Π΄ΡƒΠΊΡ†ΠΈΡ ΠΌΠ°Π³Π½ΠΈΡ‚Π½ΠΎΠ³ΠΎ поля Π½Π° оси ΠΊΡ€ΡƒΠ³ΠΎΠ²ΠΎΠ³ΠΎ ΠΏΡ€ΠΎΠ²ΠΎΠ΄Π½ΠΈΠΊΠ° с Ρ‚ΠΎΠΊΠΎΠΌ ΠΠ°ΠΏΡ€ΡΠΆΠ΅Π½Π½ΠΎΡΡ‚ΡŒ ΠΌΠ°Π³Π½ΠΈΡ‚Π½ΠΎΠ³ΠΎ поляμ ― магнитная ΠΏΡ€ΠΎΠ½ΠΈΡ†Π°Π΅ΠΌΠΎΡΡ‚ΡŒ срСды, ΞΌ0 = 4Ο€βˆ™10βˆ’7 Π“Π½/ΠΌ ― магнитная постоянная, I ― сила Ρ‚ΠΎΠΊΠ° Π² ΠΏΡ€ΠΎΠ²ΠΎΠ΄Π½ΠΈΠΊΠ΅, R ― радиус ΠΏΡ€ΠΎΠ²ΠΎΠ΄Π½ΠΈΠΊΠ°, a ― расстояниС Π΄ΠΎ плоскости ΠΏΡ€ΠΎΠ²ΠΎΠ΄Π½ΠΈΠΊΠ°
Π˜Π½Π΄ΡƒΠΊΡ†ΠΈΡ ΠΌΠ°Π³Π½ΠΈΡ‚Π½ΠΎΠ³ΠΎ поля Π²Π½ΡƒΡ‚Ρ€ΠΈ Π΄Π»ΠΈΠ½Π½ΠΎΠ³ΠΎ солСноидаμ ― магнитная ΠΏΡ€ΠΎΠ½ΠΈΡ†Π°Π΅ΠΌΠΎΡΡ‚ΡŒ срСды, ΞΌ0 = 4Ο€βˆ™10βˆ’7 Π“Π½/ΠΌ ― магнитная постоянная, I ― сила Ρ‚ΠΎΠΊΠ° Π² ΠΏΡ€ΠΎΠ²ΠΎΠ΄Π½ΠΈΠΊΠ΅, N ― количСство Π²ΠΈΡ‚ΠΊΠΎΠ², l ― Π΄Π»ΠΈΠ½Π° солСноида
ΠœΠ°Π³Π½ΠΈΡ‚Π½Π°Ρ индукция поля, создаваСмая ΠΎΡ‚Ρ€Π΅Π·ΠΊΠΎΠΌ ΠΏΡ€ΠΎΠ²ΠΎΠ΄Π½ΠΈΠΊΠ°ΞΌ ― магнитная ΠΏΡ€ΠΎΠ½ΠΈΡ†Π°Π΅ΠΌΠΎΡΡ‚ΡŒ срСды, ΞΌ0 = 4Ο€βˆ™10βˆ’7 Π“Π½/ΠΌ ― магнитная постоянная, a ― расстояниС Π΄ΠΎ оси ΠΏΡ€ΠΎΠ²ΠΎΠ΄Π½ΠΈΠΊΠ°, Ξ±1 ΠΈ Ξ±2 ― ΡƒΠ³Π»Ρ‹ ΠΌΠ΅ΠΆΠ΄Ρƒ Π½Π°ΠΏΡ€Π°Π²Π»Π΅Π½ΠΈΠ΅ΠΌ Ρ‚ΠΎΠΊΠ° ΠΈ Π½Π°ΠΏΡ€Π°Π²Π»Π΅Π½ΠΈΠ΅ΠΌ Π½Π° Ρ‚ΠΎΡ‡ΠΊΡƒ, Π² ΠΊΠΎΡ‚ΠΎΡ€ΠΎΠΉ создано ΠΌΠ°Π³Π½ΠΈΡ‚Π½ΠΎΠ΅ ΠΏΠΎΠ»Π΅, Π²Π΅Ρ€ΡˆΠΈΠ½Π°ΠΌΠΈ ΠΊΠΎΡ‚ΠΎΡ€Ρ‹Ρ… ΡΠ²Π»ΡΡŽΡ‚ΡΡ соотвСтствСнно Π½Π°Ρ‡Π°Π»ΠΎ ΠΈ ΠΊΠΎΠ½Π΅Ρ† прямого участка ΠΏΡ€ΠΎΠ²ΠΎΠ΄Π½ΠΈΠΊΠ°
Бвязь ΠΌΠ΅ΠΆΠ΄Ρƒ Π½Π°ΠΏΡ€ΡΠΆΠ΅Π½Π½ΠΎΡΡ‚ΡŒΡŽ H ΠΈ ΠΈΠ½Π΄ΡƒΠΊΡ†ΠΈΠ΅ΠΉ B ΠΌΠ°Π³Π½ΠΈΡ‚Π½ΠΎΠ³ΠΎ поляμ ― магнитная ΠΏΡ€ΠΎΠ½ΠΈΡ†Π°Π΅ΠΌΠΎΡΡ‚ΡŒ срСды, ΞΌ0 = 4Ο€βˆ™10βˆ’7 Π“Π½/ΠΌ ― магнитная постоянная
Π˜Π½Π΄ΡƒΠΊΡ‚ΠΈΠ²Π½ΠΎΡΡ‚ΡŒ ΠΊΠ°Ρ‚ΡƒΡˆΠΊΠΈ Ρ€Π°Π²Π½Π°ΞΌ0 = 4Ο€βˆ™10βˆ’7 Π“Π½/ΠΌ ― магнитная постоянная; N ― количСство Π²ΠΈΡ‚ΠΊΠΎΠ²; N = l/d, d ― Π΄ΠΈΠ°ΠΌΠ΅Ρ‚Ρ€ ΠΏΡ€ΠΎΠ²ΠΎΠ΄Π½ΠΈΠΊΠ° ΠΊΠ°Ρ‚ΡƒΡˆΠΊΠΈ; l ― Π΄Π»ΠΈΠ½Π° ΠΊΠ°Ρ‚ΡƒΡˆΠΊΠΈ; V ― объСм ΠΊΠ°Ρ‚ΡƒΡˆΠΊΠΈ; S ― ΠΏΠ»ΠΎΡ‰Π°Π΄ΡŒ Π²ΠΈΡ‚ΠΊΠ° ΠΊΠ°Ρ‚ΡƒΡˆΠΊΠΈ
БрСдняя объСмная ΠΏΠ»ΠΎΡ‚Π½ΠΎΡΡ‚ΡŒ энСргииΡ0 = 8,85βˆ™10βˆ’12 Π€/ΠΌ ― элСктричСская постоянная, Ξ΅ ― диэлСктричСская ΠΏΡ€ΠΎΠ½ΠΈΡ†Π°Π΅ΠΌΠΎΡΡ‚ΡŒ срСды, E ― Π΄Π΅ΠΉΡΡ‚Π²ΡƒΡŽΡ‰Π΅Π΅ Π·Π½Π°Ρ‡Π΅Π½ΠΈΠ΅ напряТСнности элСктричСского поля
Π‘ΠΈΠ»Π° , Π΄Π΅ΠΉΡΡ‚Π²ΡƒΡŽΡ‰Π°Ρ Π½Π° заряд Q, двиТущийся со ΡΠΊΠΎΡ€ΠΎΡΡ‚ΡŒΡŽ Β Π² ΠΌΠ°Π³Π½ΠΈΡ‚Π½ΠΎΠΌ ΠΏΠΎΠ»Π΅ с ΠΈΠ½Π΄ΡƒΠΊΡ†ΠΈΠ΅ΠΉ (сила Π›ΠΎΡ€Π΅Π½Ρ†Π°Ξ± ― ΡƒΠ³ΠΎΠ», ΠΎΠ±Ρ€Π°Π·ΠΎΠ²Π°Π½Π½Ρ‹ΠΉ Π²Π΅ΠΊΡ‚ΠΎΡ€ΠΎΠΌ скорости двиТСния частицы ΠΈ Π²Π΅ΠΊΡ‚ΠΎΡ€ΠΎΠΌ Β ΠΈΠ½Π΄ΡƒΠΊΡ†ΠΈΠΈ ΠΌΠ°Π³Π½ΠΈΡ‚Π½ΠΎΠ³ΠΎ поля
CΠΈΠ»Π° АмпСра (сила, Π΄Π΅ΠΉΡΡ‚Π²ΡƒΡŽΡ‰Π°Ρ Π½Π° ΠΏΡ€ΠΎΠ²ΠΎΠ΄Π½ΠΈΠΊ с Ρ‚ΠΎΠΊΠΎΠΌ Π² ΠΌΠ°Π³Π½ΠΈΡ‚Π½ΠΎΠΌ ΠΏΠΎΠ»Π΅)
I ― сила Ρ‚ΠΎΠΊΠ°, l ― Π΄Π»ΠΈΠ½Π° ΠΏΡ€ΠΎΠ²ΠΎΠ΄Π½ΠΈΠΊΠ°, Π’ ― индукция ΠΌΠ°Π³Π½ΠΈΡ‚Π½ΠΎΠ³ΠΎ поля, Ξ± ― ΡƒΠ³ΠΎΠ» ΠΌΠ΅ΠΆΠ΄Ρƒ Π²Π΅ΠΊΡ‚ΠΎΡ€Π°ΠΌΠΈ
ЦикличСская частота ΠΊΠΎΠ»Π΅Π±Π°Π½ΠΈΠΉ Π² ΠΊΠΎΠ½Ρ‚ΡƒΡ€Π΅L ― ΠΈΠ½Π΄ΡƒΠΊΡ‚ΠΈΠ²Π½ΠΎΡΡ‚ΡŒ ΠΊΠΎΠ½Ρ‚ΡƒΡ€Π°; C ― Π΅ΠΌΠΊΠΎΡΡ‚ΡŒ ΠΊΠΎΠ½Ρ‚ΡƒΡ€Π°
МгновСнноС Π·Π½Π°Ρ‡Π΅Π½ΠΈΠ΅ I силы Ρ‚ΠΎΠΊΠ° Π² Ρ†Π΅ΠΏΠΈ, ΠΎΠ±Π»Π°Π΄Π°ΡŽΡ‰Π΅ΠΉ Π°ΠΊΡ‚ΠΈΠ²Π½Ρ‹ΠΌ сопротивлСниСм R ΠΈ ΠΈΠ½Π΄ΡƒΠΊΡ‚ΠΈΠ²Π½ΠΎΡΡ‚ΡŒΡŽ L, послС размыкания Ρ†Π΅ΠΏΠΈI0 ― Π·Π½Π°Ρ‡Π΅Π½ΠΈΠ΅ силы Ρ‚ΠΎΠΊΠ° Π² Ρ†Π΅ΠΏΠΈ ΠΏΡ€ΠΈ t = 0; t ― врСмя, ΠΏΡ€ΠΎΡˆΠ΅Π΄ΡˆΠ΅Π΅ с ΠΌΠΎΠΌΠ΅Π½Ρ‚Π° размыкания Ρ†Π΅ΠΏΠΈ
МгновСнноС Π·Π½Π°Ρ‡Π΅Π½ΠΈΠ΅ I силы Ρ‚ΠΎΠΊΠ° Π² Ρ†Π΅ΠΏΠΈ, ΠΎΠ±Π»Π°Π΄Π°ΡŽΡ‰Π΅ΠΉ Π°ΠΊΡ‚ΠΈΠ²Π½Ρ‹ΠΌ сопротивлСниСм R ΠΈ ΠΈΠ½Π΄ΡƒΠΊΡ‚ΠΈΠ²Π½ΠΎΡΡ‚ΡŒΡŽ L, послС замыкания Ρ†Π΅ΠΏΠΈΞ΅ ― э. Π΄.с. источника Ρ‚ΠΎΠΊΠ°; t ― врСмя, ΠΏΡ€ΠΎΡˆΠ΅Π΄ΡˆΠ΅Π΅ с ΠΌΠΎΠΌΠ΅Π½Ρ‚Π° замыкания Ρ†Π΅ΠΏΠΈ
Основной Π·Π°ΠΊΠΎΠ½ элСктромагнитной индукцииΡi ― элСктродвиТущая сила ΠΈΠ½Π΄ΡƒΠΊΡ†ΠΈΠΈ; N ― число Π²ΠΈΡ‚ΠΊΠΎΠ² ΠΊΠΎΠ½Ρ‚ΡƒΡ€Π°; Ξ¨ ― потокосцСплСниС
Π’Π΅Π»ΠΈΡ‡ΠΈΠ½Π° Π­Π”Π‘ самоиндукции ΠΏΡ€ΠΎΠΏΠΎΡ€Ρ†ΠΈΠΎΠ½Π°Π»ΡŒΠ½Π° скорости измСнСния силы Ρ‚ΠΎΠΊΠ° I:L ― ΠΈΠ½Π΄ΡƒΠΊΡ‚ΠΈΠ²Π½ΠΎΡΡ‚ΡŒ ΠΊΠΎΠ½Ρ‚ΡƒΡ€Π° ΠΈΠ»ΠΈ ΠΊΠ°Ρ‚ΡƒΡˆΠΊΠΈ
Π Π°Π±ΠΎΡ‚Π° ΠΏΠΎ ΠΏΠ΅Ρ€Π΅ΠΌΠ΅Ρ‰Π΅Π½ΠΈΡŽ ΠΏΡ€ΠΎΠ²ΠΎΠ΄Π½ΠΈΠΊΠ° ΠΈΠ»ΠΈ ΠΏΠΎ ΠΏΠΎΠ²ΠΎΡ€ΠΎΡ‚Ρƒ ΠΊΠΎΠ½Ρ‚ΡƒΡ€Π° Π² ΠΌΠ°Π³Π½ΠΈΡ‚Π½ΠΎΠΌ ΠΏΠΎΠ»Π΅I ― сила Ρ‚ΠΎΠΊΠ° Π² ΠΏΡ€ΠΎΠ²ΠΎΠ΄Π½ΠΈΠΊΠ΅, ΠΊΠΎΠ½Ρ‚ΡƒΡ€Π΅; dΠ€ ― пСрСсСкаСмый ΠΏΡ€ΠΎΠ²ΠΎΠ΄Π½ΠΈΠΊΠΎΠΌ ΠΌΠ°Π³Π½ΠΈΡ‚Π½Ρ‹ΠΉ ΠΏΠΎΡ‚ΠΎΠΊ Π»ΠΈΠ±ΠΎ ΠΈΠ·ΠΌΠ΅Π½Π΅Π½ΠΈΠ΅ ΠΌΠ°Π³Π½ΠΈΡ‚Π½ΠΎΠ³ΠΎ ΠΏΠΎΡ‚ΠΎΠΊΠ° Ρ‡Π΅Ρ€Π΅Π· Π·Π°ΠΌΠΊΠ½ΡƒΡ‚Ρ‹ΠΉ ΠΊΠΎΠ½Ρ‚ΡƒΡ€
Π’Ρ€Π°Ρ‰Π°ΡŽΡ‰ΠΈΠΉ ΠΌΠΎΠΌΠ΅Π½Ρ‚, Π΄Π΅ΠΉΡΡ‚Π²ΡƒΡŽΡ‰ΠΈΠΉ Π½Π° ΠΊΠΎΠ½Ρ‚ΡƒΡ€ с Ρ‚ΠΎΠΊΠΎΠΌ, ΠΏΠΎΠΌΠ΅Ρ‰Π΅Π½Π½Ρ‹ΠΉ Π² ΠΌΠ°Π³Π½ΠΈΡ‚Π½ΠΎΠ΅ ΠΏΠΎΠ»Π΅ Π—Π½Π°Ρ‡Π΅Π½ΠΈΠ΅ Π²Ρ€Π°Ρ‰Π°ΡŽΡ‰Π΅Π³ΠΎ ΠΌΠΎΠΌΠ΅Π½Ρ‚Π°
 индукция ΠΌΠ°Π³Π½ΠΈΡ‚Π½ΠΎΠ³ΠΎ поля; ― ΠΌΠ°Π³Π½ΠΈΡ‚Π½Ρ‹ΠΉ ΠΌΠΎΠΌΠ΅Π½Ρ‚ ΠΊΠΎΠ½Ρ‚ΡƒΡ€Π°, = IS, Π³Π΄Π΅ I ― Ρ‚ΠΎΠΊ, ΠΏΡ€ΠΎΡ‚Π΅ΠΊΠ°ΡŽΡ‰ΠΈΠΉ ΠΏΠΎ ΠΊΠΎΠ½Ρ‚ΡƒΡ€Ρƒ, S ― ΠΏΠ»ΠΎΡ‰Π°Π΄ΡŒ ΠΊΠΎΠ½Ρ‚ΡƒΡ€Π°; Ξ± ― ΡƒΠ³ΠΎΠ» ΠΌΠ΅ΠΆΠ΄Ρƒ Π²Π΅ΠΊΡ‚ΠΎΡ€Π°ΠΌΠΈΒ  ΠΈΒ 

Π Π°Π±ΠΎΡ‚Π° ΠΈ ΠΌΠΎΡ‰Π½ΠΎΡΡ‚ΡŒ Ρ‚ΠΎΠΊΠ° β€” ΡƒΡ€ΠΎΠΊ. Π€ΠΈΠ·ΠΈΠΊΠ°, 8 класс.

ΠŸΡ€ΠΈ ΠΏΡ€ΠΎΡ…ΠΎΠΆΠ΄Π΅Π½ΠΈΠΈ Ρ‚ΠΎΠΊΠ° Π² Ρ†Π΅ΠΏΠΈ элСктричСскоС ΠΏΠΎΠ»Π΅ ΡΠΎΠ²Π΅Ρ€ΡˆΠ°Π΅Ρ‚ Ρ€Π°Π±ΠΎΡ‚Ρƒ ΠΏΠΎ ΠΏΠ΅Ρ€Π΅ΠΌΠ΅Ρ‰Π΅Π½ΠΈΡŽ заряда. Π’ этом случаС Ρ€Π°Π±ΠΎΡ‚Ρƒ элСктричСского поля Π½Π°Π·Ρ‹Π²Π°ΡŽΡ‚ Ρ€Π°Π±ΠΎΡ‚ΠΎΠΉ элСктричСского Ρ‚ΠΎΠΊΠ°.

ΠŸΡ€ΠΈ ΠΏΡ€ΠΎΡ…ΠΎΠΆΠ΄Π΅Π½ΠΈΠΈ заряда \(q\) ΠΏΠΎ участку Ρ†Π΅ΠΏΠΈ элСктричСскоС ΠΏΠΎΠ»Π΅ Π±ΡƒΠ΄Π΅Ρ‚ ΡΠΎΠ²Π΅Ρ€ΡˆΠ°Ρ‚ΡŒ Ρ€Π°Π±ΠΎΡ‚Ρƒ: \(A=q\cdot U\), Π³Π΄Π΅ \(U\)Β β€” напряТСниС элСктричСского поля, \(A\)Β β€” Ρ€Π°Π±ΠΎΡ‚Π°, ΡΠΎΠ²Π΅Ρ€ΡˆΠ°Π΅ΠΌΠ°Ρ силами элСктричСского поля ΠΏΠΎ ΠΏΠ΅Ρ€Π΅ΠΌΠ΅Ρ‰Π΅Π½ΠΈΡŽ заряда \(q\) ΠΈΠ· ΠΎΠ΄Π½ΠΎΠΉ Ρ‚ΠΎΡ‡ΠΊΠΈ Π² Π΄Ρ€ΡƒΠ³ΡƒΡŽ.

Для выраТСния любой ΠΈΠ· этих Π²Π΅Π»ΠΈΡ‡ΠΈΠ½ ΠΌΠΎΠΆΠ½ΠΎ ΠΈΡΠΏΠΎΠ»ΡŒΠ·ΠΎΠ²Π°Ρ‚ΡŒ ΠΏΡ€ΠΈΠ²Π΅Π΄Ρ‘Π½Π½Ρ‹Π΅ Π½ΠΈΠΆΠ΅ рисунки.


ΠšΠΎΠ»ΠΈΡ‡Π΅ΡΡ‚Π²ΠΎ заряда, ΠΏΡ€ΠΎΡˆΠ΅Π΄ΡˆΠ΅Π΅ ΠΏΠΎ участку Ρ†Π΅ΠΏΠΈ, ΠΏΡ€ΠΎΠΏΠΎΡ€Ρ†ΠΈΠΎΠ½Π°Π»ΡŒΠ½ΠΎ силС Ρ‚ΠΎΠΊΠ° ΠΈ Π²Ρ€Π΅ΠΌΠ΅Π½ΠΈ прохоТдСния заряда: q=Iβ‹…t.

Π Π°Π±ΠΎΡ‚Π° элСктричСского Ρ‚ΠΎΠΊΠ° Π½Π° участкС Ρ†Π΅ΠΏΠΈ ΠΏΡ€ΠΎΠΏΠΎΡ€Ρ†ΠΈΠΎΠ½Π°Π»ΡŒΠ½Π° Π½Π°ΠΏΡ€ΡΠΆΠ΅Π½ΠΈΡŽ Π½Π° Π΅Ρ‘ ΠΊΠΎΠ½Ρ†Π°Ρ… ΠΈ количСству заряда, проходящСго ΠΏΠΎ этому участку: A=Uβ‹…q.

Π Π°Π±ΠΎΡ‚Π° элСктричСского Ρ‚ΠΎΠΊΠ°Β Π½Π° участкС Ρ†Π΅ΠΏΠΈ ΠΏΡ€ΠΎΠΏΠΎΡ€Ρ†ΠΈΠΎΠ½Π°Π»ΡŒΠ½Π° силС Ρ‚ΠΎΠΊΠ°, Π²Ρ€Π΅ΠΌΠ΅Π½ΠΈ прохоТдСния заряда ΠΈ Π½Π°ΠΏΡ€ΡΠΆΠ΅Π½ΠΈΡŽ Π½Π° ΠΊΠΎΠ½Ρ†Π°Ρ… участка Ρ†Π΅ΠΏΠΈ: A=Uβ‹…Iβ‹…t.

Π§Ρ‚ΠΎΠ±Ρ‹ Π²Ρ‹Ρ€Π°Π·ΠΈΡ‚ΡŒ Π»ΡŽΠ±ΡƒΡŽ ΠΈΠ· Π²Π΅Π»ΠΈΡ‡ΠΈΠ½ ΠΈΠ· Π΄Π°Π½Π½ΠΎΠΉ Ρ„ΠΎΡ€ΠΌΡƒΠ»Ρ‹, ΠΌΠΎΠΆΠ½ΠΎ Π²ΠΎΡΠΏΠΎΠ»ΡŒΠ·ΠΎΠ²Π°Ρ‚ΡŒΡΡ рисунком.

Β 

Β 

Π•Π΄ΠΈΠ½ΠΈΡ†Ρ‹ измСрСния Π²Π΅Π»ΠΈΡ‡ΠΈΠ½:

Ρ€Π°Π±ΠΎΡ‚Π° элСктричСского Ρ‚ΠΎΠΊΠ° \([A]=1\) Π”ΠΆ;

напряТСниС Π½Π° участкС Ρ†Π΅ΠΏΠΈ \([U]=1\) Π’;

сила Ρ‚ΠΎΠΊΠ°, проходящСго ΠΏΠΎ участку \([I]=1\) А;

врСмя прохоТдСния заряда (Ρ‚ΠΎΠΊΠ°) \([t]=1\) с.

Для измСрСния Ρ€Π°Π±ΠΎΡ‚Ρ‹ элСктричСского Ρ‚ΠΎΠΊΠ° Π½ΡƒΠΆΠ½Ρ‹ Π²ΠΎΠ»ΡŒΡ‚ΠΌΠ΅Ρ‚Ρ€, Π°ΠΌΠΏΠ΅Ρ€ΠΌΠ΅Ρ‚Ρ€ ΠΈ часы. НапримСр, для опрСдСлСния Ρ€Π°Π±ΠΎΡ‚Ρ‹, ΠΊΠΎΡ‚ΠΎΡ€ΡƒΡŽ ΡΠΎΠ²Π΅Ρ€ΡˆΠ°Π΅Ρ‚ элСктричСский Ρ‚ΠΎΠΊ, проходя ΠΏΠΎ спирали Π»Π°ΠΌΠΏΡ‹ накаливания, Π½Π΅ΠΎΠ±Ρ…ΠΎΠ΄ΠΈΠΌΠΎ ΡΠΎΠ±Ρ€Π°Ρ‚ΡŒ Ρ†Π΅ΠΏΡŒ, ΠΈΠ·ΠΎΠ±Ρ€Π°ΠΆΡ‘Π½Π½ΡƒΡŽ Π½Π° рисункС. Π’ΠΎΠ»ΡŒΡ‚ΠΌΠ΅Ρ‚Ρ€ΠΎΠΌ измСряСтся напряТСниС Π½Π° Π»Π°ΠΌΠΏΠ΅, Π°ΠΌΠΏΠ΅Ρ€ΠΌΠ΅Ρ‚Ρ€ΠΎΠΌΒ β€” сила Ρ‚ΠΎΠΊΠ° Π² Π½Π΅ΠΉ. А ΠΏΡ€ΠΈ ΠΏΠΎΠΌΠΎΡ‰ΠΈ часов (сСкундомСра) засСкаСтся врСмя горСния Π»Π°ΠΌΠΏΡ‹.


Β 

НапримСр:

Β 

IΒ =Β 1,2 АUΒ =Β 5Β Π’tΒ =Β 1,5Β ΠΌΠΈΠ½ =Β 90 сА =Β Uβ‹…Iβ‹…tΒ =Β 5β‹…1,2β‹…90Β =Β 540Β Π”ΠΆΒ 

Β 

ΠžΠ±Ρ€Π°Ρ‚ΠΈ Π²Π½ΠΈΠΌΠ°Π½ΠΈΠ΅!

Π Π°Π±ΠΎΡ‚Π°Β Ρ‡Π°Ρ‰Π΅ всСго выраТаСтся Π² килодТоулях ΠΈΠ»ΠΈ мСгадТоулях.

\(1\) ΠΊΠ”ΠΆ = 1000 Π”ΠΆ ΠΈΠ»ΠΈ \(1\) Π”ΠΆ = \(0,001\) ΠΊΠ”ΠΆ;
\(1\) ΠœΠ”ΠΆ = 1000000 Π”ΠΆ ΠΈΠ»ΠΈ \(1\) Π”ΠΆ = \(0,000001\) ΠœΠ”ΠΆ.

Для ΠΏΠΎΡ‚Ρ€Π΅Π±ΠΈΡ‚Π΅Π»Π΅ΠΉ элСктричСской энСргии ΡΡƒΡ‰Π΅ΡΡ‚Π²ΡƒΡŽΡ‚ ΠΏΡ€ΠΈΠ±ΠΎΡ€Ρ‹, ΠΏΠΎΠ·Π²ΠΎΠ»ΡΡŽΡ‰ΠΈΠ΅ Π² ΠΏΡ€Π΅Π΄Π΅Π»Π°Ρ… ошибки измСрСния ΠΏΠΎΠ»ΡƒΡ‡Π°Ρ‚ΡŒ числовыС Π΄Π°Π½Π½Ρ‹Π΅ ΠΎ Π΅Π΅ расходС Π² Π΅Π΄ΠΈΠ½ΠΈΡ†Ρƒ Π²Ρ€Π΅ΠΌΠ΅Π½ΠΈ.

Β 

ΠœΠ΅Ρ…Π°Π½ΠΈΡ‡Π΅ΡΠΊΠ°ΡΒ ΠΌΠΎΡ‰Π½ΠΎΡΡ‚ΡŒ числСнно Ρ€Π°Π²Π½Π° Ρ€Π°Π±ΠΎΡ‚Π΅, ΡΠΎΠ²Π΅Ρ€ΡˆΡ‘Π½Π½ΠΎΠΉ Ρ‚Π΅Π»ΠΎΠΌ Π² Π΅Π΄ΠΈΠ½ΠΈΡ†Ρƒ Π²Ρ€Π΅ΠΌΠ΅Π½ΠΈ: NΒ = Аt.Β  Π§Ρ‚ΠΎΠ±Ρ‹ Π½Π°ΠΉΡ‚ΠΈ ΠΌΠΎΡ‰Π½ΠΎΡΡ‚ΡŒ элСктричСского Ρ‚ΠΎΠΊΠ°, Π½Π°Π΄ΠΎ ΠΏΠΎΡΡ‚ΡƒΠΏΠΈΡ‚ΡŒ Ρ‚ΠΎΡ‡Π½ΠΎ Ρ‚Π°ΠΊΠΆΠ΅, Ρ‚.Π΅. Ρ€Π°Π±ΠΎΡ‚Ρƒ Ρ‚ΠΎΠΊΠ°, A=Uβ‹…Iβ‹…t,Β Ρ€Π°Π·Π΄Π΅Π»ΠΈΡ‚ΡŒ Π½Π° врСмя.

ΠœΠΎΡ‰Π½ΠΎΡΡ‚ΡŒ элСктричСского Ρ‚ΠΎΠΊΠ° ΠΎΠ±ΠΎΠ·Π½Π°Ρ‡Π°ΡŽΡ‚ Π±ΡƒΠΊΠ²ΠΎΠΉ \(Π \):

P=At=Uβ‹…Iβ‹…tt=Uβ‹…I. Π’Π°ΠΊΠΈΠΌ ΠΎΠ±Ρ€Π°Π·ΠΎΠΌ:

ΠœΠΎΡ‰Π½ΠΎΡΡ‚ΡŒ элСктричСского Ρ‚ΠΎΠΊΠ° Ρ€Π°Π²Π½Π° ΠΏΡ€ΠΎΠΈΠ·Π²Π΅Π΄Π΅Π½ΠΈΡŽ напряТСния Π½Π° силу Ρ‚ΠΎΠΊΠ°: P=Uβ‹…I.

Из этой Ρ„ΠΎΡ€ΠΌΡƒΠ»Ρ‹ ΠΌΠΎΠΆΠ½ΠΎ ΠΎΠΏΡ€Π΅Π΄Π΅Π»ΠΈΡ‚ΡŒ ΠΈ Π΄Ρ€ΡƒΠ³ΠΈΠ΅ физичСскиС Π²Π΅Π»ΠΈΡ‡ΠΈΠ½Ρ‹.
Для удобства ΠΌΠΎΠΆΠ½ΠΎ ΠΈΡΠΏΠΎΠ»ΡŒΠ·ΠΎΠ²Π°Ρ‚ΡŒ ΠΏΡ€ΠΈΠ²Π΅Π΄Ρ‘Π½Π½Ρ‹Π΅ Π½ΠΈΠΆΠ΅ рисунки.

Β 

Β 

Π—Π° Π΅Π΄ΠΈΠ½ΠΈΡ†Ρƒ мощности принят Π²Π°Ρ‚Ρ‚: \(1\) Π’Ρ‚ = \(1\) Π”ΠΆ/с.

Β 

Из Ρ„ΠΎΡ€ΠΌΡƒΠ»Ρ‹ P=Uβ‹…I слСдуСт, Ρ‡Ρ‚ΠΎ


\(1\) Π²Π°Ρ‚Ρ‚ = \(1\) Π²ΠΎΠ»ΡŒΡ‚ βˆ™ \(1\) Π°ΠΌΠΏΠ΅Ρ€, ΠΈΠ»ΠΈ \(1\) Π’Ρ‚ = \(1\) Π’ βˆ™ А.


Β 

ΠžΠ±Ρ€Π°Ρ‚ΠΈ Π²Π½ΠΈΠΌΠ°Π½ΠΈΠ΅!

Π˜ΡΠΏΠΎΠ»ΡŒΠ·ΡƒΡŽΡ‚ Ρ‚Π°ΠΊΠΆΠ΅ Π΅Π΄ΠΈΠ½ΠΈΡ†Ρ‹ мощности, ΠΊΡ€Π°Ρ‚Π½Ρ‹Π΅ Π²Π°Ρ‚Ρ‚Ρƒ: Π³Π΅ΠΊΡ‚ΠΎΠ²Π°Ρ‚Ρ‚ (Π³Π’Ρ‚), ΠΊΠΈΠ»ΠΎΠ²Π°Ρ‚Ρ‚ (ΠΊΠ’Ρ‚), ΠΌΠ΅Π³Π°Π²Π°Ρ‚Ρ‚ (ΠœΠ’Ρ‚).
\(1\) Π³Π’Ρ‚ = \(100\) Π’Ρ‚ ΠΈΠ»ΠΈ \(1\) Π’Ρ‚ = \(0,01\) Π³Π’Ρ‚;
\(1\) ΠΊΠ’Ρ‚ = \(1000\) Π’Ρ‚ ΠΈΠ»ΠΈ \(1\) Π’Ρ‚ = \(0,001\) ΠΊΠ’Ρ‚;
\(1\) ΠœΠ’Ρ‚ = \(1 000 000\) Π’Ρ‚ ΠΈΠ»ΠΈ \(1\) Π’Ρ‚ = \(0,000001\) ΠœΠ’Ρ‚.

ΠŸΡ€ΠΈΠΌΠ΅Ρ€:

Π˜Π·ΠΌΠ΅Ρ€ΠΈΠΌ силу Ρ‚ΠΎΠΊΠ° Π² Ρ†Π΅ΠΏΠΈ с ΠΏΠΎΠΌΠΎΡ‰ΡŒΡŽ Π°ΠΌΠΏΠ΅Ρ€ΠΌΠ΅Ρ‚Ρ€Π°, Π° напряТСниС Π½Π° участкС β€” с ΠΏΠΎΠΌΠΎΡ‰ΡŒΡŽΒ Π²ΠΎΠ»ΡŒΡ‚ΠΌΠ΅Ρ‚Ρ€Π°.

Β 

Β 

Π’Π°ΠΊ ΠΊΠ°ΠΊΒ ΠΌΠΎΡ‰Π½ΠΎΡΡ‚ΡŒ тока прямо ΠΏΡ€ΠΎΠΏΠΎΡ€Ρ†ΠΈΠΎΠ½Π°Π»ΡŒΠ½Π° Π½Π°ΠΏΡ€ΡΠΆΠ΅Π½ΠΈΡŽ ΠΈ силС Ρ‚ΠΎΠΊΠ°, ΠΏΡ€ΠΎΡ‚Π΅ΠΊΠ°ΡŽΡ‰Π΅Π³ΠΎ Ρ‡Π΅Ρ€Π΅Π· Π»Π°ΠΌΠΏΠΎΡ‡ΠΊΡƒ, Ρ‚ΠΎ ΠΏΠ΅Ρ€Π΅ΠΌΠ½ΠΎΠΆΠΈΠΌ ΠΈΡ… значСния:

Β 

I=1,2АU=5Π’PΒ =Uβ‹…I=5β‹…1,2=6Π’Ρ‚.

Β 

Π’Π°Ρ‚Ρ‚ΠΌΠ΅Ρ‚Ρ€Ρ‹ ΠΈΠ·ΠΌΠ΅Ρ€ΡΡŽΡ‚ ΠΌΠΎΡ‰Π½ΠΎΡΡ‚ΡŒ элСктричСского Ρ‚ΠΎΠΊΠ°, ΠΏΡ€ΠΎΡ‚Π΅ΠΊΠ°ΡŽΡ‰Π΅Π³ΠΎ Ρ‡Π΅Ρ€Π΅Π· ΠΏΡ€ΠΈΠ±ΠΎΡ€. По своСму Π½Π°Π·Π½Π°Ρ‡Π΅Π½ΠΈΡŽ ΠΈ тСхничСским характСристикам ваттмСтры Ρ€Π°Π·Π½ΠΎΠΎΠ±Ρ€Π°Π·Π½Ρ‹.

Π’ зависимости ΠΎΡ‚ сфСры примСнСния Ρƒ Π½ΠΈΡ… Ρ€Π°Π·Π»ΠΈΡ‡Π°ΡŽΡ‚ΡΡ ΠΏΡ€Π΅Π΄Π΅Π»Ρ‹ измСрСния.

Β 

Аналоговый Π²Π°Ρ‚Ρ‚ΠΌΠ΅Ρ‚Ρ€

Аналоговый Π²Π°Ρ‚Ρ‚ΠΌΠ΅Ρ‚Ρ€

Аналоговый Π²Π°Ρ‚Ρ‚ΠΌΠ΅Ρ‚Ρ€

Π¦ΠΈΡ„Ρ€ΠΎΠ²ΠΎΠΉ Π²Π°Ρ‚Ρ‚ΠΌΠ΅Ρ‚Ρ€

Β 

ΠŸΠΎΠ΄ΠΊΠ»ΡŽΡ‡ΠΈΠΌ ΠΊ Ρ†Π΅ΠΏΠΈ ΠΏΠΎ ΠΎΡ‡Π΅Ρ€Π΅Π΄ΠΈ Π΄Π²Π΅ Π»Π°ΠΌΠΏΠΎΡ‡ΠΊΠΈ накаливания, сначала ΠΎΠ΄Π½Ρƒ, Π·Π°Ρ‚Π΅ΠΌ Π΄Ρ€ΡƒΠ³ΡƒΡŽ ΠΈ ΠΈΠ·ΠΌΠ΅Ρ€ΠΈΠΌ силу Ρ‚ΠΎΠΊΠ° Π² ΠΊΠ°ΠΆΠ΄ΠΎΠΉ ΠΈΠ· Π½ΠΈΡ…. Она Π±ΡƒΠ΄Π΅Ρ‚ Ρ€Π°Π·Π½ΠΎΠΉ.

Β 

Β 

Β 

Π‘ΠΈΠ»Π° Ρ‚ΠΎΠΊΠ° Π² Π»Π°ΠΌΠΏΠΎΡ‡ΠΊΠ΅ ΠΌΠΎΡ‰Π½ΠΎΡΡ‚ΡŒΡŽ \(25\) Π²Π°Ρ‚Ρ‚ Π±ΡƒΠ΄Π΅Ρ‚ ΡΠΎΡΡ‚Π°Π²Π»ΡΡ‚ΡŒ \(0,1\) А. Π›Π°ΠΌΠΏΠΎΡ‡ΠΊΠ° ΠΌΠΎΡ‰Π½ΠΎΡΡ‚ΡŒΡŽ \(100\) Π²Π°Ρ‚Ρ‚ потрСбляСт Ρ‚ΠΎΠΊ Π² Ρ‡Π΅Ρ‚Ρ‹Ρ€Π΅ Ρ€Π°Π·Π° большС β€” \(0,4\) А. НапряТСниС Π² этом экспСримСнтС Π½Π΅ΠΈΠ·ΠΌΠ΅Π½Π½ΠΎ ΠΈ Ρ€Π°Π²Π½ΠΎ \(220\) Π’. Π›Π΅Π³ΠΊΠΎ ΠΌΠΎΠΆΠ½ΠΎ Π·Π°ΠΌΠ΅Ρ‚ΠΈΡ‚ΡŒ, Ρ‡Ρ‚ΠΎ Π»Π°ΠΌΠΏΠΎΡ‡ΠΊΠ° Π² \(100\) Π²Π°Ρ‚Ρ‚ свСтится Π³ΠΎΡ€Π°Π·Π΄ΠΎ ярчС, Ρ‡Π΅ΠΌ \(25\)-ваттовая Π»Π°ΠΌΠΏΠΎΡ‡ΠΊΠ°. Π­Ρ‚ΠΎ происходит ΠΎΡ‚Ρ‚ΠΎΠ³ΠΎ, Ρ‡Ρ‚ΠΎ Π΅Ρ‘ ΠΌΠΎΡ‰Π½ΠΎΡΡ‚ΡŒ большС. Π›Π°ΠΌΠΏΠΎΡ‡ΠΊΠ°, ΠΌΠΎΡ‰Π½ΠΎΡΡ‚ΡŒ ΠΊΠΎΡ‚ΠΎΡ€ΠΎΠΉ Π² \(4\) Ρ€Π°Π·Π° большС, потрСбляСт Π² \(4\) Ρ€Π°Π·Π° большС Ρ‚ΠΎΠΊΠ°.Β Π—Π½Π°Ρ‡ΠΈΡ‚:Β 

Β 

ΠžΠ±Ρ€Π°Ρ‚ΠΈ Π²Π½ΠΈΠΌΠ°Π½ΠΈΠ΅!

ΠœΠΎΡ‰Π½ΠΎΡΡ‚ΡŒ прямо ΠΏΡ€ΠΎΠΏΠΎΡ€Ρ†ΠΈΠΎΠ½Π°Π»ΡŒΠ½Π° силС Ρ‚ΠΎΠΊΠ°.

Π§Ρ‚ΠΎ ΠΏΡ€ΠΎΠΈΠ·ΠΎΠΉΠ΄Ρ‘Ρ‚, Ссли ΠΎΠ΄Π½Ρƒ ΠΈ Ρ‚Ρƒ ΠΆΠ΅ Π»Π°ΠΌΠΏΠΎΡ‡ΠΊΡƒ ΠΏΠΎΠ΄ΡΠΎΠ΅Π΄ΠΈΠ½ΠΈΡ‚ΡŒ ΠΊ источникам Ρ€Π°Π·Π»ΠΈΡ‡Π½ΠΎΠ³ΠΎ напряТСния? Π’ Π΄Π°Π½Π½ΠΎΠΌ случаС ΠΈΡΠΏΠΎΠ»ΡŒΠ·ΡƒΠ΅Ρ‚ΡΡ напряТСниС \(110\) Π’ ΠΈ \(220\) Π’.


Β Β 

Β 

МоТно Π·Π°ΠΌΠ΅Ρ‚ΠΈΡ‚ΡŒ, Ρ‡Ρ‚ΠΎ ΠΏΡ€ΠΈ большСм напряТСнии Π»Π°ΠΌΠΏΠΎΡ‡ΠΊΠ° свСтится ярчС, Π·Π½Π°Ρ‡ΠΈΡ‚, Π² этом случаС Π΅Ρ‘ ΠΌΠΎΡ‰Π½ΠΎΡΡ‚ΡŒ Π±ΡƒΠ΄Π΅Ρ‚ большС.Β Π‘Π»Π΅Π΄ΠΎΠ²Π°Ρ‚Π΅Π»ΡŒΠ½ΠΎ:

Β 

ΠžΠ±Ρ€Π°Ρ‚ΠΈ Π²Π½ΠΈΠΌΠ°Π½ΠΈΠ΅!

ΠœΠΎΡ‰Π½ΠΎΡΡ‚ΡŒ зависит ΠΎΡ‚ напряТСния.

РассчитаСм ΠΌΠΎΡ‰Π½ΠΎΡΡ‚ΡŒ Π»Π°ΠΌΠΏΠΎΡ‡ΠΊΠΈ Π² ΠΊΠ°ΠΆΠ΄ΠΎΠΌ случаС:

Β 

I=0,2АU=110Π’P=Uβ‹…I=110β‹…0,2=22Π’Ρ‚I=0,4АU=220Π’P=Uβ‹…I=220β‹…0,4=88Π’Ρ‚.

Β 

МоТно ΡΠ΄Π΅Π»Π°Ρ‚ΡŒ Π²Ρ‹Π²ΠΎΠ΄ ΠΎ Ρ‚ΠΎΠΌ, Ρ‡Ρ‚ΠΎ ΠΏΡ€ΠΈ ΡƒΠ²Π΅Π»ΠΈΡ‡Π΅Π½ΠΈΠΈ напряТСния Π² \(2\) Ρ€Π°Π·Π° ΠΌΠΎΡ‰Π½ΠΎΡΡ‚ΡŒ увСличиваСтся Π² \(4\) Ρ€Π°Π·Π°.
НС слСдуСт ΠΏΡƒΡ‚Π°Ρ‚ΡŒ эту ΠΌΠΎΡ‰Π½ΠΎΡΡ‚ΡŒ с номинальной ΠΌΠΎΡ‰Π½ΠΎΡΡ‚ΡŒΡŽ Π»Π°ΠΌΠΏΡ‹ (ΠΌΠΎΡ‰Π½ΠΎΡΡ‚ΡŒ, Π½Π° ΠΊΠΎΡ‚ΠΎΡ€ΡƒΡŽ рассчитана Π»Π°ΠΌΠΏΠ°). Номинальная ΠΌΠΎΡ‰Π½ΠΎΡΡ‚ΡŒ Π»Π°ΠΌΠΏΡ‹ (Π° соотвСтствСнно, Ρ‚ΠΎΠΊ Ρ‡Π΅Ρ€Π΅Π· Π½ΠΈΡ‚ΡŒ Π½Π°ΠΊΠ°Π»Π° ΠΈ Π΅Ρ‘ расчётноС сопротивлСниС) указываСтся Ρ‚ΠΎΠ»ΡŒΠΊΠΎ для номинального напряТСния Π»Π°ΠΌΠΏΡ‹ (ΡƒΠΊΠ°Π·Π°Π½ΠΎ Π½Π° Π±Π°Π»Π»ΠΎΠ½Π΅, Ρ†ΠΎΠΊΠΎΠ»Π΅ ΠΈΠ»ΠΈ ΡƒΠΏΠ°ΠΊΠΎΠ²ΠΊΠ΅).


Β 

Β 

Π’ Ρ‚Π°Π±Π»ΠΈΡ†Π΅ Π΄Π°Π½Π° ΠΌΠΎΡ‰Π½ΠΎΡΡ‚ΡŒ, потрСбляСмая Ρ€Π°Π·Π»ΠΈΡ‡Π½Ρ‹ΠΌΠΈ ΠΏΡ€ΠΈΠ±ΠΎΡ€Π°ΠΌΠΈ ΠΈ устройствами:

Β 

НазваниС

Рисунок

ΠœΠΎΡ‰Π½ΠΎΡΡ‚ΡŒ

Β ΠšΠ°Π»ΡŒΠΊΡƒΠ»ΡΡ‚ΠΎΡ€

\(0,001\) Π’Ρ‚

Β Π›Π°ΠΌΠΏΡ‹ Π΄Π½Π΅Π²Π½ΠΎΠ³ΠΎ свСта

\(15Β β€” 80\) Π’Ρ‚

Β Π›Π°ΠΌΠΏΡ‹ накаливания

\(25Β β€” 5000\) Π’Ρ‚

Β ΠšΠΎΠΌΠΏΡŒΡŽΡ‚Π΅Ρ€

\(200 β€” 450\) Π’Ρ‚

 ЭлСктричСский Ρ‡Π°ΠΉΠ½ΠΈΠΊ

\(650Β β€” 3100\) Π’Ρ‚

Β ΠŸΡ‹Π»Π΅ΡΠΎΡ

\(1500Β β€” 3000\) Π’Ρ‚

Β Π‘Ρ‚ΠΈΡ€Π°Π»ΡŒΠ½Π°Ρ машина

\(2000Β β€” 4000\) Π’Ρ‚

Β Π’Ρ€Π°ΠΌΠ²Π°ΠΉ

\(150 000Β β€” 240000\) Π’Ρ‚

Π€ΠΎΡ€ΠΌΡƒΠ»Π° мощности Ρ‚ΠΎΠΊΠ° Π² Ρ„ΠΈΠ·ΠΈΠΊΠ΅

Π‘ΠΎΠ΄Π΅Ρ€ΠΆΠ°Π½ΠΈΠ΅:

ЭлСктричСский Ρ‚ΠΎΠΊ, Π½Π° ΠΊΠ°ΠΊΠΎΠΌ ΡƒΠ³ΠΎΠ΄Π½ΠΎ участкС Ρ†Π΅ΠΏΠΈ ΡΠΎΠ²Π΅Ρ€ΡˆΠ°Π΅Ρ‚ Π½Π΅ΠΊΠΎΡ‚ΠΎΡ€ΡƒΡŽ Ρ€Π°Π±ΠΎΡ‚Ρƒ (А).{2}(6)$$

Π³Π΄Π΅ j – ΠΏΠ»ΠΎΡ‚Π½ΠΎΡΡ‚ΡŒ Ρ‚ΠΎΠΊΠ°, $\rho$ – ΡƒΠ΄Π΅Π»ΡŒΠ½ΠΎΠ΅ сопротивлСниС.

Π•Π΄ΠΈΠ½ΠΈΡ†Ρ‹ измСрСния мощности Ρ‚ΠΎΠΊΠ°

Основной Π΅Π΄ΠΈΠ½ΠΈΡ†Π΅ΠΉ измСрСния мощности Ρ‚ΠΎΠΊΠ° (ΠΊΠ°ΠΊ ΠΈ мощности Π²ΠΎΠΎΠ±Ρ‰Π΅) Π² систСмС БИ являСтся: [P]=Π’Ρ‚=Π”ΠΆ/с.

Π’ Π‘Π“Π‘: [P]=эрг/с.

1 Π’Ρ‚=107 эрг/( с).

Π’Ρ‹Ρ€Π°ΠΆΠ΅Π½ΠΈΠ΅ (4) ΠΏΡ€ΠΈΠΌΠ΅Π½ΡΡŽΡ‚ Π² систСмС БИ для Ρ‚ΠΎΠ³ΠΎ, Ρ‡Ρ‚ΠΎΠ±Ρ‹ Π΄Π°Ρ‚ΡŒ ΠΎΠΏΡ€Π΅Π΄Π΅Π»Π΅Π½ΠΈΠ΅ Π΅Π΄ΠΈΠ½ΠΈΡ†Ρ‹ напряТСния. Π’Π°ΠΊ, Π΅Π΄ΠΈΠ½ΠΈΡ†Π΅ΠΉ напряТСния (U) являСтся Π²ΠΎΠ»ΡŒΡ‚ (Π’), ΠΊΠΎΡ‚ΠΎΡ€Ρ‹ΠΉ Ρ€Π°Π²Π΅Π½: 1 Π’= (1 Π’Ρ‚)/(1 А).

Π’ΠΎΠ»ΡŒΡ‚ΠΎΠΌ Π½Π°Π·Ρ‹Π²Π°ΡŽΡ‚ элСктричСскоС напряТСниС, ΠΊΠΎΡ‚ΠΎΡ€ΠΎΠ΅ ΠΏΠΎΡ€ΠΎΠΆΠ΄Π°Π΅Ρ‚ Π² элСктроцСпи постоянный Ρ‚ΠΎΠΊ силы 1 А ΠΏΡ€ΠΈ мощности 1 Π’Ρ‚.

ΠŸΡ€ΠΈΠΌΠ΅Ρ€Ρ‹ Ρ€Π΅ΡˆΠ΅Π½ΠΈΡ Π·Π°Π΄Π°Ρ‡

ΠŸΡ€ΠΈΠΌΠ΅Ρ€

Π—Π°Π΄Π°Π½ΠΈΠ΅. Какой Π΄ΠΎΠ»ΠΆΠ½Π° Π±Ρ‹Ρ‚ΡŒ сила Ρ‚ΠΎΠΊΠ°, которая Ρ‚Π΅Ρ‡Π΅Ρ‚ Ρ‡Π΅Ρ€Π΅Π· ΠΎΠ±ΠΌΠΎΡ‚ΠΊΡƒ элСктричСского ΠΌΠΎΡ‚ΠΎΡ€Π° для Ρ‚ΠΎΠ³ΠΎ, Ρ‡Ρ‚ΠΎΠ±Ρ‹ полСзная ΠΌΠΎΡ‰Π½ΠΎΡΡ‚ΡŒ двигатСля (PA) стала максимальной?Какова максимальная полСзная ΠΌΠΎΡ‰Π½ΠΎΡΡ‚ΡŒ? Если Π΄Π²ΠΈΠ³Π°Ρ‚Π΅Π»ΡŒ постоянного Ρ‚ΠΎΠΊΠ° ΠΏΠΎΠ΄ΠΊΠ»ΡŽΡ‡Π΅Π½ ΠΊ Π½Π°ΠΏΡ€ΡΠΆΠ΅Π½ΠΈΡŽ U, сопротивлСниС ΠΎΠ±ΠΌΠΎΡ‚ΠΊΠΈ якоря – R.

РСшСниС. ΠœΠΎΡ‰Π½ΠΎΡΡ‚ΡŒ, ΠΊΠΎΡ‚ΠΎΡ€ΡƒΡŽ потрСбляСт элСктроприбор, ΠΈΠ΄Π΅Ρ‚ Π½Π° Π½Π°Π³Ρ€Π΅Π²Π°Π½ΠΈΠ΅ (PQ) ΠΈ ΡΠΎΠ²Π΅Ρ€ΡˆΠ΅Π½ΠΈΠ΅ Ρ€Π°Π±ΠΎΡ‚Ρ‹ (PA):

$$P=P_{Q}+P_{A}(1.{2}}{P_{2}}}$$

Π§ΠΈΡ‚Π°Ρ‚ΡŒ дальшС: Π€ΠΎΡ€ΠΌΡƒΠ»Π° напряТСния элСктричСского поля.

Π€ΠΎΡ€ΠΌΡƒΠ»Π° ΠΈ ΠΎΠΏΡ€Π΅Π΄Π΅Π»Π΅Π½ΠΈΠ΅ элСктричСского напряТСния Π² Ρ†Π΅ΠΏΠΈ Π² Ρ„ΠΈΠ·ΠΈΠΊΠ΅

ΠŸΡ€ΠΎΠ±ΠΎΠ²Π°Π»ΠΈ Π»ΠΈ Π²Ρ‹ ΠΊΠΎΠ³Π΄Π°-Π½ΠΈΠ±ΡƒΠ΄ΡŒ Π½Π°Π΄ΡƒΠ²Π°Ρ‚ΡŒ Π²ΠΎΠ·Π΄ΡƒΡˆΠ½Ρ‹Π΅ ΡˆΠ°Ρ€ΠΈΠΊΠΈ Π½Π° врСмя? Один Π½Π°Π΄ΡƒΠ²Π°Π΅Ρ‚ быстро, Π° Π΄Ρ€ΡƒΠ³ΠΎΠΉ Π·Π° это ΠΆΠ΅ врСмя Π½Π°Π΄ΡƒΠ²Π°Π΅Ρ‚ Π³ΠΎΡ€Π°Π·Π΄ΠΎ мСньшС. Π‘Π΅Π· сомнСния, ΠΏΠ΅Ρ€Π²Ρ‹ΠΉ ΡΠΎΠ²Π΅Ρ€ΡˆΠ°Π΅Ρ‚ Π±ΠΎΠ»ΡŒΡˆΡƒΡŽ Ρ€Π°Π±ΠΎΡ‚Ρƒ, Ρ‡Π΅ΠΌ Π²Ρ‚ΠΎΡ€ΠΎΠΉ.

Π‘ источниками напряТСния происходит Ρ‚ΠΎΡ‡Π½ΠΎ Ρ‚Π°ΠΊ ΠΆΠ΅. Π§Ρ‚ΠΎΠ±Ρ‹ ΠΎΠ±Π΅ΡΠΏΠ΅Ρ‡ΠΈΡ‚ΡŒ Π΄Π²ΠΈΠΆΠ΅Π½ΠΈΠ΅ частиц Π² ΠΏΡ€ΠΎΠ²ΠΎΠ΄Π½ΠΈΠΊΠ΅, Π½Π°Π΄ΠΎ ΡΠΎΠ²Π΅Ρ€ΡˆΠΈΡ‚ΡŒ Ρ€Π°Π±ΠΎΡ‚Ρƒ. И эту Ρ€Π°Π±ΠΎΡ‚Ρƒ ΡΠΎΠ²Π΅Ρ€ΡˆΠ°Π΅Ρ‚ источник. Π Π°Π±ΠΎΡ‚Ρƒ источника Ρ…Π°Ρ€Π°ΠΊΡ‚Π΅Ρ€ΠΈΠ·ΡƒΠ΅Ρ‚ напряТСниС. Π§Π΅ΠΌ ΠΎΠ½ΠΎ большС, Ρ‚Π΅ΠΌ Π±ΠΎΠ»ΡŒΡˆΡƒΡŽ Ρ€Π°Π±ΠΎΡ‚Ρƒ ΡΠΎΠ²Π΅Ρ€ΡˆΠ°Π΅Ρ‚ источник, Ρ‚Π΅ΠΌ ярчС Π±ΡƒΠ΄Π΅Ρ‚ Π³ΠΎΡ€Π΅Ρ‚ΡŒ Π»Π°ΠΌΠΏΠΎΡ‡ΠΊΠ° Π² Ρ†Π΅ΠΏΠΈ (ΠΏΡ€ΠΈ Π΄Ρ€ΡƒΠ³ΠΈΡ… ΠΎΠ΄ΠΈΠ½Π°ΠΊΠΎΠ²Ρ‹Ρ… условиях).

НапряТСниС Ρ€Π°Π²Π½ΠΎ ΠΎΡ‚Π½ΠΎΡˆΠ΅Π½ΠΈΡŽ Ρ€Π°Π±ΠΎΡ‚Ρ‹ элСктричСского поля ΠΏΠΎ ΠΏΠ΅Ρ€Π΅ΠΌΠ΅Ρ‰Π΅Π½ΠΈΡŽ зарядак Π²Π΅Π»ΠΈΡ‡ΠΈΠ½Π΅ ΠΏΠ΅Ρ€Π΅ΠΌΠ΅Ρ‰Π°Π΅ΠΌΠΎΠ³ΠΎ заряда Π½Π° участкС Ρ†Π΅ΠΏΠΈ.

U=Aq, Π³Π΄Π΅ (U) β€” напряТСниС,Β (A)Β β€” Ρ€Π°Π±ΠΎΡ‚Π° элСктричСского поля, (q) β€” заряд.

ΠžΠ±Ρ€Π°Ρ‚ΠΈ Π²Π½ΠΈΠΌΠ°Π½ΠΈΠ΅!

Π•Π΄ΠΈΠ½ΠΈΡ†Π° измСрСния напряТСния Π² систСмС БИ β€”Β [(U)] = (1) B (Π²ΠΎΠ»ΡŒΡ‚).

(1) Π²ΠΎΠ»ΡŒΡ‚ Ρ€Π°Π²Π΅Π½ элСктричСскому Π½Π°ΠΏΡ€ΡΠΆΠ΅Π½ΠΈΡŽ Π½Π° участкС Ρ†Π΅ΠΏΠΈ, Π³Π΄Π΅ ΠΏΡ€ΠΈ ΠΏΡ€ΠΎΡ‚Π΅ΠΊΠ°Π½ΠΈΠΈ заряда, Ρ€Π°Π²Π½ΠΎΠ³ΠΎ (1) Кл, ΡΠΎΠ²Π΅Ρ€ΡˆΠ°Π΅Ρ‚ΡΡ Ρ€Π°Π±ΠΎΡ‚Π°, равная (1) Π”ΠΆ: (1) Π’ (= 1) Π”ΠΆ/1 Кл.

ВсС Π²ΠΈΠ΄Π΅Π»ΠΈ надпись Π½Π° Π΄ΠΎΠΌΠ°ΡˆΠ½ΠΈΡ… Π±Ρ‹Ρ‚ΠΎΠ²Ρ‹Ρ… ΠΏΡ€ΠΈΠ±ΠΎΡ€Π°Ρ… Β«(220) Π’Β». Она ΠΎΠ·Π½Π°Ρ‡Π°Π΅Ρ‚, Ρ‡Ρ‚ΠΎ Π½Π° участкС Ρ†Π΅ΠΏΠΈ ΡΠΎΠ²Π΅Ρ€ΡˆΠ°Π΅Ρ‚ΡΡ Ρ€Π°Π±ΠΎΡ‚Π° (220) Π”ΠΆ ΠΏΠΎ ΠΏΠ΅Ρ€Π΅ΠΌΠ΅Ρ‰Π΅Π½ΠΈΡŽ заряда (1) Кл.

ΠšΡ€ΠΎΠΌΠ΅ Π²ΠΎΠ»ΡŒΡ‚Π°, ΠΏΡ€ΠΈΠΌΠ΅Π½ΡΡŽΡ‚ Π΄ΠΎΠ»ΡŒΠ½Ρ‹Π΅ ΠΈ ΠΊΡ€Π°Ρ‚Π½Ρ‹Π΅ Π΅ΠΌΡƒ Π΅Π΄ΠΈΠ½ΠΈΡ†Ρ‹ β€”Β ΠΌΠΈΠ»Π»ΠΈΠ²ΠΎΠ»ΡŒΡ‚ ΠΈ ΠΊΠΈΠ»ΠΎΠ²ΠΎΠ»ΡŒΡ‚.

(1) ΠΌΠ’ (= 0,001) Π’, (1) ΠΊΠ’ (= 1000) Π’ ΠΈΠ»ΠΈ (1) Π’Β (= 1000) ΠΌΠ’, (1) Π’ (= 0,001) ΠΊΠ’.

Для измСрСния напряТСния ΠΈΡΠΏΠΎΠ»ΡŒΠ·ΡƒΡŽΡ‚ ΠΏΡ€ΠΈΠ±ΠΎΡ€, ΠΊΠΎΡ‚ΠΎΡ€Ρ‹ΠΉ называСтся Π²ΠΎΠ»ΡŒΡ‚ΠΌΠ΅Ρ‚Ρ€.

ΠžΠ±ΠΎΠ·Π½Π°Ρ‡Π°ΡŽΡ‚ΡΡ всС Π²ΠΎΠ»ΡŒΡ‚ΠΌΠ΅Ρ‚Ρ€Ρ‹ латинской Π±ΡƒΠΊΠ²ΠΎΠΉΒ (V), которая наносится Π½Π° Ρ†ΠΈΡ„Π΅Ρ€Π±Π»Π°Ρ‚ ΠΏΡ€ΠΈΠ±ΠΎΡ€ΠΎΠ² ΠΈ ΠΈΡΠΏΠΎΠ»ΡŒΠ·ΡƒΠ΅Ρ‚ΡΡ Π² схСматичСском ΠΈΠ·ΠΎΠ±Ρ€Π°ΠΆΠ΅Π½ΠΈΠΈ ΠΏΡ€ΠΈΠ±ΠΎΡ€Π°.

Π’ ΡˆΠΊΠΎΠ»ΡŒΠ½Ρ‹Ρ… условиях ΠΈΡΠΏΠΎΠ»ΡŒΠ·ΡƒΡŽΡ‚ΡΡ Π²ΠΎΠ»ΡŒΡ‚ΠΌΠ΅Ρ‚Ρ€Ρ‹, ΠΈΠ·ΠΎΠ±Ρ€Π°ΠΆΡ‘Π½Π½Ρ‹Π΅ Π½Π° рисункС:

Β 

ΠžΡΠ½ΠΎΠ²Π½Ρ‹ΠΌΠΈ элСмСнтами Π²ΠΎΠ»ΡŒΡ‚ΠΌΠ΅Ρ‚Ρ€Π° ΡΠ²Π»ΡΡŽΡ‚ΡΡ корпус, шкала, стрСлка ΠΈ ΠΊΠ»Π΅ΠΌΠΌΡ‹. ΠšΠ»Π΅ΠΌΠΌΡ‹ ΠΎΠ±Ρ‹Ρ‡Π½ΠΎ подписаны плюсом ΠΈΠ»ΠΈ минусом ΠΈ для наглядности Π²Ρ‹Π΄Π΅Π»Π΅Π½Ρ‹ Ρ€Π°Π·Π½Ρ‹ΠΌΠΈ Ρ†Π²Π΅Ρ‚Π°ΠΌΠΈ: красный — плюс, Ρ‡Π΅Ρ€Π½Ρ‹ΠΉ (синий)Β β€” минус. Π‘Π΄Π΅Π»Π°Π½ΠΎ это с Ρ‚ΠΎΠΉ Ρ†Π΅Π»ΡŒΡŽ, Ρ‡Ρ‚ΠΎΠ±Ρ‹ Π·Π°Π²Π΅Π΄ΠΎΠΌΠΎ ΠΏΡ€Π°Π²ΠΈΠ»ΡŒΠ½ΠΎ ΠΏΠΎΠ΄ΠΊΠ»ΡŽΡ‡Π°Ρ‚ΡŒ ΠΊΠ»Π΅ΠΌΠΌΡ‹ ΠΏΡ€ΠΈΠ±ΠΎΡ€Π° ΠΊ ΡΠΎΠΎΡ‚Π²Π΅Ρ‚ΡΡ‚Π²ΡƒΡŽΡ‰ΠΈΠΌ ΠΏΡ€ΠΎΠ²ΠΎΠ΄Π°ΠΌ, ΠΏΠΎΠ΄ΠΊΠ»ΡŽΡ‡Ρ‘Π½Π½Ρ‹ΠΌ ΠΊ источнику.

ΠžΠ±Ρ€Π°Ρ‚ΠΈ Π²Π½ΠΈΠΌΠ°Π½ΠΈΠ΅!

Π’ ΠΎΡ‚Π»ΠΈΡ‡ΠΈΠ΅ ΠΎΡ‚ Π°ΠΌΠΏΠ΅Ρ€ΠΌΠ΅Ρ‚Ρ€Π°, ΠΊΠΎΡ‚ΠΎΡ€Ρ‹ΠΉ Π²ΠΊΠ»ΡŽΡ‡Π°Π΅Ρ‚ΡΡ Π² Ρ€Π°Π·Ρ€Ρ‹Π² Ρ†Π΅ΠΏΠΈ ΠΏΠΎΡΠ»Π΅Π΄ΠΎΠ²Π°Ρ‚Π΅Π»ΡŒΠ½ΠΎ, Π²ΠΎΠ»ΡŒΡ‚ΠΌΠ΅Ρ‚Ρ€ Π²ΠΊΠ»ΡŽΡ‡Π°Π΅Ρ‚ΡΡ Π² Ρ†Π΅ΠΏΡŒ ΠΏΠ°Ρ€Π°Π»Π»Π΅Π»ΡŒΠ½ΠΎ.

Π’ΠΊΠ»ΡŽΡ‡Π°Ρ Π²ΠΎΠ»ΡŒΡ‚ΠΌΠ΅Ρ‚Ρ€ Π² Ρ†Π΅ΠΏΡŒ постоянного Ρ‚ΠΎΠΊΠ°, Π½Π΅ΠΎΠ±Ρ…ΠΎΠ΄ΠΈΠΌΠΎ ΡΠΎΠ±Π»ΡŽΠ΄Π°Ρ‚ΡŒ ΠΏΠΎΠ»ΡΡ€Π½ΠΎΡΡ‚ΡŒ.

Π‘Π±ΠΎΡ€ΠΊΡƒ элСктричСской Ρ†Π΅ΠΏΠΈ Π»ΡƒΡ‡ΡˆΠ΅ Π½Π°Ρ‡ΠΈΠ½Π°Ρ‚ΡŒ со всСх элСмСнтов, ΠΊΡ€ΠΎΠΌΠ΅ Π²ΠΎΠ»ΡŒΡ‚ΠΌΠ΅Ρ‚Ρ€Π°, Π° Π΅Π³ΠΎ ΡƒΠΆΠ΅ ΠΏΠΎΠ΄ΠΊΠ»ΡŽΡ‡Π°Ρ‚ΡŒ Π² самом ΠΊΠΎΠ½Ρ†Π΅.

Π’ΠΎΠ»ΡŒΡ‚ΠΌΠ΅Ρ‚Ρ€Ρ‹ дСлятся Π½Π° ΠΏΡ€ΠΈΠ±ΠΎΡ€Ρ‹ постоянного Ρ‚ΠΎΠΊΠ° ΠΈ ΠΏΠ΅Ρ€Π΅ΠΌΠ΅Π½Π½ΠΎΠ³ΠΎ Ρ‚ΠΎΠΊΠ°.

Если ΠΏΡ€ΠΈΠ±ΠΎΡ€ ΠΏΡ€Π΅Π΄Π½Π°Π·Π½Π°Ρ‡Π΅Π½ для Ρ†Π΅ΠΏΠ΅ΠΉ ΠΏΠ΅Ρ€Π΅ΠΌΠ΅Π½Π½ΠΎΠ³ΠΎ Ρ‚ΠΎΠΊΠ°, Ρ‚ΠΎ Π½Π° Ρ†ΠΈΡ„Π΅Ρ€Π±Π»Π°Ρ‚Π΅ принято ΠΈΠ·ΠΎΠ±Ρ€Π°ΠΆΠ°Ρ‚ΡŒ Π²ΠΎΠ»Π½ΠΈΡΡ‚ΡƒΡŽ линию. Если ΠΏΡ€ΠΈΠ±ΠΎΡ€ ΠΏΡ€Π΅Π΄Π½Π°Π·Π½Π°Ρ‡Π΅Π½ для Ρ†Π΅ΠΏΠ΅ΠΉ постоянного Ρ‚ΠΎΠΊΠ°, Ρ‚ΠΎ линия Π±ΡƒΠ΄Π΅Ρ‚ прямой.

Π’ΠΎΠ»ΡŒΡ‚ΠΌΠ΅Ρ‚Ρ€Β ΠΏΠΎΡΡ‚ΠΎΡΠ½Π½ΠΎΠ³ΠΎ Ρ‚ΠΎΠΊΠ°Π’ΠΎΠ»ΡŒΡ‚ΠΌΠ΅Ρ‚Ρ€ ΠΏΠ΅Ρ€Π΅ΠΌΠ΅Π½Π½ΠΎΠ³ΠΎ Ρ‚ΠΎΠΊΠ°

МоТно ΠΎΠ±Ρ€Π°Ρ‚ΠΈΡ‚ΡŒ Π²Π½ΠΈΠΌΠ°Π½ΠΈΠ΅ Π½Π° ΠΊΠ»Π΅ΠΌΠΌΡ‹ ΠΏΡ€ΠΈΠ±ΠΎΡ€Π°. Если ΡƒΠΊΠ°Π·Π°Π½Π° ΠΏΠΎΠ»ΡΡ€Π½ΠΎΡΡ‚ΡŒ (Β«(+)Β» ΠΈ Β«(-)Β»), Ρ‚ΠΎ это ΠΏΡ€ΠΈΠ±ΠΎΡ€ для измСрСния постоянного напряТСния.

Иногда ΠΈΡΠΏΠΎΠ»ΡŒΠ·ΡƒΡŽΡ‚ Π±ΡƒΠΊΠ²Ρ‹ (AC/DC). Π’ ΠΏΠ΅Ρ€Π΅Π²ΠΎΠ΄Π΅ с английского (AC) (alternating current) β€” ΠΏΠ΅Ρ€Π΅ΠΌΠ΅Π½Π½Ρ‹ΠΉ Ρ‚ΠΎΠΊ, Π° (DC) (direct current)Β β€” постоянный Ρ‚ΠΎΠΊ.

Π’ Ρ†Π΅ΠΏΡŒ ΠΏΠ΅Ρ€Π΅ΠΌΠ΅Π½Π½ΠΎΠ³ΠΎ Ρ‚ΠΎΠΊΠ° Π²ΠΊΠ»ΡŽΡ‡Π°Π΅Ρ‚ΡΡ Π²ΠΎΠ»ΡŒΡ‚ΠΌΠ΅Ρ‚Ρ€ для измСрСния ΠΏΠ΅Ρ€Π΅ΠΌΠ΅Π½Π½ΠΎΠ³ΠΎ Ρ‚ΠΎΠΊΠ°. Он полярности Π½Π΅ ΠΈΠΌΠ΅Π΅Ρ‚.

ΠžΠ±Ρ€Π°Ρ‚ΠΈ Π²Π½ΠΈΠΌΠ°Π½ΠΈΠ΅!

Для измСрСния напряТСния ΠΌΠΎΠΆΠ½ΠΎ ΠΈΡΠΏΠΎΠ»ΡŒΠ·ΠΎΠ²Π°Ρ‚ΡŒ ΠΈ ΠΌΡƒΠ»ΡŒΡ‚ΠΈΠΌΠ΅Ρ‚Ρ€.

ΠŸΠ΅Ρ€Π΅Π΄ ΠΈΠ·ΠΌΠ΅Ρ€Π΅Π½ΠΈΠ΅ΠΌ Π½Π΅ΠΎΠ±Ρ…ΠΎΠ΄ΠΈΠΌΠΎ ΠΏΡ€ΠΎΡ‡ΠΈΡ‚Π°Ρ‚ΡŒ ΠΈΠ½ΡΡ‚Ρ€ΡƒΠΊΡ†ΠΈΡŽ, Ρ‡Ρ‚ΠΎΠ±Ρ‹ ΠΏΡ€Π°Π²ΠΈΠ»ΡŒΠ½ΠΎ ΠΏΠΎΠ΄ΠΊΠ»ΡŽΡ‡ΠΈΡ‚ΡŒ ΠΏΡ€ΠΈΠ±ΠΎΡ€.

  • Π‘Π»Π΅Π΄ΡƒΠ΅Ρ‚ ΠΏΠΎΠΌΠ½ΠΈΡ‚ΡŒ, Ρ‡Ρ‚ΠΎ высокоС напряТСниС опасно.
  • Π§Ρ‚ΠΎ Π±ΡƒΠ΄Π΅Ρ‚ с Ρ‡Π΅Π»ΠΎΠ²Π΅ΠΊΠΎΠΌ, ΠΊΠΎΡ‚ΠΎΡ€Ρ‹ΠΉ окаТСтся рядом с ΡƒΠΏΠ°Π²ΡˆΠΈΠΌ ΠΎΠ³ΠΎΠ»Ρ‘Π½Π½Ρ‹ΠΌ ΠΊΠ°Π±Π΅Π»Π΅ΠΌ, находящимся ΠΏΠΎΠ΄ высоким напряТСниСм?
  • Π’Π°ΠΊ ΠΊΠ°ΠΊ зСмля являСтся ΠΏΡ€ΠΎΠ²ΠΎΠ΄Π½ΠΈΠΊΠΎΠΌ элСктричСского Ρ‚ΠΎΠΊΠ°, Π²ΠΎΠΊΡ€ΡƒΠ³ ΡƒΠΏΠ°Π²ΡˆΠ΅Π³ΠΎ ΠΎΠ³ΠΎΠ»Ρ‘Π½Π½ΠΎΠ³ΠΎ кабСля, находящСгося ΠΏΠΎΠ΄ напряТСниСм, ΠΌΠΎΠΆΠ΅Ρ‚ Π²ΠΎΠ·Π½ΠΈΠΊΠ½ΡƒΡ‚ΡŒ опасноС для Ρ‡Π΅Π»ΠΎΠ²Π΅ΠΊΠ° шаговоС напряТСниС.

ΠŸΡ€ΠΈ ΠΏΠΎΠΏΠ°Π΄Π°Π½ΠΈΠΈ ΠΏΠΎΠ΄ шаговоС напряТСниС Π΄Π°ΠΆΠ΅ нСбольшого значСния Π²ΠΎΠ·Π½ΠΈΠΊΠ°ΡŽΡ‚ Π½Π΅ΠΏΡ€ΠΎΠΈΠ·Π²ΠΎΠ»ΡŒΠ½Ρ‹Π΅ судороТныС сокращСния ΠΌΡ‹ΡˆΡ† Π½ΠΎΠ³. ΠžΠ±Ρ‹Ρ‡Π½ΠΎ Ρ‡Π΅Π»ΠΎΠ²Π΅ΠΊΡƒ удаётся Π² Ρ‚Π°ΠΊΠΎΠΉ ситуации своСврСмСнно Π²Ρ‹ΠΉΡ‚ΠΈ ΠΈΠ· опасной Π·ΠΎΠ½Ρ‹.

ΠžΠ±Ρ€Π°Ρ‚ΠΈ Π²Π½ΠΈΠΌΠ°Π½ΠΈΠ΅!

Однако нСльзя Π²Ρ‹Π±Π΅Π³Π°Ρ‚ΡŒ ΠΎΡ‚Ρ‚ΡƒΠ΄Π° ΠΎΠ³Ρ€ΠΎΠΌΠ½Ρ‹ΠΌΠΈ шагами, шаговоС напряТСниС ΠΏΡ€ΠΈ этом Ρ‚ΠΎΠ»ΡŒΠΊΠΎ увСличится! Π’Ρ‹Ρ…ΠΎΠ΄ΠΈΡ‚ΡŒ Π½Π°Π΄ΠΎ ΠΎΠ±ΡΠ·Π°Ρ‚Π΅Π»ΡŒΠ½ΠΎ быстро, Π½ΠΎ ΠΎΡ‡Π΅Π½ΡŒ ΠΌΠ΅Π»ΠΊΠΈΠΌΠΈ шагами ΠΈΠ»ΠΈ скачками Π½Π° ΠΎΠ΄Π½ΠΎΠΉ Π½ΠΎΠ³Π΅!

БущСствуСт ΠΌΠ½ΠΎΠ³ΠΎ Π·Π½Π°ΠΊΠΎΠ², ΠΏΡ€Π΅Π΄ΡƒΠΏΡ€Π΅ΠΆΠ΄Π°ΡŽΡ‰ΠΈΡ… ΠΎ высоком напряТСнии. Π’ΠΎΡ‚ Π½Π΅ΠΊΠΎΡ‚ΠΎΡ€Ρ‹Π΅ ΠΈΠ· Π½ΠΈΡ….

Β Β Β 

БСзопасным напряТСниСм для Ρ‡Π΅Π»ΠΎΠ²Π΅ΠΊΠ° считаСтся напряТСниС (42) Π’ Π² Π½ΠΎΡ€ΠΌΠ°Π»ΡŒΠ½Ρ‹Ρ… условиях ΠΈ (12) Π’ Π² условиях с ΠΏΠΎΠ²Ρ‹ΡˆΠ΅Π½Π½ΠΎΠΉ ΠΎΠΏΠ°ΡΠ½ΠΎΡΡ‚ΡŒΡŽ (ΡΡ‹Ρ€ΠΎΡΡ‚ΡŒ, высокая Ρ‚Π΅ΠΌΠΏΠ΅Ρ€Π°Ρ‚ΡƒΡ€Π°, мСталличСскиС ΠΏΠΎΠ»Ρ‹ ΠΈ Π΄Ρ€.).

Π˜ΡΡ‚ΠΎΡ‡Π½ΠΈΠΊΠΈ:

ΠŸΡ‘Ρ€Ρ‹ΡˆΠΊΠΈΠ½ А.Π’. Π€ΠΈΠ·ΠΈΠΊΠ°, 8 класс// Π”Π ΠžΠ€Π, 2013.

http://class-fizika.narod.ru/8_29.htmhttp://interneturok.ru/ru/school/physics/8-klass/belektricheskie-yavleniyab/elektricheskoe-napryazhenie

http://kamenskih3.narod.ru/untitled74.htm

Π˜ΡΡ‚ΠΎΡ‡Π½ΠΈΠΊ: https://www.yaklass.ru/p/fizika/8-klass/elektricheskie-iavleniia-12351/elektricheskoe-napriazhenie-voltmetr-12361/re-9effb98b-8fe9-4eb7-9964-54c153a18241

ЭлСктричСскоС напряТСниС: ΠΎΠΏΡ€Π΅Π΄Π΅Π»Π΅Π½ΠΈΠ΅, Ρ„ΠΎΡ€ΠΌΡƒΠ»Ρ‹ ΠΈ ΠΊΠ°ΠΊ измСряСтся

Π’ Π΄Π°Π½Π½ΠΎΠΉ ΡΡ‚Π°Ρ‚ΡŒΠ΅ ΠΌΡ‹ ΠΏΠΎΠ΄Ρ€ΠΎΠ±Π½ΠΎ Ρ€Π°Π·Π±Π΅Ρ€Π΅ΠΌ Ρ‡Ρ‚ΠΎ Ρ‚Π°ΠΊΠΎΠ΅ напряТСниС, ΠΊΠ°ΠΊ просто Π΅Π³ΠΎ ΠΏΡ€Π΅Π΄ΡΡ‚Π°Π²ΠΈΡ‚ΡŒ ΠΈ ΠΈΠ·ΠΌΠ΅Ρ€ΠΈΡ‚ΡŒ.

ΠžΠΏΡ€Π΅Π΄Π΅Π»Π΅Π½ΠΈΠ΅

НапряТСниС β€” это элСктродвиТущая сила, которая Ρ‚ΠΎΠ»ΠΊΠ°Π΅Ρ‚ свободныС элСктроны ΠΎΡ‚ ΠΎΠ΄Π½ΠΎΠ³ΠΎ Π°Ρ‚ΠΎΠΌΠ° ΠΊ Π΄Ρ€ΡƒΠ³ΠΎΠΌΡƒ Π² Ρ‚ΠΎΠΌ ΠΆΠ΅ Π½Π°ΠΏΡ€Π°Π²Π»Π΅Π½ΠΈΠΈ.

Π’ ΠΏΠ΅Ρ€Π²Ρ‹Π΅ Π΄Π½ΠΈ элСктричСства напряТСниС Π±Ρ‹Π»ΠΎ извСстно ΠΊΠ°ΠΊ элСктродвиТущая сила (Π­Π”Π‘). ИмСнно поэтому Π² уравнСниях, Ρ‚Π°ΠΊΠΈΡ… ΠΊΠ°ΠΊ Π·Π°ΠΊΠΎΠ½ Ома, напряТСниС прСдставлСно символом Π•.

АлСссандро Π’ΠΎΠ»ΡŒΡ‚Π°

Π•Π΄ΠΈΠ½ΠΈΡ†Π΅ΠΉ элСктричСского ΠΏΠΎΡ‚Π΅Π½Ρ†ΠΈΠ°Π»Π° являСтся Π²ΠΎΠ»ΡŒΡ‚, Π½Π°Π·Π²Π°Π½Π½Ρ‹ΠΉ Π² Ρ‡Π΅ΡΡ‚ΡŒ АлСссандро Π’ΠΎΠ»ΡŒΡ‚Π°, ΠΈΡ‚Π°Π»ΡŒΡΠ½ΡΠΊΠΎΠ³ΠΎ Ρ„ΠΈΠ·ΠΈΠΊΠ°, ТившСго ΠΌΠ΅ΠΆΠ΄Ρƒ 1745 ΠΈ 1827 Π³ΠΎΠ΄Π°ΠΌΠΈ.

АлСссандро Π’ΠΎΠ»ΡŒΡ‚Π° Π±Ρ‹Π» ΠΎΠ΄Π½ΠΈΠΌ ΠΈΠ· ΠΏΠΈΠΎΠ½Π΅Ρ€ΠΎΠ² динамичСского элСктричСства.Β Π˜ΡΡΠ»Π΅Π΄ΡƒΡ основныС свойства элСктричСства, ΠΎΠ½ ΠΈΠ·ΠΎΠ±Ρ€Π΅Π» ΠΏΠ΅Ρ€Π²ΡƒΡŽ Π±Π°Ρ‚Π°Ρ€Π΅ΡŽ ΠΈ ΡƒΠ³Π»ΡƒΠ±ΠΈΠ» ΠΏΠΎΠ½ΠΈΠΌΠ°Π½ΠΈΠ΅ элСктричСства.

ΠŸΡ€Π΅Π΄ΡΡ‚Π°Π²Π»Π΅Π½ΠΈΠ΅ напряТСния

Π›Π΅Π³Ρ‡Π΅ всСго ΠΏΠΎΠ½ΡΡ‚ΡŒ напряТСнии, прСдставив Π΄Π°Π²Π»Π΅Π½ΠΈΠΈ Π² Ρ‚Ρ€ΡƒΠ±Π΅. ΠŸΡ€ΠΈ Π±ΠΎΠ»Π΅Π΅ высоком напряТСнии (Π΄Π°Π²Π»Π΅Π½ΠΈΠΈ) Π±ΡƒΠ΄Π΅Ρ‚ Ρ‚Π΅Ρ‡ΡŒ Π±ΠΎΠ»Π΅Π΅ ΡΠΈΠ»ΡŒΠ½Ρ‹ΠΉ Ρ‚ΠΎΠΊ. Π₯отя Π²Π°ΠΆΠ½ΠΎ ΠΏΠΎΠ½ΠΈΠΌΠ°Ρ‚ΡŒ, Ρ‡Ρ‚ΠΎ напряТСниС (Π΄Π°Π²Π»Π΅Π½ΠΈΠ΅) ΠΌΠΎΠΆΠ΅Ρ‚ ΡΡƒΡ‰Π΅ΡΡ‚Π²ΠΎΠ²Π°Ρ‚ΡŒ Π±Π΅Π· Ρ‚ΠΎΠΊΠ° (ΠΏΠΎΡ‚ΠΎΠΊΠ°), Π½ΠΎ Ρ‚ΠΎΠΊ Π½Π΅ ΠΌΠΎΠΆΠ΅Ρ‚ ΡΡƒΡ‰Π΅ΡΡ‚Π²ΠΎΠ²Π°Ρ‚ΡŒ Π±Π΅Π· напряТСния (давлСния).

НапряТСниС часто Π½Π°Π·Ρ‹Π²Π°ΡŽΡ‚ Ρ€Π°Π·Π½ΠΎΡΡ‚ΡŒΡŽ ΠΏΠΎΡ‚Π΅Π½Ρ†ΠΈΠ°Π»ΠΎΠ², ΠΏΠΎΡ‚ΠΎΠΌΡƒ Ρ‡Ρ‚ΠΎ ΠΌΠ΅ΠΆΠ΄Ρƒ Π»ΡŽΠ±Ρ‹ΠΌΠΈ двумя Ρ‚ΠΎΡ‡ΠΊΠ°ΠΌΠΈ Π² Ρ†Π΅ΠΏΠΈ Π±ΡƒΠ΄Π΅Ρ‚ ΡΡƒΡ‰Π΅ΡΡ‚Π²ΠΎΠ²Π°Ρ‚ΡŒ Ρ€Π°Π·Π½ΠΈΡ†Π° Π² ΠΏΠΎΡ‚Π΅Π½Ρ†ΠΈΠ°Π»ΡŒΠ½ΠΎΠΉ энСргии элСктронов. Когда элСктроны ΠΏΡ€ΠΎΡ‚Π΅ΠΊΠ°ΡŽΡ‚ Ρ‡Π΅Ρ€Π΅Π· Π±Π°Ρ‚Π°Ρ€Π΅ΡŽ, ΠΈΡ… ΠΏΠΎΡ‚Π΅Π½Ρ†ΠΈΠ°Π»ΡŒΠ½Π°Ρ энСргия увСличиваСтся, Π½ΠΎ ΠΊΠΎΠ³Π΄Π° ΠΎΠ½ΠΈ ΠΏΡ€ΠΎΡ‚Π΅ΠΊΠ°ΡŽΡ‚ Ρ‡Π΅Ρ€Π΅Π· Π»Π°ΠΌΠΏΠΎΡ‡ΠΊΡƒ, ΠΈΡ… ΠΏΠΎΡ‚Π΅Π½Ρ†ΠΈΠ°Π»ΡŒΠ½Π°Ρ энСргия Π±ΡƒΠ΄Π΅Ρ‚ ΡƒΠΌΠ΅Π½ΡŒΡˆΠ°Ρ‚ΡŒΡΡ, эта энСргия ΠΏΠΎΠΊΠΈΠ½Π΅Ρ‚ Ρ†Π΅ΠΏΡŒ Π² Π²ΠΈΠ΄Π΅ свСта ΠΈ Ρ‚Π΅ΠΏΠ»Π°.

Π’ΠΎΠ·ΡŒΠΌΠΈΡ‚Π΅, Π½Π°ΠΏΡ€ΠΈΠΌΠ΅Ρ€, ΠΎΠ±Ρ‹Ρ‡Π½ΡƒΡŽ 1,5-Π²ΠΎΠ»ΡŒΡ‚ΠΎΠ²ΡƒΡŽ Π±Π°Ρ‚Π°Ρ€Π΅ΡŽ AA, ΠΌΠ΅ΠΆΠ΄Ρƒ двумя ΠΊΠ»Π΅ΠΌΠΌΠ°ΠΌΠΈ (+ ΠΈ -) Π΅ΡΡ‚ΡŒ Ρ€Π°Π·Π½ΠΎΡΡ‚ΡŒ ΠΏΠΎΡ‚Π΅Π½Ρ†ΠΈΠ°Π»ΠΎΠ² 1,5 Π’ΠΎΠ»ΡŒΡ‚.

НапряТСниС ΠΈΠ»ΠΈ Ρ€Π°Π·Π½ΠΎΡΡ‚ΡŒ ΠΏΠΎΡ‚Π΅Π½Ρ†ΠΈΠ°Π»ΠΎΠ² β€” это просто ΠΈΠ·ΠΌΠ΅Ρ€Π΅Π½ΠΈΠ΅ количСства энСргии (Π² дТоулях) Π½Π° Π΅Π΄ΠΈΠ½ΠΈΡ†Ρƒ заряда (ΠΊΡƒΠ»ΠΎΠ½Π°). НапримСр, Π² 1,5-Π²ΠΎΠ»ΡŒΡ‚ΠΎΠ²ΠΎΠΉ Π±Π°Ρ‚Π°Ρ€Π΅Π΅ AA ΠΊΠ°ΠΆΠ΄Ρ‹ΠΉ ΠΊΡƒΠ»ΠΎΠ½ (заряд) Π±ΡƒΠ΄Π΅Ρ‚ ΠΏΠΎΠ»ΡƒΡ‡Π°Ρ‚ΡŒ 1,5 Π²ΠΎΠ»ΡŒΡ‚ ΠΈΠ»ΠΈ Π΄ΠΆΠΎΡƒΠ»Π΅ΠΉ энСргии.

  • НапряТСниС = [Π”ΠΆΠΎΡƒΠ»ΡŒ Γ· ΠšΡƒΠ»ΠΎΠ½]
  • 1 Π²ΠΎΠ»ΡŒΡ‚ = 1 Π΄ΠΆΠΎΡƒΠ»ΡŒ Π½Π° ΠΊΡƒΠ»ΠΎΠ½
  • 100 Π²ΠΎΠ»ΡŒΡ‚ = 100 Π΄ΠΆΠΎΡƒΠ»Π΅ΠΉ Π½Π° ΠΊΡƒΠ»ΠΎΠ½
  • 1 ΠΊΡƒΠ»ΠΎΠ½ = 6 200 000 000 000 000 000 элСктронов (6,2 Γ— 10Β 18Β )

Π’ Ρ‡Π΅ΠΌ измСряСтся напряТСниС

ΠœΡ‹ измСряСм напряТСниС Π² Π΅Π΄ΠΈΠ½ΠΈΡ†Π°Ρ… Β«Π’ΠΎΠ»ΡŒΡ‚Β», ΠΊΠΎΡ‚ΠΎΡ€Ρ‹Π΅ ΠΎΠ±Ρ‹Ρ‡Π½ΠΎ ΠΎΠ±ΠΎΠ·Π½Π°Ρ‡Π°ΡŽΡ‚ΡΡ просто Π±ΡƒΠΊΠ²ΠΎΠΉ Β«VΒ» Π½Π° Ρ‡Π΅Ρ€Ρ‚Π΅ΠΆΠ°Ρ… ΠΈ тСхничСской Π»ΠΈΡ‚Π΅Ρ€Π°Ρ‚ΡƒΡ€Π΅. Часто Π½Π΅ΠΎΠ±Ρ…ΠΎΠ΄ΠΈΠΌΠΎ количСствСнно ΠΎΠΏΡ€Π΅Π΄Π΅Π»ΠΈΡ‚ΡŒ Π²Π΅Π»ΠΈΡ‡ΠΈΠ½Ρƒ напряТСния, это дСлаСтся Π² соотвСтствии с Π΅Π΄ΠΈΠ½ΠΈΡ†Π°ΠΌΠΈ БИ, Π½Π°ΠΈΠ±ΠΎΠ»Π΅Π΅ распространСнныС Π²Π΅Π»ΠΈΡ‡ΠΈΠ½Ρ‹ напряТСния, ΠΊΠΎΡ‚ΠΎΡ€Ρ‹Π΅ Π²Ρ‹ ΡƒΠ²ΠΈΠ΄ΠΈΡ‚Π΅:

  • ΠΌΠ΅Π³Π°Π²ΠΎΠ»ΡŒΡ‚ (ΠΌΠ’)
  • ΠΊΠΈΠ»ΠΎΠ²ΠΎΠ»ΡŒΡ‚ (ΠΊΠ’)
  • Π²ΠΎΠ»ΡŒΡ‚ (Π’)
  • ΠΌΠΈΠ»Π»ΠΈΠ²ΠΎΠ»ΡŒΡ‚ (ΠΌΠ’)
  • ΠΌΠΈΠΊΡ€ΠΎΠ²ΠΎΠ»ΡŒΡ‚ (ΠΌΠΊΠ’)

НапряТСниС всСгда измСряСтся Π² Π΄Π²ΡƒΡ… Ρ‚ΠΎΡ‡ΠΊΠ°Ρ… с ΠΏΠΎΠΌΠΎΡ‰ΡŒΡŽ устройства, Π½Π°Π·Ρ‹Π²Π°Π΅ΠΌΠΎΠ³ΠΎ Π²ΠΎΠ»ΡŒΡ‚ΠΌΠ΅Ρ‚Ρ€ΠΎΠΌ. Π’ΠΎΠ»ΡŒΡ‚ΠΌΠ΅Ρ‚Ρ€Ρ‹ ΡΠ²Π»ΡΡŽΡ‚ΡΡ Π»ΠΈΠ±ΠΎ Ρ†ΠΈΡ„Ρ€ΠΎΠ²Ρ‹ΠΌΠΈ, Π»ΠΈΠ±ΠΎ Π°Π½Π°Π»ΠΎΠ³ΠΎΠ²Ρ‹ΠΌΠΈ, ΠΏΡ€ΠΈΡ‡Π΅ΠΌ послСдний являСтся Π½Π°ΠΈΠ±ΠΎΠ»Π΅Π΅ Ρ‚ΠΎΡ‡Π½Ρ‹ΠΌ.

Π’ΠΎΠ»ΡŒΡ‚ΠΌΠ΅Ρ‚Ρ€Ρ‹ ΠΎΠ±Ρ‹Ρ‡Π½ΠΎ встроСны Π² ΠΏΠΎΡ€Ρ‚Π°Ρ‚ΠΈΠ²Π½Ρ‹Π΅ Ρ†ΠΈΡ„Ρ€ΠΎΠ²Ρ‹Π΅ ΠΌΡƒΠ»ΡŒΡ‚ΠΈΠΌΠ΅Ρ‚Ρ€ΠΎΠ²Ρ‹Π΅ устройства, ΠΊΠ°ΠΊ ΠΏΠΎΠΊΠ°Π·Π°Π½ΠΎ Π½ΠΈΠΆΠ΅, ΠΎΠ½ΠΈ ΡΠ²Π»ΡΡŽΡ‚ΡΡ распространСнным ΠΈ часто Π²Π°ΠΆΠ½Ρ‹ΠΌ инструмСнтом для любого элСктрика ΠΈΠ»ΠΈ ΠΈΠ½ΠΆΠ΅Π½Π΅Ρ€Π°-элСктрика.

ΠžΠ±Ρ‹Ρ‡Π½ΠΎ Π²Ρ‹ Π½Π°ΠΉΠ΄Π΅Ρ‚Π΅ Π°Π½Π°Π»ΠΎΠ³ΠΎΠ²Ρ‹Π΅ Π²ΠΎΠ»ΡŒΡ‚ΠΌΠ΅Ρ‚Ρ€Ρ‹ Π½Π° старых элСктричСских панСлях, Ρ‚Π°ΠΊΠΈΡ… ΠΊΠ°ΠΊ Ρ€Π°ΡΠΏΡ€Π΅Π΄Π΅Π»ΠΈΡ‚Π΅Π»ΡŒΠ½Ρ‹Π΅ Ρ‰ΠΈΡ‚Ρ‹ ΠΈ Π³Π΅Π½Π΅Ρ€Π°Ρ‚ΠΎΡ€Ρ‹, Π½ΠΎ ΠΏΠΎΡ‡Ρ‚ΠΈ всС Π½ΠΎΠ²ΠΎΠ΅ ΠΎΠ±ΠΎΡ€ΡƒΠ΄ΠΎΠ²Π°Π½ΠΈΠ΅ Π±ΡƒΠ΄Π΅Ρ‚ ΠΏΠΎΡΡ‚Π°Π²Π»ΡΡ‚ΡŒΡΡ с Ρ†ΠΈΡ„Ρ€ΠΎΠ²Ρ‹ΠΌΠΈ счСтчиками Π² качСствС стандарта.

ΠŸΠΎΡ€Ρ‚Π°Ρ‚ΠΈΠ²Π½Ρ‹ΠΉ Ρ†ΠΈΡ„Ρ€ΠΎΠ²ΠΎΠΉ ΠΌΡƒΠ»ΡŒΡ‚ΠΈΠΌΠ΅Ρ‚Ρ€ с Ρ„ΡƒΠ½ΠΊΡ†ΠΈΠ΅ΠΉ Π²ΠΎΠ»ΡŒΡ‚ΠΌΠ΅Ρ‚Ρ€Π°

На элСктричСских схСмах Π²Ρ‹ ΡƒΠ²ΠΈΠ΄ΠΈΡ‚Π΅ устройства Π²ΠΎΠ»ΡŒΡ‚ΠΌΠ΅Ρ‚Ρ€Π°, ΠΎΠ±ΠΎΠ·Π½Π°Ρ‡Π΅Π½Π½Ρ‹Π΅ Π±ΡƒΠΊΠ²ΠΎΠΉ V Π²Π½ΡƒΡ‚Ρ€ΠΈ ΠΊΡ€ΡƒΠ³Π°, ΠΊΠ°ΠΊ ΠΏΠΎΠΊΠ°Π·Π°Π½ΠΎ Π½ΠΈΠΆΠ΅:

РасчСт напряТСния

Π’ элСктричСских цСпях напряТСниС ΠΌΠΎΠΆΠ΅Ρ‚ Π±Ρ‹Ρ‚ΡŒ рассчитано Π² соотвСтствии с Ρ‚Ρ€Π΅ΡƒΠ³ΠΎΠ»ΡŒΠ½ΠΈΠΊΠΎΠΌ Ома.Β Π§Ρ‚ΠΎΠ±Ρ‹ Π½Π°ΠΉΡ‚ΠΈ напряТСниС (V), просто ΡƒΠΌΠ½ΠΎΠΆΡŒΡ‚Π΅ Ρ‚ΠΎΠΊ (I) Π½Π° сопротивлСниС (R).

НапряТСниС (V) = Ρ‚ΠΎΠΊ (I) * сопротивлСниС (R)

V = I *R

ΠŸΡ€ΠΈΠΌΠ΅Ρ€

  1. Π’ΠΎΠΊ Π² Ρ†Π΅ΠΏΠΈ (I) = 10 АБопротивлСниС Ρ†Π΅ΠΏΠΈ (R) = 2 Ом
  2. НапряТСниС (V) = 10 А * 2 Ом
  3. ΠžΡ‚Π²Π΅Ρ‚: V = 20Π’

РСзюмС

  • НапряТСниС β€” это сила, которая ΠΏΠ΅Ρ€Π΅ΠΌΠ΅Ρ‰Π°Π΅Ρ‚ элСктроны ΠΎΡ‚ ΠΎΠ΄Π½ΠΎΠ³ΠΎ Π°Ρ‚ΠΎΠΌΠ° ΠΊ Π΄Ρ€ΡƒΠ³ΠΎΠΌΡƒ
  • НапряТСниС Ρ‚Π°ΠΊΠΆΠ΅ извСстно ΠΊΠ°ΠΊ Ρ€Π°Π·Π½ΠΎΡΡ‚ΡŒ ΠΏΠΎΡ‚Π΅Π½Ρ†ΠΈΠ°Π»ΠΎΠ²
  • НапряТСниС измСряСтся Π² Π΅Π΄ΠΈΠ½ΠΈΡ†Π°Ρ… Β«Π²ΠΎΠ»ΡŒΡ‚Β» (Π’)
  • Π‘Π°Ρ‚Π°Ρ€Π΅ΠΈ ΡƒΠ²Π΅Π»ΠΈΡ‡ΠΈΠ²Π°ΡŽΡ‚ ΠΏΠΎΡ‚Π΅Π½Ρ†ΠΈΠ°Π»ΡŒΠ½ΡƒΡŽ ΡΠ½Π΅Ρ€Π³ΠΈΡŽ элСктронов
  • Π›Π°ΠΌΠΏΠΎΡ‡ΠΊΠΈ ΠΈ Π΄Ρ€ΡƒΠ³ΠΈΠ΅ Π½Π°Π³Ρ€ΡƒΠ·ΠΊΠΈ ΡƒΠΌΠ΅Π½ΡŒΡˆΠ°ΡŽΡ‚ ΠΏΠΎΡ‚Π΅Π½Ρ†ΠΈΠ°Π»ΡŒΠ½ΡƒΡŽ ΡΠ½Π΅Ρ€Π³ΠΈΡŽ элСктронов
  • НапряТСниС измСряСтся с ΠΏΠΎΠΌΠΎΡ‰ΡŒΡŽ Π²ΠΎΠ»ΡŒΡ‚ΠΌΠ΅Ρ‚Ρ€Π°
  • НапряТСниС Ρ†Π΅ΠΏΠΈ ΠΌΠΎΠΆΠ½ΠΎ Ρ€Π°ΡΡΡ‡ΠΈΡ‚Π°Ρ‚ΡŒ ΠΏΡƒΡ‚Π΅ΠΌ умноТСния Ρ‚ΠΎΠΊΠ° ΠΈ сопротивлСния

Π˜ΡΡ‚ΠΎΡ‡Π½ΠΈΠΊ: https://meanders.ru/naprjazhenie.shtml

Π§Ρ‚ΠΎ Ρ‚Π°ΠΊΠΎΠ΅ ЭлСктричСскоС напряТСниС β€” ΠžΠΏΡ€Π΅Π΄Π΅Π»Π΅Π½ΠΈΠ΅, ΠΈΠ·ΠΌΠ΅Ρ€Π΅Π½ΠΈΠ΅

Π‘ΠΎΠ»ΡŒΡˆΠΈΠ½ΡΡ‚Π²ΠΎ людСй Π² Π±Ρ‹Ρ‚Ρƒ ΠΌΠΎΠ³ΡƒΡ‚ ΠΎΠΏΠ΅Ρ€ΠΈΡ€ΠΎΠ²Π°Ρ‚ΡŒ Ρ‚Π°ΠΊΠΈΠΌ понятиСм ΠΊΠ°ΠΊ элСктричСскоС напряТСниС. ΠŸΡ€Π°ΠΊΡ‚ΠΈΡ‡Π΅ΡΠΊΠΈ всС Π·Π½Π°ΡŽΡ‚, Ρ‡Ρ‚ΠΎ бытовая Ρ€ΠΎΠ·Π΅Ρ‚ΠΊΠ° находится ΠΏΠΎΠ΄ напряТСниСм 220Π’, Π° ΠΏΠ°Π»ΡŒΡ‡ΠΈΠΊΠΎΠ²Π°Ρ Π±Π°Ρ‚Π°Ρ€Π΅ΠΉΠΊΠ° Π²Ρ‹Π΄Π°Π΅Ρ‚ напряТСниС всСго Π² 1.5Π’.

ΠŸΡ€ΠΈ этом Π΄Π°Π»Π΅ΠΊΠΎ Π½Π΅ ΠΊΠ°ΠΆΠ΄Ρ‹ΠΉ Ρ‡Π΅Π»ΠΎΠ²Π΅ΠΊ, ΠΎΠΊΠΎΠ½Ρ‡ΠΈΠ²ΡˆΠΈΠΉ ΡΡ€Π΅Π΄Π½ΡŽΡŽ ΡˆΠΊΠΎΠ»Ρƒ ΠΈΠ»ΠΈ Π΄Π°ΠΆΠ΅ тСхничСский Π’Π£Π— Π² состоянии ΠΎΡ‚Π²Π΅Ρ‚ΠΈΡ‚ΡŒ, Ρ‡Ρ‚ΠΎ ΠΆΠ΅ всС-Ρ‚Π°ΠΊΠΈ ΠΎΠ·Π½Π°Ρ‡Π°Π΅Ρ‚ Ρ‚Π΅Ρ€ΠΌΠΈΠ½ элСктричСскоС напряТСниС.

Π’ этом ΠΌΠ°Ρ‚Π΅Ρ€ΠΈΠ°Π»Π΅ ΠΌΡ‹ постараСмся ΠΎΡ‚Π²Π΅Ρ‚ΠΈΡ‚ΡŒ Π½Π° этот вопрос, ΠΏΠΎ возмоТности Π½Π΅ прибСгая ΠΊ слоТной ΠΌΠ°Ρ‚Π΅ΠΌΠ°Ρ‚ΠΈΠΊΠ΅.

ΠžΠΏΡ€Π΅Π΄Π΅Π»Π΅Π½ΠΈΠ΅ элСктричСского напряТСния

Π’ ΡƒΡ‡Π΅Π±Π½ΠΈΠΊΠ°Ρ… ΠΏΠΎ Ρ„ΠΈΠ·ΠΈΠΊΠ΅ ΠΈ элСктротСхникС ΠΌΠΎΠΆΠ½ΠΎ Π²ΡΡ‚Ρ€Π΅Ρ‚ΠΈΡ‚ΡŒ Ρ€Π°Π·Π½Ρ‹Π΅ опрСдСлСния элСктричСского напряТСния. Одно ΠΈΠ· Π½ΠΈΡ… Π·Π²ΡƒΡ‡ΠΈΡ‚ ΡΠ»Π΅Π΄ΡƒΡŽΡ‰ΠΈΠΌ ΠΎΠ±Ρ€Π°Π·ΠΎΠΌ: элСктричСскоС напряТСниС ΠΌΠ΅ΠΆΠ΄Ρƒ двумя Ρ‚ΠΎΡ‡ΠΊΠ°ΠΌΠΈ пространства Ρ€Π°Π²Π½ΠΎ разности ΠΏΠΎΡ‚Π΅Π½Ρ†ΠΈΠ°Π»ΠΎΠ² элСктричСского поля Π² этих Ρ‚ΠΎΡ‡ΠΊΠ°Ρ…. ΠœΠ°Ρ‚Π΅ΠΌΠ°Ρ‚ΠΈΡ‡Π΅ΡΠΊΠΈ это записываСтся Ρ‚Π°ΠΊ:

U=Ο†_a-Ο†_b (1).

Π“Π΄Π΅ U – элСктричСскоС напряТСниС, Π°Β Ο†_a ΠΈ Ο†_bΒ ΠΏΠΎΡ‚Π΅Π½Ρ†ΠΈΠ°Π»Ρ‹ элСктричСского поля Π² Ρ‚ΠΎΡ‡ΠΊΠ°Ρ… A ΠΈ B соотвСтствСнно.

Если ΠΌΡ‹ Π½Π΅ Π·Π½Π°Π΅ΠΌ Ρ‡Ρ‚ΠΎ Ρ‚Π°ΠΊΠΎΠ΅ ΠΏΠΎΡ‚Π΅Π½Ρ†ΠΈΠ°Π» элСктричСского поля Π² Ρ‚ΠΎΡ‡ΠΊΠ΅, Ρ‚ΠΎ ΠΏΡ€ΠΈΠ²Π΅Π΄Π΅Π½Π½ΠΎΠ΅ Π²Ρ‹ΡˆΠ΅ ΠΎΠΏΡ€Π΅Π΄Π΅Π»Π΅Π½ΠΈΠ΅ ΠΌΠ°Π»ΠΎ проясняСт вопрос, Ρ‡Ρ‚ΠΎ ΠΆΠ΅ Ρ‚Π°ΠΊΠΎΠ΅ элСктричСскоС напряТСниС.

Под ΠΏΠΎΡ‚Π΅Π½Ρ†ΠΈΠ°Π»ΠΎΠΌ элСктричСского поля Π² Ρ‚ΠΎΡ‡ΠΊΠ΅ ΠΏΠΎΠ½ΠΈΠΌΠ°ΡŽΡ‚ Ρ€Π°Π±ΠΎΡ‚Ρƒ, ΠΏΠΎ ΠΏΠ΅Ρ€Π΅ΠΌΠ΅Ρ‰Π΅Π½ΠΈΡŽ Π΅Π΄ΠΈΠ½ΠΈΡ‡Π½ΠΎΠ³ΠΎ заряда ΡΠΎΠ²Π΅Ρ€ΡˆΠ°Π΅ΠΌΡƒΡŽ элСктричСским ΠΏΠΎΠ»Π΅ΠΌ ΠΈΠ· Π΄Π°Π½Π½ΠΎΠΉ Ρ‚ΠΎΡ‡ΠΊΠΈ Π² Ρ‚ΠΎΡ‡ΠΊΡƒ с Π½ΡƒΠ»Π΅Π²Ρ‹ΠΌ ΠΏΠΎΡ‚Π΅Π½Ρ†ΠΈΠ°Π»ΠΎΠΌ.

На ΠΏΠ΅Ρ€Π²Ρ‹ΠΉ взгляд ΠΎΠΏΡ€Π΅Π΄Π΅Π»Π΅Π½ΠΈΠ΅ элСктричСского ΠΏΠΎΡ‚Π΅Π½Ρ†ΠΈΠ°Π»Π° каТСтся довольно слоТным. НапримСр, Π½Π΅ совсСм понятно, Π³Π΄Π΅ находится Ρ‚ΠΎΡ‡ΠΊΠ° с Π½ΡƒΠ»Π΅Π²Ρ‹ΠΌ ΠΏΠΎΡ‚Π΅Π½Ρ†ΠΈΠ°Π»ΠΎΠΌ.

 Для Π½Π°Ρ‡Π°Π»Π° Π½ΡƒΠΆΠ½ΠΎ Π·Π°ΠΏΠΎΠΌΠ½ΠΈΡ‚ΡŒ, Ρ‡Ρ‚ΠΎ элСктричСский ΠΏΠΎΡ‚Π΅Π½Ρ†ΠΈΠ°Π» это Ρ€Π°Π±ΠΎΡ‚Π° ΠΏΠΎ пСрСносу Π΅Π΄ΠΈΠ½ΠΈΡ‡Π½ΠΎΠ³ΠΎ заряда. Если ΠΎΠ±Ρ€Π°Ρ‚ΠΈΡ‚ΡŒΡΡ ΠΊ Ρ„ΠΎΡ€ΠΌΡƒΠ»Π΅ (1) Ρ‚ΠΎ станСт ясно, Ρ‡Ρ‚ΠΎ элСктричСскоС напряТСниС Π½Π΅ Ρ‡Ρ‚ΠΎ ΠΈΠ½ΠΎΠ΅, ΠΊΠ°ΠΊ Ρ€Π°Π·Π½ΠΎΡΡ‚ΡŒ Π΄Π²ΡƒΡ… Ρ€Π°Π±ΠΎΡ‚. Π’ΠΎ Π΅ΡΡ‚ΡŒ элСктричСскоС напряТСниС, Ρ‚ΠΎΠΆΠ΅ Π΅ΡΡ‚ΡŒ Ρ€Π°Π±ΠΎΡ‚Π°.

ΠžΡ‚ΡΡŽΠ΄Π° ΠΌΡ‹ ΠΏΡ€ΠΈΡ…ΠΎΠ΄ΠΈΠΌ ΠΊΠΎ Π²Ρ‚ΠΎΡ€ΠΎΠΌΡƒ ΠΎΠΏΡ€Π΅Π΄Π΅Π»Π΅Π½ΠΈΡŽ. ЭлСктричСскоС напряТСниС числСнно Ρ€Π°Π²Π½ΠΎ Ρ€Π°Π±ΠΎΡ‚Π΅ ΠΏΠΎ пСрСносу Π΅Π΄ΠΈΠ½ΠΈΡ‡Π½ΠΎΠ³ΠΎ элСктричСского заряда ΠΈΠ· Ρ‚ΠΎΡ‡ΠΊΠΈ А Π² Ρ‚ΠΎΡ‡ΠΊΡƒ Π’. ΠŸΡ€ΠΈ этом Ο†_a ΠΈ Ο†_b это ΠΏΠΎΡ‚Π΅Π½Ρ†ΠΈΠ°Π»ΡŒΠ½Π°Ρ энСргия ΠΊΠΎΡ‚ΠΎΡ€ΠΎΠΉ ΠΎΠ±Π»Π°Π΄Π°Π΅Ρ‚ Π΅Π΄ΠΈΠ½ΠΈΡ‡Π½Ρ‹ΠΉ заряд Π² Ρ‚ΠΎΡ‡ΠΊΠ°Ρ… А ΠΈ Π’ соотвСтствСнно.

Для Π»ΡƒΡ‡ΡˆΠ΅Π³ΠΎ понимания ΠΈΠ·Π»ΠΎΠΆΠ΅Π½Π½ΠΎΠ³ΠΎ Π²Ρ‹ΡˆΠ΅ ΠΌΠΎΠΆΠ½ΠΎ привСсти ΡΠ»Π΅Π΄ΡƒΡŽΡ‰ΡƒΡŽ аналогию. Π›ΡŽΠ±ΠΎΠ΅ Ρ‚Π΅Π»ΠΎ, находящССся Π½Π° Π½Π΅ΠΊΠΎΡ‚ΠΎΡ€ΠΎΠΌ расстоянии ΠΎΡ‚ Π—Π΅ΠΌΠ»ΠΈ ΠΎΠ±Π»Π°Π΄Π°Π΅Ρ‚ ΠΏΠΎΡ‚Π΅Π½Ρ†ΠΈΠ°Π»ΡŒΠ½ΠΎΠΉ энСргиСй.

Для Ρ‚ΠΎΠ³ΠΎ Ρ‡Ρ‚ΠΎΠ±Ρ‹ ΠΏΠΎΠ΄Π½ΡΡ‚ΡŒ Ρ‚Π΅Π»ΠΎ Π²Ρ‹ΡˆΠ΅ придСтся Π²Ρ‹ΠΏΠΎΠ»Π½ΠΈΡ‚ΡŒ Π½Π΅ΠΊΠΎΡ‚ΠΎΡ€ΡƒΡŽ Ρ€Π°Π±ΠΎΡ‚Ρƒ. Π’Π΅Π»ΠΈΡ‡ΠΈΠ½Π° этой Ρ€Π°Π±ΠΎΡ‚Ρ‹ Π±ΡƒΠ΄Π΅Ρ‚ Ρ€Π°Π²Π½Π° разности ΠΏΠΎΡ‚Π΅Π½Ρ†ΠΈΠ°Π»ΡŒΠ½Ρ‹Ρ… энСргий, ΠΊΠΎΡ‚ΠΎΡ€Ρ‹ΠΌΠΈ ΠΎΠ±Π»Π°Π΄Π°Π΅Ρ‚ Ρ‚Π΅Π»ΠΎ Π½Π° Ρ€Π°Π·Π½ΠΎΠΉ высотС.

ΠŸΠΎΡ…ΠΎΠΆΡƒΡŽ ΠΊΠ°Ρ€Ρ‚ΠΈΠ½Ρƒ ΠΌΡ‹ наблюдаСм, ΠΊΠΎΠ³Π΄Π° ΠΌΡ‹ ΠΈΠΌΠ΅Π΅ΠΌ Π΄Π΅Π»ΠΎ с элСктричСским ΠΏΠΎΠ»Π΅ΠΌ.

Π§Ρ‚ΠΎ касаСтся Ρ‚ΠΎΡ‡ΠΊΠΈ пространства, Π² ΠΊΠΎΡ‚ΠΎΡ€ΠΎΠΉ элСктричСский заряд ΠΎΠ±Π»Π°Π΄Π°Π΅Ρ‚ Π½ΡƒΠ»Π΅Π²Ρ‹ΠΌ элСктричСским ΠΏΠΎΡ‚Π΅Π½Ρ†ΠΈΠ°Π»ΠΎΠΌ, Ρ‚ΠΎ Π² Ρ‚Π΅ΠΎΡ€ΠΈΠΈ элСктричСства эту Ρ‚ΠΎΡ‡ΠΊΡƒ ΠΌΠΎΠΆΠ½ΠΎ Π²Ρ‹Π±Ρ€Π°Ρ‚ΡŒ ΠΏΡ€ΠΎΠΈΠ·Π²ΠΎΠ»ΡŒΠ½ΠΎ. Бвязанно это с Ρ‚Π΅ΠΌ, Ρ‡Ρ‚ΠΎ элСктричСскоС ΠΏΠΎΠ»Π΅ Β«ΠΏΠΎΡ‚Π΅Π½Ρ†ΠΈΠ°Π»ΡŒΠ½ΠΎΒ».

Π§Ρ‚ΠΎΠ±Ρ‹ ΠΏΡ€ΠΎΡΡΠ½ΠΈΡ‚ΡŒ этот Ρ‚Π΅Ρ€ΠΌΠΈΠ½ придСтся ΠΏΡ€ΠΈΠ±Π΅Π³Π½ΡƒΡ‚ΡŒ ΠΊ Π²Ρ‹ΡΡˆΠ΅ΠΉ ΠΌΠ°Ρ‚Π΅ΠΌΠ°Ρ‚ΠΈΠΊΠ΅, Π° ΠΌΡ‹ Ρ€Π΅ΡˆΠΈΠ»ΠΈ этого ΠΈΠ·Π±Π΅ΠΆΠ°Ρ‚ΡŒ. На ΠΏΡ€Π°ΠΊΡ‚ΠΈΠΊΠ΅ спСциалисты Π² области элСктротСхники Π² качСствС Ρ‚ΠΎΡ‡Π΅ΠΊ с Π½ΡƒΠ»Π΅Π²Ρ‹ΠΌ ΠΏΠΎΡ‚Π΅Π½Ρ†ΠΈΠ°Π»ΠΎΠΌ часто Π²Ρ‹Π±ΠΈΡ€Π°ΡŽΡ‚ ΠΏΠΎΠ²Π΅Ρ€Ρ…Π½ΠΎΡΡ‚ΡŒ Π—Π΅ΠΌΠ»ΠΈ.

И ΠΌΠ½ΠΎΠ³ΠΈΠ΅ измСрСния Π²Ρ‹ΠΏΠΎΠ»Π½ΡΡŽΡ‚ ΠΎΡ‚Π½ΠΎΡΠΈΡ‚Π΅Π»ΡŒΠ½ΠΎ Π½Π΅Π΅.

ЭлСктричСскиС поля ΠΌΠΎΠ³ΡƒΡ‚ Π±Ρ‹Ρ‚ΡŒ постоянными (Π½Π΅ΠΈΠ·ΠΌΠ΅Π½Π½Ρ‹ΠΌΠΈ Π²ΠΎ Π²Ρ€Π΅ΠΌΠ΅Π½ΠΈ) ΠΈ ΠΏΠ΅Ρ€Π΅ΠΌΠ΅Π½Π½Ρ‹ΠΌΠΈ. ΠŸΠ΅Ρ€Π΅ΠΌΠ΅Π½Π½Ρ‹Π΅ элСктричСскиС поля ΠΌΠΎΠ³ΡƒΡ‚ ΠΈΠ·ΠΌΠ΅Π½ΡΡ‚ΡŒΡΡ ΠΏΠΎ Ρ€Π°Π·Π»ΠΈΡ‡Π½Ρ‹ΠΌ матСматичСским Π·Π°ΠΊΠΎΠ½Π°ΠΌ.

Π’ Ρ‚Π΅Ρ…Π½ΠΈΠΊΠ΅ Ρ‡Π°Ρ‰Π΅ всСго ΠΈΡΠΏΠΎΠ»ΡŒΠ·ΡƒΡŽΡ‚ΡΡ ΠΏΠ΅Ρ€Π΅ΠΌΠ΅Π½Π½Ρ‹Π΅ элСктричСскиС поля, ΠΊΠΎΡ‚ΠΎΡ€Ρ‹Π΅ ΠΈΠ·ΠΌΠ΅Π½ΡΡŽΡ‚ΡΡ ΠΏΠΎ Π·Π°ΠΊΠΎΠ½Ρƒ синуса.

Π’ случаС ΠΏΠ΅Ρ€Π΅ΠΌΠ΅Π½Π½ΠΎΠ³ΠΎ элСктричСского поля ΠΌΠ³Π½ΠΎΠ²Π΅Π½Π½ΠΎΠ΅ Π·Π½Π°Ρ‡Π΅Π½ΠΈΠ΅ разности ΠΏΠΎΡ‚Π΅Π½Ρ†ΠΈΠ°Π»ΠΎΠ² ΠΌΠ΅ΠΆΠ΄Ρƒ двумя Ρ‚ΠΎΡ‡ΠΊΠ°ΠΌΠΈ ΠΌΠΎΠΆΠ½ΠΎ Π²Ρ‹Ρ‡ΠΈΡΠ»ΠΈΡ‚ΡŒ ΠΏΠΎ ΡΠ»Π΅Π΄ΡƒΡŽΡ‰Π΅ΠΉ Ρ„ΠΎΡ€ΠΌΡƒΠ»Π΅:

u(t)=U_m Β sin⁑〖(Ο‰t)γ€— (2).

Π—Π΄Π΅ΡΡŒ u – ΠΌΠ³Π½ΠΎΠ²Π΅Π½Π½ΠΎΠ΅ Π·Π½Π°Ρ‡Π΅Π½ΠΈΠ΅ напряТСния; Um – максимальноС Π·Π½Π°Ρ‡Π΅Π½ΠΈΠ΅ напряТСния; ω – частота, t – врСмя.

Π˜Π·ΠΌΠ΅Ρ€Π΅Π½ΠΈΠ΅ элСктричСского напряТСния

ЭлСктричСскоС напряТСниС ΠΈΠ·ΠΌΠ΅Ρ€ΡΡŽΡ‚ с ΠΏΠΎΠΌΠΎΡ‰ΡŒΡŽ Π²ΠΎΠ»ΡŒΡ‚ΠΌΠ΅Ρ‚Ρ€ΠΎΠ². Для измСрСния напряТСния (разности ΠΏΠΎΡ‚Π΅Π½Ρ†ΠΈΠ°Π»ΠΎΠ²) Π½Π° участкС элСктричСской Ρ†Π΅ΠΏΠΈ Ρ‰ΡƒΠΏΡ‹ Π²ΠΎΠ»ΡŒΡ‚ΠΌΠ΅Ρ‚Ρ€Π° ΠΏΠΎΠ΄ΠΊΠ»ΡŽΡ‡Π°ΡŽΡ‚ ΠΊ ΠΊΠΎΠ½Ρ†Π°ΠΌ этого участка ΠΈ ΠΏΠΎ шкалС ΡΡ‡ΠΈΡ‚Ρ‹Π²Π°ΡŽΡ‚ показания ΠΏΡ€ΠΈΠ±ΠΎΡ€Π°.

БущСствуСт мноТСство Ρ‚ΠΈΠΏΠΎΠ² Π²ΠΎΠ»ΡŒΡ‚ΠΌΠ΅Ρ‚Ρ€ΠΎΠ². ΠœΡ‹ остановимся Π½Π° Π°Π½Π°Π»ΠΎΠ³ΠΎΠ²Ρ‹Ρ… Π²ΠΎΠ»ΡŒΡ‚ΠΌΠ΅Ρ‚Ρ€Π°Ρ… с магнитоэлСктричСскими ΠΈΠ·ΠΌΠ΅Ρ€ΠΈΡ‚Π΅Π»ΡŒΠ½Ρ‹ΠΌΠΈ ΠΌΠ΅Ρ…Π°Π½ΠΈΠ·ΠΌΠ°ΠΌΠΈ. Π­Ρ‚ΠΈ ΠΌΠ΅Ρ…Π°Π½ΠΈΠ·ΠΌΡ‹ довольно часто ΠΏΡ€ΠΈΠΌΠ΅Π½ΡΡŽΡ‚ Π² Ρ‰ΠΈΡ‚ΠΎΠ²Ρ‹Ρ… Π²ΠΎΠ»ΡŒΡ‚ΠΌΠ΅Ρ‚Ρ€Π°Ρ… ΠΈ ΠΌΠ½ΠΎΠ³ΠΎΡ„ΡƒΠ½ΠΊΡ†ΠΈΠΎΠ½Π°Π»ΡŒΠ½Ρ‹Ρ… ΠΈΠ·ΠΌΠ΅Ρ€ΠΈΡ‚Π΅Π»ΡŒΠ½Ρ‹Ρ… ΠΏΡ€ΠΈΠ±ΠΎΡ€Π°Ρ… – ΠΌΡƒΠ»ΡŒΡ‚ΠΈΠΌΠ΅Ρ‚Ρ€Π°Ρ….

ΠœΠ°Π³Π½ΠΈΡ‚ΠΎΡΠ»Π΅ΠΊΡ‚Ρ€ΠΈΡ‡Π΅ΡΠΊΠΈΠΉ элСктричСский ΠΌΠ΅Ρ…Π°Π½ΠΈΠ·ΠΌ прСдставляСт собой ΠΏΡ€ΠΎΠ²ΠΎΠ»ΠΎΡ‡Π½ΡƒΡŽ ΠΊΠ°Ρ‚ΡƒΡˆΠΊΡƒ, Ρ€Π°Π·ΠΌΠ΅Ρ‰Π΅Π½Π½ΡƒΡŽ ΠΌΠ΅ΠΆΠ΄Ρƒ полюсами ΠΌΠ°Π³Π½ΠΈΡ‚Π°. ΠšΠ°Ρ‚ΡƒΡˆΠΊΠ° ΠΏΠΎΠ΄Π²Π΅ΡˆΠΈΠ²Π°Π΅Ρ‚ΡΡ Π½Π° ΡΠΏΠΈΡ€Π°Π»ΡŒΠ½Ρ‹Ρ… ΠΏΡ€ΡƒΠΆΠΈΠ½Π°Ρ… ΠΎΠ±Π΅ΡΠΏΠ΅Ρ‡ΠΈΠ²Π°ΡŽΡ‰ΠΈΡ… Π²Ρ‹ΡΠΎΠΊΡƒΡŽ Ρ‡ΡƒΠ²ΡΡ‚Π²ΠΈΡ‚Π΅Π»ΡŒΠ½ΠΎΡΡ‚ΡŒ ΠΏΡ€ΠΈΠ±ΠΎΡ€Π°. Π‘ ΠΊΠ°Ρ‚ΡƒΡˆΠΊΠΎΠΉ связана ΡƒΠΊΠ°Π·Π°Ρ‚Π΅Π»ΡŒΠ½Π°Ρ стрСлка, с ΠΏΠΎΠΌΠΎΡ‰ΡŒΡŽ ΠΊΠΎΡ‚ΠΎΡ€ΠΎΠΉ осущСствляСтся отсчСт ΠΏΠΎΠΊΠ°Π·Π°Π½ΠΈΠΉ Π½Π° шкалС ΠΏΡ€ΠΈΠ±ΠΎΡ€Π°.

НиТС Π½Π° рисункС ΠΏΠΎΠΊΠ°Π·Π°Π½ΠΎ устройство магнитоэлСктричСского ΠΌΠ΅Ρ…Π°Π½ΠΈΠ·ΠΌΠ°.

ΠœΠ°Π³Π½ΠΈΡ‚ΠΎΡΠ»Π΅ΠΊΡ‚Ρ€ΠΈΡ‡Π΅ΡΠΊΠΈΠ΅ ΠΈΠ·ΠΌΠ΅Ρ€ΠΈΡ‚Π΅Π»ΡŒΠ½Ρ‹Π΅ ΠΌΠ΅Ρ…Π°Π½ΠΈΠ·ΠΌΡ‹ ΠΈΠΌΠ΅ΡŽΡ‚ Π²Ρ‹ΡΠΎΠΊΡƒΡŽ Ρ‡ΡƒΠ²ΡΡ‚Π²ΠΈΡ‚Π΅Π»ΡŒΠ½ΠΎΡΡ‚ΡŒ. Π‘ ΠΈΡ… ΠΏΠΎΠΌΠΎΡ‰ΡŒΡŽ ΠΌΠΎΠΆΠ½ΠΎ ΠΈΠ·ΠΌΠ΅Ρ€ΠΈΡ‚ΡŒ напряТСния ΡΠΎΡΡ‚Π°Π²Π»ΡΡŽΡ‰ΠΈΠ΅ сотыС Π΄ΠΎΠ»ΠΈ Π²ΠΎΠ»ΡŒΡ‚Π°. Для Ρ€Π°ΡΡˆΠΈΡ€Π΅Π½ΠΈΡ ΠΏΡ€Π΅Π΄Π΅Π»ΠΎΠ² измСрСния ΠΏΠΎΡΠ»Π΅Π΄ΠΎΠ²Π°Ρ‚Π΅Π»ΡŒΠ½ΠΎ с ΠΈΠ·ΠΌΠ΅Ρ€ΠΈΡ‚Π΅Π»ΡŒΠ½Ρ‹ΠΌ ΠΌΠ΅Ρ…Π°Π½ΠΈΠ·ΠΌΠΎΠΌ Π²ΠΊΠ»ΡŽΡ‡Π°ΡŽΡ‚ Π΄ΠΎΠ±Π°Π²ΠΎΡ‡Π½Ρ‹Π΅ сопротивлСния. Π‘Ρ…Π΅ΠΌΠ° ΠΏΡ€ΠΎΡΡ‚Π΅ΠΉΡˆΠ΅Π³ΠΎ Π²ΠΎΠ»ΡŒΡ‚ΠΌΠ΅Ρ‚Ρ€Π° постоянного Ρ‚ΠΎΠΊΠ° ΠΏΠΎΠΊΠ°Π·Π°Π½Π° Π½Π° рисункС.

Одним ΠΈΠ· Π²Π°ΠΆΠ½Π΅ΠΉΡˆΠΈΡ… ΠΏΠ°Ρ€Π°ΠΌΠ΅Ρ‚Ρ€ΠΎΠΌ Π²ΠΎΠ»ΡŒΡ‚ΠΌΠ΅Ρ‚Ρ€Π° являСтся Π΅Π³ΠΎ Π²Π½ΡƒΡ‚Ρ€Π΅Π½Π½Π΅Π΅ сопротивлСниС. Π§Π΅ΠΌ большС Π·Π½Π°Ρ‡Π΅Π½ΠΈΠ΅ Π²Π½ΡƒΡ‚Ρ€Π΅Π½Π½Π΅Π³ΠΎ сопротивлСния Π²ΠΎΠ»ΡŒΡ‚ΠΌΠ΅Ρ‚Ρ€Π°, Ρ‚Π΅ΠΌ ΠΌΠ΅Π½ΡŒΡˆΡƒΡŽ ΠΏΠΎΠ³Ρ€Π΅ΡˆΠ½ΠΎΡΡ‚ΡŒ ΠΌΠΎΠΆΠ½ΠΎ ΠΏΠΎΠ»ΡƒΡ‡ΠΈΡ‚ΡŒ Π² процСссС измСрСния.

Для Π°Π½Π°Π»ΠΎΠ³ΠΎΠ²Ρ‹Ρ… Π²ΠΎΠ»ΡŒΡ‚ΠΌΠ΅Ρ‚Ρ€ΠΎΠ² Π²Π½ΡƒΡ‚Ρ€Π΅Π½Π½Π΅Π΅ сопротивлСниС ΠΎΠ±Ρ‹Ρ‡Π½ΠΎ составляСт 20кОм Π½Π° Π²ΠΎΠ»ΡŒΡ‚.

Если Π½Π΅ΠΎΠ±Ρ…ΠΎΠ΄ΠΈΠΌΠΎ ΠΏΠΎΠ»ΡƒΡ‡ΠΈΡ‚ΡŒ большСС Π·Π½Π°Ρ‡Π΅Π½ΠΈΠ΅ сопротивлСния для ΠΈΠ·ΠΌΠ΅Ρ€Π΅Π½ΠΈΠΉ ΠΏΡ€ΠΈΠΌΠ΅Π½ΡΡŽΡ‚ элСктронныС Π²ΠΎΠ»ΡŒΡ‚ΠΌΠ΅Ρ‚Ρ€Ρ‹, Ρ†ΠΈΡ„Ρ€ΠΎΠ²Ρ‹Π΅ ΠΈΠ»ΠΈ Π°Π½Π°Π»ΠΎΠ³ΠΎΠ²Ρ‹Π΅.

Для измСрСния ΠΏΠ΅Ρ€Π΅ΠΌΠ΅Π½Π½ΠΎΠ³ΠΎ напряТСния Π² ΠΊΠΎΠ½ΡΡ‚Ρ€ΡƒΠΊΡ†ΠΈΡŽ Π²ΠΎΠ»ΡŒΡ‚ΠΌΠ΅Ρ‚Ρ€ΠΎΠ² Π²ΠΊΠ»ΡŽΡ‡Π°ΡŽΡ‚ выпрямитСли, ΠΊΠΎΡ‚ΠΎΡ€Ρ‹Π΅ ΠΏΡ€Π΅ΠΎΠ±Ρ€Π°Π·ΡƒΡŽΡ‚ ΠΏΠ΅Ρ€Π΅ΠΌΠ΅Π½Π½ΠΎΠ΅ напряТСниС Π² постоянноС. Π¨ΠΊΠ°Π»Ρ‹ Π²ΠΎΠ»ΡŒΡ‚ΠΌΠ΅Ρ‚Ρ€ΠΎΠ² для измСрСния ΠΏΠ΅Ρ€Π΅ΠΌΠ΅Π½Π½ΠΎΠ³ΠΎ напряТСния ΠΎΠ±Ρ‹Ρ‡Π½ΠΎ Π³Ρ€Π°Π΄ΡƒΠΈΡ€ΡƒΡŽΡ‚ Π² Π΄Π΅ΠΉΡΡ‚Π²ΡƒΡŽΡ‰ΠΈΡ… (эффСктивных) значСниях напряТСния. Π”Π΅ΠΉΡΡ‚Π²ΡƒΡŽΡ‰Π΅Π΅ Π·Π½Π°Ρ‡Π΅Π½ΠΈΠ΅ ΠΏΠ΅Ρ€Π΅ΠΌΠ΅Π½Π½ΠΎΠ³ΠΎ Ρ‚ΠΎΠΊΠ° связано с ΠΌΠ°ΠΊΡΠΈΠΌΠ°Π»ΡŒΠ½Ρ‹ΠΌ ΡΠ»Π΅Π΄ΡƒΡŽΡ‰ΠΈΠΌ ΡΠΎΠΎΡ‚Π½ΠΎΡˆΠ΅Π½ΠΈΠ΅ΠΌ.

U=1/√2 U_m=0,707U_m (3)

Π”Π΅ΠΉΡΡ‚Π²ΡƒΡŽΡ‰Π΅Π΅ Π·Π½Π°Ρ‡Π΅Π½ΠΈΠ΅ ΡƒΠ΄ΠΎΠ±Π½ΠΎ ΠΏΡ€ΠΈΠΌΠ΅Π½ΡΡ‚ΡŒ ΠΏΡ€ΠΈ вычислСнии мощности элСктричСской Ρ†Π΅ΠΏΠΈ. Когда ΠΌΡ‹ Π³ΠΎΠ²ΠΎΡ€ΠΈΠΌ, Ρ‡Ρ‚ΠΎ Π² элСктричСской Ρ€ΠΎΠ·Π΅Ρ‚ΠΊΠ΅ присутствуСт напряТСниС 220Π’, Ρ€Π΅Ρ‡ΡŒ ΠΈΠ΄Π΅Ρ‚ ΠΈΠΌΠ΅Π½Π½ΠΎ ΠΎ Π΄Π΅ΠΉΡΡ‚Π²ΡƒΡŽΡ‰Π΅ΠΌ Π·Π½Π°Ρ‡Π΅Π½ΠΈΠΈ напряТСния.

Π’ ΠΊΠΎΡ€ΠΎΡ‚ΠΊΠΎΠΌ ΠΌΠ°Ρ‚Π΅Ρ€ΠΈΠ°Π»Π΅ Ρ‚Ρ€ΡƒΠ΄Π½ΠΎ Ρ€Π°ΡΡΠΊΠ°Π·Π°Ρ‚ΡŒ ΠΎΠ±ΠΎ всСх Π½ΡŽΠ°Π½ΡΠ°Ρ… связанных с элСктричСским напряТСниСм ΠΈ способах Π΅Π³ΠΎ измСрСния. Но ΠΌΡ‹ надССмся, Ρ‡Ρ‚ΠΎ тСкст окаТСтся ΠΏΠΎΠ»Π΅Π·Π΅Π½ Ρ‡ΠΈΡ‚Π°Ρ‚Π΅Π»ΡŽ.

Π˜ΡΡ‚ΠΎΡ‡Π½ΠΈΠΊ: https://Elektrika.ru/articles/elektroprovodka/elektricheskoe_napryazhenie/

ЭлСктричСскоС напряТСниС Ρ†Π΅ΠΏΠΈ

ΠŸΡ€ΠΈ описании ΠΏΡ€ΠΎΡ‚Π΅ΠΊΠ°ΡŽΡ‰ΠΈΡ… Π² элСктроцСпи процСссов Π² элСктротСхникС ΠΏΡ€ΠΈΠΌΠ΅Π½ΡΡŽΡ‚ Ρ‚Π°ΠΊΠΈΠ΅ понятия, ΠΊΠ°ΠΊ сопротивлСниС, напряТСниС ΠΈ Ρ‚ΠΎΠΊ. ΠšΠ°ΠΆΠ΄ΠΎΠΌΡƒ ΠΈΠ· этих понятий свойствСнны свои спСцифичСскиС характСристики, ΠΈ ΠΎΠ½ΠΈ ΠΈΠΌΠ΅ΡŽΡ‚ ΡΠΎΠΎΡ‚Π²Π΅Ρ‚ΡΡ‚Π²ΡƒΡŽΡ‰Π΅Π΅ Π½Π°Π·Π½Π°Ρ‡Π΅Π½ΠΈΠ΅.

ΠžΠ±ΡΠ·Π°Ρ‚Π΅Π»ΡŒΠ½Ρ‹ΠΌ для протСкания зарядов Ρ‚Ρ€Π΅Π±ΠΎΠ²Π°Π½ΠΈΠ΅ΠΌ считаСтся Π½Π°Π»ΠΈΡ‡ΠΈΠ΅ Ρ†Π΅ΠΏΠΈ (Π·Π°ΠΌΠΊΠ½ΡƒΡ‚ΠΎΠ³ΠΎ ΠΊΠΎΠ½Ρ‚ΡƒΡ€Π°, ΠΎΠ±Π΅ΡΠΏΠ΅Ρ‡ΠΈΠ²Π°ΡŽΡ‰Π΅Π³ΠΎ всС Π½Π΅ΠΎΠ±Ρ…ΠΎΠ΄ΠΈΠΌΡ‹Π΅ условия для ΠΈΡ… пСрСдвиТСния). ΠŸΡ€ΠΈ Ρ„ΠΎΡ€ΠΌΠΈΡ€ΠΎΠ²Π°Π½ΠΈΠΈ Ρ€Π°Π·Ρ€Ρ‹Π²Π° Π²Π½ΡƒΡ‚Ρ€ΠΈ двиТущихся частиц ΠΈΡ… Π½Π°ΠΏΡ€Π°Π²Π»Π΅Π½Π½ΠΎΠ΅ ΠΏΠ΅Ρ€Π΅ΠΌΠ΅Ρ‰Π΅Π½ΠΈΠ΅ Ρ€Π΅Π·ΠΊΠΎ прСкращаСтся.

По Ρ‚Π°ΠΊΠΎΠΌΡƒ ΠΏΡ€ΠΈΠ½Ρ†ΠΈΠΏΡƒ Ρ€Π°Π±ΠΎΡ‚Π°ΡŽΡ‚ всС Ρ‚ΠΈΠΏΡ‹ Π²Ρ‹ΠΊΠ»ΡŽΡ‡Π°Ρ‚Π΅Π»Π΅ΠΉ ΠΈ ΠΈΡΠΏΠΎΠ»ΡŒΠ·ΡƒΠ΅ΠΌΡ‹Π΅ Π² элСктрикС Π·Π°Ρ‰ΠΈΡ‚Ρ‹. Они ΠΎΡΡƒΡ‰Π΅ΡΡ‚Π²Π»ΡΡŽΡ‚ Ρ€Π°Π·Π΄Π΅Π»Π΅Π½ΠΈΠ΅ ΠΌΠ΅ΠΆΠ΄Ρƒ собой Π·Π° счСт ΠΏΠΎΠ΄Π²ΠΈΠΆΠ½Ρ‹Ρ… ΠΊΠΎΠ½Ρ‚Π°ΠΊΡ‚ΠΎΠ² токопроводящих частСй. Π­Ρ‚ΠΎ дСйствиС ΠΈ способствуСт ΠΏΡ€Π΅Ρ€Ρ‹Π²Π°Π½ΠΈΡŽ процСсса протСкания элСктричСского Ρ‚ΠΎΠΊΠ° послС ΠΎΡ‚ΠΊΠ»ΡŽΡ‡Π΅Π½ΠΈΡ элСктроприбора.

ΠŸΠΎΠ½ΡΡ‚ΠΈΠ΅ элСктричСского напряТСния Π² Ρ„ΠΈΠ·ΠΈΠΊΠ΅

ЭлСктричСским Ρ‚ΠΎΠΊΠΎΠΌ Π² Ρ„ΠΈΠ·ΠΈΠΊΠ΅ считаСтся Π½Π°ΠΏΡ€Π°Π²Π»Π΅Π½Π½ΠΎΠ΅ ΠΏΠ΅Ρ€Π΅ΠΌΠ΅Ρ‰Π΅Π½ΠΈΠ΅ заряТСнных частиц, создаваСмоС элСктрополСм, ΡΠΎΠ²Π΅Ρ€ΡˆΠ°ΡŽΡ‰ΠΈΠΌ ΠΏΡ€ΠΈ этом ΠΎΠΏΡ€Π΅Π΄Π΅Π»Π΅Π½Π½ΡƒΡŽ Ρ€Π°Π±ΠΎΡ‚Ρƒ.

ΠžΠΏΡ€Π΅Π΄Π΅Π»Π΅Π½ΠΈΠ΅ 1

Π Π°Π±ΠΎΡ‚Π° ΡΠΎΠ·Π΄Π°ΡŽΡ‰Π΅Π³ΠΎ Ρ‚ΠΎΠΊ элСктрополя называСтся Ρ€Π°Π±ΠΎΡ‚ΠΎΠΉ Ρ‚ΠΎΠΊΠ° ($A$). Вакая Ρ€Π°Π±ΠΎΡ‚Π° ΠΌΠΎΠΆΠ΅Ρ‚ Π½Π° Ρ€Π°Π·Π½Ρ‹Ρ… участках Ρ†Π΅ΠΏΠΈ ΠΎΡ‚Π»ΠΈΡ‡Π°Ρ‚ΡŒΡΡ, ΠΎΠ΄Π½Π°ΠΊΠΎ ΠΏΡ€ΠΈ этом ΠΎΠ½Π° Π±ΡƒΠ΄Π΅Ρ‚ ΠΏΡ€ΠΎΠΏΠΎΡ€Ρ†ΠΈΠΎΠ½Π°Π»ΡŒΠ½ΠΎΠΉ проходящСму Ρ‡Π΅Ρ€Π΅Π· Π½Π΅Π³ΠΎ заряду.

НичСго нСпонятно?

ΠŸΠΎΠΏΡ€ΠΎΠ±ΡƒΠΉ ΠΎΠ±Ρ€Π°Ρ‚ΠΈΡ‚ΡŒΡΡ Π·Π° ΠΏΠΎΠΌΠΎΡ‰ΡŒΡŽ ΠΊ прСподаватСлям

  • ЀизичСской Π²Π΅Π»ΠΈΡ‡ΠΈΠ½ΠΎΠΉ Ρ€Π°Π±ΠΎΡ‚Ρ‹ Ρ‚ΠΎΠΊΠ° Π½Π° ΠΊΠΎΠ½ΠΊΡ€Π΅Ρ‚Π½ΠΎΠΌ участкС ΠΏΡ€ΠΈ ΠΏΠ΅Ρ€Π΅ΠΌΠ΅Ρ‰Π΅Π½ΠΈΠΈ ΠΏΠΎ Π½Π΅ΠΌΡƒ заряда 1 Кл считаСтся элСктричСскоС напряТСниС ($U$).
  • Для опрСдСлСния напряТСния Π½Π° ΠΎΡ‚Π΄Π΅Π»ΡŒΠ½ΠΎ взятом участкС сущСствуСт ΡΠ»Π΅Π΄ΡƒΡŽΡ‰Π°Ρ Ρ„ΠΎΡ€ΠΌΡƒΠ»Π°:
  • $U =frac{A}{q}$, Π³Π΄Π΅:
  • $A$ β€” Ρ€Π°Π±ΠΎΡ‚Π° Ρ‚ΠΎΠΊΠ°,
  • $q$ β€” ΠΏΡ€ΠΎΡˆΠ΅Π΄ΡˆΠΈΠΉ ΠΏΠΎ участку заряд.

Π’ΠΎΠ·Π½ΠΈΠΊΠ½ΠΎΠ²Π΅Π½ΠΈΠ΅ Ρ‚ΠΎΠΊΠ° Π² элСктричСской Ρ†Π΅ΠΏΠΈ

Π—Π°ΠΌΠ΅Ρ‡Π°Π½ΠΈΠ΅ 1

Π­Π»Π΅ΠΊΡ‚Ρ€ΠΈΡ‡Π΅ΡΠΊΡƒΡŽ Ρ†Π΅ΠΏΡŒ Ρ…Π°Ρ€Π°ΠΊΡ‚Π΅Ρ€ΠΈΠ·ΡƒΠ΅Ρ‚ комплСкс устройств, ΠΎΠ±Π΅ΡΠΏΠ΅Ρ‡ΠΈΠ²Π°ΡŽΡ‰ΠΈΡ… ΠΏΡƒΡ‚ΡŒ для ΠΏΡ€ΠΎΡ‚Π΅ΠΊΠ°ΡŽΡ‰Π΅Π³ΠΎ элСктричСского Ρ‚ΠΎΠΊΠ° ΠΈ соСдинСнных ΠΎΠΏΡ€Π΅Π΄Π΅Π»Π΅Π½Π½Ρ‹ΠΌ ΠΎΠ±Ρ€Π°Π·ΠΎΠΌ. Π’ качСствС элСмСнтов элСктроцСпи слуТат: Π½Π°Π³Ρ€ΡƒΠ·ΠΊΠ°, ΠΏΡ€ΠΎΠ²ΠΎΠ΄Π½ΠΈΠΊΠΈ ΠΈ источник Ρ‚ΠΎΠΊΠ°. Π’ составС элСктричСской Ρ†Π΅ΠΏΠΈ ΠΌΠΎΠ³ΡƒΡ‚ Π±Ρ‹Ρ‚ΡŒ ΠΈ Π΄Ρ€ΡƒΠ³ΠΈΠ΅ элСмСнты, ΠΊΠ°ΠΊ, Π½Π°ΠΏΡ€ΠΈΠΌΠ΅Ρ€, устройства Π·Π°Ρ‰ΠΈΡ‚Ρ‹ ΠΈ ΠΊΠΎΠΌΠΌΡƒΡ‚Π°Ρ†ΠΈΠΈ.

НСобходимым условиСм возникновСния Ρ‚ΠΎΠΊΠ° Π±ΡƒΠ΄Π΅Ρ‚ соСдинСниС Π΄Π²ΡƒΡ… Ρ‚ΠΎΡ‡Π΅ΠΊ, Ρƒ ΠΎΠ΄Π½ΠΎΠΉ ΠΈΠ· ΠΊΠΎΡ‚ΠΎΡ€Ρ‹Ρ… ΠΎΡ‡Π΅Π½ΡŒ ΠΌΠ½ΠΎΠ³ΠΎ элСктронов Π² ΠΎΡ‚Π»ΠΈΡ‡ΠΈΠ΅ ΠΎΡ‚ Π΄Ρ€ΡƒΠ³ΠΎΠΉ. Π˜Π½Ρ‹ΠΌΠΈ словами, потрСбуСтся ΠΎΠ±Ρ€Π°Π·ΠΎΠ²Π°Π½ΠΈΠ΅ разности ΠΏΠΎΡ‚Π΅Π½Ρ†ΠΈΠ°Π»ΠΎΠ² ΠΌΠ΅ΠΆΠ΄Ρƒ ΡƒΠΊΠ°Π·Π°Π½Π½Ρ‹ΠΌΠΈ Ρ‚ΠΎΡ‡ΠΊΠ°ΠΌΠΈ. Π‘ этой Ρ†Π΅Π»ΡŒΡŽ Π² Ρ†Π΅ΠΏΠΈ ΠΈΡΠΏΠΎΠ»ΡŒΠ·ΡƒΠ΅Ρ‚ΡΡ источник Ρ‚ΠΎΠΊΠ°. Π’Π°ΠΊΠΈΠΌ источником ΠΌΠΎΠ³ΡƒΡ‚ ΡΠ»ΡƒΠΆΠΈΡ‚ΡŒ устройства Π² Π²ΠΈΠ΄Π΅ Π³Π΅Π½Π΅Ρ€Π°Ρ‚ΠΎΡ€ΠΎΠ², Π±Π°Ρ‚Π°Ρ€Π΅ΠΉ, химичСских элСмСнтов ΠΈ Π΄Ρ€.

Π’ качСствС Π½Π°Π³Ρ€ΡƒΠ·ΠΊΠΈ Π² элСктроцСпи выступаСт Π°Π±ΡΠΎΠ»ΡŽΡ‚Π½ΠΎ любой ΠΏΠΎΡ‚Ρ€Π΅Π±ΠΈΡ‚Π΅Π»ΡŒ элСктроэнСргии. Нагрузка способна ΠΎΠΊΠ°Π·Ρ‹Π²Π°Ρ‚ΡŒ сопротивлСниС элСктричСскому Ρ‚ΠΎΠΊΡƒ. ΠžΡ‚ Π²Π΅Π»ΠΈΡ‡ΠΈΠ½Ρ‹ Ρ‚Π°ΠΊΠΎΠ³ΠΎ сопротивлСния Π±ΡƒΠ΄Π΅Ρ‚ Π·Π°Π²ΠΈΡΠ΅Ρ‚ΡŒ Π²Π΅Π»ΠΈΡ‡ΠΈΠ½Π° Ρ‚ΠΎΠΊΠ°. Π’ΠΎΠΊ Ρ‚Π΅Ρ‡Π΅Ρ‚ ΠΏΠΎ ΠΏΡ€ΠΎΠ²ΠΎΠ΄Π½ΠΈΠΊΠ°ΠΌ ΠΎΡ‚ источника Ρ‚ΠΎΠΊΠ° ΠΊ Π½Π°Π³Ρ€ΡƒΠ·ΠΊΠ΅. ΠŸΡ€ΠΎΠ²ΠΎΠ΄Π½ΠΈΠΊΠ°ΠΌΠΈ, Π² свою ΠΎΡ‡Π΅Ρ€Π΅Π΄ΡŒ, слуТат ΠΌΠ°Ρ‚Π΅Ρ€ΠΈΠ°Π»Ρ‹, ΠΈΠΌΠ΅ΡŽΡ‰ΠΈΠ΅ наимСньшСС сопротивлСниС, Ρ‚Π°ΠΊΠΈΠ΅, ΠΊΠ°ΠΊ Π·ΠΎΠ»ΠΎΡ‚ΠΎ, сСрСбро, мСдь.

Π’ΠΈΠΏΡ‹ соСдинСния элСмСнтов Π² элСктричСской Ρ†Π΅ΠΏΠΈ

Π’ элСктротСхникС, Π² зависимости ΠΎΡ‚ Ρ‚ΠΈΠΏΠ° соСдинСния элСмСнтов элСктроцСпи, ΡΡƒΡ‰Π΅ΡΡ‚Π²ΡƒΡŽΡ‚ Ρ‚Π°ΠΊΠΈΠ΅ Π²ΠΈΠ΄Ρ‹ элСктричСских Ρ†Π΅ΠΏΠ΅ΠΉ:

  • ΠΏΠΎΡΠ»Π΅Π΄ΠΎΠ²Π°Ρ‚Π΅Π»ΡŒΠ½Π°Ρ;
  • ΠΏΠ°Ρ€Π°Π»Π»Π΅Π»ΡŒΠ½Π°Ρ элСктричСская Ρ†Π΅ΠΏΡŒ;
  • ΠΏΠΎΡΠ»Π΅Π΄ΠΎΠ²Π°Ρ‚Π΅Π»ΡŒΠ½ΠΎ-ΠΏΠ°Ρ€Π°Π»Π»Π΅Π»ΡŒΠ½Π°Ρ.

Π’ элСктричСской Ρ†Π΅ΠΏΠΈ ΠΏΠΎΡΠ»Π΅Π΄ΠΎΠ²Π°Ρ‚Π΅Π»ΡŒΠ½ΠΎΠ³ΠΎ Ρ‚ΠΈΠΏΠ° соСдинСнии всС элСмСнты соСдинСны Π΄Ρ€ΡƒΠ³ с Π΄Ρ€ΡƒΠ³ΠΎΠΌ ΠΏΠΎΡΠ»Π΅Π΄ΠΎΠ²Π°Ρ‚Π΅Π»ΡŒΠ½ΠΎ. Π­Ρ‚ΠΎ ΠΎΠ·Π½Π°Ρ‡Π°Π΅Ρ‚, Ρ‡Ρ‚ΠΎ ΠΊΠΎΠ½Π΅Ρ† ΠΏΠ΅Ρ€Π²ΠΎΠ³ΠΎ элСмСнта соСдиняСтся с Π½Π°Ρ‡Π°Π»ΠΎΠΌ Π²Ρ‚ΠΎΡ€ΠΎΠ³ΠΎ ΠΈ Ρ‚.Π΄.

  1. Для Ρ‚ΠΎΠΊΠ° Ρ‚Π°ΠΊΠΎΠ΅ соСдинСниС элСмСнтов Π΄Π°Π΅Ρ‚ Ρ‚ΠΎΠ»ΡŒΠΊΠΎ ΠΎΠ΄ΠΈΠ½ ΠΏΡƒΡ‚ΡŒ протСкания ΠΎΡ‚ источника ΠΊ Π½Π°Π³Ρ€ΡƒΠ·ΠΊΠ΅. ΠžΠ±Ρ‰ΠΈΠΉ Ρ‚ΠΎΠΊ Ρ†Π΅ΠΏΠΈ ΠΏΡ€ΠΈ этом Π±ΡƒΠ΄Π΅Ρ‚ Ρ€Π°Π²Π΅Π½ Ρ‚ΠΎΠΊΡƒ, ΠΊΠΎΡ‚ΠΎΡ€Ρ‹ΠΉ ΠΏΡ€ΠΎΡ…ΠΎΠ΄ΠΈΡ‚ Ρ‡Π΅Ρ€Π΅Π· ΠΊΠ°ΠΆΠ΄Ρ‹ΠΉ элСмСнт Ρ†Π΅ΠΏΠΈ:
  2. $I_{ΠΎΠ±Ρ‰} = I_1=I_2=I_3$
  3. ΠŸΡ€ΠΈ ΠΏΠ°Π΄Π°ΡŽΡ‰Π΅ΠΌ напряТСнии вдоль всСй Ρ†Π΅ΠΏΠΈ ΠΎΠ½ΠΎ Π±ΡƒΠ΄Π΅Ρ‚ Ρ€Π°Π²Π½ΡΡ‚ΡŒΡΡ ΠΏΡ€ΠΈΠ»ΠΎΠΆΠ΅Π½Π½ΠΎΠΌΡƒ ΠΊ рассматриваСмому участку (AB) Π½Π°ΠΏΡ€ΡΠΆΠ΅Π½ΠΈΡŽ $E$ ΠΈ суммС ΠΏΠ°Π΄Π΅Π½ΠΈΠΉ напряТСний Π½Π° всСх участках элСктроцСпи (рСзисторах). Π­Ρ‚ΠΎ Π²Ρ‹Ρ€Π°ΠΆΠ°Π΅Ρ‚ ΡΠ»Π΅Π΄ΡƒΡŽΡ‰Π°Ρ Ρ„ΠΎΡ€ΠΌΡƒΠ»Π°:
  4. $E=U(A-B)=U_1+U_2+U_3$
  5. Π­Π»Π΅ΠΌΠ΅Π½Ρ‚Ρ‹ Π² ΠΏΠ°Ρ€Π°Π»Π»Π΅Π»ΡŒΠ½ΠΎΠΉ элСктричСской Ρ†Π΅ΠΏΠΈ соСдинСны Ρ‚Π°ΠΊ, Ρ‡Ρ‚ΠΎ Π½Π°Ρ‡Π°Π»ΠΎ ΠΊΠ°ΠΆΠ΄ΠΎΠ³ΠΎ ΠΈΠ· Π½ΠΈΡ… соСдиняСтся Π² ΠΎΠ΄Π½Ρƒ ΠΎΠ±Ρ‰ΡƒΡŽ Ρ‚ΠΎΡ‡ΠΊΡƒ, Π° ΠΊΠΎΠ½Ρ†Ρ‹ ΠΏΡ€ΠΈ этом β€” Π² Π΄Ρ€ΡƒΠ³ΡƒΡŽ.
  6. Для Ρ‚ΠΎΠΊΠ° Π² этом случаС сущСствуСт нСсколько ΠΏΡƒΡ‚Π΅ΠΉ протСкания ΠΊ Π½Π°Π³Ρ€ΡƒΠ·ΠΊΠ°ΠΌ ΠΎΡ‚ источника. ΠŸΡ€ΠΈ этом ΠΎΠ±Ρ‰ΠΈΠΉ Ρ‚ΠΎΠΊ Ρ†Π΅ΠΏΠΈ $I_{ΠΎΠ±Ρ‰}$ ΠΏΠΎΠ»ΡƒΡ‡Π΅Π½ посрСдством Ρ„ΠΎΡ€ΠΌΡƒΠ»Ρ‹:
  7. $I_{ΠΎΠ±Ρ‰}=I_1+I_2+I_3$
  8. ПадСниС напряТСния Π½Π° всСх рСзисторах Π²Ρ‹Ρ€Π°ΠΆΠ°Π΅Ρ‚ ΡΠ»Π΅Π΄ΡƒΡŽΡ‰Π°Ρ Ρ„ΠΎΡ€ΠΌΡƒΠ»Π°: $E=U_1=U_2=U_3$

ΠŸΠΎΡΠ»Π΅Π΄ΠΎΠ²Π°Ρ‚Π΅Π»ΡŒΠ½ΠΎ-ΠΏΠ°Ρ€Π°Π»Π»Π΅Π»ΡŒΠ½Π°Ρ ΡΠ»Π΅ΠΊΡ‚Ρ€ΠΎΡ†Π΅ΠΏΡŒ прСдставляСт ΠΊΠΎΠΌΠ±ΠΈΠ½Π°Ρ†ΠΈΡŽ Ρ†Π΅ΠΏΠΈ ΠΏΠΎΡΠ»Π΅Π΄ΠΎΠ²Π°Ρ‚Π΅Π»ΡŒΠ½ΠΎΠ³ΠΎ ΠΈ ΠΏΠ°Ρ€Π°Π»Π»Π΅Π»ΡŒΠ½ΠΎΠ³ΠΎ Ρ‚ΠΈΠΏΠ° соСдинСния. Π”Ρ€ΡƒΠ³ΠΈΠΌΠΈ словами, Π΅Π΅ элСмСнты ΠΌΠΎΠ³ΡƒΡ‚ Π²ΠΊΠ»ΡŽΡ‡Π°Ρ‚ΡŒΡΡ, ΠΊΠ°ΠΊ ΠΏΠΎΡΠ»Π΅Π΄ΠΎΠ²Π°Ρ‚Π΅Π»ΡŒΠ½Ρ‹ΠΌ, Ρ‚Π°ΠΊ ΠΈ ΠΏΠ°Ρ€Π°Π»Π»Π΅Π»ΡŒΠ½Ρ‹ΠΌ ΠΎΠ±Ρ€Π°Π·ΠΎΠΌ.

ЭлСктричСскоС напряТСниС Π² цСпях постоянного, ΠΏΠ΅Ρ€Π΅ΠΌΠ΅Π½Π½ΠΎΠ³ΠΎ ΠΈ Ρ‚Ρ€Π΅Ρ…Ρ„Π°Π·Π½ΠΎΠ³ΠΎ Ρ‚ΠΎΠΊΠ°

ΠžΠΏΡ€Π΅Π΄Π΅Π»Π΅Π½ΠΈΠ΅ 2

НапряТСниСм Π² Ρ†Π΅ΠΏΠΈ постоянного Ρ‚ΠΎΠΊΠ° Π½Π° участкС ΠΌΠ΅ΠΆΠ΄Ρƒ Ρ‚ΠΎΡ‡ΠΊΠ°ΠΌΠΈ A ΠΈ B считаСтся ΡΠΎΠ²Π΅Ρ€ΡˆΠ°Π΅ΠΌΠ°Ρ элСктричСским ΠΏΠΎΠ»Π΅ΠΌ Ρ€Π°Π±ΠΎΡ‚Π° Π² ΠΌΠΎΠΌΠ΅Π½Ρ‚ пСрСноса ΠΏΡ€ΠΎΠ±Π½ΠΎΠ³ΠΎ ΠΏΠΎΠ»ΠΎΠΆΠΈΡ‚Π΅Π»ΡŒΠ½ΠΎΠ³ΠΎ заряда ΠΈΠ· ΠΏΠ΅Ρ€Π²ΠΎΠΉ Ρ‚ΠΎΡ‡ΠΊΠΈ Π²ΠΎ Π²Ρ‚ΠΎΡ€ΡƒΡŽ.

ΠŸΡ€ΠΈ описании Ρ†Π΅ΠΏΠ΅ΠΉ ΠΏΠ΅Ρ€Π΅ΠΌΠ΅Π½Π½ΠΎΠ³ΠΎ Ρ‚ΠΎΠΊΠ° ΠΈΡΠΏΠΎΠ»ΡŒΠ·ΡƒΡŽΡ‚ Ρ‚Π°ΠΊΠΈΠ΅ Π²ΠΈΠ΄Ρ‹ напряТСний: ΠΌΠ³Π½ΠΎΠ²Π΅Π½Π½ΠΎΠ΅, Π°ΠΌΠΏΠ»ΠΈΡ‚ΡƒΠ΄Π½ΠΎΠ΅, срСднСС, срСднСквадратичноС.

МгновСнноС напряТСниС прСдставляСт Ρ€Π°Π·Π½ΠΎΡΡ‚ΡŒ ΠΏΠΎΡ‚Π΅Π½Ρ†ΠΈΠ°Π»ΠΎΠ² Π΄Π²ΡƒΡ… Ρ‚ΠΎΡ‡Π΅ΠΊ, которая Π±Ρ‹Π»Π° ΠΈΠ·ΠΌΠ΅Ρ€Π΅Π½Π° Π² ΠΊΠΎΠ½ΠΊΡ€Π΅Ρ‚Π½Ρ‹ΠΉ ΠΌΠΎΠΌΠ΅Π½Ρ‚ Π²Ρ€Π΅ΠΌΠ΅Π½ΠΈ. Π”Π°Π½Π½Ρ‹ΠΉ Π²ΠΈΠ΄ напряТСния Π±ΡƒΠ΄Π΅Ρ‚ Π·Π°Π²ΠΈΡΠ΅Ρ‚ΡŒ ΠΎΡ‚ Π²Ρ€Π΅ΠΌΠ΅Π½ΠΈ.

Амплитудным считаСтся максимальноС ΠΏΠΎ ΠΌΠΎΠ΄ΡƒΠ»ΡŽ Π·Π½Π°Ρ‡Π΅Π½ΠΈΠ΅ ΠΌΠ³Π½ΠΎΠ²Π΅Π½Π½ΠΎΠ³ΠΎ напряТСния, взятоС Π·Π° вСсь ΠΏΠ΅Ρ€ΠΈΠΎΠ΄ ΠΊΠΎΠ»Π΅Π±Π°Π½ΠΈΠΉ:

$U_M=max(u(t))$

Π’ цСпях Ρ‚Ρ€Π΅Ρ…Ρ„Π°Π·Π½ΠΎΠ³ΠΎ Ρ‚ΠΎΠΊΠ° сущСствуСт напряТСниС Ρ„Π°Π·Π½ΠΎΠ³ΠΎ ΠΈ Π»ΠΈΠ½Π΅ΠΉΠ½ΠΎΠ³ΠΎ Ρ‚ΠΈΠΏΠ°. Под Ρ„Π°Π·Π½Ρ‹ΠΌ понимаСтся срСднСквадратичноС Π·Π½Π°Ρ‡Π΅Π½ΠΈΠ΅ напряТСния Π½Π° ΠΊΠ°ΠΆΠ΄ΠΎΠΉ ΠΎΡ‚Π΄Π΅Π»ΡŒΠ½ΠΎΠΉ Ρ„Π°Π·Π΅ Π½Π°Π³Ρ€ΡƒΠ·ΠΊΠΈ. Π›ΠΈΠ½Π΅ΠΉΠ½Ρ‹ΠΌ считаСтся напряТСниС ΠΌΠ΅ΠΆΠ΄Ρƒ подводящими Ρ„Π°Π·Π½Ρ‹ΠΌΠΈ ΠΏΡ€ΠΎΠ²ΠΎΠ΄Π°ΠΌΠΈ. Если Π½Π°Π³Ρ€ΡƒΠ·ΠΊΠ° соСдиняСтся Π² Ρ‚Ρ€Π΅ΡƒΠ³ΠΎΠ»ΡŒΠ½ΠΈΠΊ, Ρ„Π°Π·Π½ΠΎΠ΅ ΠΈ Π»ΠΈΠ½Π΅ΠΉΠ½ΠΎΠ΅ напряТСниС Π±ΡƒΠ΄ΡƒΡ‚ Ρ€Π°Π²Π½Ρ‹.

Π˜ΡΡ‚ΠΎΡ‡Π½ΠΈΠΊ: https://spravochnick.ru/fizika/napryazhenie_elektricheskogo_toka/elektricheskoe_napryazhenie_cepi/

НапряТСниС элСктричСского Ρ‚ΠΎΠΊΠ° ΠΈ Π²ΠΎΠ»ΡŒΡ‚ΠΌΠ΅Ρ‚Ρ€

ЭлСктричСский Ρ‚ΠΎΠΊ – это проходящиС Ρ‡Π΅Ρ€Π΅Π· ΠΏΡ€ΠΎΠ²ΠΎΠ΄Π½ΠΈΠΊ элСктроны, нСсущиС ΠΎΡ‚Ρ€ΠΈΡ†Π°Ρ‚Π΅Π»ΡŒΠ½Ρ‹ΠΉ заряд. ОбъСм этого заряда ΠΈΠ»ΠΈ, ΠΈΠ½Ρ‹ΠΌΠΈ словами, количСство элСктричСства Ρ…Π°Ρ€Π°ΠΊΡ‚Π΅Ρ€ΠΈΠ·ΡƒΠ΅Ρ‚ силу Ρ‚ΠΎΠΊΠ°. ΠœΡ‹ Π·Π½Π°Π΅ΠΌ, Ρ‡Ρ‚ΠΎ сила Ρ‚ΠΎΠΊΠ° ΠΎΠ΄ΠΈΠ½Π°ΠΊΠΎΠ²Π° Π²ΠΎ всСх мСстах Ρ†Π΅ΠΏΠΈ.

Π­Π»Π΅ΠΊΡ‚Ρ€ΠΎΠ½Ρ‹ Π½Π΅ ΠΌΠΎΠ³ΡƒΡ‚ ΠΈΡΡ‡Π΅Π·Π°Ρ‚ΡŒ ΠΈΠ»ΠΈ Β«ΡΠΏΡ€Ρ‹Π³ΠΈΠ²Π°Ρ‚ΡŒΒ» с ΠΏΡ€ΠΎΠ²ΠΎΠ΄ΠΎΠ² ΠΈ Π½Π°Π³Ρ€ΡƒΠ·ΠΊΠΈ. ΠŸΠΎΡΡ‚ΠΎΠΌΡƒ, силу Ρ‚ΠΎΠΊΠ° ΠΌΡ‹ ΠΌΠΎΠΆΠ΅ΠΌ ΠΈΠ·ΠΌΠ΅Ρ€ΠΈΡ‚ΡŒ Π² любом мСстС элСктричСской Ρ†Π΅ΠΏΠΈ. Однако, Π±ΡƒΠ΄Π΅Ρ‚ Π»ΠΈ ΠΎΠ΄ΠΈΠ½Π°ΠΊΠΎΠ²Ρ‹ΠΌ дСйствиС Ρ‚ΠΎΠΊΠ° Π½Π° Ρ€Π°Π·Π½Ρ‹Π΅ участки этой Ρ†Π΅ΠΏΠΈ? Π”Π°Π²Π°ΠΉΡ‚Π΅ разбСрСмся.

ΠŸΡ€ΠΎΡ…ΠΎΠ΄Ρ ΠΏΠΎ ΠΏΡ€ΠΎΠ²ΠΎΠ΄Π°ΠΌ, Ρ‚ΠΎΠΊ лишь слСгка ΠΈΡ… Π½Π°Π³Ρ€Π΅Π²Π°Π΅Ρ‚, ΠΎΠ΄Π½Π°ΠΊΠΎ Π½Π΅ ΡΠΎΠ²Π΅Ρ€ΡˆΠ°Π΅Ρ‚ ΠΏΡ€ΠΈ этом большой Ρ€Π°Π±ΠΎΡ‚Ρ‹.

ΠŸΡ€ΠΎΡ…ΠΎΠ΄Ρ ΠΆΠ΅ Ρ‡Π΅Ρ€Π΅Π· ΡΠΏΠΈΡ€Π°Π»ΡŒ элСктричСской Π»Π°ΠΌΠΏΠΎΡ‡ΠΊΠΈ, Ρ‚ΠΎΠΊ Π½Π΅ просто сильно Π½Π°Π³Ρ€Π΅Π²Π°Π΅Ρ‚ Π΅Π΅, ΠΎΠ½ Π½Π°Π³Ρ€Π΅Π²Π°Π΅Ρ‚ Π΅Π΅ Π΄ΠΎ Ρ‚Π°ΠΊΠΎΠΉ стСпСни, Ρ‡Ρ‚ΠΎ ΠΎΠ½Π°, Ρ€Π°ΡΠΊΠ°Π»ΡΡΡΡŒ, Π½Π°Ρ‡ΠΈΠ½Π°Π΅Ρ‚ ΡΠ²Π΅Ρ‚ΠΈΡ‚ΡŒΡΡ.

Π’ΠΎ Π΅ΡΡ‚ΡŒ Π² Π΄Π°Π½Π½ΠΎΠΌ случаС Ρ‚ΠΎΠΊ ΡΠΎΠ²Π΅Ρ€ΡˆΠ°Π΅Ρ‚ ΠΌΠ΅Ρ…Π°Π½ΠΈΡ‡Π΅ΡΠΊΡƒΡŽ Ρ€Π°Π±ΠΎΡ‚Ρƒ, ΠΈ довольно ΠΏΡ€ΠΈΠ»ΠΈΡ‡Π½ΡƒΡŽ Ρ€Π°Π±ΠΎΡ‚Ρƒ. Π’ΠΎΠΊ Ρ‚Ρ€Π°Ρ‚ΠΈΡ‚ свою ΡΠ½Π΅Ρ€Π³ΠΈΡŽ.Β Π­Π»Π΅ΠΊΡ‚Ρ€ΠΎΠ½Ρ‹ Π² Ρ‚ΠΎΠΌ ΠΆΠ΅ количСствС ΠΏΡ€ΠΎΠ΄ΠΎΠ»ΠΆΠ°ΡŽΡ‚ Π±Π΅ΠΆΠ°Ρ‚ΡŒ дальшС, Π½ΠΎ энСргии Ρƒ Π½ΠΈΡ… ΡƒΠΆΠ΅ помСньшС.

ΠžΠΏΡ€Π΅Π΄Π΅Π»Π΅Π½ΠΈΠ΅ элСктричСского напряТСния

Π’ΠΎ Π΅ΡΡ‚ΡŒ элСктричСскоС ΠΏΠΎΠ»Π΅ Π΄ΠΎΠ»ΠΆΠ½ΠΎ Π±Ρ‹Π»ΠΎ Β«ΠΏΡ€ΠΎΡ‚Π°Ρ‰ΠΈΡ‚ΡŒΒ» элСктроны Ρ‡Π΅Ρ€Π΅Π· Π½Π°Π³Ρ€ΡƒΠ·ΠΊΡƒ, ΠΈ энСргия, которая ΠΏΡ€ΠΈ этом ΠΈΠ·Ρ€Π°ΡΡ…ΠΎΠ΄ΠΎΠ²Π°Π»Π°ΡΡŒ, характСризуСтся Π²Π΅Π»ΠΈΡ‡ΠΈΠ½ΠΎΠΉ, Π½Π°Π·Ρ‹Π²Π°Π΅ΠΌΠΎΠΉ элСктричСским напряТСниСм.

Π­Ρ‚Π° ΠΆΠ΅ энСргия ΠΏΠΎΡ‚Ρ€Π°Ρ‚ΠΈΠ»Π°ΡΡŒ Π½Π° ΠΊΠ°ΠΊΠΎΠ΅-Ρ‚ΠΎ ΠΈΠ·ΠΌΠ΅Π½Π΅Π½ΠΈΠ΅ состояния вСщСства Π½Π°Π³Ρ€ΡƒΠ·ΠΊΠΈ. ЭнСргия, ΠΊΠ°ΠΊ ΠΌΡ‹ Π·Π½Π°Π΅ΠΌ, Π½Π΅ ΠΏΡ€ΠΎΠΏΠ°Π΄Π°Π΅Ρ‚ Π² Π½ΠΈΠΊΡƒΠ΄Π° ΠΈ Π½Π΅ появляСтся ΠΈΠ· Π½ΠΈΠΎΡ‚ΠΊΡƒΠ΄Π°. Об этом гласит Π—Π°ΠΊΠΎΠ½ сохранСния энСргии.

Π’ΠΎ Π΅ΡΡ‚ΡŒ, Ссли Ρ‚ΠΎΠΊ ΠΏΠΎΡ‚Ρ€Π°Ρ‚ΠΈΠ» ΡΠ½Π΅Ρ€Π³ΠΈΡŽ Π½Π° ΠΏΡ€ΠΎΡ…ΠΎΠΆΠ΄Π΅Π½ΠΈΠ΅ Ρ‡Π΅Ρ€Π΅Π· Π½Π°Π³Ρ€ΡƒΠ·ΠΊΡƒ, эту ΡΠ½Π΅Ρ€Π³ΠΈΡŽ ΠΏΡ€ΠΈΠΎΠ±Ρ€Π΅Π»Π° Π½Π°Π³Ρ€ΡƒΠ·ΠΊΠ° ΠΈ, Π½Π°ΠΏΡ€ΠΈΠΌΠ΅Ρ€, Π½Π°Π³Ρ€Π΅Π»Π°ΡΡŒ.

Π’ΠΎ Π΅ΡΡ‚ΡŒ, ΠΏΡ€ΠΈΡ…ΠΎΠ΄ΠΈΠΌ ΠΊ ΠΎΠΏΡ€Π΅Π΄Π΅Π»Π΅Π½ΠΈΡŽ: напряТСниС элСктричСского Ρ‚ΠΎΠΊΠ° – это Π²Π΅Π»ΠΈΡ‡ΠΈΠ½Π°, ΠΏΠΎΠΊΠ°Π·Ρ‹Π²Π°ΡŽΡ‰Π°Ρ, ΠΊΠ°ΠΊΡƒΡŽ Ρ€Π°Π±ΠΎΡ‚Ρƒ ΡΠΎΠ²Π΅Ρ€ΡˆΠΈΠ»ΠΎ ΠΏΠΎΠ»Π΅ ΠΏΡ€ΠΈ ΠΏΠ΅Ρ€Π΅ΠΌΠ΅Ρ‰Π΅Π½ΠΈΠΈ заряда ΠΎΡ‚ ΠΎΠ΄Π½ΠΎΠΉ Ρ‚ΠΎΡ‡ΠΊΠΈ Π΄ΠΎ Π΄Ρ€ΡƒΠ³ΠΎΠΉ. НапряТСниС Π² Ρ€Π°Π·Π½Ρ‹Ρ… участках Ρ†Π΅ΠΏΠΈ Π±ΡƒΠ΄Π΅Ρ‚ Ρ€Π°Π·Π»ΠΈΡ‡Π½Ρ‹ΠΌ.

НапряТСниС Π½Π° участкС пустого ΠΏΡ€ΠΎΠ²ΠΎΠ΄Π° Π±ΡƒΠ΄Π΅Ρ‚ совсСм нСбольшим, Π° напряТСниС Π½Π° участкС с ΠΊΠ°ΠΊΠΎΠΉ-Π»ΠΈΠ±ΠΎ Π½Π°Π³Ρ€ΡƒΠ·ΠΊΠΎΠΉ Π±ΡƒΠ΄Π΅Ρ‚ Π³ΠΎΡ€Π°Π·Π΄ΠΎ большим, ΠΈ Π·Π°Π²ΠΈΡΠ΅Ρ‚ΡŒ Π²Π΅Π»ΠΈΡ‡ΠΈΠ½Π° напряТСния Π±ΡƒΠ΄Π΅Ρ‚ ΠΎΡ‚ Π²Π΅Π»ΠΈΡ‡ΠΈΠ½Ρ‹ Ρ€Π°Π±ΠΎΡ‚Ρ‹, ΠΏΡ€ΠΎΠΈΠ·Π²Π΅Π΄Π΅Π½Π½ΠΎΠΉ Ρ‚ΠΎΠΊΠΎΠΌ. Π˜Π·ΠΌΠ΅Ρ€ΡΡŽΡ‚ напряТСниС Π² Π²ΠΎΠ»ΡŒΡ‚Π°Ρ… (1 Π’).

Для опрСдСлСния напряТСния сущСствуСт Ρ„ΠΎΡ€ΠΌΡƒΠ»Π°:Β 

U=A/q,

Π³Π΄Π΅ U β€” напряТСниС,A – Ρ€Π°Π±ΠΎΡ‚Π°, ΡΠΎΠ²Π΅Ρ€ΡˆΠ΅Π½Π½Π°Ρ Ρ‚ΠΎΠΊΠΎΠΌ ΠΏΠΎ ΠΏΠ΅Ρ€Π΅ΠΌΠ΅Ρ‰Π΅Π½ΠΈΡŽ заряда q Π½Π° Π½Π΅ΠΊΠΈΠΉ участок Ρ†Π΅ΠΏΠΈ.

НапряТСниС Π½Π° ΠΏΠΎΠ»ΡŽΡΠ°Ρ… источника Ρ‚ΠΎΠΊΠ°

Π§Ρ‚ΠΎ касаСтся напряТСния Π½Π° участкС Ρ†Π΅ΠΏΠΈ – всС понятно.

А Ρ‡Ρ‚ΠΎ ΠΆΠ΅ Ρ‚ΠΎΠ³Π΄Π° ΠΎΠ·Π½Π°Ρ‡Π°Π΅Ρ‚ напряТСниС Π½Π° ΠΏΠΎΠ»ΡŽΡΠ°Ρ… источника Ρ‚ΠΎΠΊΠ°? Π’ Π΄Π°Π½Π½ΠΎΠΌ случаС это напряТСниС ΠΎΠ·Π½Π°Ρ‡Π°Π΅Ρ‚ ΠΏΠΎΡ‚Π΅Π½Ρ†ΠΈΠ°Π»ΡŒΠ½ΡƒΡŽ Π²Π΅Π»ΠΈΡ‡ΠΈΠ½Ρƒ энСргии, ΠΊΠΎΡ‚ΠΎΡ€ΡƒΡŽ ΠΌΠΎΠΆΠ΅Ρ‚ источник ΠΏΡ€ΠΈΠ΄Π°Ρ‚ΡŒ Ρ‚ΠΎΠΊΡƒ. Π­Ρ‚ΠΎ ΠΊΠ°ΠΊ Π΄Π°Π²Π»Π΅Π½ΠΈΠ΅ Π²ΠΎΠ΄Ρ‹ Π² Ρ‚Ρ€ΡƒΠ±Π°Ρ….

Π­Ρ‚Π° Π²Π΅Π»ΠΈΡ‡ΠΈΠ½Π° энСргии, которая Π±ΡƒΠ΄Π΅Ρ‚ израсходована, Ссли ΠΊ источнику ΠΏΠΎΠ΄ΠΊΠ»ΡŽΡ‡ΠΈΡ‚ΡŒ Π½Π΅ΠΊΡƒΡŽ Π½Π°Π³Ρ€ΡƒΠ·ΠΊΡƒ. ΠŸΠΎΡΡ‚ΠΎΠΌΡƒ, Ρ‡Π΅ΠΌ большСС напряТСниС Ρƒ источника Ρ‚ΠΎΠΊΠ°, Ρ‚Π΅ΠΌ Π±ΠΎΠ»ΡŒΡˆΡƒΡŽ Ρ€Π°Π±ΠΎΡ‚Ρƒ ΠΌΠΎΠΆΠ΅Ρ‚ ΡΠΎΠ²Π΅Ρ€ΡˆΠΈΡ‚ΡŒ Ρ‚ΠΎΠΊ.

Π’ΠΎΠ»ΡŒΡ‚ΠΌΠ΅Ρ‚Ρ€

Для измСрСния напряТСния сущСствуСт ΠΏΡ€ΠΈΠ±ΠΎΡ€, Π½Π°Π·Ρ‹Π²Π°Π΅ΠΌΡ‹ΠΉ Π²ΠΎΠ»ΡŒΡ‚ΠΌΠ΅Ρ‚Ρ€ΠΎΠΌ. Π’ ΠΎΡ‚Π»ΠΈΡ‡ΠΈΠ΅ ΠΎΡ‚ Π°ΠΌΠΏΠ΅Ρ€ΠΌΠ΅Ρ‚Ρ€Π°, ΠΎΠ½ ΠΏΠΎΠ΄ΠΊΠ»ΡŽΡ‡Π°Π΅Ρ‚ΡΡ Π½Π΅ ΠΏΡ€ΠΎΠΈΠ·Π²ΠΎΠ»ΡŒΠ½ΠΎ Π² любом мСстС Ρ†Π΅ΠΏΠΈ, Π° ΠΏΠ°Ρ€Π°Π»Π»Π΅Π»ΡŒΠ½ΠΎ Π½Π°Π³Ρ€ΡƒΠ·ΠΊΠ΅, Π΄ΠΎ Π½Π΅Π΅ ΠΈ послС. Π’ Ρ‚Π°ΠΊΠΎΠΌ случаС Π²ΠΎΠ»ΡŒΡ‚ΠΌΠ΅Ρ‚Ρ€ ΠΏΠΎΠΊΠ°Π·Ρ‹Π²Π°Π΅Ρ‚ Π²Π΅Π»ΠΈΡ‡ΠΈΠ½Ρƒ напряТСния, ΠΏΡ€ΠΈΠ»ΠΎΠΆΠ΅Π½Π½ΠΎΠ³ΠΎ ΠΊ Π½Π°Π³Ρ€ΡƒΠ·ΠΊΠ΅. Для измСрСния напряТСния Π½Π° ΠΏΠΎΠ»ΡŽΡΠ°Ρ… источника Ρ‚ΠΎΠΊΠ°, Π²ΠΎΠ»ΡŒΡ‚ΠΌΠ΅Ρ‚Ρ€ ΠΏΠΎΠ΄ΠΊΠ»ΡŽΡ‡Π°ΡŽΡ‚ нСпосрСдствСнно ΠΊ полюсам ΠΏΡ€ΠΈΠ±ΠΎΡ€Π°.

НуТна ΠΏΠΎΠΌΠΎΡ‰ΡŒ Π² ΡƒΡ‡Π΅Π±Π΅?

ΠŸΡ€Π΅Π΄Ρ‹Π΄ΡƒΡ‰Π°Ρ Ρ‚Π΅ΠΌΠ°: Π‘ΠΈΠ»Π° Ρ‚ΠΎΠΊΠ°: ΠΏΡ€ΠΈΡ€ΠΎΠ΄Π°, Ρ„ΠΎΡ€ΠΌΡƒΠ»Π°, ΠΈΠ·ΠΌΠ΅Ρ€Π΅Π½ΠΈΠ΅ Π°ΠΌΠΏΠ΅Ρ€ΠΌΠ΅Ρ‚Ρ€ΠΎΠΌ
Π‘Π»Π΅Π΄ΡƒΡŽΡ‰Π°Ρ Ρ‚Π΅ΠΌΠ°:Β Β Β Π‘ΠΎΠΏΡ€ΠΎΡ‚ΠΈΠ²Π»Π΅Π½ΠΈΠ΅ Ρ‚ΠΎΠΊΠ°: притяТСниС ядСр, ΠΏΡ€ΠΎΠ²ΠΎΠ΄Π½ΠΈΠΊΠΈ ΠΈ Π½Π΅ΠΏΡ€ΠΎΠ²ΠΎΠ΄Π½ΠΈΠΊΠΈ

Π˜ΡΡ‚ΠΎΡ‡Π½ΠΈΠΊ: http://www.nado5.ru/e-book/ehlnapryazhenie-voltmetr

ΠŸΠΎΡΡ‚ΠΎΡΠ½Π½Ρ‹ΠΉ элСктричСский Ρ‚ΠΎΠΊ

Автор ΡΡ‚Π°Ρ‚ΡŒΠΈ β€” ΠΏΡ€ΠΎΡ„Π΅ΡΡΠΈΠΎΠ½Π°Π»ΡŒΠ½Ρ‹ΠΉ Ρ€Π΅ΠΏΠ΅Ρ‚ΠΈΡ‚ΠΎΡ€, Π°Π²Ρ‚ΠΎΡ€ ΡƒΡ‡Π΅Π±Π½Ρ‹Ρ… пособий для ΠΏΠΎΠ΄Π³ΠΎΡ‚ΠΎΠ²ΠΊΠΈ ΠΊ Π•Π“Π­ Π˜Π³ΠΎΡ€ΡŒ ВячСславович Π―ΠΊΠΎΠ²Π»Π΅Π²

Π’Π΅ΠΌΡ‹ ΠΊΠΎΠ΄ΠΈΡ„ΠΈΠΊΠ°Ρ‚ΠΎΡ€Π° Π•Π“Π­: постоянный элСктричСский Ρ‚ΠΎΠΊ, сила Ρ‚ΠΎΠΊΠ°, напряТСниС

ЭлСктричСский Ρ‚ΠΎΠΊ обСспСчиваСт ΠΊΠΎΠΌΡ„ΠΎΡ€Ρ‚ΠΎΠΌ Тизнь соврСмСнного Ρ‡Π΅Π»ΠΎΠ²Π΅ΠΊΠ°. ВСхнологичСскиС достиТСния Ρ†ΠΈΠ²ΠΈΠ»ΠΈΠ·Π°Ρ†ΠΈΠΈ β€” энСргСтика, транспорт, Ρ€Π°Π΄ΠΈΠΎ, Ρ‚Π΅Π»Π΅Π²ΠΈΠ΄Π΅Π½ΠΈΠ΅, ΠΊΠΎΠΌΠΏΡŒΡŽΡ‚Π΅Ρ€Ρ‹, мобильная связь β€” основаны Π½Π° использовании элСктричСского Ρ‚ΠΎΠΊΠ°.

ЭлСктричСский Ρ‚ΠΎΠΊ β€” это Π½Π°ΠΏΡ€Π°Π²Π»Π΅Π½Π½ΠΎΠ΅ Π΄Π²ΠΈΠΆΠ΅Π½ΠΈΠ΅ заряТСнных частиц, ΠΏΡ€ΠΈ ΠΊΠΎΡ‚ΠΎΡ€ΠΎΠΌ происходит пСрСнос заряда ΠΈΠ· ΠΎΠ΄Π½ΠΈΡ… областСй пространства Π² Π΄Ρ€ΡƒΠ³ΠΈΠ΅.

ЭлСктричСский Ρ‚ΠΎΠΊ ΠΌΠΎΠΆΠ΅Ρ‚ Π²ΠΎΠ·Π½ΠΈΠΊΠ°Ρ‚ΡŒ Π² самых Ρ€Π°Π·Π»ΠΈΡ‡Π½Ρ‹Ρ… срСдах: Ρ‚Π²Ρ‘Ρ€Π΄Ρ‹Ρ… Ρ‚Π΅Π»Π°Ρ…, Тидкостях, Π³Π°Π·Π°Ρ…. ΠŸΠΎΡ€ΠΎΠΉ ΠΈ срСды Π½ΠΈΠΊΠ°ΠΊΠΎΠΉ Π½Π΅ Π½ΡƒΠΆΠ½ΠΎ β€” Ρ‚ΠΎΠΊ ΠΌΠΎΠΆΠ΅Ρ‚ ΡΡƒΡ‰Π΅ΡΡ‚Π²ΠΎΠ²Π°Ρ‚ΡŒ Π΄Π°ΠΆΠ΅ Π² Π²Π°ΠΊΡƒΡƒΠΌΠ΅! ΠœΡ‹ ΠΏΠΎΠ³ΠΎΠ²ΠΎΡ€ΠΈΠΌ ΠΎΠ± этом Π² своё врСмя, Π° ΠΏΠΎΠΊΠ° ΠΏΡ€ΠΈΠ²Π΅Π΄Ρ‘ΠΌ лишь Π½Π΅ΠΊΠΎΡ‚ΠΎΡ€Ρ‹Π΅ ΠΏΡ€ΠΈΠΌΠ΅Ρ€Ρ‹.

β€’ Π—Π°ΠΌΠΊΠ½Ρ‘ΠΌ полюса Π±Π°Ρ‚Π°Ρ€Π΅ΠΉΠΊΠΈ мСталличСским ΠΏΡ€ΠΎΠ²ΠΎΠ΄ΠΎΠΌ. Π‘Π²ΠΎΠ±ΠΎΠ΄Π½Ρ‹Π΅ элСктроны ΠΏΡ€ΠΎΠ²ΠΎΠ΄Π° Π½Π°Ρ‡Π½ΡƒΡ‚ Π½Π°ΠΏΡ€Π°Π²Π»Π΅Π½Π½ΠΎΠ΅ Π΄Π²ΠΈΠΆΠ΅Π½ΠΈΠ΅ ΠΎΡ‚ «минуса» Π±Π°Ρ‚Π°Ρ€Π΅ΠΉΠΊΠΈ ΠΊ Β«ΠΏΠ»ΡŽΡΡƒΒ».
Π­Ρ‚ΠΎ β€” ΠΏΡ€ΠΈΠΌΠ΅Ρ€ Ρ‚ΠΎΠΊΠ° Π² ΠΌΠ΅Ρ‚Π°Π»Π»Π°Ρ….

β€’ Бросим Π² стакан Π²ΠΎΠ΄Ρ‹ Ρ‰Π΅ΠΏΠΎΡ‚ΠΊΡƒ ΠΏΠΎΠ²Π°Ρ€Π΅Π½Π½ΠΎΠΉ соли . ΠœΠΎΠ»Π΅ΠΊΡƒΠ»Ρ‹ соли Π΄ΠΈΡΡΠΎΡ†ΠΈΠΈΡ€ΡƒΡŽΡ‚ Π½Π° ΠΈΠΎΠ½Ρ‹, Ρ‚Π°ΠΊ Ρ‡Ρ‚ΠΎ Π² растворС появятся свободныС заряды: ΠΏΠΎΠ»ΠΎΠΆΠΈΡ‚Π΅Π»ΡŒΠ½Ρ‹Π΅ ΠΈΠΎΠ½Ρ‹ ΠΈ ΠΎΡ‚Ρ€ΠΈΡ†Π°Ρ‚Π΅Π»ΡŒΠ½Ρ‹Π΅ ΠΈΠΎΠ½Ρ‹ . Π’Π΅ΠΏΠ΅Ρ€ΡŒ засунСм Π² Π²ΠΎΠ΄Ρƒ Π΄Π²Π° элСктрода, соСдинённыС с полюсами Π±Π°Ρ‚Π°Ρ€Π΅ΠΉΠΊΠΈ. Π˜ΠΎΠ½Ρ‹ Π½Π°Ρ‡Π½ΡƒΡ‚ Π½Π°ΠΏΡ€Π°Π²Π»Π΅Π½Π½ΠΎΠ΅ Π΄Π²ΠΈΠΆΠ΅Π½ΠΈΠ΅ ΠΊ ΠΎΡ‚Ρ€ΠΈΡ†Π°Ρ‚Π΅Π»ΡŒΠ½ΠΎΠΌΡƒ элСктроду, Π° ΠΈΠΎΠ½Ρ‹ β€” ΠΊ ΠΏΠΎΠ»ΠΎΠΆΠΈΡ‚Π΅Π»ΡŒΠ½ΠΎΠΌΡƒ.
Π­Ρ‚ΠΎ β€” ΠΏΡ€ΠΈΠΌΠ΅Ρ€ прохоТдСния Ρ‚ΠΎΠΊΠ° Ρ‡Π΅Ρ€Π΅Π· раствор элСктролита.

β€’ Π“Ρ€ΠΎΠ·ΠΎΠ²Ρ‹Π΅ Ρ‚ΡƒΡ‡ΠΈ ΡΠΎΠ·Π΄Π°ΡŽΡ‚ ΡΡ‚ΠΎΠ»ΡŒ ΠΌΠΎΡ‰Π½Ρ‹Π΅ элСктричСскиС поля, Ρ‡Ρ‚ΠΎ оказываСтся Π²ΠΎΠ·ΠΌΠΎΠΆΠ½Ρ‹ΠΌ ΠΏΡ€ΠΎΠ±ΠΎΠΉ Π²ΠΎΠ·Π΄ΡƒΡˆΠ½ΠΎΠ³ΠΎ ΠΏΡ€ΠΎΠΌΠ΅ΠΆΡƒΡ‚ΠΊΠ° Π΄Π»ΠΈΠ½ΠΎΠΉ Π² нСсколько ΠΊΠΈΠ»ΠΎΠΌΠ΅Ρ‚Ρ€ΠΎΠ². Π’ Ρ€Π΅Π·ΡƒΠ»ΡŒΡ‚Π°Ρ‚Π΅ сквозь Π²ΠΎΠ·Π΄ΡƒΡ… ΠΏΡ€ΠΎΡ…ΠΎΠ΄ΠΈΡ‚ гигантский разряд β€” молния.
Π­Ρ‚ΠΎ β€” ΠΏΡ€ΠΈΠΌΠ΅Ρ€ элСктричСского Ρ‚ΠΎΠΊΠ° Π² Π³Π°Π·Π΅.

Π’ΠΎ всСх Ρ‚Ρ€Ρ‘Ρ… рассмотрСнных ΠΏΡ€ΠΈΠΌΠ΅Ρ€Π°Ρ… элСктричСский Ρ‚ΠΎΠΊ обусловлСн Π΄Π²ΠΈΠΆΠ΅Π½ΠΈΠ΅ΠΌ заряТСнных частиц Π²Π½ΡƒΡ‚Ρ€ΠΈ Ρ‚Π΅Π»Π° ΠΈ называСтся Ρ‚ΠΎΠΊΠΎΠΌ проводимости.

β€’ Π’ΠΎΡ‚ нСсколько ΠΈΠ½ΠΎΠΉ ΠΏΡ€ΠΈΠΌΠ΅Ρ€. Π‘ΡƒΠ΄Π΅ΠΌ ΠΏΠ΅Ρ€Π΅ΠΌΠ΅Ρ‰Π°Ρ‚ΡŒ Π² пространствС заряТСнноС Ρ‚Π΅Π»ΠΎ. Вакая ситуация согласуСтся с ΠΎΠΏΡ€Π΅Π΄Π΅Π»Π΅Π½ΠΈΠ΅ΠΌ Ρ‚ΠΎΠΊΠ°! НаправлСнноС Π΄Π²ΠΈΠΆΠ΅Π½ΠΈΠ΅ зарядов β€” Π΅ΡΡ‚ΡŒ, пСрСнос заряда Π² пространствС β€” присутствуСт. Π’ΠΎΠΊ, созданный Π΄Π²ΠΈΠΆΠ΅Π½ΠΈΠ΅ΠΌ макроскопичСского заряТСнного Ρ‚Π΅Π»Π°, называСтся ΠΊΠΎΠ½Π²Π΅ΠΊΡ†ΠΈΠΎΠ½Π½Ρ‹ΠΌ.

Π—Π°ΠΌΠ΅Ρ‚ΠΈΠΌ, Ρ‡Ρ‚ΠΎ Π½Π΅ всякоС Π΄Π²ΠΈΠΆΠ΅Π½ΠΈΠ΅ заряТСнных частиц ΠΎΠ±Ρ€Π°Π·ΡƒΠ΅Ρ‚ Ρ‚ΠΎΠΊ.

НапримСр, хаотичСскоС Ρ‚Π΅ΠΏΠ»ΠΎΠ²ΠΎΠ΅ Π΄Π²ΠΈΠΆΠ΅Π½ΠΈΠ΅ зарядов ΠΏΡ€ΠΎΠ²ΠΎΠ΄Π½ΠΈΠΊΠ° β€” Π½Π΅ Π½Π°ΠΏΡ€Π°Π²Π»Π΅Π½Π½ΠΎΠ΅ (ΠΎΠ½ΠΎ ΡΠΎΠ²Π΅Ρ€ΡˆΠ°Π΅Ρ‚ΡΡ Π² ΠΊΠ°ΠΊΠΈΡ… ΡƒΠ³ΠΎΠ΄Π½ΠΎ направлСниях), ΠΈ ΠΏΠΎΡ‚ΠΎΠΌΡƒ Ρ‚ΠΎΠΊΠΎΠΌ Π½Π΅ являСтся (ΠΏΡ€ΠΈ Π²ΠΎΠ·Π½ΠΈΠΊΠ½ΠΎΠ²Π΅Π½ΠΈΠΈ Ρ‚ΠΎΠΊΠ° свободныС заряды ΠΏΡ€ΠΎΠ΄ΠΎΠ»ΠΆΠ°ΡŽΡ‚ ΡΠΎΠ²Π΅Ρ€ΡˆΠ°Ρ‚ΡŒ Ρ‚Π΅ΠΏΠ»ΠΎΠ²ΠΎΠ΅ Π΄Π²ΠΈΠΆΠ΅Π½ΠΈΠ΅! ΠŸΡ€ΠΎΡΡ‚ΠΎ Π² этом случаС ΠΊ хаотичСским пСрСмСщСниям заряТСнных частиц добавляСтся ΠΈΡ… упорядочСнный Π΄Ρ€Π΅ΠΉΡ„ Π² ΠΎΠΏΡ€Π΅Π΄Π΅Π»Ρ‘Π½Π½ΠΎΠΌ
Π½Π°ΠΏΡ€Π°Π²Π»Π΅Π½ΠΈΠΈ).

НС Π±ΡƒΠ΄Π΅Ρ‚ Ρ‚ΠΎΠΊΠΎΠΌ ΠΈ ΠΏΠΎΡΡ‚ΡƒΠΏΠ°Ρ‚Π΅Π»ΡŒΠ½ΠΎΠ΅ Π΄Π²ΠΈΠΆΠ΅Π½ΠΈΠ΅ элСктричСски Π½Π΅ΠΉΡ‚Ρ€Π°Π»ΡŒΠ½ΠΎΠ³ΠΎ Ρ‚Π΅Π»Π°: хотя заряТСнныС частицы Π² Π΅Π³ΠΎ Π°Ρ‚ΠΎΠΌΠ°Ρ… ΠΈ ΡΠΎΠ²Π΅Ρ€ΡˆΠ°ΡŽΡ‚ Π½Π°ΠΏΡ€Π°Π²Π»Π΅Π½Π½ΠΎΠ΅ Π΄Π²ΠΈΠΆΠ΅Π½ΠΈΠ΅, Π½Π΅ происходит пСрСноса заряда ΠΈΠ· ΠΎΠ΄Π½ΠΈΡ… участков пространства Π² Π΄Ρ€ΡƒΠ³ΠΈΠ΅.

НаправлСниС элСктричСского Ρ‚ΠΎΠΊΠ°

НаправлСниС двиТСния заряТСнных частиц, ΠΎΠ±Ρ€Π°Π·ΡƒΡŽΡ‰ΠΈΡ… Ρ‚ΠΎΠΊ, зависит ΠΎΡ‚ Π·Π½Π°ΠΊΠ° ΠΈΡ… заряда. ΠŸΠΎΠ»ΠΎΠΆΠΈΡ‚Π΅Π»ΡŒΠ½ΠΎ заряТСнныС частицы Π±ΡƒΠ΄ΡƒΡ‚ Π΄Π²ΠΈΠ³Π°Ρ‚ΡŒΡΡ ΠΎΡ‚ «плюса» ΠΊ «минусу», Π° ΠΎΡ‚Ρ€ΠΈΡ†Π°Ρ‚Π΅Π»ΡŒΠ½ΠΎ заряТСнныС β€” Π½Π°ΠΎΠ±ΠΎΡ€ΠΎΡ‚, ΠΎΡ‚ «минуса» ΠΊ Β«ΠΏΠ»ΡŽΡΡƒΒ».

Π’ элСктролитах ΠΈ Π³Π°Π·Π°Ρ…, Π½Π°ΠΏΡ€ΠΈΠΌΠ΅Ρ€, ΠΏΡ€ΠΈΡΡƒΡ‚ΡΡ‚Π²ΡƒΡŽΡ‚ ΠΊΠ°ΠΊ ΠΏΠΎΠ»ΠΎΠΆΠΈΡ‚Π΅Π»ΡŒΠ½Ρ‹Π΅, Ρ‚Π°ΠΊ ΠΈ ΠΎΡ‚Ρ€ΠΈΡ†Π°Ρ‚Π΅Π»ΡŒΠ½Ρ‹Π΅ свободныС заряды, ΠΈ Ρ‚ΠΎΠΊ создаётся ΠΈΡ… встрСчным Π΄Π²ΠΈΠΆΠ΅Π½ΠΈΠ΅ΠΌ Π² ΠΎΠ±ΠΎΠΈΡ… направлСниях.

КакоС ΠΆΠ΅ ΠΈΠ· этих Π½Π°ΠΏΡ€Π°Π²Π»Π΅Π½ΠΈΠΉ ΠΏΡ€ΠΈΠ½ΡΡ‚ΡŒ Π·Π° Π½Π°ΠΏΡ€Π°Π²Π»Π΅Π½ΠΈΠ΅ элСктричСского Ρ‚ΠΎΠΊΠ°?

НаправлСниСм Ρ‚ΠΎΠΊΠ° принято ΡΡ‡ΠΈΡ‚Π°Ρ‚ΡŒ Π½Π°ΠΏΡ€Π°Π²Π»Π΅Π½ΠΈΠ΅ двиТСния ΠΏΠΎΠ»ΠΎΠΆΠΈΡ‚Π΅Π»ΡŒΠ½Ρ‹Ρ… зарядов.

ΠŸΠΎΠΏΡ€ΠΎΡΡ‚Ρƒ говоря, ΠΏΠΎ соглашСнию Ρ‚ΠΎΠΊ Ρ‚Π΅Ρ‡Ρ‘Ρ‚ ΠΎΡ‚ «плюса» ΠΊ «минусу» (рис. 1; ΠΏΠΎΠ»ΠΎΠΆΠΈΡ‚Π΅Π»ΡŒΠ½Π°Ρ ΠΊΠ»Π΅ΠΌΠΌΠ° источника Ρ‚ΠΎΠΊΠ° ΠΈΠ·ΠΎΠ±Ρ€Π°ΠΆΠ΅Π½Π° Π΄Π»ΠΈΠ½Π½ΠΎΠΉ Ρ‡Π΅Ρ€Ρ‚ΠΎΠΉ, ΠΎΡ‚Ρ€ΠΈΡ†Π°Ρ‚Π΅Π»ΡŒΠ½Π°Ρ ΠΊΠ»Π΅ΠΌΠΌΠ° β€” ΠΊΠΎΡ€ΠΎΡ‚ΠΊΠΎΠΉ).

Рис. 1. НаправлСниС Ρ‚ΠΎΠΊΠ°

Π”Π°Π½Π½ΠΎΠ΅ соглашСниС вступаСт Π² Π½Π΅ΠΊΠΎΡ‚ΠΎΡ€ΠΎΠ΅ ΠΏΡ€ΠΎΡ‚ΠΈΠ²ΠΎΡ€Π΅Ρ‡ΠΈΠ΅ с Π½Π°ΠΈΠ±ΠΎΠ»Π΅Π΅ распространённым случаСм мСталличСских ΠΏΡ€ΠΎΠ²ΠΎΠ΄Π½ΠΈΠΊΠΎΠ². Π’ ΠΌΠ΅Ρ‚Π°Π»Π»Π΅ носитСлями заряда ΡΠ²Π»ΡΡŽΡ‚ΡΡ свободныС элСктроны, ΠΈ Π΄Π²ΠΈΠ³Π°ΡŽΡ‚ΡΡ ΠΎΠ½ΠΈ ΠΎΡ‚ «минуса» ΠΊ Β«ΠΏΠ»ΡŽΡΡƒΒ». Но Π² соотвСтствии с соглашСниСм ΠΌΡ‹ Π²Ρ‹Π½ΡƒΠΆΠ΄Π΅Π½Ρ‹ ΡΡ‡ΠΈΡ‚Π°Ρ‚ΡŒ, Ρ‡Ρ‚ΠΎ Π½Π°ΠΏΡ€Π°Π²Π»Π΅Π½ΠΈΠ΅ Ρ‚ΠΎΠΊΠ° Π² мСталличСском ΠΏΡ€ΠΎΠ²ΠΎΠ΄Π½ΠΈΠΊΠ΅ ΠΏΡ€ΠΎΡ‚ΠΈΠ²ΠΎΠΏΠΎΠ»ΠΎΠΆΠ½ΠΎ двиТСнию свободных элСктронов. Π­Ρ‚ΠΎ, ΠΊΠΎΠ½Π΅Ρ‡Π½ΠΎ, Π½Π΅ ΠΎΡ‡Π΅Π½ΡŒ ΡƒΠ΄ΠΎΠ±Π½ΠΎ.

Π’ΡƒΡ‚, ΠΎΠ΄Π½Π°ΠΊΠΎ, Π½ΠΈΡ‡Π΅Π³ΠΎ Π½Π΅ подСлаСшь β€” придётся ΠΏΡ€ΠΈΠ½ΡΡ‚ΡŒ эту ΡΠΈΡ‚ΡƒΠ°Ρ†ΠΈΡŽ ΠΊΠ°ΠΊ Π΄Π°Π½Π½ΠΎΡΡ‚ΡŒ. Π’Π°ΠΊ ΡƒΠΆ историчСски слоТилось.

Π’Ρ‹Π±ΠΎΡ€ направлСния Ρ‚ΠΎΠΊΠ° Π±Ρ‹Π» ΠΏΡ€Π΅Π΄Π»ΠΎΠΆΠ΅Π½ АмпСром (Π΄ΠΎΠ³ΠΎΠ²ΠΎΡ€Ρ‘Π½Π½ΠΎΡΡ‚ΡŒ ΠΎ Π½Π°ΠΏΡ€Π°Π²Π»Π΅Π½ΠΈΠΈ Ρ‚ΠΎΠΊΠ° понадобилась АмпСру для Ρ‚ΠΎΠ³ΠΎ, Ρ‡Ρ‚ΠΎΠ±Ρ‹ Π΄Π°Ρ‚ΡŒ Ρ‡Ρ‘Ρ‚ΠΊΠΎΠ΅ ΠΏΡ€Π°Π²ΠΈΠ»ΠΎ опрСдСлСния направлСния силы, Π΄Π΅ΠΉΡΡ‚Π²ΡƒΡŽΡ‰Π΅ΠΉ Π½Π° ΠΏΡ€ΠΎΠ²ΠΎΠ΄Π½ΠΈΠΊ с Ρ‚ΠΎΠΊΠΎΠΌ Π² ΠΌΠ°Π³Π½ΠΈΡ‚Π½ΠΎΠΌ ΠΏΠΎΠ»Π΅.

БСгодня эту силу ΠΌΡ‹ Π½Π°Π·Ρ‹Π²Π°Π΅ΠΌ силой АмпСра, Π½Π°ΠΏΡ€Π°Π²Π»Π΅Π½ΠΈΠ΅ ΠΊΠΎΡ‚ΠΎΡ€ΠΎΠΉ опрСдСляСтся ΠΏΠΎ ΠΏΡ€Π°Π²ΠΈΠ»Ρƒ Π»Π΅Π²ΠΎΠΉ Ρ€ΡƒΠΊΠΈ) Π² ΠΏΠ΅Ρ€Π²ΠΎΠΉ ΠΏΠΎΠ»ΠΎΠ²ΠΈΠ½Π΅ XIX Π²Π΅ΠΊΠ°, Π·Π° 70 Π»Π΅Ρ‚ Π΄ΠΎ открытия элСктрона. К этому Π²Ρ‹Π±ΠΎΡ€Ρƒ всС ΠΏΡ€ΠΈΠ²Ρ‹ΠΊΠ»ΠΈ, ΠΈ ΠΊΠΎΠ³Π΄Π° Π² 1916 Π³ΠΎΠ΄Ρƒ Π²Ρ‹ΡΡΠ½ΠΈΠ»ΠΎΡΡŒ, Ρ‡Ρ‚ΠΎ Ρ‚ΠΎΠΊ Π² ΠΌΠ΅Ρ‚Π°Π»Π»Π°Ρ… Π²Ρ‹Π·Π²Π°Π½ Π΄Π²ΠΈΠΆΠ΅Π½ΠΈΠ΅ΠΌ свободных элСктронов, Π½ΠΈΡ‡Π΅Π³ΠΎ ΠΌΠ΅Π½ΡΡ‚ΡŒ ΡƒΠΆΠ΅ Π½Π΅ стали.

ДСйствия элСктричСского Ρ‚ΠΎΠΊΠ°

Как ΠΌΡ‹ ΠΌΠΎΠΆΠ΅ΠΌ ΠΎΠΏΡ€Π΅Π΄Π΅Π»ΠΈΡ‚ΡŒ, ΠΏΡ€ΠΎΡ‚Π΅ΠΊΠ°Π΅Ρ‚ элСктричСский Ρ‚ΠΎΠΊ ΠΈΠ»ΠΈ Π½Π΅Ρ‚? О Π²ΠΎΠ·Π½ΠΈΠΊΠ½ΠΎΠ²Π΅Π½ΠΈΠΈ элСктричСского Ρ‚ΠΎΠΊΠ° ΠΌΠΎΠΆΠ½ΠΎ ΡΡƒΠ΄ΠΈΡ‚ΡŒ ΠΏΠΎ ΡΠ»Π΅Π΄ΡƒΡŽΡ‰ΠΈΠΌ Π΅Π³ΠΎ проявлСниям.

1. Π’Π΅ΠΏΠ»ΠΎΠ²ΠΎΠ΅ дСйствиС Ρ‚ΠΎΠΊΠ°. ЭлСктричСский Ρ‚ΠΎΠΊ Π²Ρ‹Π·Ρ‹Π²Π°Π΅Ρ‚ Π½Π°Π³Ρ€Π΅Π²Π°Π½ΠΈΠ΅ вСщСства, Π² ΠΊΠΎΡ‚ΠΎΡ€ΠΎΠΌ ΠΎΠ½ ΠΏΡ€ΠΎΡ‚Π΅ΠΊΠ°Π΅Ρ‚. ИмСнно Ρ‚Π°ΠΊ Π½Π°Π³Ρ€Π΅Π²Π°ΡŽΡ‚ΡΡ спирали Π½Π°Π³Ρ€Π΅Π²Π°Ρ‚Π΅Π»ΡŒΠ½Ρ‹Ρ… ΠΏΡ€ΠΈΠ±ΠΎΡ€ΠΎΠ² ΠΈ Π»Π°ΠΌΠΏ накаливания. ИмСнно поэтому ΠΌΡ‹ Π²ΠΈΠ΄ΠΈΠΌ молнию. Π’ основС дСйствия Ρ‚Π΅ΠΏΠ»ΠΎΠ²Ρ‹Ρ… Π°ΠΌΠΏΠ΅Ρ€ΠΌΠ΅Ρ‚Ρ€ΠΎΠ² Π»Π΅ΠΆΠΈΡ‚ Ρ‚Π΅ΠΏΠ»ΠΎΠ²ΠΎΠ΅ Ρ€Π°ΡΡˆΠΈΡ€Π΅Π½ΠΈΠ΅ ΠΏΡ€ΠΎΠ²ΠΎΠ΄Π½ΠΈΠΊΠ° с Ρ‚ΠΎΠΊΠΎΠΌ, приводящСС ΠΊ ΠΏΠ΅Ρ€Π΅ΠΌΠ΅Ρ‰Π΅Π½ΠΈΡŽ стрСлки ΠΏΡ€ΠΈΠ±ΠΎΡ€Π°.

2. ΠœΠ°Π³Π½ΠΈΡ‚Π½ΠΎΠ΅ дСйствиС Ρ‚ΠΎΠΊΠ°. ЭлСктричСский Ρ‚ΠΎΠΊ создаёт ΠΌΠ°Π³Π½ΠΈΡ‚Π½ΠΎΠ΅ ΠΏΠΎΠ»Π΅: стрСлка компаса, располоТСнная рядом с ΠΏΡ€ΠΎΠ²ΠΎΠ΄ΠΎΠΌ, ΠΏΡ€ΠΈ Π²ΠΊΠ»ΡŽΡ‡Π΅Π½ΠΈΠΈ Ρ‚ΠΎΠΊΠ° поворачиваСтся пСрпСндикулярно ΠΏΡ€ΠΎΠ²ΠΎΠ΄Ρƒ.

ΠœΠ°Π³Π½ΠΈΡ‚Π½ΠΎΠ΅ ΠΏΠΎΠ»Π΅ Ρ‚ΠΎΠΊΠ° ΠΌΠΎΠΆΠ½ΠΎ ΠΌΠ½ΠΎΠ³ΠΎΠΊΡ€Π°Ρ‚Π½ΠΎ ΡƒΡΠΈΠ»ΠΈΡ‚ΡŒ, Ссли ΠΎΠ±ΠΌΠΎΡ‚Π°Ρ‚ΡŒ ΠΏΡ€ΠΎΠ²ΠΎΠ΄ Π²ΠΎΠΊΡ€ΡƒΠ³ ΠΆΠ΅Π»Π΅Π·Π½ΠΎΠ³ΠΎ стСрТня β€” получится элСктромагнит.

На этом ΠΏΡ€ΠΈΠ½Ρ†ΠΈΠΏΠ΅ основано дСйствиС Π°ΠΌΠΏΠ΅Ρ€ΠΌΠ΅Ρ‚Ρ€ΠΎΠ² магнитоэлСктричСской систСмы: элСктромагнит поворачиваСтся Π² ΠΏΠΎΠ»Π΅ постоянного ΠΌΠ°Π³Π½ΠΈΡ‚Π°, Π² Ρ€Π΅Π·ΡƒΠ»ΡŒΡ‚Π°Ρ‚Π΅ Ρ‡Π΅Π³ΠΎ стрСлка ΠΏΡ€ΠΈΠ±ΠΎΡ€Π° пСрСмСщаСтся ΠΏΠΎ шкалС.

3. Π₯имичСскоС дСйствиС Ρ‚ΠΎΠΊΠ°. ΠŸΡ€ΠΈ ΠΏΡ€ΠΎΡ…ΠΎΠΆΠ΄Π΅Π½ΠΈΠΈ Ρ‚ΠΎΠΊΠ° Ρ‡Π΅Ρ€Π΅Π· элСктролиты ΠΌΠΎΠΆΠ½ΠΎ Π½Π°Π±Π»ΡŽΠ΄Π°Ρ‚ΡŒ ΠΈΠ·ΠΌΠ΅Π½Π΅Π½ΠΈΠ΅ химичСского состава вСщСства. Π’Π°ΠΊ, Π² растворС ΠΏΠΎΠ»ΠΎΠΆΠΈΡ‚Π΅Π»ΡŒΠ½Ρ‹Π΅ ΠΈΠΎΠ½Ρ‹ Π΄Π²ΠΈΠ³Π°ΡŽΡ‚ΡΡ ΠΊ ΠΎΡ‚Ρ€ΠΈΡ†Π°Ρ‚Π΅Π»ΡŒΠ½ΠΎΠΌΡƒ элСктроду, ΠΈ этот элСктрод покрываСтся мСдью.

ЭлСктричСский Ρ‚ΠΎΠΊ называСтся постоянным, Ссли Π·Π° Ρ€Π°Π²Π½Ρ‹Π΅ ΠΏΡ€ΠΎΠΌΠ΅ΠΆΡƒΡ‚ΠΊΠΈ Π²Ρ€Π΅ΠΌΠ΅Π½ΠΈ Ρ‡Π΅Ρ€Π΅Π· ΠΏΠΎΠΏΠ΅Ρ€Π΅Ρ‡Π½ΠΎΠ΅ сСчСниС ΠΏΡ€ΠΎΠ²ΠΎΠ΄Π½ΠΈΠΊΠ° ΠΏΡ€ΠΎΡ…ΠΎΠ΄ΠΈΡ‚ ΠΎΠ΄ΠΈΠ½Π°ΠΊΠΎΠ²Ρ‹ΠΉ заряд.

ΠŸΠΎΡΡ‚ΠΎΡΠ½Π½Ρ‹ΠΉ Ρ‚ΠΎΠΊ Π½Π°ΠΈΠ±ΠΎΠ»Π΅Π΅ прост для изучСния. Π‘ Π½Π΅Π³ΠΎ ΠΌΡ‹ ΠΈ Π½Π°Ρ‡ΠΈΠ½Π°Π΅ΠΌ.

Π‘ΠΈΠ»Π° ΠΈ ΠΏΠ»ΠΎΡ‚Π½ΠΎΡΡ‚ΡŒ Ρ‚ΠΎΠΊΠ°

ΠšΠΎΠ»ΠΈΡ‡Π΅ΡΡ‚Π²Π΅Π½Π½ΠΎΠΉ характСристикой элСктричСского Ρ‚ΠΎΠΊΠ° являСтся сила Ρ‚ΠΎΠΊΠ°. Π’ случаС постоянного Ρ‚ΠΎΠΊΠ° Π°Π±ΡΠΎΠ»ΡŽΡ‚Π½Π°Ρ Π²Π΅Π»ΠΈΡ‡ΠΈΠ½Π° силы Ρ‚ΠΎΠΊΠ° Π΅ΡΡ‚ΡŒ ΠΎΡ‚Π½ΠΎΡˆΠ΅Π½ΠΈΠ΅ Π°Π±ΡΠΎΠ»ΡŽΡ‚Π½ΠΎΠΉ Π²Π΅Π»ΠΈΡ‡ΠΈΠ½Ρ‹ заряда , ΠΏΡ€ΠΎΡˆΠ΅Π΄ΡˆΠ΅Π³ΠΎ Ρ‡Π΅Ρ€Π΅Π· ΠΏΠΎΠΏΠ΅Ρ€Π΅Ρ‡Π½ΠΎΠ΅ сСчСниС ΠΏΡ€ΠΎΠ²ΠΎΠ΄Π½ΠΈΠΊΠ° Π·Π° врСмя , ΠΊ этому самому Π²Ρ€Π΅ΠΌΠ΅Π½ΠΈ:

(1)

Π˜Π·ΠΌΠ΅Ρ€ΡΠ΅Ρ‚ΡΡ сила Ρ‚ΠΎΠΊΠ° Π² Π°ΠΌΠΏΠ΅Ρ€Π°Ρ… (A). ΠŸΡ€ΠΈ силС Ρ‚ΠΎΠΊΠ° Π² А Ρ‡Π΅Ρ€Π΅Π· ΠΏΠΎΠΏΠ΅Ρ€Π΅Ρ‡Π½ΠΎΠ΅ сСчСниС ΠΏΡ€ΠΎΠ²ΠΎΠ΄Π½ΠΈΠΊΠ° Π·Π° с ΠΏΡ€ΠΎΡ…ΠΎΠ΄ΠΈΡ‚ заряд Π² Кл.

ΠŸΠΎΠ΄Ρ‡Π΅Ρ€ΠΊΠ½Ρ‘ΠΌ, Ρ‡Ρ‚ΠΎ Ρ„ΠΎΡ€ΠΌΡƒΠ»Π° (1) опрСдСляСт Π°Π±ΡΠΎΠ»ΡŽΡ‚Π½ΡƒΡŽ Π²Π΅Π»ΠΈΡ‡ΠΈΠ½Ρƒ, ΠΈΠ»ΠΈ ΠΌΠΎΠ΄ΡƒΠ»ΡŒ силы Ρ‚ΠΎΠΊΠ°.
Π‘ΠΈΠ»Π° Ρ‚ΠΎΠΊΠ° ΠΌΠΎΠΆΠ΅Ρ‚ ΠΈΠΌΠ΅Ρ‚ΡŒ Π΅Ρ‰Ρ‘ ΠΈ Π·Π½Π°ΠΊ! Π­Ρ‚ΠΎΡ‚ Π·Π½Π°ΠΊ Π½Π΅ связан со Π·Π½Π°ΠΊΠΎΠΌ зарядов, ΠΎΠ±Ρ€Π°Π·ΡƒΡŽΡ‰ΠΈΡ… Ρ‚ΠΎΠΊ, ΠΈ выбираСтся ΠΈΠ· ΠΈΠ½Ρ‹Ρ… сообраТСний.

А ΠΈΠΌΠ΅Π½Π½ΠΎ, Π² рядС ситуаций (Π½Π°ΠΏΡ€ΠΈΠΌΠ΅Ρ€, Ссли Π·Π°Ρ€Π°Π½Π΅Π΅ Π½Π΅ ясно, ΠΊΡƒΠ΄Π° ΠΏΠΎΡ‚Π΅Ρ‡Ρ‘Ρ‚ Ρ‚ΠΎΠΊ) ΡƒΠ΄ΠΎΠ±Π½ΠΎ Π·Π°Ρ„ΠΈΠΊΡΠΈΡ€ΠΎΠ²Π°Ρ‚ΡŒ Π½Π΅ΠΊΠΎΡ‚ΠΎΡ€ΠΎΠ΅ Π½Π°ΠΏΡ€Π°Π²Π»Π΅Π½ΠΈΠ΅ ΠΎΠ±Ρ…ΠΎΠ΄Π° Ρ†Π΅ΠΏΠΈ (скаТСм, ΠΏΡ€ΠΎΡ‚ΠΈΠ² часовой стрСлки) ΠΈ ΡΡ‡ΠΈΡ‚Π°Ρ‚ΡŒ силу Ρ‚ΠΎΠΊΠ° ΠΏΠΎΠ»ΠΎΠΆΠΈΡ‚Π΅Π»ΡŒΠ½ΠΎΠΉ, Ссли Π½Π°ΠΏΡ€Π°Π²Π»Π΅Π½ΠΈΠ΅ Ρ‚ΠΎΠΊΠ° совпадаСт с Π½Π°ΠΏΡ€Π°Π²Π»Π΅Π½ΠΈΠ΅ΠΌ ΠΎΠ±Ρ…ΠΎΠ΄Π°, ΠΈ ΠΎΡ‚Ρ€ΠΈΡ†Π°Ρ‚Π΅Π»ΡŒΠ½ΠΎΠΉ, Ссли Ρ‚ΠΎΠΊ Ρ‚Π΅Ρ‡Ρ‘Ρ‚ ΠΏΡ€ΠΎΡ‚ΠΈΠ² направлСния ΠΎΠ±Ρ…ΠΎΠ΄Π° (сравнитС с тригономСтричСским ΠΊΡ€ΡƒΠ³ΠΎΠΌ: ΡƒΠ³Π»Ρ‹ ΡΡ‡ΠΈΡ‚Π°ΡŽΡ‚ΡΡ ΠΏΠΎΠ»ΠΎΠΆΠΈΡ‚Π΅Π»ΡŒΠ½Ρ‹ΠΌΠΈ, Ссли ΠΎΡ‚ΡΡ‡ΠΈΡ‚Ρ‹Π²Π°ΡŽΡ‚ΡΡ ΠΏΡ€ΠΎΡ‚ΠΈΠ² часовой стрСлки, ΠΈ ΠΎΡ‚Ρ€ΠΈΡ†Π°Ρ‚Π΅Π»ΡŒΠ½Ρ‹ΠΌΠΈ, Ссли ΠΏΠΎ часовой стрСлкС).

Π’ случаС постоянного Ρ‚ΠΎΠΊΠ° сила Ρ‚ΠΎΠΊΠ° Π΅ΡΡ‚ΡŒ Π²Π΅Π»ΠΈΡ‡ΠΈΠ½Π° постоянная. Она ΠΏΠΎΠΊΠ°Π·Ρ‹Π²Π°Π΅Ρ‚, ΠΊΠ°ΠΊΠΎΠΉ заряд ΠΏΡ€ΠΎΡ…ΠΎΠ΄ΠΈΡ‚ Ρ‡Π΅Ρ€Π΅Π· ΠΏΠΎΠΏΠ΅Ρ€Π΅Ρ‡Π½ΠΎΠ΅ сСчСниС ΠΏΡ€ΠΎΠ²ΠΎΠ΄Π½ΠΈΠΊΠ° Π·Π° с.

  • Часто Π±Ρ‹Π²Π°Π΅Ρ‚ ΡƒΠ΄ΠΎΠ±Π½ΠΎ Π½Π΅ ΡΠ²ΡΠ·Ρ‹Π²Π°Ρ‚ΡŒΡΡ с ΠΏΠ»ΠΎΡ‰Π°Π΄ΡŒΡŽ ΠΏΠΎΠΏΠ΅Ρ€Π΅Ρ‡Π½ΠΎΠ³ΠΎ сСчСния ΠΈ ввСсти Π²Π΅Π»ΠΈΡ‡ΠΈΠ½Ρƒ плотности Ρ‚ΠΎΠΊΠ°:
  • (2)
  • Π³Π΄Π΅ β€” сила Ρ‚ΠΎΠΊΠ°, β€” ΠΏΠ»ΠΎΡ‰Π°Π΄ΡŒ ΠΏΠΎΠΏΠ΅Ρ€Π΅Ρ‡Π½ΠΎΠ³ΠΎ сСчСния ΠΏΡ€ΠΎΠ²ΠΎΠ΄Π½ΠΈΠΊΠ° (разумССтся, это сСчСниС пСрпСндикулярно Π½Π°ΠΏΡ€Π°Π²Π»Π΅Π½ΠΈΡŽ Ρ‚ΠΎΠΊΠ°). Π‘ ΡƒΡ‡Ρ‘Ρ‚ΠΎΠΌ Ρ„ΠΎΡ€ΠΌΡƒΠ»Ρ‹ (1) ΠΈΠΌΠ΅Π΅ΠΌ Ρ‚Π°ΠΊΠΆΠ΅:

ΠŸΠ»ΠΎΡ‚Π½ΠΎΡΡ‚ΡŒ Ρ‚ΠΎΠΊΠ° ΠΏΠΎΠΊΠ°Π·Ρ‹Π²Π°Π΅Ρ‚, ΠΊΠ°ΠΊΠΎΠΉ заряд ΠΏΡ€ΠΎΡ…ΠΎΠ΄ΠΈΡ‚ Π·Π° Π΅Π΄ΠΈΠ½ΠΈΡ†Ρƒ Π²Ρ€Π΅ΠΌΠ΅Π½ΠΈ Ρ‡Π΅Ρ€Π΅Π· Π΅Π΄ΠΈΠ½ΠΈΡ†Ρƒ ΠΏΠ»ΠΎΡ‰Π°Π΄ΠΈ ΠΏΠΎΠΏΠ΅Ρ€Π΅Ρ‡Π½ΠΎΠ³ΠΎ сСчСния ΠΏΡ€ΠΎΠ²ΠΎΠ΄Π½ΠΈΠΊΠ°. Богласно Ρ„ΠΎΡ€ΠΌΡƒΠ»Π΅ (2), ΠΏΠ»ΠΎΡ‚Π½ΠΎΡΡ‚ΡŒ Ρ‚ΠΎΠΊΠ° измСряСтся Π² А/ΠΌ2.

Π‘ΠΊΠΎΡ€ΠΎΡΡ‚ΡŒ Π½Π°ΠΏΡ€Π°Π²Π»Π΅Π½Π½ΠΎΠ³ΠΎ двиТСния зарядов

Когда ΠΌΡ‹ Π²ΠΊΠ»ΡŽΡ‡Π°Π΅ΠΌ Π² ΠΊΠΎΠΌΠ½Π°Ρ‚Π΅ свСт, Π½Π°ΠΌ каТСтся, Ρ‡Ρ‚ΠΎ Π»Π°ΠΌΠΏΠΎΡ‡ΠΊΠ° загораСтся ΠΌΠ³Π½ΠΎΠ²Π΅Π½Π½ΠΎ. Π‘ΠΊΠΎΡ€ΠΎΡΡ‚ΡŒ распространСния Ρ‚ΠΎΠΊΠ° ΠΏΠΎ ΠΏΡ€ΠΎΠ²ΠΎΠ΄Π°ΠΌ ΠΎΡ‡Π΅Π½ΡŒ Π²Π΅Π»ΠΈΠΊΠ°: ΠΎΠ½Π° Π±Π»ΠΈΠ·ΠΊΠ° ΠΊ ΠΊΠΌ/с (скорости свСта Π² Π²Π°ΠΊΡƒΡƒΠΌΠ΅). Если Π±Ρ‹ Π»Π°ΠΌΠΏΠΎΡ‡ΠΊΠ° Π½Π°Ρ…ΠΎΠ΄ΠΈΠ»Π°ΡΡŒ Π½Π° Π›ΡƒΠ½Π΅, ΠΎΠ½Π° заТглась Π±Ρ‹ Ρ‡Π΅Ρ€Π΅Π· сСкунду с нСбольшим.

Однако Π½Π΅ слСдуСт Π΄ΡƒΠΌΠ°Ρ‚ΡŒ, Ρ‡Ρ‚ΠΎ с Ρ‚Π°ΠΊΠΎΠΉ Π³Ρ€Π°Π½Π΄ΠΈΠΎΠ·Π½ΠΎΠΉ ΡΠΊΠΎΡ€ΠΎΡΡ‚ΡŒΡŽ Π΄Π²ΠΈΠ³Π°ΡŽΡ‚ΡΡ свободныС заряды, ΠΎΠ±Ρ€Π°Π·ΡƒΡŽΡ‰ΠΈΠ΅ Ρ‚ΠΎΠΊ. ΠžΠΊΠ°Π·Ρ‹Π²Π°Π΅Ρ‚ΡΡ, ΠΈΡ… ΡΠΊΠΎΡ€ΠΎΡΡ‚ΡŒ составляСт всСго-навсСго Π΄ΠΎΠ»ΠΈ ΠΌΠΈΠ»Π»ΠΈΠΌΠ΅Ρ‚Ρ€Π° Π² сСкунду.

ΠŸΠΎΡ‡Π΅ΠΌΡƒ ΠΆΠ΅ Ρ‚ΠΎΠΊ распространяСтся ΠΏΠΎ ΠΏΡ€ΠΎΠ²ΠΎΠ΄Π°ΠΌ Ρ‚Π°ΠΊ быстро? Π”Π΅Π»ΠΎ Π² Ρ‚ΠΎΠΌ, Ρ‡Ρ‚ΠΎ свободныС заряды Π²Π·Π°ΠΈΠΌΠΎΠ΄Π΅ΠΉΡΡ‚Π²ΡƒΡŽΡ‚ Π΄Ρ€ΡƒΠ³ с Π΄Ρ€ΡƒΠ³ΠΎΠΌ ΠΈ, Π½Π°Ρ…ΠΎΠ΄ΡΡΡŒ ΠΏΠΎΠ΄ дСйствиСм элСктричСского поля источника Ρ‚ΠΎΠΊΠ°, ΠΏΡ€ΠΈ Π·Π°ΠΌΡ‹ΠΊΠ°Π½ΠΈΠΈ Ρ†Π΅ΠΏΠΈ приходят Π² Π΄Π²ΠΈΠΆΠ΅Π½ΠΈΠ΅ ΠΏΠΎΡ‡Ρ‚ΠΈ ΠΎΠ΄Π½ΠΎΠ²Ρ€Π΅ΠΌΠ΅Π½Π½ΠΎ вдоль всСго ΠΏΡ€ΠΎΠ²ΠΎΠ΄Π½ΠΈΠΊΠ°. Π‘ΠΊΠΎΡ€ΠΎΡΡ‚ΡŒ распространСния Ρ‚ΠΎΠΊΠ° Π΅ΡΡ‚ΡŒ ΡΠΊΠΎΡ€ΠΎΡΡ‚ΡŒ ΠΏΠ΅Ρ€Π΅Π΄Π°Ρ‡ΠΈ элСктричСского взаимодСйствия ΠΌΠ΅ΠΆΠ΄Ρƒ свободными зарядами, ΠΈ ΠΎΠ½Π° Π±Π»ΠΈΠ·ΠΊΠ° ΠΊ скорости свСта Π² Π²Π°ΠΊΡƒΡƒΠΌΠ΅. Π‘ΠΊΠΎΡ€ΠΎΡΡ‚ΡŒ ΠΆΠ΅, с ΠΊΠΎΡ‚ΠΎΡ€ΠΎΠΉ сами заряды ΠΏΠ΅Ρ€Π΅ΠΌΠ΅Ρ‰Π°ΡŽΡ‚ΡΡ Π²Π½ΡƒΡ‚Ρ€ΠΈ ΠΏΡ€ΠΎΠ²ΠΎΠ΄Π½ΠΈΠΊΠ°, ΠΌΠΎΠΆΠ΅Ρ‚ Π±Ρ‹Ρ‚ΡŒ Π½Π° ΠΌΠ½ΠΎΠ³ΠΎ порядков мСньшС.

Π˜Ρ‚Π°ΠΊ, ΠΏΠΎΠ΄Ρ‡Π΅Ρ€ΠΊΠ½Ρ‘ΠΌ Π΅Ρ‰Ρ‘ Ρ€Π°Π·, Ρ‡Ρ‚ΠΎ ΠΌΡ‹ Ρ€Π°Π·Π»ΠΈΡ‡Π°Π΅ΠΌ Π΄Π²Π΅ скорости.

1. Π‘ΠΊΠΎΡ€ΠΎΡΡ‚ΡŒ распространСния Ρ‚ΠΎΠΊΠ°. Π­Ρ‚ΠΎ β€” ΡΠΊΠΎΡ€ΠΎΡΡ‚ΡŒ ΠΏΠ΅Ρ€Π΅Π΄Π°Ρ‡ΠΈ элСктричСского сигнала ΠΏΠΎ Ρ†Π΅ΠΏΠΈ. Π‘Π»ΠΈΠ·ΠΊΠ° ΠΊ ΠΊΠΌ/с.

2. Π‘ΠΊΠΎΡ€ΠΎΡΡ‚ΡŒ Π½Π°ΠΏΡ€Π°Π²Π»Π΅Π½Π½ΠΎΠ³ΠΎ двиТСния свободных зарядов. Π­Ρ‚ΠΎ β€” срСдняя ΡΠΊΠΎΡ€ΠΎΡΡ‚ΡŒ пСрСмСщСния зарядов, ΠΎΠ±Ρ€Π°Π·ΡƒΡŽΡ‰ΠΈΡ… Ρ‚ΠΎΠΊ. НазываСтся Π΅Ρ‰Ρ‘ ΡΠΊΠΎΡ€ΠΎΡΡ‚ΡŒΡŽ Π΄Ρ€Π΅ΠΉΡ„Π°.

ΠœΡ‹ сСйчас Π²Ρ‹Π²Π΅Π΄Π΅ΠΌ Ρ„ΠΎΡ€ΠΌΡƒΠ»Ρƒ, Π²Ρ‹Ρ€Π°ΠΆΠ°ΡŽΡ‰ΡƒΡŽ силу Ρ‚ΠΎΠΊΠ° Ρ‡Π΅Ρ€Π΅Π· ΡΠΊΠΎΡ€ΠΎΡΡ‚ΡŒ Π½Π°ΠΏΡ€Π°Π²Π»Π΅Π½Π½ΠΎΠ³ΠΎ двиТСния зарядов ΠΏΡ€ΠΎΠ²ΠΎΠ΄Π½ΠΈΠΊΠ°.

ΠŸΡƒΡΡ‚ΡŒ ΠΏΡ€ΠΎΠ²ΠΎΠ΄Π½ΠΈΠΊ ΠΈΠΌΠ΅Π΅Ρ‚ ΠΏΠ»ΠΎΡ‰Π°Π΄ΡŒ ΠΏΠΎΠΏΠ΅Ρ€Π΅Ρ‡Π½ΠΎΠ³ΠΎ сСчСния (рис. 2). Π‘Π²ΠΎΠ±ΠΎΠ΄Π½Ρ‹Π΅ заряды ΠΏΡ€ΠΎΠ²ΠΎΠ΄Π½ΠΈΠΊΠ° Π±ΡƒΠ΄Π΅ΠΌ ΡΡ‡ΠΈΡ‚Π°Ρ‚ΡŒ ΠΏΠΎΠ»ΠΎΠΆΠΈΡ‚Π΅Π»ΡŒΠ½Ρ‹ΠΌΠΈ; Π²Π΅Π»ΠΈΡ‡ΠΈΠ½Ρƒ свободного заряда ΠΎΠ±ΠΎΠ·Π½Π°Ρ‡ΠΈΠΌ (Π² Π½Π°ΠΈΠ±ΠΎΠ»Π΅Π΅ Π²Π°ΠΆΠ½ΠΎΠΌ для ΠΏΡ€Π°ΠΊΡ‚ΠΈΠΊΠΈ случая мСталличСского ΠΏΡ€ΠΎΠ²ΠΎΠ΄Π½ΠΈΠΊΠ° это Π΅ΡΡ‚ΡŒ заряд элСктрона). ΠšΠΎΠ½Ρ†Π΅Π½Ρ‚Ρ€Π°Ρ†ΠΈΡ свободных зарядов (Ρ‚. Π΅. ΠΈΡ… число Π² Π΅Π΄ΠΈΠ½ΠΈΡ†Π΅ ΠΎΠ±ΡŠΡ‘ΠΌΠ°) Ρ€Π°Π²Π½Π° .

Рис. 2. К Π²Ρ‹Π²ΠΎΠ΄Ρƒ Ρ„ΠΎΡ€ΠΌΡƒΠ»Ρ‹

  1. Какой заряд ΠΏΡ€ΠΎΠΉΠ΄Ρ‘Ρ‚ Ρ‡Π΅Ρ€Π΅Π· ΠΏΠΎΠΏΠ΅Ρ€Π΅Ρ‡Π½ΠΎΠ΅ сСчСниС нашСго ΠΏΡ€ΠΎΠ²ΠΎΠ΄Π½ΠΈΠΊΠ° Π·Π° врСмя ?
  2. Π‘ ΠΎΠ΄Π½ΠΎΠΉ стороны, разумССтся,
  3. (3)
  4. Π‘ Π΄Ρ€ΡƒΠ³ΠΎΠΉ стороны, сСчСниС пСрСсСкут всС Ρ‚Π΅ свободныС заряды, ΠΊΠΎΡ‚ΠΎΡ€Ρ‹Π΅ спустя врСмя окаТутся Π²Π½ΡƒΡ‚Ρ€ΠΈ Ρ†ΠΈΠ»ΠΈΠ½Π΄Ρ€Π° с высотой . Π˜Ρ… число Ρ€Π°Π²Π½ΠΎ:
  5. Π‘Π»Π΅Π΄ΠΎΠ²Π°Ρ‚Π΅Π»ΡŒΠ½ΠΎ, ΠΈΡ… ΠΎΠ±Ρ‰ΠΈΠΉ заряд Π±ΡƒΠ΄Π΅Ρ‚ Ρ€Π°Π²Π΅Π½:
  6. (4)
  7. ΠŸΡ€ΠΈΡ€Π°Π²Π½ΠΈΠ²Π°Ρ ΠΏΡ€Π°Π²Ρ‹Π΅ части Ρ„ΠΎΡ€ΠΌΡƒΠ» (3) ΠΈ (4) ΠΈ сокращая Π½Π° , ΠΏΠΎΠ»ΡƒΡ‡ΠΈΠΌ:
  8. (5)
  9. БоотвСтствСнно, ΠΏΠ»ΠΎΡ‚Π½ΠΎΡΡ‚ΡŒ Ρ‚ΠΎΠΊΠ° оказываСтся Ρ€Π°Π²Π½Π°:
  10. Π”Π°Π²Π°ΠΉΡ‚Π΅ Π² качСствС ΠΏΡ€ΠΈΠΌΠ΅Ρ€Π° посчитаСм, ΠΊΠ°ΠΊΠΎΠ²Π° ΡΠΊΠΎΡ€ΠΎΡΡ‚ΡŒ двиТСния свободных элСктронов Π² ΠΌΠ΅Π΄Π½ΠΎΠΌ ΠΏΡ€ΠΎΠ²ΠΎΠ΄Π΅ ΠΏΡ€ΠΈ силС Ρ‚ΠΎΠΊΠ° A.
  11. Заряд элСктрона извСстСн: Кл.

Π§Π΅ΠΌΡƒ Ρ€Π°Π²Π½Π° концСнтрация свободных элСктронов? Она совпадаСт с ΠΊΠΎΠ½Ρ†Π΅Π½Ρ‚Ρ€Π°Ρ†ΠΈΠ΅ΠΉ Π°Ρ‚ΠΎΠΌΠΎΠ² ΠΌΠ΅Π΄ΠΈ, ΠΏΠΎΡΠΊΠΎΠ»ΡŒΠΊΡƒ ΠΎΡ‚ ΠΊΠ°ΠΆΠ΄ΠΎΠ³ΠΎ Π°Ρ‚ΠΎΠΌΠ° отщСпляСтся ΠΏΠΎ ΠΎΠ΄Π½ΠΎΠΌΡƒ Π²Π°Π»Π΅Π½Ρ‚Π½ΠΎΠΌΡƒ элСктрону. Ну Π° ΠΊΠΎΠ½Ρ†Π΅Π½Ρ‚Ρ€Π°Ρ†ΠΈΡŽ Π°Ρ‚ΠΎΠΌΠΎΠ² ΠΌΡ‹ Π½Π°Ρ…ΠΎΠ΄ΠΈΡ‚ΡŒ ΡƒΠΌΠ΅Π΅ΠΌ:

  • ΠΌ
  • ПолоТим ΠΌΠΌ . Из Ρ„ΠΎΡ€ΠΌΡƒΠ»Ρ‹ (5) ΠΏΠΎΠ»ΡƒΡ‡ΠΈΠΌ:
  • ΠΌ/с.
  • Π­Ρ‚ΠΎ порядка ΠΎΠ΄Π½ΠΎΠΉ дСсятой ΠΌΠΈΠ»Π»ΠΈΠΌΠ΅Ρ‚Ρ€Π° Π² сСкунду.

Π‘Ρ‚Π°Ρ†ΠΈΠΎΠ½Π°Ρ€Π½ΠΎΠ΅ элСктричСскоС ΠΏΠΎΠ»Π΅

ΠœΡ‹ всё врСмя Π³ΠΎΠ²ΠΎΡ€ΠΈΠΌ ΠΎ Π½Π°ΠΏΡ€Π°Π²Π»Π΅Π½Π½ΠΎΠΌ Π΄Π²ΠΈΠΆΠ΅Π½ΠΈΠΈ зарядов, Π½ΠΎ Π΅Ρ‰Ρ‘ Π½Π΅ касались вопроса ΠΎ Ρ‚ΠΎΠΌ, ΠΏΠΎΡ‡Π΅ΠΌΡƒ свободныС заряды ΡΠΎΠ²Π΅Ρ€ΡˆΠ°ΡŽΡ‚ Ρ‚Π°ΠΊΠΎΠ΅ Π΄Π²ΠΈΠΆΠ΅Π½ΠΈΠ΅. ΠŸΠΎΡ‡Π΅ΠΌΡƒ, собствСнно, Π²ΠΎΠ·Π½ΠΈΠΊΠ°Π΅Ρ‚ элСктричСский Ρ‚ΠΎΠΊ?

Для упорядочСнного пСрСмСщСния зарядов Π²Π½ΡƒΡ‚Ρ€ΠΈ ΠΏΡ€ΠΎΠ²ΠΎΠ΄Π½ΠΈΠΊΠ° Π½Π΅ΠΎΠ±Ρ…ΠΎΠ΄ΠΈΠΌΠ° сила, Π΄Π΅ΠΉΡΡ‚Π²ΡƒΡŽΡ‰Π°Ρ Π½Π° заряды Π² ΠΎΠΏΡ€Π΅Π΄Π΅Π»Ρ‘Π½Π½ΠΎΠΌ Π½Π°ΠΏΡ€Π°Π²Π»Π΅Π½ΠΈΠΈ. ΠžΡ‚ΠΊΡƒΠ΄Π° бСрётся эта сила? Π‘ΠΎ стороны элСктричСского поля!

Π§Ρ‚ΠΎΠ±Ρ‹ Π² ΠΏΡ€ΠΎΠ²ΠΎΠ΄Π½ΠΈΠΊΠ΅ ΠΏΡ€ΠΎΡ‚Π΅ΠΊΠ°Π» постоянный Ρ‚ΠΎΠΊ, Π²Π½ΡƒΡ‚Ρ€ΠΈ ΠΏΡ€ΠΎΠ²ΠΎΠ΄Π½ΠΈΠΊΠ° Π΄ΠΎΠ»ΠΆΠ½ΠΎ ΡΡƒΡ‰Π΅ΡΡ‚Π²ΠΎΠ²Π°Ρ‚ΡŒ стационарноС (Ρ‚ΠΎ Π΅ΡΡ‚ΡŒ β€” постоянноС, Π½Π΅ зависящСС ΠΎΡ‚ Π²Ρ€Π΅ΠΌΠ΅Π½ΠΈ) элСктричСскоС ΠΏΠΎΠ»Π΅. Π˜Π½Ρ‹ΠΌΠΈ словами, ΠΌΠ΅ΠΆΠ΄Ρƒ ΠΊΠΎΠ½Ρ†Π°ΠΌΠΈ ΠΏΡ€ΠΎΠ²ΠΎΠ΄Π½ΠΈΠΊΠ° Π½ΡƒΠΆΠ½ΠΎ ΠΏΠΎΠ΄Π΄Π΅Ρ€ΠΆΠΈΠ²Π°Ρ‚ΡŒ ΠΏΠΎΡΡ‚ΠΎΡΠ½Π½ΡƒΡŽ Ρ€Π°Π·Π½ΠΎΡΡ‚ΡŒ ΠΏΠΎΡ‚Π΅Π½Ρ†ΠΈΠ°Π»ΠΎΠ².

Π‘Ρ‚Π°Ρ†ΠΈΠΎΠ½Π°Ρ€Π½ΠΎΠ΅ элСктричСскоС ΠΏΠΎΠ»Π΅ Π΄ΠΎΠ»ΠΆΠ½ΠΎ ΡΠΎΠ·Π΄Π°Π²Π°Ρ‚ΡŒΡΡ зарядами ΠΏΡ€ΠΎΠ²ΠΎΠ΄Π½ΠΈΠΊΠΎΠ², входящих Π² ΡΠ»Π΅ΠΊΡ‚Ρ€ΠΈΡ‡Π΅ΡΠΊΡƒΡŽ Ρ†Π΅ΠΏΡŒ. Однако заряТСнныС ΠΏΡ€ΠΎΠ²ΠΎΠ΄Π½ΠΈΠΊΠΈ сами ΠΏΠΎ сСбС Π½Π΅ смогут ΠΎΠ±Π΅ΡΠΏΠ΅Ρ‡ΠΈΡ‚ΡŒ ΠΏΡ€ΠΎΡ‚Π΅ΠΊΠ°Π½ΠΈΠ΅ постоянного Ρ‚ΠΎΠΊΠ°.

Рассмотрим, ΠΊ ΠΏΡ€ΠΈΠΌΠ΅Ρ€Ρƒ, Π΄Π²Π° проводящих ΡˆΠ°Ρ€Π°, заряТСнных Ρ€Π°Π·Π½ΠΎΠΈΠΌΡ‘Π½Π½ΠΎ. Π‘ΠΎΠ΅Π΄ΠΈΠ½ΠΈΠΌ ΠΈΡ… ΠΏΡ€ΠΎΠ²ΠΎΠ΄ΠΎΠΌ. ΠœΠ΅ΠΆΠ΄Ρƒ ΠΊΠΎΠ½Ρ†Π°ΠΌΠΈ ΠΏΡ€ΠΎΠ²ΠΎΠ΄Π° Π²ΠΎΠ·Π½ΠΈΠΊΠ½Π΅Ρ‚ Ρ€Π°Π·Π½ΠΎΡΡ‚ΡŒ ΠΏΠΎΡ‚Π΅Π½Ρ†ΠΈΠ°Π»ΠΎΠ², Π° Π²Π½ΡƒΡ‚Ρ€ΠΈ ΠΏΡ€ΠΎΠ²ΠΎΠ΄Π° β€” элСктричСскоС ΠΏΠΎΠ»Π΅. По ΠΏΡ€ΠΎΠ²ΠΎΠ΄Ρƒ ΠΏΠΎΡ‚Π΅Ρ‡Ρ‘Ρ‚ Ρ‚ΠΎΠΊ.

Но ΠΏΠΎ ΠΌΠ΅Ρ€Π΅ прохоТдСния Ρ‚ΠΎΠΊΠ° Ρ€Π°Π·Π½ΠΎΡΡ‚ΡŒ ΠΏΠΎΡ‚Π΅Π½Ρ†ΠΈΠ°Π»ΠΎΠ² ΠΌΠ΅ΠΆΠ΄Ρƒ ΡˆΠ°Ρ€Π°ΠΌΠΈ Π±ΡƒΠ΄Π΅Ρ‚ ΡƒΠΌΠ΅Π½ΡŒΡˆΠ°Ρ‚ΡŒΡΡ, вслСд Π·Π° Π½Π΅ΠΉ станСт ΡƒΠ±Ρ‹Π²Π°Ρ‚ΡŒ ΠΈ Π½Π°ΠΏΡ€ΡΠΆΡ‘Π½Π½ΠΎΡΡ‚ΡŒ поля Π² ΠΏΡ€ΠΎΠ²ΠΎΠ΄Π΅. Π’ ΠΊΠΎΠ½Ρ†Π΅ ΠΊΠΎΠ½Ρ†ΠΎΠ² ΠΏΠΎΡ‚Π΅Π½Ρ†ΠΈΠ°Π»Ρ‹ ΡˆΠ°Ρ€ΠΎΠ² станут Ρ€Π°Π²Π½Ρ‹ Π΄Ρ€ΡƒΠ³ Π΄Ρ€ΡƒΠ³Ρƒ, ΠΏΠΎΠ»Π΅ Π² ΠΏΡ€ΠΎΠ²ΠΎΠ΄Π΅ обратится Π² Π½ΡƒΠ»ΡŒ, ΠΈ Ρ‚ΠΎΠΊ исчСзнСт.

ΠœΡ‹ оказались Π² элСктростатикС: ΡˆΠ°Ρ€Ρ‹ плюс ΠΏΡ€ΠΎΠ²ΠΎΠ΄ ΠΎΠ±Ρ€Π°Π·ΡƒΡŽΡ‚ Π΅Π΄ΠΈΠ½Ρ‹ΠΉ ΠΏΡ€ΠΎΠ²ΠΎΠ΄Π½ΠΈΠΊ, Π² ΠΊΠ°ΠΆΠ΄ΠΎΠΉ Ρ‚ΠΎΡ‡ΠΊΠ΅ ΠΊΠΎΡ‚ΠΎΡ€ΠΎΠ³ΠΎ ΠΏΠΎΡ‚Π΅Π½Ρ†ΠΈΠ°Π» ΠΏΡ€ΠΈΠ½ΠΈΠΌΠ°Π΅Ρ‚ ΠΎΠ΄Π½ΠΎ ΠΈ Ρ‚ΠΎ ΠΆΠ΅ Π·Π½Π°Ρ‡Π΅Π½ΠΈΠ΅; Π½Π°ΠΏΡ€ΡΠΆΡ‘Π½Π½ΠΎΡΡ‚ΡŒ
поля Π²Π½ΡƒΡ‚Ρ€ΠΈ ΠΏΡ€ΠΎΠ²ΠΎΠ΄Π½ΠΈΠΊΠ° Ρ€Π°Π²Π½Π° Π½ΡƒΠ»ΡŽ, Π½ΠΈΠΊΠ°ΠΊΠΎΠ³ΠΎ Ρ‚ΠΎΠΊΠ° Π½Π΅Ρ‚.

Π’ΠΎ, Ρ‡Ρ‚ΠΎ элСктростатичСскоС ΠΏΠΎΠ»Π΅ само ΠΏΠΎ сСбС Π½Π΅ годится Π½Π° Ρ€ΠΎΠ»ΡŒ стационарного поля, ΡΠΎΠ·Π΄Π°ΡŽΡ‰Π΅Π³ΠΎ Ρ‚ΠΎΠΊ, ясно ΠΈ ΠΈΠ· Π±ΠΎΠ»Π΅Π΅ ΠΎΠ±Ρ‰ΠΈΡ… сообраТСний. Π’Π΅Π΄ΡŒ элСктростатичСскоС ΠΏΠΎΠ»Π΅ ΠΏΠΎΡ‚Π΅Π½Ρ†ΠΈΠ°Π»ΡŒΠ½ΠΎ, Π΅Π³ΠΎ Ρ€Π°Π±ΠΎΡ‚Π° ΠΏΡ€ΠΈ ΠΏΠ΅Ρ€Π΅ΠΌΠ΅Ρ‰Π΅Π½ΠΈΠΈ заряда ΠΏΠΎ Π·Π°ΠΌΠΊΠ½ΡƒΡ‚ΠΎΠΌΡƒ ΠΏΡƒΡ‚ΠΈ Ρ€Π°Π²Π½Π° Π½ΡƒΠ»ΡŽ. Π‘Π»Π΅Π΄ΠΎΠ²Π°Ρ‚Π΅Π»ΡŒΠ½ΠΎ, ΠΎΠ½ΠΎ Π½Π΅ ΠΌΠΎΠΆΠ΅Ρ‚ Π²Ρ‹Π·Ρ‹Π²Π°Ρ‚ΡŒ Ρ†ΠΈΡ€ΠΊΡƒΠ»ΠΈΡ€ΠΎΠ²Π°Π½ΠΈΠ΅ зарядов ΠΏΠΎ Π·Π°ΠΌΠΊΠ½ΡƒΡ‚ΠΎΠΉ элСктричСской Ρ†Π΅ΠΏΠΈ β€” для этого трСбуСтся ΡΠΎΠ²Π΅Ρ€ΡˆΠ°Ρ‚ΡŒ Π½Π΅Π½ΡƒΠ»Π΅Π²ΡƒΡŽ Ρ€Π°Π±ΠΎΡ‚Ρƒ.

ΠšΡ‚ΠΎ ΠΆΠ΅ Π±ΡƒΠ΄Π΅Ρ‚ ΡΠΎΠ²Π΅Ρ€ΡˆΠ°Ρ‚ΡŒ эту Π½Π΅Π½ΡƒΠ»Π΅Π²ΡƒΡŽ Ρ€Π°Π±ΠΎΡ‚Ρƒ? ΠšΡ‚ΠΎ Π±ΡƒΠ΄Π΅Ρ‚ ΠΏΠΎΠ΄Π΄Π΅Ρ€ΠΆΠΈΠ²Π°Ρ‚ΡŒ Π² Ρ†Π΅ΠΏΠΈ Ρ€Π°Π·Π½ΠΎΡΡ‚ΡŒ ΠΏΠΎΡ‚Π΅Π½Ρ†ΠΈΠ°Π»ΠΎΠ² ΠΈ ΠΎΠ±Π΅ΡΠΏΠ΅Ρ‡ΠΈΠ²Π°Ρ‚ΡŒ стационарноС элСктричСскоС ΠΏΠΎΠ»Π΅, ΡΠΎΠ·Π΄Π°ΡŽΡ‰Π΅Π΅ Ρ‚ΠΎΠΊ Π² ΠΏΡ€ΠΎΠ²ΠΎΠ΄Π½ΠΈΠΊΠ°Ρ…?

ΠžΡ‚Π²Π΅Ρ‚ β€” источник Ρ‚ΠΎΠΊΠ°, ваТнСйший элСмСнт элСктричСской Ρ†Π΅ΠΏΠΈ.

Π§Ρ‚ΠΎΠ±Ρ‹ Π² ΠΏΡ€ΠΎΠ²ΠΎΠ΄Π½ΠΈΠΊΠ΅ ΠΏΡ€ΠΎΡ‚Π΅ΠΊΠ°Π» постоянный Ρ‚ΠΎΠΊ, ΠΊΠΎΠ½Ρ†Ρ‹ ΠΏΡ€ΠΎΠ²ΠΎΠ΄Π½ΠΈΠΊΠ° Π΄ΠΎΠ»ΠΆΠ½Ρ‹ Π±Ρ‹Ρ‚ΡŒ присоСдинСны ΠΊ ΠΊΠ»Π΅ΠΌΠΌΠ°ΠΌ источника Ρ‚ΠΎΠΊΠ° (Π±Π°Ρ‚Π°Ρ€Π΅ΠΉΠΊΠΈ, аккумулятора ΠΈ Ρ‚. Π΄.).

ΠšΠ»Π΅ΠΌΠΌΡ‹ источника β€” это заряТСнныС ΠΏΡ€ΠΎΠ²ΠΎΠ΄Π½ΠΈΠΊΠΈ. Если Ρ†Π΅ΠΏΡŒ Π·Π°ΠΌΠΊΠ½ΡƒΡ‚Π°, Ρ‚ΠΎ заряды с ΠΊΠ»Π΅ΠΌΠΌ ΠΏΠ΅Ρ€Π΅ΠΌΠ΅Ρ‰Π°ΡŽΡ‚ΡΡ ΠΏΠΎ Ρ†Π΅ΠΏΠΈ β€” ΠΊΠ°ΠΊ Π² рассмотрСнном Π²Ρ‹ΡˆΠ΅ ΠΏΡ€ΠΈΠΌΠ΅Ρ€Π΅ с ΡˆΠ°Ρ€Π°ΠΌΠΈ. Но Ρ‚Π΅ΠΏΠ΅Ρ€ΡŒ Ρ€Π°Π·Π½ΠΎΡΡ‚ΡŒ ΠΏΠΎΡ‚Π΅Π½Ρ†ΠΈΠ°Π»ΠΎΠ² ΠΌΠ΅ΠΆΠ΄Ρƒ ΠΊΠ»Π΅ΠΌΠΌΠ°ΠΌΠΈ Π½Π΅ ΡƒΠΌΠ΅Π½ΡŒΡˆΠ°Π΅Ρ‚ΡΡ: источник Ρ‚ΠΎΠΊΠ° Π½Π΅ΠΏΡ€Π΅Ρ€Ρ‹Π²Π½ΠΎ восполняСт заряды Π½Π° ΠΊΠ»Π΅ΠΌΠΌΠ°Ρ…, поддСрТивая Ρ€Π°Π·Π½ΠΎΡΡ‚ΡŒ ΠΏΠΎΡ‚Π΅Π½Ρ†ΠΈΠ°Π»ΠΎΠ² ΠΌΠ΅ΠΆΠ΄Ρƒ ΠΊΠΎΠ½Ρ†Π°ΠΌΠΈ Ρ†Π΅ΠΏΠΈ Π½Π° Π½Π΅ΠΈΠ·ΠΌΠ΅Π½Π½ΠΎΠΌ ΡƒΡ€ΠΎΠ²Π½Π΅.

Π’ этом ΠΈ состоит ΠΏΡ€Π΅Π΄Π½Π°Π·Π½Π°Ρ‡Π΅Π½ΠΈΠ΅ источника постоянного Ρ‚ΠΎΠΊΠ°. Π’Π½ΡƒΡ‚Ρ€ΠΈ Π½Π΅Π³ΠΎ ΠΏΡ€ΠΎΡ‚Π΅ΠΊΠ°ΡŽΡ‚ процСссы нСэлСктричСского (Ρ‡Π°Ρ‰Π΅ всСго β€” химичСского) происхоТдСния, ΠΊΠΎΡ‚ΠΎΡ€Ρ‹Π΅ ΠΎΠ±Π΅ΡΠΏΠ΅Ρ‡ΠΈΠ²Π°ΡŽΡ‚ Π½Π΅ΠΏΡ€Π΅Ρ€Ρ‹Π²Π½ΠΎΠ΅ Ρ€Π°Π·Π΄Π΅Π»Π΅Π½ΠΈΠ΅ зарядов. Π­Ρ‚ΠΈ заряды ΠΏΠΎΡΡ‚Π°Π²Π»ΡΡŽΡ‚ΡΡ Π½Π° ΠΊΠ»Π΅ΠΌΠΌΡ‹ источника Π² Π½Π΅ΠΎΠ±Ρ…ΠΎΠ΄ΠΈΠΌΠΎΠΌ количСствС.

ΠšΠΎΠ»ΠΈΡ‡Π΅ΡΡ‚Π²Π΅Π½Π½ΡƒΡŽ характСристику нСэлСктричСских процСссов раздСлСния зарядов Π²Π½ΡƒΡ‚Ρ€ΠΈ источника β€” Ρ‚Π°ΠΊ Π½Π°Π·Ρ‹Π²Π°Π΅ΠΌΡƒΡŽ Π­Π”Π‘ β€” ΠΌΡ‹ ΠΈΠ·ΡƒΡ‡ΠΈΠΌ ΠΏΠΎΠ·ΠΆΠ΅, Π² ΡΠΎΠΎΡ‚Π²Π΅Ρ‚ΡΡ‚Π²ΡƒΡŽΡ‰Π΅ΠΌ листкС.

А сСйчас вСрнёмся ΠΊ стационарному элСктричСскому полю. Каким ΠΆΠ΅ ΠΎΠ±Ρ€Π°Π·ΠΎΠΌ ΠΎΠ½ΠΎ Π²ΠΎΠ·Π½ΠΈΠΊΠ°Π΅Ρ‚ Π² ΠΏΡ€ΠΎΠ²ΠΎΠ΄Π½ΠΈΠΊΠ°Ρ… Ρ†Π΅ΠΏΠΈ ΠΏΡ€ΠΈ Π½Π°Π»ΠΈΡ‡ΠΈΠΈ источника Ρ‚ΠΎΠΊΠ°?

ЗаряТСнныС ΠΊΠ»Π΅ΠΌΠΌΡ‹ источника ΡΠΎΠ·Π΄Π°ΡŽΡ‚ Π½Π° ΠΊΠΎΠ½Ρ†Π°Ρ… ΠΏΡ€ΠΎΠ²ΠΎΠ΄Π½ΠΈΠΊΠ° элСктричСскоС ΠΏΠΎΠ»Π΅. Π‘Π²ΠΎΠ±ΠΎΠ΄Π½Ρ‹Π΅ заряды ΠΏΡ€ΠΎΠ²ΠΎΠ΄Π½ΠΈΠΊΠ°, находящиСся Π²Π±Π»ΠΈΠ·ΠΈ ΠΊΠ»Π΅ΠΌΠΌ, приходят Π² Π΄Π²ΠΈΠΆΠ΅Π½ΠΈΠ΅ ΠΈ Π΄Π΅ΠΉΡΡ‚Π²ΡƒΡŽΡ‚ своим элСктричСским ΠΏΠΎΠ»Π΅ΠΌ Π½Π° сосСдниС заряды.

Π‘ΠΎ ΡΠΊΠΎΡ€ΠΎΡΡ‚ΡŒΡŽ, Π±Π»ΠΈΠ·ΠΊΠΎΠΉ ΠΊ скорости свСта, это взаимодСйствиС пСрСдаётся вдоль всСй Ρ†Π΅ΠΏΠΈ, ΠΈ Π² Ρ†Π΅ΠΏΠΈ устанавливаСтся постоянный элСктричСский Ρ‚ΠΎΠΊ. БтабилизируСтся ΠΈ элСктричСскоС ΠΏΠΎΠ»Π΅, создаваСмоС двиТущимися зарядами.

Π‘Ρ‚Π°Ρ†ΠΈΠΎΠ½Π°Ρ€Π½ΠΎΠ΅ элСктричСскоС ΠΏΠΎΠ»Π΅ β€” это ΠΏΠΎΠ»Π΅ свободных зарядов ΠΏΡ€ΠΎΠ²ΠΎΠ΄Π½ΠΈΠΊΠ°, ΡΠΎΠ²Π΅Ρ€ΡˆΠ°ΡŽΡ‰ΠΈΡ… Π½Π°ΠΏΡ€Π°Π²Π»Π΅Π½Π½ΠΎΠ΅ Π΄Π²ΠΈΠΆΠ΅Π½ΠΈΠ΅.

Π‘Ρ‚Π°Ρ†ΠΈΠΎΠ½Π°Ρ€Π½ΠΎΠ΅ элСктричСскоС ΠΏΠΎΠ»Π΅ Π½Π΅ мСняСтся со Π²Ρ€Π΅ΠΌΠ΅Π½Π΅ΠΌ ΠΏΠΎΡ‚ΠΎΠΌΡƒ, Ρ‡Ρ‚ΠΎ ΠΏΡ€ΠΈ постоянном Ρ‚ΠΎΠΊΠ΅ Π½Π΅ мСняСтся ΠΊΠ°Ρ€Ρ‚ΠΈΠ½Π° распрСдСлСния зарядов Π² ΠΏΡ€ΠΎΠ²ΠΎΠ΄Π½ΠΈΠΊΠ΅: Π½Π° мСсто заряда, ΠΏΠΎΠΊΠΈΠ½ΡƒΠ²ΡˆΠ΅Π³ΠΎ Π΄Π°Π½Π½Ρ‹ΠΉ участок ΠΏΡ€ΠΎΠ²ΠΎΠ΄Π½ΠΈΠΊΠ°, Π² ΡΠ»Π΅Π΄ΡƒΡŽΡ‰ΠΈΠΉ ΠΌΠΎΠΌΠ΅Π½Ρ‚ Π²Ρ€Π΅ΠΌΠ΅Π½ΠΈ поступаСт Ρ‚ΠΎΡ‡Π½ΠΎ Ρ‚Π°ΠΊΠΎΠΉ ΠΆΠ΅ заряд. По этой ΠΏΡ€ΠΈΡ‡ΠΈΠ½Π΅ стационарноС ΠΏΠΎΠ»Π΅ Π²ΠΎ ΠΌΠ½ΠΎΠ³ΠΎΠΌ (Π½ΠΎ Π½Π΅ Π²ΠΎ всём) Π°Π½Π°Π»ΠΎΠ³ΠΈΡ‡Π½ΠΎ полю элСктростатичСскому.

А ΠΈΠΌΠ΅Π½Π½ΠΎ, справСдливы ΡΠ»Π΅Π΄ΡƒΡŽΡ‰ΠΈΠ΅ Π΄Π²Π° утвСрТдСния, ΠΊΠΎΡ‚ΠΎΡ€Ρ‹Π΅ понадобятся Π½Π°ΠΌ Π² дальнСйшСм (ΠΈΡ… Π΄ΠΎΠΊΠ°Π·Π°Ρ‚Π΅Π»ΡŒΡΡ‚Π²ΠΎ даётся Π² вузовском курсС Ρ„ΠΈΠ·ΠΈΠΊΠΈ).

1. Как ΠΈ элСктростатичСскоС ΠΏΠΎΠ»Π΅, стационарноС элСктричСскоС ΠΏΠΎΠ»Π΅ ΠΏΠΎΡ‚Π΅Π½Ρ†ΠΈΠ°Π»ΡŒΠ½ΠΎ. Π­Ρ‚ΠΎ позволяСт Π³ΠΎΠ²ΠΎΡ€ΠΈΡ‚ΡŒ ΠΎ разности ΠΏΠΎΡ‚Π΅Π½Ρ†ΠΈΠ°Π»ΠΎΠ² (Ρ‚. Π΅. напряТСнии) Π½Π° любом участкС Ρ†Π΅ΠΏΠΈ (ΠΈΠΌΠ΅Π½Π½ΠΎ эту Ρ€Π°Π·Π½ΠΎΡΡ‚ΡŒ ΠΏΠΎΡ‚Π΅Π½Ρ†ΠΈΠ°Π»ΠΎΠ² ΠΌΡ‹ измСряСм Π²ΠΎΠ»ΡŒΡ‚ΠΌΠ΅Ρ‚Ρ€ΠΎΠΌ).

ΠŸΠΎΡ‚Π΅Π½Ρ†ΠΈΠ°Π»ΡŒΠ½ΠΎΡΡ‚ΡŒ, Π½Π°ΠΏΠΎΠΌΠ½ΠΈΠΌ, ΠΎΠ·Π½Π°Ρ‡Π°Π΅Ρ‚, Ρ‡Ρ‚ΠΎ Ρ€Π°Π±ΠΎΡ‚Π° стационарного поля ΠΏΠΎ ΠΏΠ΅Ρ€Π΅ΠΌΠ΅Ρ‰Π΅Π½ΠΈΡŽ заряда Π½Π΅ зависит ΠΎΡ‚ Ρ„ΠΎΡ€ΠΌΡ‹ Ρ‚Ρ€Π°Π΅ΠΊΡ‚ΠΎΡ€ΠΈΠΈ.

ИмСнно поэтому ΠΏΡ€ΠΈ ΠΏΠ°Ρ€Π°Π»Π»Π΅Π»ΡŒΠ½ΠΎΠΌ соСдинСнии ΠΏΡ€ΠΎΠ²ΠΎΠ΄Π½ΠΈΠΊΠΎΠ² напряТСниС Π½Π° ΠΊΠ°ΠΆΠ΄ΠΎΠΌ ΠΈΠ· Π½ΠΈΡ… ΠΎΠ΄ΠΈΠ½Π°ΠΊΠΎΠ²ΠΎ: ΠΎΠ½ΠΎ Ρ€Π°Π²Π½ΠΎ разности ΠΏΠΎΡ‚Π΅Π½Ρ†ΠΈΠ°Π»ΠΎΠ² стационарного поля ΠΌΠ΅ΠΆΠ΄Ρƒ Ρ‚Π΅ΠΌΠΈ двумя Ρ‚ΠΎΡ‡ΠΊΠ°ΠΌΠΈ, ΠΊ ΠΊΠΎΡ‚ΠΎΡ€Ρ‹ΠΌ ΠΏΠΎΠ΄ΠΊΠ»ΡŽΡ‡Π΅Π½Ρ‹ ΠΏΡ€ΠΎΠ²ΠΎΠ΄Π½ΠΈΠΊΠΈ.
2.

Π’ ΠΎΡ‚Π»ΠΈΡ‡ΠΈΠ΅ ΠΎΡ‚ элСктростатичСского поля, стационарноС ΠΏΠΎΠ»Π΅ двиТущихся зарядов ΠΏΡ€ΠΎΠ½ΠΈΠΊΠ°Π΅Ρ‚ Π²Π½ΡƒΡ‚Ρ€ΡŒ ΠΏΡ€ΠΎΠ²ΠΎΠ΄Π½ΠΈΠΊΠ° (Π΄Π΅Π»ΠΎ Π² Ρ‚ΠΎΠΌ, Ρ‡Ρ‚ΠΎ свободныС заряды, участвуя Π² Π½Π°ΠΏΡ€Π°Π²Π»Π΅Π½Π½ΠΎΠΌ Π΄Π²ΠΈΠΆΠ΅Π½ΠΈΠΈ, Π½Π΅ ΡƒΡΠΏΠ΅Π²Π°ΡŽΡ‚ Π΄ΠΎΠ»ΠΆΠ½Ρ‹ΠΌ ΠΎΠ±Ρ€Π°Π·ΠΎΠΌ ΠΏΠ΅Ρ€Π΅ΡΡ‚Ρ€Π°ΠΈΠ²Π°Ρ‚ΡŒΡΡ ΠΈ ΠΏΡ€ΠΈΠ½ΠΈΠΌΠ°Ρ‚ΡŒ «элСктростатичСскиС» ΠΊΠΎΠ½Ρ„ΠΈΠ³ΡƒΡ€Π°Ρ†ΠΈΠΈ).

Π›ΠΈΠ½ΠΈΠΈ напряТённости стационарного поля Π²Π½ΡƒΡ‚Ρ€ΠΈ ΠΏΡ€ΠΎΠ²ΠΎΠ΄Π½ΠΈΠΊΠ° ΠΏΠ°Ρ€Π°Π»Π»Π΅Π»ΡŒΠ½Ρ‹ Π΅Π³ΠΎ повСрхности, ΠΊΠ°ΠΊ Π±Ρ‹ Π½ΠΈ изгибался ΠΏΡ€ΠΎΠ²ΠΎΠ΄Π½ΠΈΠΊ. ΠŸΠΎΡΡ‚ΠΎΠΌΡƒ, ΠΊΠ°ΠΊ ΠΈ Π² ΠΎΠ΄Π½ΠΎΡ€ΠΎΠ΄Π½ΠΎΠΌ элСктростатичСском ΠΏΠΎΠ»Π΅, справСдлива Ρ„ΠΎΡ€ΠΌΡƒΠ»Π° , Π³Π΄Π΅ β€” напряТСниС Π½Π° ΠΊΠΎΠ½Ρ†Π°Ρ… ΠΏΡ€ΠΎΠ²ΠΎΠ΄Π½ΠΈΠΊΠ°, β€” Π½Π°ΠΏΡ€ΡΠΆΡ‘Π½Π½ΠΎΡΡ‚ΡŒ стационарного поля Π² ΠΏΡ€ΠΎΠ²ΠΎΠ΄Π½ΠΈΠΊΠ΅, β€” Π΄Π»ΠΈΠ½Π° ΠΏΡ€ΠΎΠ²ΠΎΠ΄Π½ΠΈΠΊΠ°.

Π˜ΡΡ‚ΠΎΡ‡Π½ΠΈΠΊ: https://ege-study.ru/ru/ege/materialy/fizika/postoyannyj-elektricheskij-tok/

ЭлСктричСскоС напряТСниС β€” это… Π§Ρ‚ΠΎ Ρ‚Π°ΠΊΠΎΠ΅ ЭлСктричСскоС напряТСниС?

ЭлСктри́чСскоС напряТС́ниС ΠΌΠ΅ΠΆΠ΄Ρƒ Ρ‚ΠΎΡ‡ΠΊΠ°ΠΌΠΈ A ΠΈ B элСктричСской Ρ†Π΅ΠΏΠΈ ΠΈΠ»ΠΈ элСктричСского поля β€” физичСская Π²Π΅Π»ΠΈΡ‡ΠΈΠ½Π°, Π·Π½Π°Ρ‡Π΅Π½ΠΈΠ΅ ΠΊΠΎΡ‚ΠΎΡ€ΠΎΠΉ Ρ€Π°Π²Π½ΠΎ ΠΎΡ‚Π½ΠΎΡˆΠ΅Π½ΠΈΡŽ Ρ€Π°Π±ΠΎΡ‚Ρ‹ элСктричСского поля, ΡΠΎΠ²Π΅Ρ€ΡˆΠ°Π΅ΠΌΠΎΠΉ ΠΏΡ€ΠΈ пСрСносС ΠΏΡ€ΠΎΠ±Π½ΠΎΠ³ΠΎ элСктричСского заряда ΠΈΠ· Ρ‚ΠΎΡ‡ΠΊΠΈ A Π² Ρ‚ΠΎΡ‡ΠΊΡƒ B, ΠΊ Π²Π΅Π»ΠΈΡ‡ΠΈΠ½Π΅ ΠΏΡ€ΠΎΠ±Π½ΠΎΠ³ΠΎ заряда.

ΠŸΡ€ΠΈ этом считаСтся, Ρ‡Ρ‚ΠΎ пСрСнос ΠΏΡ€ΠΎΠ±Π½ΠΎΠ³ΠΎ заряда Π½Π΅ измСняСт распрСдСлСния зарядов Π½Π° источниках поля (ΠΏΠΎ ΠΎΠΏΡ€Π΅Π΄Π΅Π»Π΅Π½ΠΈΡŽ ΠΏΡ€ΠΎΠ±Π½ΠΎΠ³ΠΎ заряда).

Π’ ΠΏΠΎΡ‚Π΅Π½Ρ†ΠΈΠ°Π»ΡŒΠ½ΠΎΠΌ элСктричСском ΠΏΠΎΠ»Π΅ эта Ρ€Π°Π±ΠΎΡ‚Π° Π½Π΅ зависит ΠΎΡ‚ ΠΏΡƒΡ‚ΠΈ, ΠΏΠΎ ΠΊΠΎΡ‚ΠΎΡ€ΠΎΠΌΡƒ пСрСмСщаСтся заряд. Π’ этом случаС элСктричСскоС напряТСниС ΠΌΠ΅ΠΆΠ΄Ρƒ двумя Ρ‚ΠΎΡ‡ΠΊΠ°ΠΌΠΈ совпадаСт с Ρ€Π°Π·Π½ΠΎΡΡ‚ΡŒΡŽ ΠΏΠΎΡ‚Π΅Π½Ρ†ΠΈΠ°Π»ΠΎΠ² ΠΌΠ΅ΠΆΠ΄Ρƒ Π½ΠΈΠΌΠΈ.

β€” ΠΈΠ½Ρ‚Π΅Π³Ρ€Π°Π» ΠΎΡ‚ ΠΏΡ€ΠΎΠ΅ΠΊΡ†ΠΈΠΈ поля эффСктивной напряТённости поля (Π²ΠΊΠ»ΡŽΡ‡Π°ΡŽΡ‰Π΅Π³ΠΎ сторонниС поля) Π½Π° расстояниС ΠΌΠ΅ΠΆΠ΄Ρƒ Ρ‚ΠΎΡ‡ΠΊΠ°ΠΌΠΈ A ΠΈ B вдоль Π·Π°Π΄Π°Π½Π½ΠΎΠΉ Ρ‚Ρ€Π°Π΅ΠΊΡ‚ΠΎΡ€ΠΈΠΈ, ΠΈΠ΄ΡƒΡ‰Π΅ΠΉ ΠΈΠ· Ρ‚ΠΎΡ‡ΠΊΠΈ A Π² Ρ‚ΠΎΡ‡ΠΊΡƒ B. Π’ элСктростатичСском ΠΏΠΎΠ»Π΅ Π·Π½Π°Ρ‡Π΅Π½ΠΈΠ΅ этого ΠΈΠ½Ρ‚Π΅Π³Ρ€Π°Π»Π° Π½Π΅ зависит ΠΎΡ‚ ΠΏΡƒΡ‚ΠΈ интСгрирования ΠΈ совпадаСт с Ρ€Π°Π·Π½ΠΎΡΡ‚ΡŒΡŽ ΠΏΠΎΡ‚Π΅Π½Ρ†ΠΈΠ°Π»ΠΎΠ².

Π•Π΄ΠΈΠ½ΠΈΡ†Π΅ΠΉ измСрСния напряТСния Π² систСмС БИ являСтся Π²ΠΎΠ»ΡŒΡ‚.

НапряТСниС Π² цСпях постоянного Ρ‚ΠΎΠΊΠ°

НапряТСниС Π² Ρ†Π΅ΠΏΠΈ постоянного Ρ‚ΠΎΠΊΠ° опрСдСляСтся Ρ‚Π°ΠΊ ΠΆΠ΅, ΠΊΠ°ΠΊ ΠΈ Π² элСктростатикС.

НапряТСниС Π² цСпях ΠΏΠ΅Ρ€Π΅ΠΌΠ΅Π½Π½ΠΎΠ³ΠΎ Ρ‚ΠΎΠΊΠ°

Для описания Ρ†Π΅ΠΏΠ΅ΠΉ ΠΏΠ΅Ρ€Π΅ΠΌΠ΅Π½Π½ΠΎΠ³ΠΎ Ρ‚ΠΎΠΊΠ° ΠΏΡ€ΠΈΠΌΠ΅Π½ΡΡŽΡ‚ΡΡ ΡΠ»Π΅Π΄ΡƒΡŽΡ‰ΠΈΠ΅ понятия:

МгновСнноС напряТСниС

МгновСнноС напряТСниС Π΅ΡΡ‚ΡŒ Ρ€Π°Π·Π½ΠΎΡΡ‚ΡŒ ΠΏΠΎΡ‚Π΅Π½Ρ†ΠΈΠ°Π»ΠΎΠ² ΠΌΠ΅ΠΆΠ΄Ρƒ двумя Ρ‚ΠΎΡ‡ΠΊΠ°ΠΌΠΈ, измСрСнная Π² Π΄Π°Π½Π½Ρ‹ΠΉ ΠΌΠΎΠΌΠ΅Π½Ρ‚ Π²Ρ€Π΅ΠΌΠ΅Π½ΠΈ. Оно являСтся Ρ„ΡƒΠ½ΠΊΡ†ΠΈΠ΅ΠΉ Π²Ρ€Π΅ΠΌΠ΅Π½ΠΈ:

АмплитудноС Π·Π½Π°Ρ‡Π΅Π½ΠΈΠ΅ напряТСния

Амплитуда напряТСния Π΅ΡΡ‚ΡŒ максимальноС ΠΏΠΎ ΠΌΠΎΠ΄ΡƒΠ»ΡŽ Π·Π½Π°Ρ‡Π΅Π½ΠΈΠ΅ ΠΌΠ³Π½ΠΎΠ²Π΅Π½Π½ΠΎΠ³ΠΎ напряТСния Π·Π° вСсь ΠΏΠ΅Ρ€ΠΈΠΎΠ΄ ΠΊΠΎΠ»Π΅Π±Π°Π½ΠΈΠΉ:

Для гармоничСских (ΡΠΈΠ½ΡƒΡΠΎΠΈΠ΄Π°Π»ΡŒΠ½Ρ‹Ρ…) ΠΊΠΎΠ»Π΅Π±Π°Π½ΠΈΠΉ напряТСния ΠΌΠ³Π½ΠΎΠ²Π΅Π½Π½ΠΎΠ΅ Π·Π½Π°Ρ‡Π΅Π½ΠΈΠ΅ напряТСния выраТаСтся ΠΊΠ°ΠΊ:

Для сСти ΠΏΠ΅Ρ€Π΅ΠΌΠ΅Π½Π½ΠΎΠ³ΠΎ ΡΠΈΠ½ΡƒΡΠΎΠΈΠ΄Π°Π»ΡŒΠ½ΠΎΠ³ΠΎ напряТСния со срСднСквадратичным Π·Π½Π°Ρ‡Π΅Π½ΠΈΠ΅ΠΌ 220 Π’ Π°ΠΌΠΏΠ»ΠΈΡ‚ΡƒΠ΄Π½ΠΎΠ΅ Ρ€Π°Π²Π½ΠΎ ΠΏΡ€ΠΈΠ±Π»ΠΈΠ·ΠΈΡ‚Π΅Π»ΡŒΠ½ΠΎ 311,127 Π’.

АмплитудноС напряТСниС ΠΌΠΎΠΆΠ½ΠΎ ΠΈΠ·ΠΌΠ΅Ρ€ΠΈΡ‚ΡŒ с ΠΏΠΎΠΌΠΎΡ‰ΡŒΡŽ осциллографа.

Π‘Ρ€Π΅Π΄Π½Π΅Π΅ Π·Π½Π°Ρ‡Π΅Π½ΠΈΠ΅ напряТСния

Π‘Ρ€Π΅Π΄Π½Π΅Π΅ Π·Π½Π°Ρ‡Π΅Π½ΠΈΠ΅ напряТСния (постоянная ΡΠΎΡΡ‚Π°Π²Π»ΡΡŽΡ‰Π°Ρ напряТСния) опрСдСляСтся Π·Π° вСсь ΠΏΠ΅Ρ€ΠΈΠΎΠ΄ ΠΊΠΎΠ»Π΅Π±Π°Π½ΠΈΠΉ, ΠΊΠ°ΠΊ:

Для чистой синусоиды срСднСС Π·Π½Π°Ρ‡Π΅Π½ΠΈΠ΅ напряТСния Ρ€Π°Π²Π½ΠΎ Π½ΡƒΠ»ΡŽ.

Π‘Ρ€Π΅Π΄Π½Π΅ΠΊΠ²Π°Π΄Ρ€Π°Ρ‚ΠΈΡ‡Π½ΠΎΠ΅ Π·Π½Π°Ρ‡Π΅Π½ΠΈΠ΅ напряТСния

Π‘Ρ€Π΅Π΄Π½Π΅ΠΊΠ²Π°Π΄Ρ€Π°Ρ‚ΠΈΡ‡Π½ΠΎΠ΅ Π·Π½Π°Ρ‡Π΅Π½ΠΈΠ΅ (ΡƒΡΡ‚Π°Ρ€Π΅Π²ΡˆΠ΅Π΅ Π½Π°ΠΈΠΌΠ΅Π½ΠΎΠ²Π°Π½ΠΈΠ΅: Π΄Π΅ΠΉΡΡ‚Π²ΡƒΡŽΡ‰Π΅Π΅, эффСктивноС) Π½Π°ΠΈΠ±ΠΎΠ»Π΅Π΅ ΡƒΠ΄ΠΎΠ±Π½ΠΎ для практичСских расчётов, Ρ‚Π°ΠΊ ΠΊΠ°ΠΊ Π½Π° Π»ΠΈΠ½Π΅ΠΉΠ½ΠΎΠΉ Π°ΠΊΡ‚ΠΈΠ²Π½ΠΎΠΉ Π½Π°Π³Ρ€ΡƒΠ·ΠΊΠ΅ ΠΎΠ½ΠΎ ΡΠΎΠ²Π΅Ρ€ΡˆΠ°Π΅Ρ‚ Ρ‚Ρƒ ΠΆΠ΅ Ρ€Π°Π±ΠΎΡ‚Ρƒ (Π½Π°ΠΏΡ€ΠΈΠΌΠ΅Ρ€, Π»Π°ΠΌΠΏΠ° накаливания ΠΈΠΌΠ΅Π΅Ρ‚ Ρ‚Ρƒ ΠΆΠ΅ ΡΡ€ΠΊΠΎΡΡ‚ΡŒ свСчСния, Π½Π°Π³Ρ€Π΅Π²Π°Ρ‚Π΅Π»ΡŒΠ½Ρ‹ΠΉ элСмСнт выдСляСт ΡΡ‚ΠΎΠ»ΡŒΠΊΠΎ ΠΆΠ΅ Ρ‚Π΅ΠΏΠ»Π°), Ρ‡Ρ‚ΠΎ ΠΈ Ρ€Π°Π²Π½ΠΎΠ΅ Π΅ΠΌΡƒ постоянноС напряТСниС:

Для ΡΠΈΠ½ΡƒΡΠΎΠΈΠ΄Π°Π»ΡŒΠ½ΠΎΠ³ΠΎ напряТСния справСдливо равСнство:

Π’ Ρ‚Π΅Ρ…Π½ΠΈΠΊΠ΅ ΠΈ Π±Ρ‹Ρ‚Ρƒ ΠΏΡ€ΠΈ использовании ΠΏΠ΅Ρ€Π΅ΠΌΠ΅Π½Π½ΠΎΠ³ΠΎ Ρ‚ΠΎΠΊΠ° ΠΏΠΎΠ΄ Ρ‚Π΅Ρ€ΠΌΠΈΠ½ΠΎΠΌ «напряТСниС» имССтся Π² Π²ΠΈΠ΄Ρƒ ΠΈΠΌΠ΅Π½Π½ΠΎ эта Π²Π΅Π»ΠΈΡ‡ΠΈΠ½Π°, ΠΈ всС Π²ΠΎΠ»ΡŒΡ‚ΠΌΠ΅Ρ‚Ρ€Ρ‹ ΠΏΡ€ΠΎΠ³Ρ€Π°Π΄ΡƒΠΈΡ€ΠΎΠ²Π°Π½Ρ‹ исходя ΠΈΠ· Π΅Ρ‘ опрСдСлСния.

Однако конструктивно Π±ΠΎΠ»ΡŒΡˆΠΈΠ½ΡΡ‚Π²ΠΎ ΠΏΡ€ΠΈΠ±ΠΎΡ€ΠΎΠ² фактичСски ΠΈΠ·ΠΌΠ΅Ρ€ΡΡŽΡ‚ Π½Π΅ срСднСквадратичноС, Π° срСднСвыпрямлСнноС (см.

Π½ΠΈΠΆΠ΅) Π·Π½Π°Ρ‡Π΅Π½ΠΈΠ΅ напряТСния, поэтому для Π½Π΅ΡΠΈΠ½ΡƒΡΠΎΠΈΠ΄Π°Π»ΡŒΠ½ΠΎΠ³ΠΎ сигнала ΠΈΡ… показания ΠΌΠΎΠ³ΡƒΡ‚ ΠΎΡ‚Π»ΠΈΡ‡Π°Ρ‚ΡŒΡΡ ΠΎΡ‚ истинного значСния.

БрСднСвыпрямлСнноС Π·Π½Π°Ρ‡Π΅Π½ΠΈΠ΅ напряТСния

БрСднСвыпрямлСнноС Π·Π½Π°Ρ‡Π΅Π½ΠΈΠ΅ Π΅ΡΡ‚ΡŒ срСднСС Π·Π½Π°Ρ‡Π΅Π½ΠΈΠ΅ модуля напряТСния:

Для ΡΠΈΠ½ΡƒΡΠΎΠΈΠ΄Π°Π»ΡŒΠ½ΠΎΠ³ΠΎ напряТСния справСдливо равСнство:

На ΠΏΡ€Π°ΠΊΡ‚ΠΈΠΊΠ΅ ΠΈΡΠΏΠΎΠ»ΡŒΠ·ΡƒΠ΅Ρ‚ΡΡ Ρ€Π΅Π΄ΠΊΠΎ, ΠΎΠ΄Π½Π°ΠΊΠΎ Π±ΠΎΠ»ΡŒΡˆΠΈΠ½ΡΡ‚Π²ΠΎ Π²ΠΎΠ»ΡŒΡ‚ΠΌΠ΅Ρ‚Ρ€ΠΎΠ² ΠΏΠ΅Ρ€Π΅ΠΌΠ΅Π½Π½ΠΎΠ³ΠΎ Ρ‚ΠΎΠΊΠ° (Ρ‚Π΅, Π² ΠΊΠΎΡ‚ΠΎΡ€Ρ‹Ρ… Ρ‚ΠΎΠΊ ΠΏΠ΅Ρ€Π΅Π΄ ΠΈΠ·ΠΌΠ΅Ρ€Π΅Π½ΠΈΠ΅ΠΌ выпрямляСтся) фактичСски ΠΈΠ·ΠΌΠ΅Ρ€ΡΡŽΡ‚ ΠΈΠΌΠ΅Π½Π½ΠΎ эту Π²Π΅Π»ΠΈΡ‡ΠΈΠ½Ρƒ, хотя ΠΈΡ… шкала ΠΈ ΠΏΡ€ΠΎΠ³Ρ€Π°Π΄ΡƒΠΈΡ€ΠΎΠ²Π°Π½Π° ΠΏΠΎ срСднСквадратичным значСниям.

НапряТСниС Π² цСпях Ρ‚Ρ€Ρ‘Ρ…Ρ„Π°Π·Π½ΠΎΠ³ΠΎ Ρ‚ΠΎΠΊΠ°

Π’ цСпях Ρ‚Ρ€Ρ‘Ρ…Ρ„Π°Π·Π½ΠΎΠ³ΠΎ Ρ‚ΠΎΠΊΠ° Ρ€Π°Π·Π»ΠΈΡ‡Π°ΡŽΡ‚ Ρ„Π°Π·Π½ΠΎΠ΅ ΠΈ Π»ΠΈΠ½Π΅ΠΉΠ½ΠΎΠ΅ напряТСния.

Под Ρ„Π°Π·Π½Ρ‹ΠΌ напряТСниСм ΠΏΠΎΠ½ΠΈΠΌΠ°ΡŽΡ‚ срСднСквадратичноС Π·Π½Π°Ρ‡Π΅Π½ΠΈΠ΅ напряТСния Π½Π° ΠΊΠ°ΠΆΠ΄ΠΎΠΉ ΠΈΠ· Ρ„Π°Π· Π½Π°Π³Ρ€ΡƒΠ·ΠΊΠΈ, Π° ΠΏΠΎΠ΄ Π»ΠΈΠ½Π΅ΠΉΠ½Ρ‹ΠΌΒ β€” напряТСниС ΠΌΠ΅ΠΆΠ΄Ρƒ подводящими Ρ„Π°Π·Π½Ρ‹ΠΌΠΈ ΠΏΡ€ΠΎΠ²ΠΎΠ΄Π°ΠΌΠΈ.

ΠŸΡ€ΠΈ соСдинСнии Π½Π°Π³Ρ€ΡƒΠ·ΠΊΠΈ Π² Ρ‚Ρ€Π΅ΡƒΠ³ΠΎΠ»ΡŒΠ½ΠΈΠΊ Ρ„Π°Π·Π½ΠΎΠ΅ напряТСниС Ρ€Π°Π²Π½ΠΎ Π»ΠΈΠ½Π΅ΠΉΠ½ΠΎΠΌΡƒ, Π° ΠΏΡ€ΠΈ соСдинСнии Π² Π·Π²Π΅Π·Π΄Ρƒ (ΠΏΡ€ΠΈ симмСтричной Π½Π°Π³Ρ€ΡƒΠ·ΠΊΠ΅ ΠΈΠ»ΠΈ ΠΏΡ€ΠΈ Π³Π»ΡƒΡ…ΠΎΠ·Π°Π·Π΅ΠΌΠ»Ρ‘Π½Π½ΠΎΠΉ Π½Π΅ΠΉΡ‚Ρ€Π°Π»ΠΈ) Π»ΠΈΠ½Π΅ΠΉΠ½ΠΎΠ΅ напряТСниС Π² Ρ€Π°Π· большС Ρ„Π°Π·Π½ΠΎΠ³ΠΎ.

На ΠΏΡ€Π°ΠΊΡ‚ΠΈΠΊΠ΅ напряТСниС Ρ‚Ρ€Ρ‘Ρ…Ρ„Π°Π·Π½ΠΎΠΉ сСти ΠΎΠ±ΠΎΠ·Π½Π°Ρ‡Π°ΡŽΡ‚ Π΄Ρ€ΠΎΠ±ΡŒΡŽ, Π² Π·Π½Π°ΠΌΠ΅Π½Π°Ρ‚Π΅Π»Π΅ ΠΊΠΎΡ‚ΠΎΡ€ΠΎΠΉ стоит Π»ΠΈΠ½Π΅ΠΉΠ½ΠΎΠ΅ напряТСниС, Π° Π² числитСлС — Ρ„Π°Π·Π½ΠΎΠ΅ ΠΏΡ€ΠΈ соСдинСнии Π² Π·Π²Π΅Π·Π΄Ρƒ (ΠΈΠ»ΠΈ, Ρ‡Ρ‚ΠΎ Ρ‚ΠΎ ΠΆΠ΅ самоС, ΠΏΠΎΡ‚Π΅Π½Ρ†ΠΈΠ°Π» ΠΊΠ°ΠΆΠ΄ΠΎΠΉ ΠΈΠ· Π»ΠΈΠ½ΠΈΠΉ ΠΎΡ‚Π½ΠΎΡΠΈΡ‚Π΅Π»ΡŒΠ½ΠΎ Π·Π΅ΠΌΠ»ΠΈ). Π’Π°ΠΊ, Π² России Π½Π°ΠΈΠ±ΠΎΠ»Π΅Π΅ распространСны сСти с напряТСниСм 220/380Β Π’; Ρ‚Π°ΠΊΠΆΠ΅ ΠΈΠ½ΠΎΠ³Π΄Π° ΠΈΡΠΏΠΎΠ»ΡŒΠ·ΡƒΡŽΡ‚ΡΡ сСти 127/220Β Π’ ΠΈ 380/660Β Π’.

Π‘Ρ‚Π°Π½Π΄Π°Ρ€Ρ‚Ρ‹

ΠžΠ±ΡŠΠ΅ΠΊΡ‚
Π’ΠΈΠΏ напряТСния
Π—Π½Π°Ρ‡Π΅Π½ΠΈΠ΅ (Π½Π° Π²Π²ΠΎΠ΄Π΅ потрСбитСля)
Π—Π½Π°Ρ‡Π΅Π½ΠΈΠ΅ (Π½Π° Π²Ρ‹Ρ…ΠΎΠ΄Π΅ источника)
Π­Π»Π΅ΠΊΡ‚Ρ€ΠΎΠΊΠ°Ρ€Π΄ΠΈΠΎΠ³Ρ€Π°ΠΌΠΌΠ°Π˜ΠΌΠΏΡƒΠ»ΡŒΡΠ½ΠΎΠ΅1-2 ΠΌΠ’β€”
ВСлСвизионная Π°Π½Ρ‚Π΅Π½Π½Π°ΠŸΠ΅Ρ€Π΅ΠΌΠ΅Π½Π½ΠΎΠ΅ высокочастотноС1-100 ΠΌΠ’β€”
Π‘Π°Ρ‚Π°Ρ€Π΅ΠΉΠΊΠ° AA (Β«ΠΏΠ°Π»ΡŒΡ‡ΠΈΠΊΠΎΠ²Π°ΡΒ»)ΠŸΠΎΡΡ‚ΠΎΡΠ½Π½ΠΎΠ΅1,5 Π’β€”
ЛитиСвая Π±Π°Ρ‚Π°Ρ€Π΅ΠΉΠΊΠ°ΠŸΠΎΡΡ‚ΠΎΡΠ½Π½ΠΎΠ΅3 Π’ β€” 1,8 Π’ (Π² исполнСнии ΠΏΠ°Π»ΡŒΡ‡ΠΈΠΊΠΎΠ²ΠΎΠΉ Π±Π°Ρ‚Π°Ρ€Π΅ΠΉΠΊΠΈ , Π½Π° ΠΏΡ€ΠΈΠΌΠ΅Ρ€Π΅ Varta Professional Lithium, AA)β€”
Π£ΠΏΡ€Π°Π²Π»ΡΡŽΡ‰ΠΈΠ΅ сигналы ΠΊΠΎΠΌΠΏΡŒΡŽΡ‚Π΅Ρ€Π½Ρ‹Ρ… ΠΊΠΎΠΌΠΏΠΎΠ½Π΅Π½Ρ‚ΠΎΠ²Π˜ΠΌΠΏΡƒΠ»ΡŒΡΠ½ΠΎΠ΅3,5 Π’, 5 Π’β€”
Π‘Π°Ρ‚Π°Ρ€Π΅ΠΉΠΊΠ° Ρ‚ΠΈΠΏΠ° 6F22 (Β«ΠšΡ€ΠΎΠ½Π°Β»)ΠŸΠΎΡΡ‚ΠΎΡΠ½Π½ΠΎΠ΅9 Π’β€”
Π‘ΠΈΠ»ΠΎΠ²ΠΎΠ΅ ΠΏΠΈΡ‚Π°Π½ΠΈΠ΅ ΠΊΠΎΠΌΠΏΡŒΡŽΡ‚Π΅Ρ€Π½Ρ‹Ρ… ΠΊΠΎΠΌΠΏΠΎΠ½Π΅Π½Ρ‚ΠΎΠ²ΠŸΠΎΡΡ‚ΠΎΡΠ½Π½ΠΎΠ΅12 Π’β€”
Π­Π»Π΅ΠΊΡ‚Ρ€ΠΎΠΎΠ±ΠΎΡ€ΡƒΠ΄ΠΎΠ²Π°Π½ΠΈΠ΅ Π°Π²Ρ‚ΠΎΠΌΠΎΠ±ΠΈΠ»ΡΠŸΠΎΡΡ‚ΠΎΡΠ½Π½ΠΎΠ΅12/24 Π’β€”
Π‘Π»ΠΎΠΊ питания Π½ΠΎΡƒΡ‚Π±ΡƒΠΊΠ° ΠΈ ТидкокристалличСских ΠΌΠΎΠ½ΠΈΡ‚ΠΎΡ€ΠΎΠ²ΠŸΠΎΡΡ‚ΠΎΡΠ½Π½ΠΎΠ΅19 Π’β€”
Π‘Π΅Ρ‚ΡŒ «бСзопасного» ΠΏΠΎΠ½ΠΈΠΆΠ΅Π½Π½ΠΎΠ³ΠΎ напряТСния для Ρ€Π°Π±ΠΎΡ‚Ρ‹ Π² опасных ΡƒΡΠ»ΠΎΠ²ΠΈΡΡ…ΠŸΠ΅Ρ€Π΅ΠΌΠ΅Π½Π½ΠΎΠ΅36-42 Π’β€”
НапряТСниС Π½Π°ΠΈΠ±ΠΎΠ»Π΅Π΅ ΡΡ‚Π°Π±ΠΈΠ»ΡŒΠ½ΠΎΠ³ΠΎ горСния свСчи Π―Π±Π»ΠΎΡ‡ΠΊΠΎΠ²Π°ΠŸΠΎΡΡ‚ΠΎΡΠ½Π½ΠΎΠ΅55 Π’β€”
НапряТСниС Π² Ρ‚Π΅Π»Π΅Ρ„ΠΎΠ½Π½ΠΎΠΉ Π»ΠΈΠ½ΠΈΠΈ (ΠΏΡ€ΠΈ ΠΎΠΏΡƒΡ‰Π΅Π½Π½ΠΎΠΉ Ρ‚Ρ€ΡƒΠ±ΠΊΠ΅)ΠŸΠΎΡΡ‚ΠΎΡΠ½Π½ΠΎΠ΅60 Π’β€”
НапряТСниС Π² элСктросСти Π―ΠΏΠΎΠ½ΠΈΠΈΠŸΠ΅Ρ€Π΅ΠΌΠ΅Π½Π½ΠΎΠ΅ Ρ‚Ρ€Ρ‘Ρ…Ρ„Π°Π·Π½ΠΎΠ΅100/172 Π’β€”
НапряТСниС Π² Π΄ΠΎΠΌΠ°ΡˆΠ½ΠΈΡ… элСктросСтях Π‘Π¨ΠΠŸΠ΅Ρ€Π΅ΠΌΠ΅Π½Π½ΠΎΠ΅ Ρ‚Ρ€Ρ‘Ρ…Ρ„Π°Π·Π½ΠΎΠ΅120 Π’ / 240 Π’ (сплит-Ρ„Π°Π·Π°)β€”
НапряТСниС Π² элСктросСти Π ΠΎΡΡΠΈΠΈΠŸΠ΅Ρ€Π΅ΠΌΠ΅Π½Π½ΠΎΠ΅ Ρ‚Ρ€Ρ‘Ρ…Ρ„Π°Π·Π½ΠΎΠ΅220/380 Π’230/400 Π’
Разряд элСктричСского ΡΠΊΠ°Ρ‚Π°ΠŸΠΎΡΡ‚ΠΎΡΠ½Π½ΠΎΠ΅Π΄ΠΎ 200β€”250 Π’β€”
ΠšΠΎΠ½Ρ‚Π°ΠΊΡ‚Π½Π°Ρ ΡΠ΅Ρ‚ΡŒ трамвая ΠΈ Ρ‚Ρ€ΠΎΠ»Π»Π΅ΠΉΠ±ΡƒΡΠ°ΠŸΠΎΡΡ‚ΠΎΡΠ½Π½ΠΎΠ΅550 Π’600 Π’
Разряд элСктричСского ΡƒΠ³Ρ€ΡΠŸΠΎΡΡ‚ΠΎΡΠ½Π½ΠΎΠ΅Π΄ΠΎ 650 Π’β€”
ΠšΠΎΠ½Ρ‚Π°ΠΊΡ‚Π½Π°Ρ ΡΠ΅Ρ‚ΡŒ ΠΌΠ΅Ρ‚Ρ€ΠΎΠΏΠΎΠ»ΠΈΡ‚Π΅Π½Π°ΠŸΠΎΡΡ‚ΠΎΡΠ½Π½ΠΎΠ΅750 Π’825 Π’
ΠšΠΎΠ½Ρ‚Π°ΠΊΡ‚Π½Π°Ρ ΡΠ΅Ρ‚ΡŒ элСктрифицированной ΠΆΠ΅Π»Π΅Π·Π½ΠΎΠΉ Π΄ΠΎΡ€ΠΎΠ³ΠΈ (Россия, постоянный Ρ‚ΠΎΠΊ)ΠŸΠΎΡΡ‚ΠΎΡΠ½Π½ΠΎΠ΅3 ΠΊΠ’3,3 ΠΊΠ’
Π Π°ΡΠΏΡ€Π΅Π΄Π΅Π»ΠΈΡ‚Π΅Π»ΡŒΠ½Π°Ρ Π²ΠΎΠ·Π΄ΡƒΡˆΠ½Π°Ρ линия элСктропСрСдачи нСбольшой ΠΌΠΎΡ‰Π½ΠΎΡΡ‚ΠΈΠŸΠ΅Ρ€Π΅ΠΌΠ΅Π½Π½ΠΎΠ΅ Ρ‚Ρ€Ρ‘Ρ…Ρ„Π°Π·Π½ΠΎΠ΅6-20 ΠΊΠ’6,6-22 ΠΊΠ’
Π“Π΅Π½Π΅Ρ€Π°Ρ‚ΠΎΡ€Ρ‹ элСктростанций, ΠΌΠΎΡ‰Π½Ρ‹Π΅ ΡΠ»Π΅ΠΊΡ‚Ρ€ΠΎΠ΄Π²ΠΈΠ³Π°Ρ‚Π΅Π»ΠΈΠŸΠ΅Ρ€Π΅ΠΌΠ΅Π½Π½ΠΎΠ΅ Ρ‚Ρ€Ρ‘Ρ…Ρ„Π°Π·Π½ΠΎΠ΅10-35 ΠΊΠ’β€”
Анод ΠΊΠΈΠ½Π΅ΡΠΊΠΎΠΏΠ°ΠŸΠΎΡΡ‚ΠΎΡΠ½Π½ΠΎΠ΅7-30 ΠΊΠ’β€”
БтатичСскоС ΡΠ»Π΅ΠΊΡ‚Ρ€ΠΈΡ‡Π΅ΡΡ‚Π²ΠΎΠŸΠΎΡΡ‚ΠΎΡΠ½Π½ΠΎΠ΅1-100 ΠΊΠ’β€”
Π‘Π²Π΅Ρ‡Π° заТигания Π°Π²Ρ‚ΠΎΠΌΠΎΠ±ΠΈΠ»ΡΠ˜ΠΌΠΏΡƒΠ»ΡŒΡΠ½ΠΎΠ΅10-25 ΠΊΠ’β€”
ΠšΠΎΠ½Ρ‚Π°ΠΊΡ‚Π½Π°Ρ ΡΠ΅Ρ‚ΡŒ элСктрифицированной ΠΆΠ΅Π»Π΅Π·Π½ΠΎΠΉ Π΄ΠΎΡ€ΠΎΠ³ΠΈ (Россия, ΠΏΠ΅Ρ€Π΅ΠΌΠ΅Π½Π½Ρ‹ΠΉ Ρ‚ΠΎΠΊ)ΠŸΠ΅Ρ€Π΅ΠΌΠ΅Π½Π½ΠΎΠ΅25 ΠΊΠ’27,5 ΠΊΠ’
ΠŸΡ€ΠΎΠ±ΠΎΠΉ Π²ΠΎΠ·Π΄ΡƒΡ…Π° Π½Π° расстоянии 1 см10-20 ΠΊΠ’β€”
ΠšΠ°Ρ‚ΡƒΡˆΠΊΠ° Π ΡƒΠΌΠΊΠΎΡ€Ρ„Π°Π˜ΠΌΠΏΡƒΠ»ΡŒΡΠ½ΠΎΠ΅Π΄ΠΎ 50 ΠΊΠ’β€”
ΠŸΡ€ΠΎΠ±ΠΎΠΉ трансформаторного масла Π½Π° расстоянии 1 см100-200 ΠΊΠ’β€”
Π’ΠΎΠ·Π΄ΡƒΡˆΠ½Π°Ρ линия элСктропСрСдачи большой ΠΌΠΎΡ‰Π½ΠΎΡΡ‚ΠΈΠŸΠ΅Ρ€Π΅ΠΌΠ΅Π½Π½ΠΎΠ΅ Ρ‚Ρ€Ρ‘Ρ…Ρ„Π°Π·Π½ΠΎΠ΅35 ΠΊΠ’, 110 ΠΊΠ’, 220 ΠΊΠ’, 330 ΠΊΠ’38 ΠΊΠ’, 120 ΠΊΠ’, 240 ΠΊΠ’, 360 ΠΊΠ’
ЭлСктрофорная ΠΌΠ°ΡˆΠΈΠ½Π°ΠŸΠΎΡΡ‚ΠΎΡΠ½Π½ΠΎΠ΅50-500 ΠΊΠ’β€”
Π’ΠΎΠ·Π΄ΡƒΡˆΠ½Π°Ρ линия элСктропСрСдачи свСрхвысокого напряТСния (мСТсистСмныС)ΠŸΠ΅Ρ€Π΅ΠΌΠ΅Π½Π½ΠΎΠ΅ Ρ‚Ρ€Ρ‘Ρ…Ρ„Π°Π·Π½ΠΎΠ΅500 ΠΊΠ’, 750 ΠΊΠ’, 1150 ΠΊΠ’545 ΠΊΠ’, 800 ΠΊΠ’, 1250 ΠΊΠ’
Врансформатор Π’Π΅ΡΠ»Π°Π˜ΠΌΠΏΡƒΠ»ΡŒΡΠ½ΠΎΠ΅ высокочастотноСдо Π½Π΅ΡΠΊΠΎΠ»ΡŒΠΊΠΈΡ… ΠœΠ’β€”
Π“Π΅Π½Π΅Ρ€Π°Ρ‚ΠΎΡ€ Π’Π°Π½ Π΄Π΅ Π“Ρ€Π°Π°Ρ„Π°ΠŸΠΎΡΡ‚ΠΎΡΠ½Π½ΠΎΠ΅Π΄ΠΎ 7 ΠœΠ’β€”
Π“Ρ€ΠΎΠ·ΠΎΠ²ΠΎΠ΅ ΠΎΠ±Π»Π°ΠΊΠΎΠŸΠΎΡΡ‚ΠΎΡΠ½Π½ΠΎΠ΅ΠžΡ‚ 2 Π΄ΠΎ 10 Π“Π’β€”

Π‘ΠΌ. Ρ‚Π°ΠΊΠΆΠ΅

Бсылки

Π˜ΡΡ‚ΠΎΡ‡Π½ΠΈΠΊ: https://dic.academic.ru/dic.nsf/ruwiki/15264

ЭлСктричСство ΠΈ ΠΌΠ°Π³Π½Π΅Ρ‚ΠΈΠ·ΠΌ

Π’Π΅Π»ΠΈΡ‡ΠΈΠ½Π° ΠžΠ±ΠΎΠ·Π½Π°Ρ‡Π΅Π½ΠΈΠ΅ Π•Π΄ΠΈΠ½ΠΈΡ†Π° измСрСния Π² систСмС БИ
Π‘ΠΈΠ»Π° Ρ‚ΠΎΠΊΠ° I Π°ΠΌΠΏΠ΅Ρ€ А
ΠŸΠ»ΠΎΡ‚Π½ΠΎΡΡ‚ΡŒ Ρ‚ΠΎΠΊΠ° j Π°ΠΌΠΏΠ΅Ρ€ Π½Π° ΠΊΠ²Π°Π΄Ρ€Π°Ρ‚Π½Ρ‹ΠΉ ΠΌΠ΅Ρ‚Ρ€ А/ΠΌ2
ЭлСктричСский заряд Q, q ΠΊΡƒΠ»ΠΎΠ½ Кл
ЭлСктричСский Π΄ΠΈΠΏΠΎΠ»ΡŒΠ½Ρ‹ΠΉ ΠΌΠΎΠΌΠ΅Π½Ρ‚ p ΠΊΡƒΠ»ΠΎΠ½-ΠΌΠ΅Ρ‚Ρ€ Кл βˆ™ ΠΌ
ΠŸΠΎΠ»ΡΡ€ΠΈΠ·ΠΎΠ²Π°Π½Π½ΠΎΡΡ‚ΡŒ P ΠΊΡƒΠ»ΠΎΠ½ Π½Π° ΠΊΠ²Π°Π΄Ρ€Π°Ρ‚Π½Ρ‹ΠΉ ΠΌΠ΅Ρ‚Ρ€ Кл/ΠΌ2
НапряТСниС, ΠΏΠΎΡ‚Π΅Π½Ρ†ΠΈΠ°Π», Π­Π”Π‘ U, Ο†, Ξ΅ Π²ΠΎΠ»ΡŒΡ‚ Π’
ΠΠ°ΠΏΡ€ΡΠΆΠ΅Π½Π½ΠΎΡΡ‚ΡŒ элСктричСского поля E Π²ΠΎΠ»ΡŒΡ‚ Π½Π° ΠΌΠ΅Ρ‚Ρ€ Π’/ΠΌ
ЭлСктричСская Π΅ΠΌΠΊΠΎΡΡ‚ΡŒ C Ρ„Π°Ρ€Π°Π΄ Π€
ЭлСктричСскоС сопротивлСниС R, r ΠΎΠΌ Ом
УдСльноС элСктричСскоС сопротивлСниС ρ ΠΎΠΌ-ΠΌΠ΅Ρ‚Ρ€ Ом βˆ™ ΠΌ
ЭлСктричСская ΠΏΡ€ΠΎΠ²ΠΎΠ΄ΠΈΠΌΠΎΡΡ‚ΡŒ G симСнс Π‘ΠΌ
ΠœΠ°Π³Π½ΠΈΡ‚Π½Π°Ρ индукция B тСсла Π’Π»
ΠœΠ°Π³Π½ΠΈΡ‚Π½Ρ‹ΠΉ ΠΏΠΎΡ‚ΠΎΠΊ Π€ Π²Π΅Π±Π΅Ρ€ Π’Π±
ΠΠ°ΠΏΡ€ΡΠΆΠ΅Π½Π½ΠΎΡΡ‚ΡŒ ΠΌΠ°Π³Π½ΠΈΡ‚Π½ΠΎΠ³ΠΎ поля H Π°ΠΌΠΏΠ΅Ρ€ Π½Π° ΠΌΠ΅Ρ‚Ρ€ А/ΠΌ
ΠœΠ°Π³Π½ΠΈΡ‚Π½Ρ‹ΠΉ ΠΌΠΎΠΌΠ΅Π½Ρ‚ pm Π°ΠΌΠΏΠ΅Ρ€-ΠΊΠ²Π°Π΄Ρ€Π°Ρ‚Π½Ρ‹ΠΉ ΠΌΠ΅Ρ‚Ρ€ А βˆ™ ΠΌ2
ΠΠ°ΠΌΠ°Π³Π½ΠΈΡ‡Π΅Π½Π½ΠΎΡΡ‚ΡŒ J Π°ΠΌΠΏΠ΅Ρ€ Π½Π° ΠΌΠ΅Ρ‚Ρ€ А/ΠΌ
Π˜Π½Π΄ΡƒΠΊΡ‚ΠΈΠ²Π½ΠΎΡΡ‚ΡŒ L Π³Π΅Π½Ρ€ΠΈ Π“Π½
ЭлСктромагнитная энСргия N Π΄ΠΆΠΎΡƒΠ»ΡŒ Π”ΠΆ
ОбъСмная ΠΏΠ»ΠΎΡ‚Π½ΠΎΡΡ‚ΡŒ энСргии w Π΄ΠΆΠΎΡƒΠ»ΡŒ Π½Π° кубичСский ΠΌΠ΅Ρ‚Ρ€ Π”ΠΆ/ΠΌ3
Активная ΠΌΠΎΡ‰Π½ΠΎΡΡ‚ΡŒ P Π²Π°Ρ‚Ρ‚ Π’Ρ‚
РСактивная ΠΌΠΎΡ‰Π½ΠΎΡΡ‚ΡŒ Q Π²Π°Ρ€ Π²Π°Ρ€
Полная ΠΌΠΎΡ‰Π½ΠΎΡΡ‚ΡŒ S Π²Π°Ρ‚Ρ‚-Π°ΠΌΠΏΠ΅Ρ€ Π’Ρ‚ βˆ™ А

ΠžΡΠ½ΠΎΠ²Π½Ρ‹Π΅ элСктричСскиС Π·Π°ΠΊΠΎΠ½Ρ‹. Π‘Π°Π·ΠΎΠ²Ρ‹Π΅ Ρ„ΠΎΡ€ΠΌΡƒΠ»Ρ‹ ΠΈ расчСты

Π’ ΠΏΡ€Π΅Π΄Ρ‹Π΄ΡƒΡ‰Π΅ΠΉ ΡΡ‚Π°Ρ‚ΡŒΠ΅ ΠΌΡ‹ познакомились с основными элСктричСскими понятиями, Ρ‚Π°ΠΊΠΈΠΌΠΈ ΠΊΠ°ΠΊ элСктричСский Ρ‚ΠΎΠΊ, напряТСниС, сопротивлСниС ΠΈ ΠΌΠΎΡ‰Π½ΠΎΡΡ‚ΡŒ. Настал Ρ‡Π΅Ρ€Π΅Π΄ основных элСктричСских Π·Π°ΠΊΠΎΠ½ΠΎΠ², Ρ‚Π°ΠΊ ΡΠΊΠ°Π·Π°Ρ‚ΡŒ, базиса, Π±Π΅Π· знания ΠΈ понимания ΠΊΠΎΡ‚ΠΎΡ€Ρ‹Ρ… Π½Π΅Π²ΠΎΠ·ΠΌΠΎΠΆΠ½ΠΎ ΠΈΠ·ΡƒΡ‡Π΅Π½ΠΈΠ΅ ΠΈ ΠΏΠΎΠ½ΠΈΠΌΠ°Π½ΠΈΠ΅ элСктронных схСм ΠΈ устройств.

Π—Π°ΠΊΠΎΠ½ Ома

ЭлСктричСский Ρ‚ΠΎΠΊ, напряТСниС, сопротивлСниС ΠΈ ΠΌΠΎΡ‰Π½ΠΎΡΡ‚ΡŒ, бСзусловно, ΠΌΠ΅ΠΆΠ΄Ρƒ собой связаны. А взаимосвязь ΠΌΠ΅ΠΆΠ΄Ρƒ Π½ΠΈΠΌΠΈ описываСтся, Π±Π΅Π· сомнСния, самым Π³Π»Π°Π²Π½Ρ‹ΠΌ элСктричСским Π·Π°ΠΊΠΎΠ½ΠΎΠΌ – Π·Π°ΠΊΠΎΠ½ΠΎΠΌ Ома. Π’ ΡƒΠΏΡ€ΠΎΡ‰Π΅Π½Π½ΠΎΠΌ Π²ΠΈΠ΄Π΅ этот Π·Π°ΠΊΠΎΠ½ называСтся: Π·Π°ΠΊΠΎΠ½ Ома для участка Ρ†Π΅ΠΏΠΈ. И Π·Π²ΡƒΡ‡ΠΈΡ‚ этот Π·Π°ΠΊΠΎΠ½ ΡΠ»Π΅Π΄ΡƒΡŽΡ‰Π΅ΠΌ ΠΎΠ±Ρ€Π°Π·ΠΎΠΌ:

Β«Π‘ΠΈΠ»Π° Ρ‚ΠΎΠΊΠ° Π² участкС Ρ†Π΅ΠΏΠΈ прямо ΠΏΡ€ΠΎΠΏΠΎΡ€Ρ†ΠΈΠΎΠ½Π°Π»ΡŒΠ½Π° Π½Π°ΠΏΡ€ΡΠΆΠ΅Π½ΠΈΡŽ ΠΈ ΠΎΠ±Ρ€Π°Ρ‚Π½ΠΎ ΠΏΡ€ΠΎΠΏΠΎΡ€Ρ†ΠΈΠΎΠ½Π°Π»ΡŒΠ½Π° элСктричСскому ΡΠΎΠΏΡ€ΠΎΡ‚ΠΈΠ²Π»Π΅Π½ΠΈΡŽ Π΄Π°Π½Π½ΠΎΠ³ΠΎ участка Ρ†Π΅ΠΏΠΈΒ».

Для практичСского примСнСния Ρ„ΠΎΡ€ΠΌΡƒΠ»Ρƒ Π·Π°ΠΊΠΎΠ½Π° Ома ΠΌΠΎΠΆΠ½ΠΎ ΠΏΡ€Π΅Π΄ΡΡ‚Π°Π²ΠΈΡ‚ΡŒ Π² Π²ΠΈΠ΄Π΅ Π²ΠΎΡ‚ Ρ‚Π°ΠΊΠΎΠ³ΠΎ Ρ‚Ρ€Π΅ΡƒΠ³ΠΎΠ»ΡŒΠ½ΠΈΠΊΠ°, ΠΊΠΎΡ‚ΠΎΡ€Ρ‹ΠΉ ΠΏΠΎΠΌΠΈΠΌΠΎ основного прСдставлСния Ρ„ΠΎΡ€ΠΌΡƒΠ»Ρ‹, ΠΏΠΎΠΌΠΎΠΆΠ΅Ρ‚ ΠΎΠΏΡ€Π΅Π΄Π΅Π»ΠΈΡ‚ΡŒ ΠΈ ΠΎΡΡ‚Π°Π»ΡŒΠ½Ρ‹Π΅ Π²Π΅Π»ΠΈΡ‡ΠΈΠ½Ρ‹.

Π Π°Π±ΠΎΡ‚Π°Π΅Ρ‚ Ρ‚Ρ€Π΅ΡƒΠ³ΠΎΠ»ΡŒΠ½ΠΈΠΊ ΡΠ»Π΅Π΄ΡƒΡŽΡ‰ΠΈΠΌ ΠΎΠ±Ρ€Π°Π·ΠΎΠΌ. Π§Ρ‚ΠΎΠ±Ρ‹ Π²Ρ‹Ρ‡ΠΈΡΠ»ΠΈΡ‚ΡŒ ΠΎΠ΄Π½Ρƒ ΠΈΠ· Π²Π΅Π»ΠΈΡ‡ΠΈΠ½, достаточно Π·Π°ΠΊΡ€Ρ‹Ρ‚ΡŒ Π΅Π΅ ΠΏΠ°Π»ΡŒΡ†Π΅ΠΌ. НапримСр:

Π’ ΠΏΡ€Π΅Π΄Ρ‹Π΄ΡƒΡ‰Π΅ΠΉ ΡΡ‚Π°Ρ‚ΡŒΠ΅ ΠΌΡ‹ ΠΏΡ€ΠΎΠ²ΠΎΠ΄ΠΈΠ»ΠΈ аналогию ΠΌΠ΅ΠΆΠ΄Ρƒ элСктричСством ΠΈ Π²ΠΎΠ΄ΠΎΠΉ, ΠΈ выявили взаимосвязь ΠΌΠ΅ΠΆΠ΄Ρƒ напряТСниСм, Ρ‚ΠΎΠΊΠΎΠΌ ΠΈ сопротивлСниСм. Π’Π°ΠΊΠΆΠ΅ Ρ…ΠΎΡ€ΠΎΡˆΠ΅ΠΉ ΠΈΠ½Ρ‚Π΅Ρ€ΠΏΡ€Π΅Ρ‚Π°Ρ†ΠΈΠ΅ΠΉ Π·Π°ΠΊΠΎΠ½Π° Ома ΠΌΠΎΠΆΠ΅Ρ‚ ΠΏΠΎΡΠ»ΡƒΠΆΠΈΡ‚ΡŒ ΡΠ»Π΅Π΄ΡƒΡŽΡ‰ΠΈΠΉ рисунок, наглядно ΠΎΡ‚ΠΎΠ±Ρ€Π°ΠΆΠ°ΡŽΡ‰ΠΈΠΉ ΡΡƒΡ‰Π½ΠΎΡΡ‚ΡŒ Π·Π°ΠΊΠΎΠ½Π°:

На Π½Π΅ΠΌ ΠΌΡ‹ Π²ΠΈΠ΄ΠΈΠΌ, Ρ‡Ρ‚ΠΎ Ρ‡Π΅Π»ΠΎΠ²Π΅Ρ‡Π΅ΠΊ Β«Π’ΠΎΠ»ΡŒΡ‚Β» (напряТСниС) ΠΏΡ€ΠΎΡ‚Π°Π»ΠΊΠΈΠ²Π°Π΅Ρ‚ Ρ‡Π΅Π»ΠΎΠ²Π΅Ρ‡ΠΊΠ° «АмпСра» (Ρ‚ΠΎΠΊ) Ρ‡Π΅Ρ€Π΅Π· ΠΏΡ€ΠΎΠ²ΠΎΠ΄Π½ΠΈΠΊ, ΠΊΠΎΡ‚ΠΎΡ€Ρ‹ΠΉ стягиваСт Ρ‡Π΅Π»ΠΎΠ²Π΅Ρ‡Π΅ΠΊ «Ом» (сопротивлСниС). Π’ΠΎΡ‚ ΠΈ получаСтся, Ρ‡Ρ‚ΠΎ Ρ‡Π΅ΠΌ сильнСС сопротивлСниС сТимаСт ΠΏΡ€ΠΎΠ²ΠΎΠ΄Π½ΠΈΠΊ, Ρ‚Π΅ΠΌ тяТСлСС Ρ‚ΠΎΠΊΡƒ Ρ‡Π΅Ρ€Π΅Π· Π½Π΅Π³ΠΎ ΠΏΡ€ΠΎΡ…ΠΎΠ΄ΠΈΡ‚ΡŒ («сила Ρ‚ΠΎΠΊΠ° ΠΎΠ±Ρ€Π°Ρ‚Π½ΠΎ ΠΏΡ€ΠΎΠΏΠΎΡ€Ρ†ΠΈΠΎΠ½Π°Π»ΡŒΠ½Π° ΡΠΎΠΏΡ€ΠΎΡ‚ΠΈΠ²Π»Π΅Π½ΠΈΡŽ участка Ρ†Π΅ΠΏΠΈΒ» – ΠΈΠ»ΠΈ Ρ‡Π΅ΠΌ большС сопротивлСниС, Ρ‚Π΅ΠΌ Ρ…ΡƒΠΆΠ΅ приходится Ρ‚ΠΎΠΊΡƒ ΠΈ Ρ‚Π΅ΠΌ ΠΎΠ½ мСньшС). Но напряТСниС Π½Π΅ спит ΠΈ Ρ‚ΠΎΠ»ΠΊΠ°Π΅Ρ‚ Ρ‚ΠΎΠΊ ΠΈΠ·ΠΎ всСх сил (Ρ‡Π΅ΠΌ Π²Ρ‹ΡˆΠ΅ напряТСниС, Ρ‚Π΅ΠΌ большС Ρ‚ΠΎΠΊ ΠΈΠ»ΠΈ – «сила Ρ‚ΠΎΠΊΠ° Π² участкС Ρ†Π΅ΠΏΠΈ прямо ΠΏΡ€ΠΎΠΏΠΎΡ€Ρ†ΠΈΠΎΠ½Π°Π»ΡŒΠ½Π° Π½Π°ΠΏΡ€ΡΠΆΠ΅Π½ΠΈΡŽΒ»).

Когда Ρ„ΠΎΠ½Π°Ρ€ΠΈΠΊ Π½Π°Ρ‡ΠΈΠ½Π°Π΅Ρ‚ слабо ΡΠ²Π΅Ρ‚ΠΈΡ‚ΡŒ, ΠΌΡ‹ Π³ΠΎΠ²ΠΎΡ€ΠΈΠΌ – Β«Ρ€Π°Π·Ρ€ΡΠ΄ΠΈΠ»Π°ΡΡŒ Π±Π°Ρ‚Π°Ρ€Π΅ΠΉΠΊΠ°Β». Π§Ρ‚ΠΎ с Π½Π΅ΠΉ ΠΏΡ€ΠΎΠΈΠ·ΠΎΡˆΠ»ΠΎ, Ρ‡Ρ‚ΠΎ Π·Π½Π°Ρ‡ΠΈΡ‚ Ρ€Π°Π·Ρ€ΡΠ΄ΠΈΠ»Π°ΡΡŒ? А Π·Π½Π°Ρ‡ΠΈΡ‚ это, Ρ‡Ρ‚ΠΎ напряТСниС Π±Π°Ρ‚Π°Ρ€Π΅ΠΉΠΊΠΈ снизилось ΠΈ ΠΎΠ½ΠΎ большС Π½Π΅ Π² состоянии Β«ΠΏΠΎΠΌΠΎΠ³Π°Ρ‚ΡŒΒ» Ρ‚ΠΎΠΊΡƒ ΠΏΡ€Π΅ΠΎΠ΄ΠΎΠ»Π΅Π²Π°Ρ‚ΡŒ сопротивлСниС Ρ†Π΅ΠΏΠ΅ΠΉ Ρ„ΠΎΠ½Π°Ρ€ΠΈΠΊΠ° ΠΈ Π»Π°ΠΌΠΏΠΎΡ‡ΠΊΠΈ. Π’ΠΎΡ‚ ΠΈ получаСтся, Ρ‡Ρ‚ΠΎ Ρ‡Π΅ΠΌ большС напряТСниС – Ρ‚Π΅ΠΌ большС Ρ‚ΠΎΠΊ.

ΠŸΠΎΡΠ»Π΅Π΄ΠΎΠ²Π°Ρ‚Π΅Π»ΡŒΠ½ΠΎΠ΅ ΠΏΠΎΠ΄ΠΊΠ»ΡŽΡ‡Π΅Π½ΠΈΠ΅ – ΠΏΠΎΡΠ»Π΅Π΄ΠΎΠ²Π°Ρ‚Π΅Π»ΡŒΠ½Π°Ρ Ρ†Π΅ΠΏΡŒ

ΠŸΡ€ΠΈ ΠΏΠΎΡΠ»Π΅Π΄ΠΎΠ²Π°Ρ‚Π΅Π»ΡŒΠ½ΠΎΠΌ ΠΏΠΎΠ΄ΠΊΠ»ΡŽΡ‡Π΅Π½ΠΈΠΈ ΠΏΠΎΡ‚Ρ€Π΅Π±ΠΈΡ‚Π΅Π»Π΅ΠΉ, Π½Π°ΠΏΡ€ΠΈΠΌΠ΅Ρ€ ΠΎΠ±Ρ‹Ρ‡Π½Ρ‹Ρ… Π»Π°ΠΌΠΏΠΎΡ‡Π΅ΠΊ, сила Ρ‚ΠΎΠΊΠ° Π² ΠΊΠ°ΠΆΠ΄ΠΎΠΌ ΠΏΠΎΡ‚Ρ€Π΅Π±ΠΈΡ‚Π΅Π»Π΅ одинаковая, Π° Π²ΠΎΡ‚ напряТСниС Π±ΡƒΠ΄Π΅Ρ‚ ΠΎΡ‚Π»ΠΈΡ‡Π°Ρ‚ΡŒΡΡ. На ΠΊΠ°ΠΆΠ΄ΠΎΠΌ ΠΈΠ· ΠΏΠΎΡ‚Ρ€Π΅Π±ΠΈΡ‚Π΅Π»Π΅ΠΉ напряТСниС Π±ΡƒΠ΄Π΅Ρ‚ ΠΏΠ°Π΄Π°Ρ‚ΡŒ (ΡΠ½ΠΈΠΆΠ°Ρ‚ΡŒΡΡ).

А Π·Π°ΠΊΠΎΠ½ Ома Π² ΠΏΠΎΡΠ»Π΅Π΄ΠΎΠ²Π°Ρ‚Π΅Π»ΡŒΠ½ΠΎΠΉ Ρ†Π΅ΠΏΠΈ Π±ΡƒΠ΄Π΅Ρ‚ ΠΈΠΌΠ΅Ρ‚ΡŒ Π²ΠΈΠ΄:

ΠŸΡ€ΠΈ ΠΏΠΎΡΠ»Π΅Π΄ΠΎΠ²Π°Ρ‚Π΅Π»ΡŒΠ½ΠΎΠΌ соСдинСнии сопротивлСния ΠΏΠΎΡ‚Ρ€Π΅Π±ΠΈΡ‚Π΅Π»Π΅ΠΉ ΡΠΊΠ»Π°Π΄Ρ‹Π²Π°ΡŽΡ‚ΡΡ. Π€ΠΎΡ€ΠΌΡƒΠ»Π° для расчСта ΠΎΠ±Ρ‰Π΅Π³ΠΎ сопротивлСния:

ΠŸΠ°Ρ€Π°Π»Π»Π΅Π»ΡŒΠ½ΠΎΠ΅ ΠΏΠΎΠ΄ΠΊΠ»ΡŽΡ‡Π΅Π½ΠΈΠ΅ – ΠΏΠ°Ρ€Π°Π»Π»Π΅Π»ΡŒΠ½Π°Ρ Ρ†Π΅ΠΏΡŒ

ΠŸΡ€ΠΈ ΠΏΠ°Ρ€Π°Π»Π»Π΅Π»ΡŒΠ½ΠΎΠΌ ΠΏΠΎΠ΄ΠΊΠ»ΡŽΡ‡Π΅Π½ΠΈΠΈ, ΠΊ ΠΊΠ°ΠΆΠ΄ΠΎΠΌΡƒ ΠΏΠΎΡ‚Ρ€Π΅Π±ΠΈΡ‚Π΅Π»ΡŽ прикладываСтся ΠΎΠ΄ΠΈΠ½Π°ΠΊΠΎΠ²ΠΎΠ΅ напряТСниС, Π° Π²ΠΎΡ‚ Ρ‚ΠΎΠΊ Ρ‡Π΅Ρ€Π΅Π· ΠΊΠ°ΠΆΠ΄Ρ‹ΠΉ ΠΈΠ· ΠΏΠΎΡ‚Ρ€Π΅Π±ΠΈΡ‚Π΅Π»Π΅ΠΉ, Π² случаС, Ссли ΠΈΡ… сопротивлСниС отличаСтся – Π±ΡƒΠ΄Π΅Ρ‚ ΠΎΡ‚Π»ΠΈΡ‡Π°Ρ‚ΡŒΡΡ.

Π—Π°ΠΊΠΎΠ½ Ома для ΠΏΠ°Ρ€Π°Π»Π»Π΅Π»ΡŒΠ½ΠΎΠΉ Ρ†Π΅ΠΏΠΈ, состоящСй ΠΈΠ· Ρ‚Ρ€Π΅Ρ… ΠΏΠΎΡ‚Ρ€Π΅Π±ΠΈΡ‚Π΅Π»Π΅ΠΉ, Π±ΡƒΠ΄Π΅Ρ‚ ΠΈΠΌΠ΅Ρ‚ΡŒ Π²ΠΈΠ΄:

ΠŸΡ€ΠΈ ΠΏΠ°Ρ€Π°Π»Π»Π΅Π»ΡŒΠ½ΠΎΠΌ соСдинСнии ΠΎΠ±Ρ‰Π΅Π΅ сопротивлСниС Ρ†Π΅ΠΏΠΈ всСгда Π±ΡƒΠ΄Π΅Ρ‚ мСньшС значСния самого малСнького ΠΎΡ‚Π΄Π΅Π»ΡŒΠ½ΠΎΠ³ΠΎ сопротивлСния. Или Π΅Ρ‰Π΅ говорят, Ρ‡Ρ‚ΠΎ «сопротивлСниС Π±ΡƒΠ΄Π΅Ρ‚ мСньшС наимСньшСго».

ΠžΠ±Ρ‰Π΅Π΅ сопротивлСниС Ρ†Π΅ΠΏΠΈ, состоящСй ΠΈΠ· Π΄Π²ΡƒΡ… ΠΏΠΎΡ‚Ρ€Π΅Π±ΠΈΡ‚Π΅Π»Π΅ΠΉ, ΠΏΡ€ΠΈ ΠΏΠ°Ρ€Π°Π»Π»Π΅Π»ΡŒΠ½ΠΎΠΌ соСдинСнии:

ΠžΠ±Ρ‰Π΅Π΅ сопротивлСниС Ρ†Π΅ΠΏΠΈ, состоящСй ΠΈΠ· Ρ‚Ρ€Π΅Ρ… ΠΏΠΎΡ‚Ρ€Π΅Π±ΠΈΡ‚Π΅Π»Π΅ΠΉ, ΠΏΡ€ΠΈ ΠΏΠ°Ρ€Π°Π»Π»Π΅Π»ΡŒΠ½ΠΎΠΌ соСдинСнии:


Для большСго числа ΠΏΠΎΡ‚Ρ€Π΅Π±ΠΈΡ‚Π΅Π»Π΅ΠΉ расчСт производится исходя ΠΈΠ· Ρ‚ΠΎΠ³ΠΎ, Ρ‡Ρ‚ΠΎ ΠΏΡ€ΠΈ ΠΏΠ°Ρ€Π°Π»Π»Π΅Π»ΡŒΠ½ΠΎΠΌ соСдинСнии ΠΏΡ€ΠΎΠ²ΠΎΠ΄ΠΈΠΌΠΎΡΡ‚ΡŒ (Π²Π΅Π»ΠΈΡ‡ΠΈΠ½Π° обратная ΡΠΎΠΏΡ€ΠΎΡ‚ΠΈΠ²Π»Π΅Π½ΠΈΡŽ) рассчитываСтся ΠΊΠ°ΠΊ сумма проводимостСй ΠΊΠ°ΠΆΠ΄ΠΎΠ³ΠΎ потрСбитСля.

ЭлСктричСская ΠΌΠΎΡ‰Π½ΠΎΡΡ‚ΡŒ

ΠœΠΎΡ‰Π½ΠΎΡΡ‚ΡŒ – это физичСская Π²Π΅Π»ΠΈΡ‡ΠΈΠ½Π°, Ρ…Π°Ρ€Π°ΠΊΡ‚Π΅Ρ€ΠΈΠ·ΡƒΡŽΡ‰Π°Ρ ΡΠΊΠΎΡ€ΠΎΡΡ‚ΡŒ ΠΏΠ΅Ρ€Π΅Π΄Π°Ρ‡ΠΈ ΠΈΠ»ΠΈ прСобразования элСктричСской энСргии. РассчитываСтся ΠΌΠΎΡ‰Π½ΠΎΡΡ‚ΡŒ ΠΏΠΎ ΡΠ»Π΅Π΄ΡƒΡŽΡ‰Π΅ΠΉ Ρ„ΠΎΡ€ΠΌΡƒΠ»Π΅:

Π’Π°ΠΊΠΈΠΌ ΠΎΠ±Ρ€Π°Π·ΠΎΠΌ зная, напряТСниС источника ΠΈ ΠΈΠ·ΠΌΠ΅Ρ€ΠΈΠ² потрСбляСмый Ρ‚ΠΎΠΊ, ΠΌΡ‹ ΠΌΠΎΠΆΠ΅ΠΌ ΠΎΠΏΡ€Π΅Π΄Π΅Π»ΠΈΡ‚ΡŒ ΠΌΠΎΡ‰Π½ΠΎΡΡ‚ΡŒ ΠΏΠΎΡ‚Ρ€Π΅Π±Π»ΡΠ΅ΠΌΡƒΡŽ элСктроприбором. И Π½Π°ΠΎΠ±ΠΎΡ€ΠΎΡ‚, зная ΠΌΠΎΡ‰Π½ΠΎΡΡ‚ΡŒ элСктроприбора ΠΈ напряТСниС сСти, ΠΌΠΎΠΆΠ΅ΠΌ ΠΎΠΏΡ€Π΅Π΄Π΅Π»ΠΈΡ‚ΡŒ Π²Π΅Π»ΠΈΡ‡ΠΈΠ½Ρƒ потрСбляСмого Ρ‚ΠΎΠΊΠ°. Π’Π°ΠΊΠΈΠ΅ вычислСния ΠΏΠΎΡ€ΠΎΠΉ Π½Π΅ΠΎΠ±Ρ…ΠΎΠ΄ΠΈΠΌΡ‹. НапримСр, для Π·Π°Ρ‰ΠΈΡ‚Ρ‹ элСктроприборов ΠΈΡΠΏΠΎΠ»ΡŒΠ·ΡƒΡŽΡ‚ΡΡ ΠΏΡ€Π΅Π΄ΠΎΡ…Ρ€Π°Π½ΠΈΡ‚Π΅Π»ΠΈ ΠΈΠ»ΠΈ автоматичСскиС Π²Ρ‹ΠΊΠ»ΡŽΡ‡Π°Ρ‚Π΅Π»ΠΈ. Π§Ρ‚ΠΎΠ±Ρ‹ ΠΏΡ€Π°Π²ΠΈΠ»ΡŒΠ½ΠΎ ΠΏΠΎΠ΄ΠΎΠ±Ρ€Π°Ρ‚ΡŒ срСдство Π·Π°Ρ‰ΠΈΡ‚Ρ‹ Π½ΡƒΠΆΠ½ΠΎ Π·Π½Π°Ρ‚ΡŒ потрСбляСмый Ρ‚ΠΎΠΊ. ΠŸΡ€Π΅Π΄ΠΎΡ…Ρ€Π°Π½ΠΈΡ‚Π΅Π»ΠΈ, примСняСмыС Π² Π±Ρ‹Ρ‚ΠΎΠ²ΠΎΠΉ Ρ‚Π΅Ρ…Π½ΠΈΠΊΠ΅, ΠΊΠ°ΠΊ ΠΏΡ€Π°Π²ΠΈΠ»ΠΎ ΠΏΠΎΠ΄Π»Π΅ΠΆΠ°Ρ‚ Ρ€Π΅ΠΌΠΎΠ½Ρ‚Ρƒ ΠΈ для ΠΈΡ… восстановлСния достаточно ΠΏΠΎΠ΄ΠΎΠ±Ρ€Π°Ρ‚ΡŒ ΠΈ Π·Π°ΠΌΠ΅Π½ΠΈΡ‚ΡŒ ΠΏΡ€ΠΎΠ²ΠΎΠ»ΠΎΠΊΡƒ.

ΠŸΡ€ΠΈΠΌΠ΅Π½ΠΈΠ² Π·Π°ΠΊΠΎΠ½ Ома, ΠΌΠΎΠΆΠ½ΠΎ Ρ€Π°ΡΡΡ‡ΠΈΡ‚Π°Ρ‚ΡŒ ΠΌΠΎΡ‰Π½ΠΎΡΡ‚ΡŒ ΠΈ ΠΏΠΎ Π΄Ρ€ΡƒΠ³ΠΎΠΉ Ρ„ΠΎΡ€ΠΌΡƒΠ»Π΅:

ΠŸΡ€ΠΈ расчСтах Π½Π°Π΄ΠΎ ΡƒΡ‡ΠΈΡ‚Ρ‹Π²Π°Ρ‚ΡŒ, Ρ‡Ρ‚ΠΎ Ρ‡Π°ΡΡ‚ΡŒ потрСбляСмой элСктроэнСргии расходуСтся Π½Π° Π½Π°Π³Ρ€Π΅Π² ΠΈ прСобразуСтся Π² Ρ‚Π΅ΠΏΠ»ΠΎ. ΠŸΡ€ΠΈ Ρ€Π°Π±ΠΎΡ‚Π΅ Π³Ρ€Π΅ΡŽΡ‚ΡΡ Π½Π΅ Ρ‚ΠΎΠ»ΡŒΠΊΠΎ элСктрообогрСватСли, Π½ΠΎ ΠΈ Ρ‚Π΅Π»Π΅Π²ΠΈΠ·ΠΎΡ€Ρ‹, ΠΈ ΠΊΠΎΠΌΠΏΡŒΡŽΡ‚Π΅Ρ€Ρ‹ ΠΈ другая бытовая Ρ‚Π΅Ρ…Π½ΠΈΠΊΠ°.

И Π² Π·Π°Π²Π΅Ρ€ΡˆΠ΅Π½ΠΈΠ΅, Π² качСствС бонуса, Π²ΠΎΡ‚ такая ΡˆΠΏΠ°Ρ€Π³Π°Π»ΠΊΠ°, которая ΠΏΠΎΠΌΠΎΠΆΠ΅Ρ‚ ΠΎΠΏΡ€Π΅Π΄Π΅Π»ΠΈΡ‚ΡŒ любой ΠΈΠ· основных элСктричСских ΠΏΠ°Ρ€Π°ΠΌΠ΅Ρ‚Ρ€ΠΎΠ², ΠΏΠΎ ΡƒΠΆΠ΅ извСстным.

Π’Π΅Π±-сайт класса Ρ„ΠΈΠ·ΠΈΠΊΠΈ

ЭлСктричСскиС схСмы: ΠΎΠ±Π·ΠΎΡ€ Π½Π°Π±ΠΎΡ€Π° ΠΏΡ€ΠΎΠ±Π»Π΅ΠΌ

Π­Ρ‚ΠΎΡ‚ Π½Π°Π±ΠΎΡ€ ΠΈΠ· 34 Π·Π°Π΄Π°Ρ‡ Π½Π°Ρ†Π΅Π»Π΅Π½ Π½Π° Π²Π°ΡˆΡƒ ΡΠΏΠΎΡΠΎΠ±Π½ΠΎΡΡ‚ΡŒ ΠΎΠΏΡ€Π΅Π΄Π΅Π»ΡΡ‚ΡŒ Ρ‚Π°ΠΊΠΈΠ΅ Π²Π΅Π»ΠΈΡ‡ΠΈΠ½Ρ‹ Ρ†Π΅ΠΏΠΈ, ΠΊΠ°ΠΊ Ρ‚ΠΎΠΊ, сопротивлСниС, Ρ€Π°Π·Π½ΠΎΡΡ‚ΡŒ элСктричСских ΠΏΠΎΡ‚Π΅Π½Ρ†ΠΈΠ°Π»ΠΎΠ², ΠΌΠΎΡ‰Π½ΠΎΡΡ‚ΡŒ ΠΈ элСктричСская энСргия, Π½Π° основС словСсных описаний ΠΈ Π΄ΠΈΠ°Π³Ρ€Π°ΠΌΠΌ физичСских ситуаций, относящихся ΠΊ элСктричСским цСпям. ΠŸΡ€ΠΎΠ±Π»Π΅ΠΌΡ‹ Π²Π°Ρ€ΡŒΠΈΡ€ΡƒΡŽΡ‚ΡΡ ΠΏΠΎ слоТности ΠΎΡ‚ ΠΎΡ‡Π΅Π½ΡŒ простых ΠΈ простых Π΄ΠΎ ΠΎΡ‡Π΅Π½ΡŒ слоТных ΠΈ слоТных.Π‘ΠΎΠ»Π΅Π΅ слоТныС Π·Π°Π΄Π°Ρ‡ΠΈ ΠΎΠ±ΠΎΠ·Π½Π°Ρ‡Π΅Π½Ρ‹ Ρ†Π²Π΅Ρ‚ΠΎΠΌ , синиС Π·Π°Π΄Π°Ρ‡ΠΈ .

Π’Π΅ΠΊΡƒΡ‰ΠΈΠΉ

Когда заряд ΠΏΡ€ΠΎΡ…ΠΎΠ΄ΠΈΡ‚ ΠΏΠΎ ΠΏΡ€ΠΎΠ²ΠΎΠ΄Π°ΠΌ элСктричСской Ρ†Π΅ΠΏΠΈ , считаСтся, Ρ‡Ρ‚ΠΎ Π² ΠΏΡ€ΠΎΠ²ΠΎΠ΄Π°Ρ… присутствуСт Ρ‚ΠΎΠΊ. ЭлСктричСский Ρ‚ΠΎΠΊ – это ΠΈΠ·ΠΌΠ΅Ρ€ΠΈΠΌΠΎΠ΅ понятиС, ΠΊΠΎΡ‚ΠΎΡ€ΠΎΠ΅ опрСдСляСтся ΠΊΠ°ΠΊ ΡΠΊΠΎΡ€ΠΎΡΡ‚ΡŒ , с ΠΊΠΎΡ‚ΠΎΡ€ΠΎΠΉ заряд ΠΏΡ€ΠΎΡ…ΠΎΠ΄ΠΈΡ‚ Ρ‡Π΅Ρ€Π΅Π· Ρ‚ΠΎΡ‡ΠΊΡƒ Π² Ρ†Π΅ΠΏΠΈ. Π•Π³ΠΎ ΠΌΠΎΠΆΠ½ΠΎ ΠΎΠΏΡ€Π΅Π΄Π΅Π»ΠΈΡ‚ΡŒ, ΠΈΠ·ΠΌΠ΅Ρ€ΠΈΠ² количСство заряда, ΠΏΡ€ΠΎΡ‚Π΅ΠΊΠ°ΡŽΡ‰Π΅Π³ΠΎ ΠΏΠΎ ΠΏΠ»ΠΎΡ‰Π°Π΄ΠΈ ΠΏΠΎΠΏΠ΅Ρ€Π΅Ρ‡Π½ΠΎΠ³ΠΎ сСчСния ΠΏΡ€ΠΎΠ²ΠΎΠ΄Π° Π² Ρ†Π΅ΠΏΠΈ.Как Π²Π΅Π»ΠΈΡ‡ΠΈΠ½Π° скорости, Ρ‚ΠΎΠΊ (I) выраТаСтся ΡΠ»Π΅Π΄ΡƒΡŽΡ‰ΠΈΠΌ ΡƒΡ€Π°Π²Π½Π΅Π½ΠΈΠ΅ΠΌ

I = Q / Ρ‚

Π³Π΄Π΅ Q – количСство заряда, ΠΏΡ€ΠΎΡ‚Π΅ΠΊΠ°ΡŽΡ‰Π΅Π³ΠΎ Ρ‡Π΅Ρ€Π΅Π· Ρ‚ΠΎΡ‡ΠΊΡƒ Π·Π° ΠΏΠ΅Ρ€ΠΈΠΎΠ΄ Π²Ρ€Π΅ΠΌΠ΅Π½ΠΈ t. Π‘Ρ‚Π°Π½Π΄Π°Ρ€Ρ‚Π½ΠΎΠΉ мСтричСской Π΅Π΄ΠΈΠ½ΠΈΡ†Π΅ΠΉ измСрСния Π²Π΅Π»ΠΈΡ‡ΠΈΠ½Ρ‹ Ρ‚ΠΎΠΊΠ° являСтся Π°ΠΌΠΏΠ΅Ρ€, часто сокращСнно АмпСр ΠΈΠ»ΠΈ А. Π’ΠΎΠΊ Π² 1 Π°ΠΌΠΏΠ΅Ρ€ эквивалСнтСн 1 ΠΊΡƒΠ»ΠΎΠ½Ρƒ заряда, ΠΏΡ€ΠΎΡ‚Π΅ΠΊΠ°ΡŽΡ‰Π΅ΠΌΡƒ Ρ‡Π΅Ρ€Π΅Π· Ρ‚ΠΎΡ‡ΠΊΡƒ Π·Π° 1 сСкунду. ΠŸΠΎΡΠΊΠΎΠ»ΡŒΠΊΡƒ количСство заряда, проходящСго Ρ‡Π΅Ρ€Π΅Π· Ρ‚ΠΎΡ‡ΠΊΡƒ Π² Ρ†Π΅ΠΏΠΈ, связано с количСством ΠΌΠΎΠ±ΠΈΠ»ΡŒΠ½Ρ‹Ρ… носитСлСй заряда (элСктронов), ΠΊΠΎΡ‚ΠΎΡ€Ρ‹Π΅ проходят Ρ‡Π΅Ρ€Π΅Π· эту Ρ‚ΠΎΡ‡ΠΊΡƒ, Ρ‚ΠΎΠΊ Ρ‚Π°ΠΊΠΆΠ΅ ΠΌΠΎΠΆΠ΅Ρ‚ Π±Ρ‹Ρ‚ΡŒ связан с количСством элСктронов ΠΈ Π²Ρ€Π΅ΠΌΠ΅Π½Π΅ΠΌ.Π§Ρ‚ΠΎΠ±Ρ‹ ΡƒΡΡ‚Π°Π½ΠΎΠ²ΠΈΡ‚ΡŒ связь ΠΌΠ΅ΠΆΠ΄Ρƒ Ρ‚ΠΎΠΊΠΎΠΌ ΠΈ числом элСктронов, Π½ΡƒΠΆΠ½ΠΎ Π·Π½Π°Ρ‚ΡŒ количСство заряда Π½Π° ΠΎΠ΄Π½ΠΎΠΌ элСктронС.

Q элСктрон = 1,6 x 10 -19 C

Π‘ΠΎΠΏΡ€ΠΎΡ‚ΠΈΠ²Π»Π΅Π½ΠΈΠ΅

Когда заряд Ρ‚Π΅Ρ‡Π΅Ρ‚ ΠΏΠΎ Ρ†Π΅ΠΏΠΈ, ΠΎΠ½ встрСчаСт сопротивлСниС ΠΈΠ»ΠΈ прСпятствиС для Π΅Π³ΠΎ прохоТдСния. Как ΠΈ Ρ‚ΠΎΠΊ, сопротивлСниС – это ΠΈΠ·ΠΌΠ΅Ρ€ΠΈΠΌΡ‹ΠΉ Ρ‚Π΅Ρ€ΠΌΠΈΠ½. Π’Π΅Π»ΠΈΡ‡ΠΈΠ½Π° сопротивлСния, обСспСчиваСмого сСчСниСм ΠΏΡ€ΠΎΠ²ΠΎΠ΄Π°, зависит ΠΎΡ‚ Ρ‚Ρ€Π΅Ρ… ΠΏΠ΅Ρ€Π΅ΠΌΠ΅Π½Π½Ρ‹Ρ… – ΠΌΠ°Ρ‚Π΅Ρ€ΠΈΠ°Π»Π°, ΠΈΠ· ΠΊΠΎΡ‚ΠΎΡ€ΠΎΠ³ΠΎ сдСлан ΠΏΡ€ΠΎΠ²ΠΎΠ΄, Π΄Π»ΠΈΠ½Ρ‹ ΠΏΡ€ΠΎΠ²ΠΎΠ΄Π° ΠΈ ΠΏΠ»ΠΎΡ‰Π°Π΄ΠΈ ΠΏΠΎΠΏΠ΅Ρ€Π΅Ρ‡Π½ΠΎΠ³ΠΎ сСчСния ΠΏΡ€ΠΎΠ²ΠΎΠ΄Π°.Одним ΠΈΠ· физичСских свойств ΠΌΠ°Ρ‚Π΅Ρ€ΠΈΠ°Π»Π° являСтся Π΅Π³ΠΎ ΡƒΠ΄Π΅Π»ΡŒΠ½ΠΎΠ΅ сопротивлСниС – ΠΌΠ΅Ρ€Π° Ρ‚Π΅Π½Π΄Π΅Π½Ρ†ΠΈΠΈ этого ΠΌΠ°Ρ‚Π΅Ρ€ΠΈΠ°Π»Π° ΠΏΡ€ΠΎΡ‚ΠΈΠ²ΠΎΡΡ‚ΠΎΡΡ‚ΡŒ ΠΏΡ€ΠΎΡ…ΠΎΠΆΠ΄Π΅Π½ΠΈΡŽ заряда Ρ‡Π΅Ρ€Π΅Π· Π½Π΅Π³ΠΎ. ЗначСния ΡƒΠ΄Π΅Π»ΡŒΠ½ΠΎΠ³ΠΎ сопротивлСния для Ρ€Π°Π·Π»ΠΈΡ‡Π½Ρ‹Ρ… проводящих ΠΌΠ°Ρ‚Π΅Ρ€ΠΈΠ°Π»ΠΎΠ² ΠΎΠ±Ρ‹Ρ‡Π½ΠΎ ΡƒΠΊΠ°Π·Π°Π½Ρ‹ Π² ΡƒΡ‡Π΅Π±Π½ΠΈΠΊΠ°Ρ… ΠΈ справочниках. Зная Π·Π½Π°Ρ‡Π΅Π½ΠΈΠ΅ ΡƒΠ΄Π΅Π»ΡŒΠ½ΠΎΠ³ΠΎ сопротивлСния (ρ) ΠΌΠ°Ρ‚Π΅Ρ€ΠΈΠ°Π»Π°, ΠΈΠ· ΠΊΠΎΡ‚ΠΎΡ€ΠΎΠ³ΠΎ состоит ΠΏΡ€ΠΎΠ²ΠΎΠ΄, Π° Ρ‚Π°ΠΊΠΆΠ΅ Π΅Π³ΠΎ Π΄Π»ΠΈΠ½Ρƒ (L) ΠΈ ΠΏΠ»ΠΎΡ‰Π°Π΄ΡŒ ΠΏΠΎΠΏΠ΅Ρ€Π΅Ρ‡Π½ΠΎΠ³ΠΎ сСчСния (A), Π΅Π³ΠΎ сопротивлСниС (R) ΠΌΠΎΠΆΠ½ΠΎ ΠΎΠΏΡ€Π΅Π΄Π΅Π»ΠΈΡ‚ΡŒ с ΠΏΠΎΠΌΠΎΡ‰ΡŒΡŽ ΠΏΡ€ΠΈΠ²Π΅Π΄Π΅Π½Π½ΠΎΠ³ΠΎ Π½ΠΈΠΆΠ΅ уравнСния.

R = ρ β€’ L / A

Бтандартная мСтричСская Π΅Π΄ΠΈΠ½ΠΈΡ†Π° измСрСния сопротивлСния – Ом (сокращСнно грСчСской Π±ΡƒΠΊΠ²ΠΎΠΉ Ом ).

Основная Ρ‚Ρ€ΡƒΠ΄Π½ΠΎΡΡ‚ΡŒ ΠΏΡ€ΠΈ использовании ΠΏΡ€ΠΈΠ²Π΅Π΄Π΅Π½Π½ΠΎΠ³ΠΎ Π²Ρ‹ΡˆΠ΅ уравнСния связана с Π΅Π΄ΠΈΠ½ΠΈΡ†Π°ΠΌΠΈ выраТСния Ρ€Π°Π·Π»ΠΈΡ‡Π½Ρ‹Ρ… Π²Π΅Π»ΠΈΡ‡ΠΈΠ½. УдСльноС сопротивлСниС (ρ) ΠΎΠ±Ρ‹Ρ‡Π½ΠΎ выраТаСтся Π² Ом β€’ ΠΌ. Π’Π°ΠΊΠΈΠΌ ΠΎΠ±Ρ€Π°Π·ΠΎΠΌ, Π΄Π»ΠΈΠ½Π° Π΄ΠΎΠ»ΠΆΠ½Π° Π±Ρ‹Ρ‚ΡŒ Π²Ρ‹Ρ€Π°ΠΆΠ΅Π½Π° Π² ΠΌΠ΅Ρ‚Ρ€Π°Ρ…, Π° ΠΏΠ»ΠΎΡ‰Π°Π΄ΡŒ ΠΏΠΎΠΏΠ΅Ρ€Π΅Ρ‡Π½ΠΎΠ³ΠΎ сСчСния – Π² ΠΌΠ΅Ρ‚Ρ€Π°Ρ… 2 . МногиС ΠΏΡ€ΠΎΠ²ΠΎΠ΄Π° ΠΊΡ€ΡƒΠ³Π»Ρ‹Π΅ ΠΈ ΠΈΠΌΠ΅ΡŽΡ‚ ΠΊΡ€ΡƒΠ³Π»ΠΎΠ΅ сСчСниС. Π’Π°ΠΊΠΈΠΌ ΠΎΠ±Ρ€Π°Π·ΠΎΠΌ, ΠΏΠ»ΠΎΡ‰Π°Π΄ΡŒ ΠΏΠΎΠΏΠ΅Ρ€Π΅Ρ‡Π½ΠΎΠ³ΠΎ сСчСния Π² ΠΏΡ€ΠΈΠ²Π΅Π΄Π΅Π½Π½ΠΎΠΌ Π²Ρ‹ΡˆΠ΅ ΡƒΡ€Π°Π²Π½Π΅Π½ΠΈΠΈ ΠΌΠΎΠΆΠ½ΠΎ Ρ€Π°ΡΡΡ‡ΠΈΡ‚Π°Ρ‚ΡŒ, зная радиус ΠΈΠ»ΠΈ Π΄ΠΈΠ°ΠΌΠ΅Ρ‚Ρ€ ΠΏΡ€ΠΎΠ²ΠΎΠ΄Π°, ΠΈΡΠΏΠΎΠ»ΡŒΠ·ΡƒΡ Ρ„ΠΎΡ€ΠΌΡƒΠ»Ρƒ для ΠΏΠ»ΠΎΡ‰Π°Π΄ΠΈ ΠΊΡ€ΡƒΠ³Π°.

A = Ο€ β€’ R 2 = Ο€ β€’ D 2 /4

Π‘ΠΎΠΎΡ‚Π½ΠΎΡˆΠ΅Π½ΠΈΠ΅ напряТСниС-Ρ‚ΠΎΠΊ-сопротивлСниС

Π’Π΅Π»ΠΈΡ‡ΠΈΠ½Π° Ρ‚ΠΎΠΊΠ°, ΠΏΡ€ΠΎΡ‚Π΅ΠΊΠ°ΡŽΡ‰Π΅Π³ΠΎ Π² Ρ†Π΅ΠΏΠΈ, зависит ΠΎΡ‚ Π΄Π²ΡƒΡ… ΠΏΠ΅Ρ€Π΅ΠΌΠ΅Π½Π½Ρ‹Ρ…. Π’ΠΎΠΊ ΠΎΠ±Ρ€Π°Ρ‚Π½ΠΎ ΠΏΡ€ΠΎΠΏΠΎΡ€Ρ†ΠΈΠΎΠ½Π°Π»Π΅Π½ ΠΎΠ±Ρ‰Π΅ΠΌΡƒ ΡΠΎΠΏΡ€ΠΎΡ‚ΠΈΠ²Π»Π΅Π½ΠΈΡŽ (R) Ρ†Π΅ΠΏΠΈ ΠΈ прямо ΠΏΡ€ΠΎΠΏΠΎΡ€Ρ†ΠΈΠΎΠ½Π°Π»Π΅Π½ разности элСктричСских ΠΏΠΎΡ‚Π΅Π½Ρ†ΠΈΠ°Π»ΠΎΠ², ΠΏΡ€ΠΈΠ»ΠΎΠΆΠ΅Π½Π½ΠΎΠΉ ΠΊ Ρ†Π΅ΠΏΠΈ. Π Π°Π·Π½ΠΎΡΡ‚ΡŒ элСктричСских ΠΏΠΎΡ‚Π΅Π½Ρ†ΠΈΠ°Π»ΠΎΠ² (Ξ”V), прилоТСнная ΠΊ Ρ†Π΅ΠΏΠΈ, – это просто напряТСниС, ΠΏΠΎΠ΄Π°Π²Π°Π΅ΠΌΠΎΠ΅ источником энСргии (Π±Π°Ρ‚Π°Ρ€Π΅ΠΈ, Ρ€ΠΎΠ·Π΅Ρ‚ΠΊΠΈ ΠΈ Ρ‚. Π”.).). Для Π΄ΠΎΠΌΠΎΠ² Π² БША это Π·Π½Π°Ρ‡Π΅Π½ΠΈΠ΅ Π±Π»ΠΈΠ·ΠΊΠΎ ΠΊ 110–120 Π²ΠΎΠ»ΡŒΡ‚. ΠœΠ°Ρ‚Π΅ΠΌΠ°Ρ‚ΠΈΡ‡Π΅ΡΠΊΠ°Ρ взаимосвязь ΠΌΠ΅ΠΆΠ΄Ρƒ Ρ‚ΠΎΠΊΠΎΠΌ (I), напряТСниСм ΠΈ сопротивлСниСм выраТаСтся ΡΠ»Π΅Π΄ΡƒΡŽΡ‰ΠΈΠΌ ΡƒΡ€Π°Π²Π½Π΅Π½ΠΈΠ΅ΠΌ (ΠΊΠΎΡ‚ΠΎΡ€ΠΎΠ΅ ΠΈΠ½ΠΎΠ³Π΄Π° Π½Π°Π·Ρ‹Π²Π°ΡŽΡ‚ ΡƒΡ€Π°Π²Π½Π΅Π½ΠΈΠ΅ΠΌ Π·Π°ΠΊΠΎΠ½Π° Ома ).

ΠœΠΎΡ‰Π½ΠΎΡΡ‚ΡŒ

ЭлСктричСскиС схСмы – это энСргия. ЭнСргия Π²ΠΊΠ»ΡŽΡ‡Π°Π΅Ρ‚ΡΡ Π² Ρ†Π΅ΠΏΡŒ аккумулятором ΠΈΠ»ΠΈ коммСрчСским поставщиком элСктроэнСргии.Π­Π»Π΅ΠΌΠ΅Π½Ρ‚Ρ‹ схСмы (освСщСниС, ΠΎΠ±ΠΎΠ³Ρ€Π΅Π²Π°Ρ‚Π΅Π»ΠΈ, Π΄Π²ΠΈΠ³Π°Ρ‚Π΅Π»ΠΈ, Ρ…ΠΎΠ»ΠΎΠ΄ΠΈΠ»ΡŒΠ½ΠΈΠΊΠΈ ΠΈ Π΄Π°ΠΆΠ΅ ΠΏΡ€ΠΎΠ²ΠΎΠ΄Π°) ΠΏΡ€Π΅ΠΎΠ±Ρ€Π°Π·ΡƒΡŽΡ‚ эту ΡΠ»Π΅ΠΊΡ‚Ρ€ΠΈΡ‡Π΅ΡΠΊΡƒΡŽ ΠΏΠΎΡ‚Π΅Π½Ρ†ΠΈΠ°Π»ΡŒΠ½ΡƒΡŽ ΡΠ½Π΅Ρ€Π³ΠΈΡŽ Π² Π΄Ρ€ΡƒΠ³ΠΈΠ΅ Ρ„ΠΎΡ€ΠΌΡ‹ энСргии, Ρ‚Π°ΠΊΠΈΠ΅ ΠΊΠ°ΠΊ свСтовая энСргия, звуковая энСргия, тСпловая энСргия ΠΈ мСханичСская энСргия. ΠœΠΎΡ‰Π½ΠΎΡΡ‚ΡŒ ΠΎΠ·Π½Π°Ρ‡Π°Π΅Ρ‚ ΡΠΊΠΎΡ€ΠΎΡΡ‚ΡŒ, с ΠΊΠΎΡ‚ΠΎΡ€ΠΎΠΉ энСргия пСрСдаСтся ΠΈΠ»ΠΈ прСобразуСтся устройством ΠΈΠ»ΠΈ Ρ†Π΅ΠΏΡŒΡŽ. Π­Ρ‚ΠΎ ΡΠΊΠΎΡ€ΠΎΡΡ‚ΡŒ, с ΠΊΠΎΡ‚ΠΎΡ€ΠΎΠΉ энСргия тСряСтся ΠΈΠ»ΠΈ приобрСтаСтся Π² любом Π·Π°Π΄Π°Π½Π½ΠΎΠΌ мСстС Π² Ρ†Π΅ΠΏΠΈ. Π’Π°ΠΊΠΈΠΌ ΠΎΠ±Ρ€Π°Π·ΠΎΠΌ, ΠΎΠ±Ρ‰Π΅Π΅ ΡƒΡ€Π°Π²Π½Π΅Π½ΠΈΠ΅ мощности –

.

P = Ξ”E / Ρ‚

ΠŸΠΎΡ‚Π΅Ρ€Ρ (ΠΈΠ»ΠΈ усилСниС) энСргии – это просто ΠΏΡ€ΠΎΠΈΠ·Π²Π΅Π΄Π΅Π½ΠΈΠ΅ разности элСктричСских ΠΏΠΎΡ‚Π΅Π½Ρ†ΠΈΠ°Π»ΠΎΠ² ΠΌΠ΅ΠΆΠ΄Ρƒ двумя Ρ‚ΠΎΡ‡ΠΊΠ°ΠΌΠΈ ΠΈ количСства заряда, ΠΏΠ΅Ρ€Π΅ΠΌΠ΅Ρ‰Π°ΡŽΡ‰Π΅Π³ΠΎΡΡ ΠΌΠ΅ΠΆΠ΄Ρƒ этими двумя Ρ‚ΠΎΡ‡ΠΊΠ°ΠΌΠΈ Π·Π° ΠΏΠ΅Ρ€ΠΈΠΎΠ΄ Π²Ρ€Π΅ΠΌΠ΅Π½ΠΈ t.Π’Π°ΠΊΠΈΠΌ ΠΎΠ±Ρ€Π°Π·ΠΎΠΌ, потСря (ΠΈΠ»ΠΈ усилСниС) энСргии – это просто Ξ”V β€’ Q. Когда это Π²Ρ‹Ρ€Π°ΠΆΠ΅Π½ΠΈΠ΅ подставляСтся Π² Π²Ρ‹ΡˆΠ΅ΠΏΡ€ΠΈΠ²Π΅Π΄Π΅Π½Π½ΠΎΠ΅ ΡƒΡ€Π°Π²Π½Π΅Π½ΠΈΠ΅, ΡƒΡ€Π°Π²Π½Π΅Π½ΠΈΠ΅ мощности становится

P = Ξ”V β€’ Q / Ρ‚

ΠŸΠΎΡΠΊΠΎΠ»ΡŒΠΊΡƒ ΠΎΡ‚Π½ΠΎΡˆΠ΅Π½ΠΈΠ΅ Q / t, Π½Π°ΠΉΠ΄Π΅Π½Π½ΠΎΠ΅ Π² ΠΏΡ€ΠΈΠ²Π΅Π΄Π΅Π½Π½ΠΎΠΌ Π²Ρ‹ΡˆΠ΅ ΡƒΡ€Π°Π²Π½Π΅Π½ΠΈΠΈ, Ρ€Π°Π²Π½ΠΎ Ρ‚ΠΎΠΊΡƒ (I), ΠΏΡ€ΠΈΠ²Π΅Π΄Π΅Π½Π½ΠΎΠ΅ Π²Ρ‹ΡˆΠ΅ ΡƒΡ€Π°Π²Π½Π΅Π½ΠΈΠ΅ Ρ‚Π°ΠΊΠΆΠ΅ ΠΌΠΎΠΆΠ½ΠΎ Π·Π°ΠΏΠΈΡΠ°Ρ‚ΡŒ ΠΊΠ°ΠΊ

P = Ξ”V β€’ I

ΠšΠΎΠΌΠ±ΠΈΠ½ΠΈΡ€ΡƒΡ ΡƒΡ€Π°Π²Π½Π΅Π½ΠΈΠ΅ Π·Π°ΠΊΠΎΠ½Π° Ома с ΠΏΡ€ΠΈΠ²Π΅Π΄Π΅Π½Π½Ρ‹ΠΌ Π²Ρ‹ΡˆΠ΅ ΡƒΡ€Π°Π²Π½Π΅Π½ΠΈΠ΅ΠΌ, ΠΌΠΎΠΆΠ½ΠΎ ΠΏΠΎΠ»ΡƒΡ‡ΠΈΡ‚ΡŒ Π΄Π²Π° Π΄Ρ€ΡƒΠ³ΠΈΡ… уравнСния мощности. Π˜Ρ…

P = I 2 β€’ R P = Ξ”V 2 / R

Бтандартная мСтричСская Π΅Π΄ΠΈΠ½ΠΈΡ†Π° измСрСния мощности – Π’Ρ‚ .Π’ Π΅Π΄ΠΈΠ½ΠΈΡ†Π°Ρ… измСрСния Π²Π°Ρ‚Ρ‚ эквивалСнтСн ΡƒΡΠΈΠ»ΠΈΡ‚Π΅Π»ΡŽ β€’ Π’ΠΎΠ»ΡŒΡ‚, ΡƒΡΠΈΠ»ΠΈΡ‚Π΅Π»ΡŽ 2 β€’ Ом ΠΈ Π²ΠΎΠ»ΡŒΡ‚ 2 / Ом.

Π—Π°Ρ‚Ρ€Π°Ρ‚Ρ‹ Π½Π° ΡΠ»Π΅ΠΊΡ‚Ρ€ΠΎΡΠ½Π΅Ρ€Π³ΠΈΡŽ

ΠšΠΎΠΌΠΌΠ΅Ρ€Ρ‡Π΅ΡΠΊΠ°Ρ энСргСтичСская компания Π²Π·ΠΈΠΌΠ°Π΅Ρ‚ с домохозяйств Π΅ΠΆΠ΅ΠΌΠ΅ΡΡΡ‡Π½ΡƒΡŽ ΠΏΠ»Π°Ρ‚Ρƒ Π·Π° ΠΏΠΎΡΡ‚Π°Π²Π»Π΅Π½Π½ΡƒΡŽ ΡΠ»Π΅ΠΊΡ‚Ρ€ΠΎΡΠ½Π΅Ρ€Π³ΠΈΡŽ. Π’ счСтС Π·Π° услуги ΠΎΠ±Ρ‹Ρ‡Π½ΠΎ указываСтся количСство энСргии, ΠΏΠΎΡ‚Ρ€Π΅Π±Π»Π΅Π½Π½ΠΎΠΉ Π² Ρ‚Π΅Ρ‡Π΅Π½ΠΈΠ΅ мСсяца, Π² Π΅Π΄ΠΈΠ½ΠΈΡ†Π°Ρ… ΠΊΠΈΠ»ΠΎΠ²Π°Ρ‚Ρ‚ β€’ часов . Π­Ρ‚Π° Π΅Π΄ΠΈΠ½ΠΈΡ†Π° – Π΅Π΄ΠΈΠ½ΠΈΡ†Π° мощности, умноТСнная Π½Π° Π΅Π΄ΠΈΠ½ΠΈΡ†Ρƒ Π²Ρ€Π΅ΠΌΠ΅Π½ΠΈ, – это Π΅Π΄ΠΈΠ½ΠΈΡ†Π° энСргии.Домохозяйство ΠΎΠ±Ρ‹Ρ‡Π½ΠΎ ΠΎΠΏΠ»Π°Ρ‡ΠΈΠ²Π°Π΅Ρ‚ счСт Π½Π° основС количСства ΠΊΠ’Ρ‚ β€’ Ρ‡ элСктроэнСргии, ΠΏΠΎΡ‚Ρ€Π΅Π±Π»Π΅Π½Π½ΠΎΠΉ Π² Ρ‚Π΅Ρ‡Π΅Π½ΠΈΠ΅ мСсяца. Π’Π°ΠΊΠΈΠΌ ΠΎΠ±Ρ€Π°Π·ΠΎΠΌ, Π·Π°Π΄Π°Ρ‡Π° опрСдСлСния стоимости использования ΠΊΠΎΠ½ΠΊΡ€Π΅Ρ‚Π½ΠΎΠ³ΠΎ ΠΏΡ€ΠΈΠ±ΠΎΡ€Π° Π² Ρ‚Π΅Ρ‡Π΅Π½ΠΈΠ΅ Π·Π°Π΄Π°Π½Π½ΠΎΠ³ΠΎ ΠΏΠ΅Ρ€ΠΈΠΎΠ΄Π° Π²Ρ€Π΅ΠΌΠ΅Π½ΠΈ довольно проста. Π‘Π½Π°Ρ‡Π°Π»Π° Π½Π΅ΠΎΠ±Ρ…ΠΎΠ΄ΠΈΠΌΠΎ ΠΎΠΏΡ€Π΅Π΄Π΅Π»ΠΈΡ‚ΡŒ ΠΌΠΎΡ‰Π½ΠΎΡΡ‚ΡŒ ΠΈ ΠΏΡ€Π΅ΠΎΠ±Ρ€Π°Π·ΠΎΠ²Π°Ρ‚ΡŒ Π΅Π΅ Π² ΠΊΠΈΠ»ΠΎΠ²Π°Ρ‚Ρ‚Ρ‹. Π—Π°Ρ‚Π΅ΠΌ эту ΠΌΠΎΡ‰Π½ΠΎΡΡ‚ΡŒ Π½Π΅ΠΎΠ±Ρ…ΠΎΠ΄ΠΈΠΌΠΎ ΡƒΠΌΠ½ΠΎΠΆΠΈΡ‚ΡŒ Π½Π° врСмя использования Π² часах, Ρ‡Ρ‚ΠΎΠ±Ρ‹ ΠΏΠΎΠ»ΡƒΡ‡ΠΈΡ‚ΡŒ ΠΏΠΎΡ‚Ρ€Π΅Π±Π»ΡΠ΅ΠΌΡƒΡŽ ΡΠ½Π΅Ρ€Π³ΠΈΡŽ Π² Π΅Π΄ΠΈΠ½ΠΈΡ†Π°Ρ… ΠΊΠ’Ρ‚ β€’ час. НаконСц, это количСство энСргии Π΄ΠΎΠ»ΠΆΠ½ΠΎ Π±Ρ‹Ρ‚ΡŒ ΡƒΠΌΠ½ΠΎΠΆΠ΅Π½ΠΎ Π½Π° ΡΡ‚ΠΎΠΈΠΌΠΎΡΡ‚ΡŒ элСктроэнСргии Π² ΡΠΎΠΎΡ‚Π½ΠΎΡˆΠ΅Π½ΠΈΠΈ $ / ΠΊΠ’Ρ‚ β€’ час, Ρ‡Ρ‚ΠΎΠ±Ρ‹ ΠΎΠΏΡ€Π΅Π΄Π΅Π»ΠΈΡ‚ΡŒ ΡΡ‚ΠΎΠΈΠΌΠΎΡΡ‚ΡŒ Π² Π΄ΠΎΠ»Π»Π°Ρ€Π°Ρ….

Π­ΠΊΠ²ΠΈΠ²Π°Π»Π΅Π½Ρ‚Π½ΠΎΠ΅ сопротивлСниС

Π”ΠΎΠ²ΠΎΠ»ΡŒΠ½ΠΎ часто Π² Ρ†Π΅ΠΏΠΈ ΠΈΡΠΏΠΎΠ»ΡŒΠ·ΡƒΠ΅Ρ‚ΡΡ Π±ΠΎΠ»Π΅Π΅ ΠΎΠ΄Π½ΠΎΠ³ΠΎ рСзистора. Π₯отя ΠΊΠ°ΠΆΠ΄Ρ‹ΠΉ рСзистор ΠΈΠΌΠ΅Π΅Ρ‚ собствСнноС ΠΈΠ½Π΄ΠΈΠ²ΠΈΠ΄ΡƒΠ°Π»ΡŒΠ½ΠΎΠ΅ Π·Π½Π°Ρ‡Π΅Π½ΠΈΠ΅ сопротивлСния, ΠΎΠ±Ρ‰Π΅Π΅ сопротивлСниС Ρ†Π΅ΠΏΠΈ отличаСтся ΠΎΡ‚ сопротивлСния ΠΎΡ‚Π΄Π΅Π»ΡŒΠ½Ρ‹Ρ… рСзисторов, ΡΠΎΡΡ‚Π°Π²Π»ΡΡŽΡ‰ΠΈΡ… Ρ†Π΅ΠΏΡŒ. Π’Π΅Π»ΠΈΡ‡ΠΈΠ½Π°, извСстная ΠΊΠ°ΠΊ эквивалСнтноС сопротивлСниС , ΡƒΠΊΠ°Π·Ρ‹Π²Π°Π΅Ρ‚ ΠΏΠΎΠ»Π½ΠΎΠ΅ сопротивлСниС Ρ†Π΅ΠΏΠΈ.ΠšΠΎΠ½Ρ†Π΅ΠΏΡ‚ΡƒΠ°Π»ΡŒΠ½ΠΎ эквивалСнтноС сопротивлСниС – это сопротивлСниС, ΠΊΠΎΡ‚ΠΎΡ€ΠΎΠ΅ ΠΎΠ΄ΠΈΠ½ рСзистор Π±ΡƒΠ΄Π΅Ρ‚ ΠΈΠΌΠ΅Ρ‚ΡŒ, Ρ‡Ρ‚ΠΎΠ±Ρ‹ ΠΎΠΊΠ°Π·Ρ‹Π²Π°Ρ‚ΡŒ Ρ‚Π°ΠΊΠΎΠ΅ ΠΆΠ΅ ΠΎΠ±Ρ‰Π΅Π΅ влияниС Π½Π° сопротивлСниС, ΠΊΠ°ΠΊ ΠΈ комбинация рСзисторов, ΠΊΠΎΡ‚ΠΎΡ€Ρ‹Π΅ ΠΏΡ€ΠΈΡΡƒΡ‚ΡΡ‚Π²ΡƒΡŽΡ‚. Π’Π°ΠΊΠΈΠΌ ΠΎΠ±Ρ€Π°Π·ΠΎΠΌ, Ссли Π² схСмС Π΅ΡΡ‚ΡŒ Ρ‚Ρ€ΠΈ рСзистора с эквивалСнтным сопротивлСниСм 25 Ом, Ρ‚ΠΎ ΠΎΠ΄ΠΈΠ½ рСзистор Π½Π° 25 Ом ΠΌΠΎΠΆΠ΅Ρ‚ Π·Π°ΠΌΠ΅Π½ΠΈΡ‚ΡŒ Ρ‚Ρ€ΠΈ ΠΎΡ‚Π΄Π΅Π»ΡŒΠ½Ρ‹Ρ… рСзистора ΠΈ ΠΎΠΊΠ°Π·Π°Ρ‚ΡŒ влияниС Π½Π° схСму, эквивалСнтноС эквивалСнту . Π—Π½Π°Ρ‡Π΅Π½ΠΈΠ΅ эквивалСнтного сопротивлСния (R eq ) ΡƒΡ‡ΠΈΡ‚Ρ‹Π²Π°Π΅Ρ‚ ΠΈΠ½Π΄ΠΈΠ²ΠΈΠ΄ΡƒΠ°Π»ΡŒΠ½Ρ‹Π΅ значСния сопротивлСния рСзисторов ΠΈ способ ΠΈΡ… ΠΏΠΎΠ΄ΠΊΠ»ΡŽΡ‡Π΅Π½ΠΈΡ.

Π•ΡΡ‚ΡŒ Π΄Π²Π° основных способа Π²ΠΊΠ»ΡŽΡ‡Π΅Π½ΠΈΡ рСзисторов Π² ΡΠ»Π΅ΠΊΡ‚Ρ€ΠΈΡ‡Π΅ΡΠΊΡƒΡŽ Ρ†Π΅ΠΏΡŒ. Они ΠΌΠΎΠ³ΡƒΡ‚ Π±Ρ‹Ρ‚ΡŒ ΠΏΠΎΠ΄ΠΊΠ»ΡŽΡ‡Π΅Π½Ρ‹ ΠΏΠΎΡΠ»Π΅Π΄ΠΎΠ²Π°Ρ‚Π΅Π»ΡŒΠ½ΠΎ ΠΈΠ»ΠΈ ΠΏΠ°Ρ€Π°Π»Π»Π΅Π»ΡŒΠ½ΠΎ . РСзисторы, ΠΊΠΎΡ‚ΠΎΡ€Ρ‹Π΅ соСдинСны ΠΏΠΎΡΠ»Π΅Π΄ΠΎΠ²Π°Ρ‚Π΅Π»ΡŒΠ½ΠΎ, ΠΏΠΎΠ΄ΠΊΠ»ΡŽΡ‡Π°ΡŽΡ‚ΡΡ ΠΏΠΎΡΠ»Π΅Π΄ΠΎΠ²Π°Ρ‚Π΅Π»ΡŒΠ½ΠΎ, Ρ‚Π°ΠΊ Ρ‡Ρ‚ΠΎ вСсь заряд, ΠΊΠΎΡ‚ΠΎΡ€Ρ‹ΠΉ ΠΏΡ€ΠΎΡ…ΠΎΠ΄ΠΈΡ‚ Ρ‡Π΅Ρ€Π΅Π· ΠΏΠ΅Ρ€Π²Ρ‹ΠΉ рСзистор, Ρ‚Π°ΠΊΠΆΠ΅ ΠΏΡ€ΠΎΡ…ΠΎΠ΄ΠΈΡ‚ Ρ‡Π΅Ρ€Π΅Π· Π΄Ρ€ΡƒΠ³ΠΈΠ΅ рСзисторы. ΠŸΡ€ΠΈ ΠΏΠΎΡΠ»Π΅Π΄ΠΎΠ²Π°Ρ‚Π΅Π»ΡŒΠ½ΠΎΠΌ соСдинСнии вСсь заряд, ΠΏΡ€ΠΎΡ‚Π΅ΠΊΠ°ΡŽΡ‰ΠΈΠΉ ΠΏΠΎ Ρ†Π΅ΠΏΠΈ, ΠΏΡ€ΠΎΡ…ΠΎΠ΄ΠΈΡ‚ Ρ‡Π΅Ρ€Π΅Π· всС ΠΎΡ‚Π΄Π΅Π»ΡŒΠ½Ρ‹Π΅ рСзисторы. Π’Π°ΠΊΠΈΠΌ ΠΎΠ±Ρ€Π°Π·ΠΎΠΌ, эквивалСнтноС сопротивлСниС ΠΏΠΎΡΠ»Π΅Π΄ΠΎΠ²Π°Ρ‚Π΅Π»ΡŒΠ½ΠΎ соСдинСнных рСзисторов являСтся суммой Π·Π½Π°Ρ‡Π΅Π½ΠΈΠΉ ΠΎΡ‚Π΄Π΅Π»ΡŒΠ½Ρ‹Ρ… сопротивлСний этих рСзисторов.

R экв = R 1 + R 2 + R 3 +… (ΠΏΠΎΡΠ»Π΅Π΄ΠΎΠ²Π°Ρ‚Π΅Π»ΡŒΠ½Ρ‹Π΅ соСдинСния)

ΠŸΠ°Ρ€Π°Π»Π»Π΅Π»ΡŒΠ½ΠΎ ΠΏΠΎΠ΄ΠΊΠ»ΡŽΡ‡Π΅Π½Π½Ρ‹Π΅ рСзисторы ΠΏΠΎΠ΄ΠΊΠ»ΡŽΡ‡Π°ΡŽΡ‚ΡΡ Π±ΠΎΠΊ ΠΎ Π±ΠΎΠΊ, Ρ‚Π°ΠΊ Ρ‡Ρ‚ΠΎ заряд, ΠΏΡ€ΠΈΠ±Π»ΠΈΠΆΠ°ΡŽΡ‰ΠΈΠΉΡΡ ΠΊ рСзисторам, раздСляСтся Π½Π° Π΄Π²Π° ΠΈΠ»ΠΈ Π±ΠΎΠ»Π΅Π΅ Ρ€Π°Π·Π½Ρ‹Ρ… ΠΏΡƒΡ‚ΠΈ. ΠŸΠ°Ρ€Π°Π»Π»Π΅Π»ΡŒΠ½ΠΎ ΠΏΠΎΠ΄ΠΊΠ»ΡŽΡ‡Π΅Π½Π½Ρ‹Π΅ рСзисторы Ρ…Π°Ρ€Π°ΠΊΡ‚Π΅Ρ€ΠΈΠ·ΡƒΡŽΡ‚ΡΡ Π½Π°Π»ΠΈΡ‡ΠΈΠ΅ΠΌ участков развСтвлСния, Π² ΠΊΠΎΡ‚ΠΎΡ€Ρ‹Ρ… заряд развСтвляСтся ΠΏΠΎ Ρ€Π°Π·Π½Ρ‹ΠΌ путям. Заряд, ΠΊΠΎΡ‚ΠΎΡ€Ρ‹ΠΉ ΠΏΡ€ΠΎΡ…ΠΎΠ΄ΠΈΡ‚ Ρ‡Π΅Ρ€Π΅Π· ΠΎΠ΄ΠΈΠ½ рСзистор, Π½Π΅ ΠΏΡ€ΠΎΡ…ΠΎΠ΄ΠΈΡ‚ Ρ‡Π΅Ρ€Π΅Π· Π΄Ρ€ΡƒΠ³ΠΈΠ΅ рСзисторы.Π­ΠΊΠ²ΠΈΠ²Π°Π»Π΅Π½Ρ‚Π½ΠΎΠ΅ сопротивлСниС ΠΏΠ°Ρ€Π°Π»Π»Π΅Π»ΡŒΠ½ΠΎ Π²ΠΊΠ»ΡŽΡ‡Π΅Π½Π½Ρ‹Ρ… рСзисторов мСньшС Π·Π½Π°Ρ‡Π΅Π½ΠΈΠΉ сопротивлСний всСх ΠΎΡ‚Π΄Π΅Π»ΡŒΠ½Ρ‹Ρ… рСзисторов Π² Ρ†Π΅ΠΏΠΈ. Π₯отя это ΠΌΠΎΠΆΠ΅Ρ‚ Π±Ρ‹Ρ‚ΡŒ Π½Π΅ совсСм ΠΈΠ½Ρ‚ΡƒΠΈΡ‚ΠΈΠ²Π½ΠΎ понятным, ΡƒΡ€Π°Π²Π½Π΅Π½ΠΈΠ΅ эквивалСнтного сопротивлСния ΠΏΠ°Ρ€Π°Π»Π»Π΅Π»ΡŒΠ½ΠΎ соСдинСнных рСзисторов даСтся ΡƒΡ€Π°Π²Π½Π΅Π½ΠΈΠ΅ΠΌ с нСсколькими Π²Π·Π°ΠΈΠΌΠ½Ρ‹ΠΌΠΈ Ρ‡Π»Π΅Π½Π°ΠΌΠΈ.

1 / R экв. = 1 / R 1 + 1 / R 2 + 1 / R 3 +… (ΠΏΠ°Ρ€Π°Π»Π»Π΅Π»ΡŒΠ½ΠΎΠ΅ соСдинСниС)

Анализ ΠΏΠΎΡΠ»Π΅Π΄ΠΎΠ²Π°Ρ‚Π΅Π»ΡŒΠ½ΠΎΠΉ Ρ†Π΅ΠΏΠΈ

НСкоторыС ΠΏΡ€ΠΎΠ±Π»Π΅ΠΌΡ‹ Π²Ρ‚ΠΎΡ€ΠΎΠΉ ΠΏΠΎΠ»ΠΎΠ²ΠΈΠ½Ρ‹ этого Π½Π°Π±ΠΎΡ€Π° относятся ΠΊ ΠΏΠΎΡΠ»Π΅Π΄ΠΎΠ²Π°Ρ‚Π΅Π»ΡŒΠ½Ρ‹ΠΌ цСпям.НСрСдко ΠΏΡ€ΠΎΠ±Π»Π΅ΠΌΠ° сопровоТдаСтся рисунком ΠΈΠ»ΠΈ схСматичСской Π΄ΠΈΠ°Π³Ρ€Π°ΠΌΠΌΠΎΠΉ, ΠΏΠΎΠΊΠ°Π·Ρ‹Π²Π°ΡŽΡ‰Π΅ΠΉ располоТСниС Π±Π°Ρ‚Π°Ρ€Π΅ΠΉ ΠΈ рСзисторов. Π§Π΅Ρ€Ρ‚Π΅ΠΆ ΠΈ ΡΠΎΠΎΡ‚Π²Π΅Ρ‚ΡΡ‚Π²ΡƒΡŽΡ‰Π°Ρ ΠΏΡ€ΠΈΠ½Ρ†ΠΈΠΏΠΈΠ°Π»ΡŒΠ½Π°Ρ схСма Π½ΠΈΠΆΠ΅ ΠΏΡ€Π΅Π΄ΡΡ‚Π°Π²Π»ΡΡŽΡ‚ ΠΏΠΎΡΠ»Π΅Π΄ΠΎΠ²Π°Ρ‚Π΅Π»ΡŒΠ½ΡƒΡŽ Ρ†Π΅ΠΏΡŒ, ΠΏΠΈΡ‚Π°Π΅ΠΌΡƒΡŽ трСмя ячСйками ΠΈ ΠΈΠΌΠ΅ΡŽΡ‰ΡƒΡŽ Ρ‚Ρ€ΠΈ ΠΏΠΎΡΠ»Π΅Π΄ΠΎΠ²Π°Ρ‚Π΅Π»ΡŒΠ½ΠΎ соСдинСнных рСзистора (Π»Π°ΠΌΠΏΠΎΡ‡ΠΊΠΈ).

Если ΠΏΡ€Π΅Π΄ΡΡ‚Π°Π²ΠΈΡ‚ΡŒ сСбС заряд, ΠΏΠΎΠΊΠΈΠ΄Π°ΡŽΡ‰ΠΈΠΉ ΠΏΠΎΠ»ΠΎΠΆΠΈΡ‚Π΅Π»ΡŒΠ½Ρ‹ΠΉ полюс Π±Π°Ρ‚Π°Ρ€Π΅ΠΈ ΠΈ ΡΠ»Π΅Π΄ΡƒΡŽΡ‰ΠΈΠΉ ΠΏΠΎ своСму ΠΏΡƒΡ‚ΠΈ, ΠΊΠΎΠ³Π΄Π° ΠΎΠ½ пСрСсСкаСт ΠΏΠΎΠ»Π½Ρ‹ΠΉ ΠΊΠΎΠ½Ρ‚ΡƒΡ€, становится ΠΎΡ‡Π΅Π²ΠΈΠ΄Π½Ρ‹ΠΌ, Ρ‡Ρ‚ΠΎ заряд ΠΏΡ€ΠΎΡ…ΠΎΠ΄ΠΈΡ‚ Ρ‡Π΅Ρ€Π΅Π· всС рСзисторы ΠΏΠΎΡΠ»Π΅Π΄ΠΎΠ²Π°Ρ‚Π΅Π»ΡŒΠ½ΠΎ.Π’Π°ΠΊΠΈΠΌ ΠΎΠ±Ρ€Π°Π·ΠΎΠΌ, ΠΎΠ½ соотвСтствуСт критСриям ΠΏΠΎΡΠ»Π΅Π΄ΠΎΠ²Π°Ρ‚Π΅Π»ΡŒΠ½ΠΎΠΉ Ρ†Π΅ΠΏΠΈ. Π—Π½Π°Π½ΠΈΠ΅ Ρ‚ΠΎΠ³ΠΎ, Ρ‡Ρ‚ΠΎ схСма являСтся ΠΏΠΎΡΠ»Π΅Π΄ΠΎΠ²Π°Ρ‚Π΅Π»ΡŒΠ½ΠΎΠΉ, позволяСт ΡΠ²ΡΠ·Π°Ρ‚ΡŒ ΠΎΠ±Ρ‰Π΅Π΅ ΠΈΠ»ΠΈ эквивалСнтноС сопротивлСниС Ρ†Π΅ΠΏΠΈ с ΠΎΡ‚Π΄Π΅Π»ΡŒΠ½Ρ‹ΠΌΠΈ значСниями сопротивлСния с ΠΏΠΎΠΌΠΎΡ‰ΡŒΡŽ уравнСния эквивалСнтного сопротивлСния, описанного Π²Ρ‹ΡˆΠ΅.

R экв = R 1 + R 2 + R 3 +… (ΠΏΠΎΡΠ»Π΅Π΄ΠΎΠ²Π°Ρ‚Π΅Π»ΡŒΠ½Ρ‹Π΅ соСдинСния)

Π’ΠΎΠΊ ΠΏΠΎΡΠ»Π΅Π΄ΠΎΠ²Π°Ρ‚Π΅Π»ΡŒΠ½ΠΎΠΉ Ρ†Π΅ΠΏΠΈ Π² рСзисторах Ρ‚Π°ΠΊΠΎΠΉ ΠΆΠ΅, ΠΊΠ°ΠΊ ΠΈ Π² Π±Π°Ρ‚Π°Ρ€Π΅Π΅. ΠŸΠΎΡΠΊΠΎΠ»ΡŒΠΊΡƒ Π½Π΅Ρ‚ ΠΎΡ‚Π²Π΅Ρ‚Π²Π»Π΅Π½ΠΈΠΉ Π² мСстах, Π³Π΄Π΅ заряд дСлится Π½Π° ΠΏΡƒΡ‚ΠΈ, ΠΌΠΎΠΆΠ½ΠΎ ΡΠΊΠ°Π·Π°Ρ‚ΡŒ, Ρ‡Ρ‚ΠΎ Ρ‚ΠΎΠΊ Π² Π±Π°Ρ‚Π°Ρ€Π΅Π΅ Ρ€Π°Π²Π΅Π½ Ρ‚ΠΎΠΊΡƒ Π² рСзисторС 1, Ρ€Π°Π²Π΅Π½ Ρ‚ΠΎΠΊΡƒ Π² рСзисторС 2 ΠΈ Ρ€Π°Π²Π΅Π½ Ρ‚ΠΎΠΊΡƒ Π² рСзисторС 3…. Π’ Ρ„ΠΎΡ€ΠΌΠ΅ уравнСния ΠΌΠΎΠΆΠ½ΠΎ Π·Π°ΠΏΠΈΡΠ°Ρ‚ΡŒ, Ρ‡Ρ‚ΠΎ

I аккумулятор = I 1 = I 2 = I 3 =… (ΠΏΠΎΡΠ»Π΅Π΄ΠΎΠ²Π°Ρ‚Π΅Π»ΡŒΠ½Ρ‹Π΅ Ρ†Π΅ΠΏΠΈ)

Когда заряд ΠΏΡ€ΠΎΡ…ΠΎΠ΄ΠΈΡ‚ Ρ‡Π΅Ρ€Π΅Π· рСзисторы Π² ΠΏΠΎΡΠ»Π΅Π΄ΠΎΠ²Π°Ρ‚Π΅Π»ΡŒΠ½ΠΎΠΉ Ρ†Π΅ΠΏΠΈ, происходит ΠΏΠ°Π΄Π΅Π½ΠΈΠ΅ элСктричСского ΠΏΠΎΡ‚Π΅Π½Ρ†ΠΈΠ°Π»Π°, ΠΊΠΎΠ³Π΄Π° ΠΎΠ½ ΠΏΡ€ΠΎΡ…ΠΎΠ΄ΠΈΡ‚ Ρ‡Π΅Ρ€Π΅Π· ΠΊΠ°ΠΆΠ΄Ρ‹ΠΉ рСзистор. Π­Ρ‚ΠΎ ΠΏΠ°Π΄Π΅Π½ΠΈΠ΅ элСктричСского ΠΏΠΎΡ‚Π΅Π½Ρ†ΠΈΠ°Π»Π° Π½Π° ΠΊΠ°ΠΆΠ΄ΠΎΠΌ рСзисторС опрСдСляСтся Ρ‚ΠΎΠΊΠΎΠΌ Ρ‡Π΅Ρ€Π΅Π· рСзистор ΠΈ сопротивлСниСм рСзистора. Π­Ρ‚ΠΎ согласуСтся с ΡƒΡ€Π°Π²Π½Π΅Π½ΠΈΠ΅ΠΌ Π·Π°ΠΊΠΎΠ½Π° Ома, описанным Π²Ρ‹ΡˆΠ΅ (Ξ”V = I β€’ R).ΠŸΠΎΡΠΊΠΎΠ»ΡŒΠΊΡƒ Ρ‚ΠΎΠΊ (I) Π² ΠΊΠ°ΠΆΠ΄ΠΎΠΌ ΠΎΡ‚Π΄Π΅Π»ΡŒΠ½ΠΎΠΌ рСзисторС ΠΎΠ΄ΠΈΠ½Π°ΠΊΠΎΠ², Π»ΠΎΠ³ΠΈΡ‡Π½ΠΎ ΡΠ΄Π΅Π»Π°Ρ‚ΡŒ Π²Ρ‹Π²ΠΎΠ΄, Ρ‡Ρ‚ΠΎ рСзисторы с наибольшим сопротивлСниСм (R) Π±ΡƒΠ΄ΡƒΡ‚ ΠΈΠΌΠ΅Ρ‚ΡŒ Π½Π°ΠΈΠ±ΠΎΠ»ΡŒΡˆΡƒΡŽ Ρ€Π°Π·Π½ΠΎΡΡ‚ΡŒ элСктричСских ΠΏΠΎΡ‚Π΅Π½Ρ†ΠΈΠ°Π»ΠΎΠ² (Ξ”V), ΠΏΡ€ΠΈΠ»ΠΎΠΆΠ΅Π½Π½ΡƒΡŽ ΠΊ Π½ΠΈΠΌ.

Π Π°Π·Π½ΠΎΡΡ‚ΡŒ элСктричСских ΠΏΠΎΡ‚Π΅Π½Ρ†ΠΈΠ°Π»ΠΎΠ² Π½Π° ΠΎΡ‚Π΄Π΅Π»ΡŒΠ½Ρ‹Ρ… рСзисторах Ρ†Π΅ΠΏΠΈ часто обозначаСтся ΠΊΠ°ΠΊ падСния напряТСния . Π­Ρ‚ΠΈ падСния напряТСния ΠΏΠΎΡΠ»Π΅Π΄ΠΎΠ²Π°Ρ‚Π΅Π»ΡŒΠ½ΠΎ соСдинСнных рСзисторов матСматичСски связаны с элСктричСским ΠΏΠΎΡ‚Π΅Π½Ρ†ΠΈΠ°Π»ΠΎΠΌ ΠΈΠ»ΠΈ Π½ΠΎΠΌΠΈΠ½Π°Π»ΡŒΠ½Ρ‹ΠΌ напряТСниСм элСмСнтов ΠΈΠ»ΠΈ Π±Π°Ρ‚Π°Ρ€Π΅ΠΈ, ΠΊΠΎΡ‚ΠΎΡ€Ρ‹Π΅ ΠΏΠΈΡ‚Π°ΡŽΡ‚ Ρ†Π΅ΠΏΡŒ.Если заряд ΠΏΡ€ΠΈΠΎΠ±Ρ€Π΅Ρ‚Π°Π΅Ρ‚ 12 Π’ элСктричСского ΠΏΠΎΡ‚Π΅Π½Ρ†ΠΈΠ°Π»Π° ΠΏΡ€ΠΈ ΠΏΡ€ΠΎΡ…ΠΎΠΆΠ΄Π΅Π½ΠΈΠΈ Ρ‡Π΅Ρ€Π΅Π· Π±Π°Ρ‚Π°Ρ€Π΅ΡŽ элСктричСской Ρ†Π΅ΠΏΠΈ, Ρ‚ΠΎ ΠΎΠ½ тСряСт 12 Π’ ΠΏΡ€ΠΈ ΠΏΡ€ΠΎΡ…ΠΎΠΆΠ΄Π΅Π½ΠΈΠΈ Ρ‡Π΅Ρ€Π΅Π· внСшнюю Ρ†Π΅ΠΏΡŒ. Π­Ρ‚ΠΎ ΠΏΠ°Π΄Π΅Π½ΠΈΠ΅ элСктричСского ΠΏΠΎΡ‚Π΅Π½Ρ†ΠΈΠ°Π»Π° Π½Π° 12 Π’ являСтся Ρ€Π΅Π·ΡƒΠ»ΡŒΡ‚Π°Ρ‚ΠΎΠΌ сСрии ΠΎΡ‚Π΄Π΅Π»ΡŒΠ½Ρ‹Ρ… ΠΏΠ°Π΄Π΅Π½ΠΈΠΉ элСктричСского ΠΏΠΎΡ‚Π΅Π½Ρ†ΠΈΠ°Π»Π°, проходящих Ρ‡Π΅Ρ€Π΅Π· ΠΎΡ‚Π΄Π΅Π»ΡŒΠ½Ρ‹Π΅ рСзисторы ΠΏΠΎΡΠ»Π΅Π΄ΠΎΠ²Π°Ρ‚Π΅Π»ΡŒΠ½ΠΎΠΉ Ρ†Π΅ΠΏΠΈ. Π­Ρ‚ΠΈ ΠΎΡ‚Π΄Π΅Π»ΡŒΠ½Ρ‹Π΅ падСния напряТСния (Ρ€Π°Π·Π½ΠΎΡΡ‚ΡŒ элСктричСских ΠΏΠΎΡ‚Π΅Π½Ρ†ΠΈΠ°Π»ΠΎΠ²) Π² суммС Π΄Π°ΡŽΡ‚ ΠΎΠ±Ρ‰Π΅Π΅ ΠΏΠ°Π΄Π΅Π½ΠΈΠ΅ напряТСния Π² Ρ†Π΅ΠΏΠΈ. Π’ Ρ„ΠΎΡ€ΠΌΠ΅ уравнСния ΠΌΠΎΠΆΠ½ΠΎ ΡΠΊΠ°Π·Π°Ρ‚ΡŒ, Ρ‡Ρ‚ΠΎ

Ξ”V аккумулятор = Ξ”V 1 + Ξ”V 2 + Ξ”V 3 +… (ΠΏΠΎΡΠ»Π΅Π΄ΠΎΠ²Π°Ρ‚Π΅Π»ΡŒΠ½Ρ‹Π΅ Ρ†Π΅ΠΏΠΈ)

Π³Π΄Π΅ Ξ”V аккумулятор – элСктричСский ΠΏΠΎΡ‚Π΅Π½Ρ†ΠΈΠ°Π», Π½Π°ΠΊΠΎΠΏΠ»Π΅Π½Π½Ρ‹ΠΉ Π² аккумуляторС, Π° Ξ”V 1 , Ξ”V 2 ΠΈ Ξ”V 3 – падСния напряТСния (ΠΈΠ»ΠΈ разности элСктричСских ΠΏΠΎΡ‚Π΅Π½Ρ†ΠΈΠ°Π»ΠΎΠ²) Π½Π° ΠΎΡ‚Π΄Π΅Π»ΡŒΠ½Ρ‹Ρ… рСзисторах.

Π‘ΠΎΠ»Π΅Π΅ ΠΏΠΎΠ΄Ρ€ΠΎΠ±Π½ΠΎΠ΅ ΠΈ ΠΈΡΡ‡Π΅Ρ€ΠΏΡ‹Π²Π°ΡŽΡ‰Π΅Π΅ обсуТдСниС ΠΏΠΎΡΠ»Π΅Π΄ΠΎΠ²Π°Ρ‚Π΅Π»ΡŒΠ½Ρ‹Ρ… схСм ΠΈ ΠΈΡ… Π°Π½Π°Π»ΠΈΠ·Π° ΠΌΠΎΠΆΠ½ΠΎ Π½Π°ΠΉΡ‚ΠΈ Π² Π£Ρ‡Π΅Π±Π½ΠΎΠΌ пособии ΠΏΠΎ Ρ„ΠΈΠ·ΠΈΠΊΠ΅.

Анализ ΠΏΠ°Ρ€Π°Π»Π»Π΅Π»ΡŒΠ½Ρ‹Ρ… Ρ†Π΅ΠΏΠ΅ΠΉ

Π‘Π°ΠΌΡ‹Π΅ послСдниС ΠΏΡ€ΠΎΠ±Π»Π΅ΠΌΡ‹ Π² этом Π½Π°Π±ΠΎΡ€Π΅ относятся ΠΊ ΠΏΠ°Ρ€Π°Π»Π»Π΅Π»ΡŒΠ½Ρ‹ΠΌ цСпям. ΠžΠΏΡΡ‚ΡŒ ΠΆΠ΅, Π½Π΅Ρ‚ Π½ΠΈΡ‡Π΅Π³ΠΎ Π½Π΅ΠΎΠ±Ρ‹Ρ‡Π½ΠΎΠ³ΠΎ Π² Ρ‚ΠΎΠΌ, Ρ‡Ρ‚ΠΎ ΠΏΡ€ΠΎΠ±Π»Π΅ΠΌΠ° сопровоТдаСтся Ρ‡Π΅Ρ€Ρ‚Π΅ΠΆΠΎΠΌ ΠΈΠ»ΠΈ схСматичСской Π΄ΠΈΠ°Π³Ρ€Π°ΠΌΠΌΠΎΠΉ, ΠΏΠΎΠΊΠ°Π·Ρ‹Π²Π°ΡŽΡ‰Π΅ΠΉ располоТСниС Π±Π°Ρ‚Π°Ρ€Π΅ΠΉ ΠΈ рСзисторов.Π§Π΅Ρ€Ρ‚Π΅ΠΆ ΠΈ ΡΠΎΠΎΡ‚Π²Π΅Ρ‚ΡΡ‚Π²ΡƒΡŽΡ‰Π°Ρ ΠΏΡ€ΠΈΠ½Ρ†ΠΈΠΏΠΈΠ°Π»ΡŒΠ½Π°Ρ схСма Π½ΠΈΠΆΠ΅ ΠΏΡ€Π΅Π΄ΡΡ‚Π°Π²Π»ΡΡŽΡ‚ собой ΠΏΠ°Ρ€Π°Π»Π»Π΅Π»ΡŒΠ½ΡƒΡŽ Ρ†Π΅ΠΏΡŒ с ΠΏΠΈΡ‚Π°Π½ΠΈΠ΅ΠΌ ΠΎΡ‚ Ρ‚Ρ€Π΅Ρ… ячССк ΠΈ ΠΈΠΌΠ΅ΡŽΡ‰ΡƒΡŽ Ρ‚Ρ€ΠΈ ΠΏΠ°Ρ€Π°Π»Π»Π΅Π»ΡŒΠ½ΠΎ соСдинСнных рСзистора (Π»Π°ΠΌΠΏΠΎΡ‡ΠΊΠΈ).

Если ΠΏΡ€Π΅Π΄ΡΡ‚Π°Π²ΠΈΡ‚ΡŒ заряд, ΠΏΠΎΠΊΠΈΠ΄Π°ΡŽΡ‰ΠΈΠΉ ΠΏΠΎΠ»ΠΎΠΆΠΈΡ‚Π΅Π»ΡŒΠ½Ρ‹ΠΉ полюс Π±Π°Ρ‚Π°Ρ€Π΅ΠΈ ΠΈ ΡΠ»Π΅Π΄ΡƒΡŽΡ‰ΠΈΠΉ ΠΏΠΎ Π΅Π³ΠΎ ΠΏΡƒΡ‚ΠΈ, ΠΊΠΎΠ³Π΄Π° ΠΎΠ½ ΠΏΡ€ΠΎΡ…ΠΎΠ΄ΠΈΡ‚ Ρ‡Π΅Ρ€Π΅Π· ΠΏΠΎΠ»Π½Ρ‹ΠΉ ΠΊΠΎΠ½Ρ‚ΡƒΡ€, становится ΠΎΡ‡Π΅Π²ΠΈΠ΄Π½Ρ‹ΠΌ, Ρ‡Ρ‚ΠΎ заряд достигаСт мСста развСтвлСния Π΄ΠΎ Ρ‚ΠΎΠ³ΠΎ, ΠΊΠ°ΠΊ достигнСт рСзистора. Π’ мСстС развСтвлСния, ΠΊΠΎΡ‚ΠΎΡ€ΠΎΠ΅ ΠΈΠ½ΠΎΠ³Π΄Π° Π½Π°Π·Ρ‹Π²Π°ΡŽΡ‚ ΡƒΠ·Π»ΠΎΠΌ, заряд ΠΏΡ€ΠΎΡ…ΠΎΠ΄ΠΈΡ‚ ΠΏΠΎ ΠΎΠ΄Π½ΠΎΠΌΡƒ ΠΈΠ· Ρ‚Ρ€Π΅Ρ… Π²ΠΎΠ·ΠΌΠΎΠΆΠ½Ρ‹Ρ… ΠΏΡƒΡ‚Π΅ΠΉ Ρ‡Π΅Ρ€Π΅Π· рСзисторы.ВмСсто Ρ‚ΠΎΠ³ΠΎ, Ρ‡Ρ‚ΠΎΠ±Ρ‹ ΠΏΡ€ΠΎΡ…ΠΎΠ΄ΠΈΡ‚ΡŒ Ρ‡Π΅Ρ€Π΅Π· ΠΊΠ°ΠΆΠ΄Ρ‹ΠΉ рСзистор, ΠΎΠ΄ΠΈΠ½ заряд Π±ΡƒΠ΄Π΅Ρ‚ ΠΏΡ€ΠΎΡ…ΠΎΠ΄ΠΈΡ‚ΡŒ Ρ‡Π΅Ρ€Π΅Π· СдинствСнный рСзистор Π²ΠΎ врСмя ΠΏΠΎΠ»Π½ΠΎΠ³ΠΎ Ρ†ΠΈΠΊΠ»Π° Π²ΠΎΠΊΡ€ΡƒΠ³ Ρ†Π΅ΠΏΠΈ. Π’Π°ΠΊΠΈΠΌ ΠΎΠ±Ρ€Π°Π·ΠΎΠΌ, ΠΎΠ½ соотвСтствуСт критСриям ΠΏΠ°Ρ€Π°Π»Π»Π΅Π»ΡŒΠ½ΠΎΠΉ Ρ†Π΅ΠΏΠΈ. Π—Π½Π°Π½ΠΈΠ΅ Ρ‚ΠΎΠ³ΠΎ, Ρ‡Ρ‚ΠΎ схСма являСтся ΠΏΠ°Ρ€Π°Π»Π»Π΅Π»ΡŒΠ½ΠΎΠΉ, позволяСт ΡΠ²ΡΠ·Π°Ρ‚ΡŒ ΠΎΠ±Ρ‰Π΅Π΅ ΠΈΠ»ΠΈ эквивалСнтноС сопротивлСниС Ρ†Π΅ΠΏΠΈ с ΠΎΡ‚Π΄Π΅Π»ΡŒΠ½Ρ‹ΠΌΠΈ значСниями сопротивлСния с ΠΏΠΎΠΌΠΎΡ‰ΡŒΡŽ уравнСния эквивалСнтного сопротивлСния, описанного Π²Ρ‹ΡˆΠ΅.

1 / R экв. = 1 / R 1 + 1 / R 2 + 1 / R 3 +… (ΠΏΠ°Ρ€Π°Π»Π»Π΅Π»ΡŒΠ½ΠΎΠ΅ соСдинСниС)

Π’ мСстС развСтвлСния заряд раздСляСтся Π½Π° ΠΎΡ‚Π΄Π΅Π»ΡŒΠ½Ρ‹Π΅ ΠΏΡƒΡ‚ΠΈ.Π’Π°ΠΊΠΈΠΌ ΠΎΠ±Ρ€Π°Π·ΠΎΠΌ, Ρ‚ΠΎΠΊ Π² ΠΎΡ‚Π΄Π΅Π»ΡŒΠ½Ρ‹Ρ… путях Π±ΡƒΠ΄Π΅Ρ‚ мСньшС, Ρ‡Π΅ΠΌ Ρ‚ΠΎΠΊ Π²Π½Π΅ ΠΏΡƒΡ‚Π΅ΠΉ. ΠžΠ±Ρ‰ΠΈΠΉ Ρ‚ΠΎΠΊ Π² Ρ†Π΅ΠΏΠΈ ΠΈ Ρ‚ΠΎΠΊ Π² Π±Π°Ρ‚Π°Ρ€Π΅Π΅ Ρ€Π°Π²Π½Ρ‹ суммС Ρ‚ΠΎΠΊΠ° Π² ΠΎΡ‚Π΄Π΅Π»ΡŒΠ½Ρ‹Ρ… цСпях. Π’ Ρ„ΠΎΡ€ΠΌΠ΅ уравнСния ΠΌΠΎΠΆΠ½ΠΎ Π·Π°ΠΏΠΈΡΠ°Ρ‚ΡŒ, Ρ‡Ρ‚ΠΎ

I аккумулятор = I 1 + I 2 + I 3 +… (ΠΏΠ°Ρ€Π°Π»Π»Π΅Π»ΡŒΠ½Ρ‹Π΅ Ρ†Π΅ΠΏΠΈ)

Π’Π΅ΠΊΡƒΡ‰ΠΈΠ΅ значСния этих ΠΎΡ‚Π΄Π΅Π»ΡŒΠ½Ρ‹Ρ… Π²Π΅Ρ‚Π²Π΅ΠΉ ΠΊΠΎΠ½Ρ‚Ρ€ΠΎΠ»ΠΈΡ€ΡƒΡŽΡ‚ΡΡ двумя Π²Π΅Π»ΠΈΡ‡ΠΈΠ½Π°ΠΌΠΈ – сопротивлСниСм рСзистора Π² Π²Π΅Ρ‚Π²ΠΈ ΠΈ Ρ€Π°Π·Π½ΠΎΡΡ‚ΡŒΡŽ элСктричСских ΠΏΠΎΡ‚Π΅Π½Ρ†ΠΈΠ°Π»ΠΎΠ² (Ξ”V), ΠΏΡ€ΠΈΠ»ΠΎΠΆΠ΅Π½Π½ΠΎΠΉ ΠΊ Π²Π΅Ρ‚Π²ΠΈ.Π’ соотвСтствии с ΡƒΡ€Π°Π²Π½Π΅Π½ΠΈΠ΅ΠΌ Π·Π°ΠΊΠΎΠ½Π° Ома, рассмотрСнным Π²Ρ‹ΡˆΠ΅, ΠΌΠΎΠΆΠ½ΠΎ ΡΠΊΠ°Π·Π°Ρ‚ΡŒ, Ρ‡Ρ‚ΠΎ Ρ‚ΠΎΠΊ Π² Π²Π΅Ρ‚Π²ΠΈ 1 Ρ€Π°Π²Π΅Π½ разности элСктричСских ΠΏΠΎΡ‚Π΅Π½Ρ†ΠΈΠ°Π»ΠΎΠ² Π½Π° Π²Π΅Ρ‚Π²ΠΈ 1, Π΄Π΅Π»Π΅Π½Π½ΠΎΠΉ Π½Π° сопротивлСниС Π²Π΅Ρ‚Π²ΠΈ 1. АналогичныС утвСрТдСния ΠΌΠΎΠΆΠ½ΠΎ ΡΠ΄Π΅Π»Π°Ρ‚ΡŒ ΠΈ для Π΄Ρ€ΡƒΠ³ΠΈΡ… Π²Π΅Ρ‚Π²Π΅ΠΉ. Π’ Ρ„ΠΎΡ€ΠΌΠ΅ уравнСния ΠΌΠΎΠΆΠ½ΠΎ ΡƒΠΊΠ°Π·Π°Ρ‚ΡŒ, Ρ‡Ρ‚ΠΎ

I 1 = Ξ”V 1 / R 1 I 2 = Ξ”V 2 / R 2 I 3 = Ξ”V 3 / R 3

ЭклСктичСскиС разности ΠΏΠΎΡ‚Π΅Π½Ρ†ΠΈΠ°Π»ΠΎΠ² (Ξ”V 1 , Ξ”V 2 ΠΈ Ξ”V 3 ) Π½Π° ΠΎΡ‚Π΄Π΅Π»ΡŒΠ½Ρ‹Ρ… рСзисторах часто Π½Π°Π·Ρ‹Π²Π°ΡŽΡ‚ падСниями напряТСния.Подобно ΠΏΠΎΡΠ»Π΅Π΄ΠΎΠ²Π°Ρ‚Π΅Π»ΡŒΠ½Ρ‹ΠΌ цСпям, любой заряд, ΠΏΠΎΠΊΠΈΠ΄Π°ΡŽΡ‰ΠΈΠΉ Π±Π°Ρ‚Π°Ρ€Π΅ΡŽ, Π΄ΠΎΠ»ΠΆΠ΅Π½ ΠΈΡΠΏΡ‹Ρ‚Ρ‹Π²Π°Ρ‚ΡŒ Ρ‚Π°ΠΊΠΎΠ΅ ΠΆΠ΅ ΠΏΠ°Π΄Π΅Π½ΠΈΠ΅ напряТСния, ΠΊΠ°ΠΊ ΠΈ коэффициСнт усилСния, ΠΊΠΎΡ‚ΠΎΡ€Ρ‹ΠΉ ΠΎΠ½ ΠΏΠΎΠ»ΡƒΡ‡Π°Π΅Ρ‚ ΠΏΡ€ΠΈ ΠΏΡ€ΠΎΡ…ΠΎΠΆΠ΄Π΅Π½ΠΈΠΈ Ρ‡Π΅Ρ€Π΅Π· Π±Π°Ρ‚Π°Ρ€Π΅ΡŽ. Но Π² ΠΎΡ‚Π»ΠΈΡ‡ΠΈΠ΅ ΠΎΡ‚ ΠΏΠΎΡΠ»Π΅Π΄ΠΎΠ²Π°Ρ‚Π΅Π»ΡŒΠ½Ρ‹Ρ… Ρ†Π΅ΠΏΠ΅ΠΉ, Π² ΠΏΠ°Ρ€Π°Π»Π»Π΅Π»ΡŒΠ½ΠΎΠΉ Ρ†Π΅ΠΏΠΈ заряд ΠΏΡ€ΠΎΡ…ΠΎΠ΄ΠΈΡ‚ Ρ‚ΠΎΠ»ΡŒΠΊΠΎ Ρ‡Π΅Ρ€Π΅Π· ΠΎΠ΄ΠΈΠ½ рСзистор. Π’Π°ΠΊΠΈΠΌ ΠΎΠ±Ρ€Π°Π·ΠΎΠΌ, ΠΏΠ°Π΄Π΅Π½ΠΈΠ΅ напряТСния Π½Π° этом рСзисторС Π΄ΠΎΠ»ΠΆΠ½ΠΎ Ρ€Π°Π²Π½ΡΡ‚ΡŒΡΡ разности элСктричСских ΠΏΠΎΡ‚Π΅Π½Ρ†ΠΈΠ°Π»ΠΎΠ² Π½Π° Π±Π°Ρ‚Π°Ρ€Π΅Π΅. Π’ Ρ„ΠΎΡ€ΠΌΠ΅ уравнСния ΠΌΠΎΠΆΠ½ΠΎ ΡƒΠΊΠ°Π·Π°Ρ‚ΡŒ, Ρ‡Ρ‚ΠΎ

Ξ”V аккумулятор = Ξ”V 1 = Ξ”V 2 = Ξ”V 3 +… (ΠΏΠ°Ρ€Π°Π»Π»Π΅Π»ΡŒΠ½Ρ‹Π΅ Ρ†Π΅ΠΏΠΈ)

Π³Π΄Π΅ Ξ”V аккумулятор – элСктричСский ΠΏΠΎΡ‚Π΅Π½Ρ†ΠΈΠ°Π», Π½Π°ΠΊΠΎΠΏΠ»Π΅Π½Π½Ρ‹ΠΉ Π² аккумуляторС, Π° Ξ”V 1 , Ξ”V 2 ΠΈ Ξ”V 3 – падСния напряТСния (ΠΈΠ»ΠΈ разности элСктричСских ΠΏΠΎΡ‚Π΅Π½Ρ†ΠΈΠ°Π»ΠΎΠ²) Π½Π° ΠΎΡ‚Π΄Π΅Π»ΡŒΠ½Ρ‹Ρ… рСзисторах.

Π‘ΠΎΠ»Π΅Π΅ ΠΏΠΎΠ΄Ρ€ΠΎΠ±Π½ΠΎΠ΅ ΠΈ ΠΈΡΡ‡Π΅Ρ€ΠΏΡ‹Π²Π°ΡŽΡ‰Π΅Π΅ обсуТдСниС ΠΏΠ°Ρ€Π°Π»Π»Π΅Π»ΡŒΠ½Ρ‹Ρ… схСм ΠΈ ΠΈΡ… Π°Π½Π°Π»ΠΈΠ·Π° ΠΌΠΎΠΆΠ½ΠΎ Π½Π°ΠΉΡ‚ΠΈ Π² ΡƒΡ‡Π΅Π±Π½ΠΎΠΌ пособии ΠΏΠΎ Ρ„ΠΈΠ·ΠΈΠΊΠ΅.

ΠŸΡ€ΠΈΠ²Ρ‹Ρ‡ΠΊΠΈ эффСктивно Ρ€Π΅ΡˆΠ°Ρ‚ΡŒ ΠΏΡ€ΠΎΠ±Π»Π΅ΠΌΡ‹

Π­Ρ„Ρ„Π΅ΠΊΡ‚ΠΈΠ²Π½Ρ‹ΠΉ Ρ€Π΅ΡˆΠ°Ρ‚Π΅Π»ΡŒ ΠΏΡ€ΠΎΠ±Π»Π΅ΠΌ ΠΏΠΎ ΠΏΡ€ΠΈΠ²Ρ‹Ρ‡ΠΊΠ΅ ΠΏΠΎΠ΄Ρ…ΠΎΠ΄ΠΈΡ‚ ΠΊ физичСским ΠΏΡ€ΠΎΠ±Π»Π΅ΠΌΠ°ΠΌ Ρ‚Π°ΠΊΠΈΠΌ ΠΎΠ±Ρ€Π°Π·ΠΎΠΌ, Ρ‡Ρ‚ΠΎΠ±Ρ‹ ΠΎΡ‚Ρ€Π°ΠΆΠ°Ρ‚ΡŒ Π½Π°Π±ΠΎΡ€ дисциплинированных ΠΏΡ€ΠΈΠ²Ρ‹Ρ‡Π΅ΠΊ. Π₯отя Π½Π΅ всС эффСктивныС спСциалисты ΠΏΠΎ Ρ€Π΅ΡˆΠ΅Π½ΠΈΡŽ ΠΏΡ€ΠΎΠ±Π»Π΅ΠΌ ΠΈΡΠΏΠΎΠ»ΡŒΠ·ΡƒΡŽΡ‚ ΠΎΠ΄ΠΈΠ½ ΠΈ Ρ‚ΠΎΡ‚ ΠΆΠ΅ ΠΏΠΎΠ΄Ρ…ΠΎΠ΄, всС ΠΎΠ½ΠΈ ΠΈΠΌΠ΅ΡŽΡ‚ ΠΎΠ±Ρ‰ΠΈΠ΅ ΠΏΡ€ΠΈΠ²Ρ‹Ρ‡ΠΊΠΈ.Π­Ρ‚ΠΈ ΠΏΡ€ΠΈΠ²Ρ‹Ρ‡ΠΊΠΈ ΠΊΡ€Π°Ρ‚ΠΊΠΎ описаны здСсь. Π­Ρ„Ρ„Π΅ΠΊΡ‚ΠΈΠ²Π½ΠΎΠ΅ Ρ€Π΅ΡˆΠ΅Π½ΠΈΠ΅ ΠΏΡ€ΠΎΠ±Π»Π΅ΠΌ …

  • … Π²Π½ΠΈΠΌΠ°Ρ‚Π΅Π»ΡŒΠ½ΠΎ Ρ‡ΠΈΡ‚Π°Π΅Ρ‚ Π·Π°Π΄Π°Ρ‡Ρƒ ΠΈ создаСт ΠΌΡ‹ΡΠ»Π΅Π½Π½ΡƒΡŽ ΠΊΠ°Ρ€Ρ‚ΠΈΠ½Ρƒ физичСской ситуации. ΠŸΡ€ΠΈ нСобходимости ΠΎΠ½ΠΈ Π½Π°Π±Ρ€Π°ΡΡ‹Π²Π°ΡŽΡ‚ ΠΏΡ€ΠΎΡΡ‚ΡƒΡŽ схСму физичСской ситуации, Ρ‡Ρ‚ΠΎΠ±Ρ‹ ΠΏΠΎΠΌΠΎΡ‡ΡŒ Π²ΠΈΠ·ΡƒΠ°Π»ΠΈΠ·ΠΈΡ€ΠΎΠ²Π°Ρ‚ΡŒ Π΅Π΅.
  • … ΠΈΠ΄Π΅Π½Ρ‚ΠΈΡ„ΠΈΡ†ΠΈΡ€ΡƒΠ΅Ρ‚ извСстныС ΠΈ нСизвСстныС Π²Π΅Π»ΠΈΡ‡ΠΈΠ½Ρ‹ ΠΈ записываСт ΠΈΡ… Π² ΠΎΡ€Π³Π°Π½ΠΈΠ·ΠΎΠ²Π°Π½Π½ΠΎΠΌ порядкС, часто записывая ΠΈΡ… Π½Π° самой Π΄ΠΈΠ°Π³Ρ€Π°ΠΌΠΌΠ΅. Они ΠΏΡ€ΠΈΡ€Π°Π²Π½ΠΈΠ²Π°ΡŽΡ‚ Π·Π°Π΄Π°Π½Π½Ρ‹Π΅ значСния ΠΊ символам, ΠΈΡΠΏΠΎΠ»ΡŒΠ·ΡƒΠ΅ΠΌΡ‹ΠΌ для прСдставлСния ΡΠΎΠΎΡ‚Π²Π΅Ρ‚ΡΡ‚Π²ΡƒΡŽΡ‰Π΅ΠΉ Π²Π΅Π»ΠΈΡ‡ΠΈΠ½Ρ‹ (Π½Π°ΠΏΡ€ΠΈΠΌΠ΅Ρ€,Π³., Ξ”V = 9,0 Π’; R = 0,025 Ом; Π― = ???).
  • … строит ΡΡ‚Ρ€Π°Ρ‚Π΅Π³ΠΈΡŽ Ρ€Π΅ΡˆΠ΅Π½ΠΈΡ нСизвСстной Π²Π΅Π»ΠΈΡ‡ΠΈΠ½Ρ‹; стратСгия, ΠΊΠ°ΠΊ ΠΏΡ€Π°Π²ΠΈΠ»ΠΎ, сосрСдоточСна Π²ΠΎΠΊΡ€ΡƒΠ³ использования физичСских ΡƒΡ€Π°Π²Π½Π΅Π½ΠΈΠΉ ΠΈ Π²ΠΎ ΠΌΠ½ΠΎΠ³ΠΎΠΌ зависит ΠΎΡ‚ понимания ΠΏΡ€ΠΈΠ½Ρ†ΠΈΠΏΠΎΠ² Ρ„ΠΈΠ·ΠΈΠΊΠΈ.
  • … опрСдСляСт ΠΏΠΎΠ΄Ρ…ΠΎΠ΄ΡΡ‰ΡƒΡŽ (Ρ‹Π΅) Ρ„ΠΎΡ€ΠΌΡƒΠ»Ρƒ (Ρ‹) для использования, часто записывая ΠΈΡ…. ΠŸΡ€ΠΈ нСобходимости ΠΎΠ½ΠΈ Π²Ρ‹ΠΏΠΎΠ»Π½ΡΡŽΡ‚ Π½Π΅ΠΎΠ±Ρ…ΠΎΠ΄ΠΈΠΌΠΎΠ΅ ΠΏΡ€Π΅ΠΎΠ±Ρ€Π°Π·ΠΎΠ²Π°Π½ΠΈΠ΅ количСств Π² ΠΏΡ€Π°Π²ΠΈΠ»ΡŒΠ½Ρ‹Π΅ Π΅Π΄ΠΈΠ½ΠΈΡ†Ρ‹.
  • … выполняСт подстановки ΠΈ алгСбраичСскиС манипуляции, Ρ‡Ρ‚ΠΎΠ±Ρ‹ Π½Π°ΠΉΡ‚ΠΈ Π½Π΅ΠΈΠ·Π²Π΅ΡΡ‚Π½ΡƒΡŽ Π²Π΅Π»ΠΈΡ‡ΠΈΠ½Ρƒ.

ΠŸΠΎΠ΄Ρ€ΠΎΠ±Π½Π΅Π΅ …

Π”ΠΎΠΏΠΎΠ»Π½ΠΈΡ‚Π΅Π»ΡŒΠ½Π°Ρ Π»ΠΈΡ‚Π΅Ρ€Π°Ρ‚ΡƒΡ€Π° / Π£Ρ‡Π΅Π±Π½Ρ‹Π΅ пособия:

Π‘Π»Π΅Π΄ΡƒΡŽΡ‰ΠΈΠ΅ страницы Π£Ρ‡Π΅Π±Π½ΠΎΠ³ΠΎ пособия ΠΏΠΎ Ρ„ΠΈΠ·ΠΈΠΊΠ΅ ΠΌΠΎΠ³ΡƒΡ‚ Π±Ρ‹Ρ‚ΡŒ ΠΏΠΎΠ»Π΅Π·Π½Ρ‹ для Ρ‚ΠΎΠ³ΠΎ, Ρ‡Ρ‚ΠΎΠ±Ρ‹ ΠΏΠΎΠΌΠΎΡ‡ΡŒ Π²Π°ΠΌ Π² ΠΏΠΎΠ½ΠΈΠΌΠ°Π½ΠΈΠΈ ΠΊΠΎΠ½Ρ†Π΅ΠΏΡ†ΠΈΠΉ ΠΈ ΠΌΠ°Ρ‚Π΅ΠΌΠ°Ρ‚ΠΈΠΊΠΈ, связанных с этими ΠΏΡ€ΠΎΠ±Π»Π΅ΠΌΠ°ΠΌΠΈ.

Набор ΠΏΡ€ΠΎΠ±Π»Π΅ΠΌ элСктричСских Ρ†Π΅ΠΏΠ΅ΠΉ

ΠŸΡ€ΠΎΡΠΌΠΎΡ‚Ρ€Π΅Ρ‚ΡŒ Π½Π°Π±ΠΎΡ€ Π·Π°Π΄Π°Ρ‡

ЭлСктричСскиС схСмы РСшСния с Π°ΡƒΠ΄ΠΈΠΎΠ³ΠΈΠ΄ΠΎΠΌ

ΠŸΡ€ΠΎΡΠΌΠΎΡ‚Ρ€ΠΈΡ‚Π΅ Π°ΡƒΠ΄ΠΈΠΎΠ³ΠΈΠ΄ Ρ€Π΅ΡˆΠ΅Π½ΠΈΡ ΠΏΡ€ΠΎΠ±Π»Π΅ΠΌΡ‹:
1 | 2 | 3 | 4 | 5 | 6 | 7 | 8 | 9 | 10 | 11 | 12 | 13 | 14 | 15 | 16 | 17 | 18 | 19 | 20 | 21 | 22 | 23 | 24 | 25 | 26 | 27 | 28 | 29 | 30 | 31 | 32 | 33 | 34

ЭлСктроэнСргия, Ρ€Π°Π±ΠΎΡ‚Π° ΠΈ ΠΌΠΎΡ‰Π½ΠΎΡΡ‚ΡŒ

Π§Ρ‚ΠΎΠ±Ρ‹ ΠΏΠΎΠ½ΡΡ‚ΡŒ, ΠΊΠ°ΠΊ Ρ€Π°Π±ΠΎΡ‚Π°ΡŽΡ‚ устойчивыС Ρ‚Π΅Ρ…Π½ΠΎΠ»ΠΎΠ³ΠΈΠΈ, Π²Π°ΠΆΠ½ΠΎ ΡƒΡΠ²ΠΎΠΈΡ‚ΡŒ ΠΎΠΏΡ€Π΅Π΄Π΅Π»Π΅Π½Π½Ρ‹Π΅ основныС ΠΏΡ€ΠΈΠ½Ρ†ΠΈΠΏΡ‹.Π—Π½Π°Ρ‚ΡŒ, ΠΊΠ°ΠΊ фотоэлСктричСскиС элСмСнты ΠΏΡ€Π΅ΠΎΠ±Ρ€Π°Π·ΡƒΡŽΡ‚ ΡΠΎΠ»Π½Π΅Ρ‡Π½ΡƒΡŽ ΡΠ½Π΅Ρ€Π³ΠΈΡŽ Π² элСктричСство, ΠΎΠ·Π½Π°Ρ‡Π°Π΅Ρ‚ ΠΏΠΎΠ½ΠΈΠΌΠ°Ρ‚ΡŒ основы элСктричСства ΠΈ свСта. ПониманиС Ρ‚ΠΎΠ³ΠΎ, ΠΊΠ°ΠΊ вСтряныС Ρ‚ΡƒΡ€Π±ΠΈΠ½Ρ‹ производят элСктричСство, ΠΎΠ·Π½Π°Ρ‡Π°Π΅Ρ‚ ΠΏΠΎΠ½ΠΈΠΌΠ°Π½ΠΈΠ΅ ΠΊΠΎΠ΅-Ρ‡Π΅Π³ΠΎ ΠΎ власти, Ρ€Π°Π±ΠΎΡ‚Π΅ ΠΈ элСктромагнСтизмС. Π’ этом ΠΌΠΎΠ΄ΡƒΠ»Π΅ Π±ΡƒΠ΄ΡƒΡ‚ прСдставлСны основныС ΠΊΠΎΠ½Ρ†Π΅ΠΏΡ†ΠΈΠΈ, Π½Π΅ΠΎΠ±Ρ…ΠΎΠ΄ΠΈΠΌΡ‹Π΅ для понимания Ρ‚Π΅Ρ…Π½ΠΎΠ»ΠΎΠ³ΠΈΠΉ, обсуТдаСмых Π² этом курсС. Π₯отя Ρ„ΠΎΡ€ΠΌΡƒΠ»Ρ‹ ΠΈΠ½ΠΎΠ³Π΄Π° ΠΈΡΠΏΠΎΠ»ΡŒΠ·ΡƒΡŽΡ‚ΡΡ для объяснСния основных ΠΏΡ€ΠΈΠ½Ρ†ΠΈΠΏΠΎΠ², ΡΡƒΡ‚ΡŒ Π½Π΅ Π² Ρ‚ΠΎΠΌ, Ρ‡Ρ‚ΠΎΠ±Ρ‹ ΡƒΠΌΠ΅Ρ‚ΡŒ Ρ€Π΅ΡˆΠ°Ρ‚ΡŒ количСствСнныС Π·Π°Π΄Π°Ρ‡ΠΈ. Π€ΠΎΡ€ΠΌΡƒΠ»Ρ‹ ΠΏΠΎΠΌΠΎΠ³ΡƒΡ‚ Π²Π°ΠΌ ΡƒΠ²ΠΈΠ΄Π΅Ρ‚ΡŒ взаимосвязь.

Π¦Π΅Π»ΠΈ обучСния: УчащиСся смогут:

  1. Π’Ρ‹Π΄Π΅Π»ΠΈΡ‚Π΅ различия ΠΌΠ΅ΠΆΠ΄Ρƒ энСргиСй, Ρ€Π°Π±ΠΎΡ‚ΠΎΠΉ ΠΈ ΠΌΠΎΡ‰Π½ΠΎΡΡ‚ΡŒΡŽ ΠΈ ΠΏΡ€ΠΈΠ²Π΅Π΄ΠΈΡ‚Π΅ ΠΏΡ€ΠΈΠΌΠ΅Ρ€Ρ‹ ΠΊΠ°ΠΆΠ΄ΠΎΠ³ΠΎ ΠΈΠ· Π½ΠΈΡ… с использованиСм ΡΠΎΠΎΡ‚Π²Π΅Ρ‚ΡΡ‚Π²ΡƒΡŽΡ‰ΠΈΡ… Π΅Π΄ΠΈΠ½ΠΈΡ†.
  2. Π”Π°ΠΉΡ‚Π΅ ΡΠΎΠΎΡ‚Π²Π΅Ρ‚ΡΡ‚Π²ΡƒΡŽΡ‰ΠΈΠ΅ опрСдСлСния ΡΠ»Π΅Π΄ΡƒΡŽΡ‰ΠΈΠΌ элСктричСским Ρ‚Π΅Ρ€ΠΌΠΈΠ½Π°ΠΌ: элСктрон, элСктричСский заряд, элСктричСский ΠΏΠΎΡ‚Π΅Π½Ρ†ΠΈΠ°Π», сопротивлСниС, Ρ‚ΠΎΠΊ, ΠΌΠΎΡ‰Π½ΠΎΡΡ‚ΡŒ, ΠΏΡ€ΠΎΠ²ΠΎΠ΄Π½ΠΈΠΊ, ΠΏΠΎΠ»ΡƒΠΏΡ€ΠΎΠ²ΠΎΠ΄Π½ΠΈΠΊ ΠΈ изолятор.

    Учащийся смоТСт ΡΠΎΠΏΠΎΡΡ‚Π°Π²ΠΈΡ‚ΡŒ элСктричСскиС Π²Π΅Π»ΠΈΡ‡ΠΈΠ½Ρ‹ / свойства с Ρ€Π°Π·Π»ΠΈΡ‡Π½Ρ‹ΠΌΠΈ Π΅Π΄ΠΈΠ½ΠΈΡ†Π°ΠΌΠΈ измСрСния, ΠΈΡΠΏΠΎΠ»ΡŒΠ·ΡƒΠ΅ΠΌΡ‹ΠΌΠΈ Π² элСктротСхникС (Π½Π°ΠΏΡ€ΠΈΠΌΠ΅Ρ€,Π³. Π²ΠΎΠ»ΡŒΡ‚, Π°ΠΌΠΏΠ΅Ρ€, Π²Π°Ρ‚Ρ‚, ΠΎΠΌ, Π°ΠΌΠΏΠ΅Ρ€-часы, ΠΊΠΈΠ»ΠΎΠ²Π°Ρ‚Ρ‚-часы ΠΈ Ρ‚. Π΄.)

  3. Π£ΠΊΠ°ΠΆΠΈΡ‚Π΅ элСмСнты элСктричСской Ρ†Π΅ΠΏΠΈ.
  4. Π£ΠΊΠ°ΠΆΠΈΡ‚Π΅ различия ΠΌΠ΅ΠΆΠ΄Ρƒ ΠΏΠ°Ρ€Π°Π»Π»Π΅Π»ΡŒΠ½Ρ‹ΠΌΠΈ ΠΈ ΠΏΠΎΡΠ»Π΅Π΄ΠΎΠ²Π°Ρ‚Π΅Π»ΡŒΠ½Ρ‹ΠΌΠΈ цСпями ΠΈ ΠΎΡ‚ΠΌΠ΅Ρ‚ΡŒΡ‚Π΅ влияниС Π½Π° элСктричСский ΠΏΠΎΡ‚Π΅Π½Ρ†ΠΈΠ°Π» (ΠΈΠ·ΠΌΠ΅Ρ€Π΅Π½Π½Ρ‹ΠΉ Π² Π²ΠΎΠ»ΡŒΡ‚Π°Ρ…) ΠΈ Ρ‚ΠΎΠΊ (ΠΈΠ·ΠΌΠ΅Ρ€Π΅Π½Π½Ρ‹ΠΉ Π² Π°ΠΌΠΏΠ΅Ρ€Π°Ρ…).
  5. ΠžΠ±ΡŠΡΡΠ½ΠΈΡ‚Π΅ взаимосвязь ΠΌΠ΅ΠΆΠ΄Ρƒ ΠΏΠΎΡ‚ΠΎΠΊΠΎΠΌ Ρ‚ΠΎΠΊΠ° ΠΈ ΠΌΠ°Π³Π½Π΅Ρ‚ΠΈΠ·ΠΌΠΎΠΌ ΠΈ ΠΏΠΎΠΊΠ°ΠΆΠΈΡ‚Π΅, ΠΊΠ°ΠΊ это Π»Π΅ΠΆΠΈΡ‚ Π² основС элСктродвигатСлСй ΠΈ Π³Π΅Π½Π΅Ρ€Π°Ρ‚ΠΎΡ€ΠΎΠ².
  6. Π Π°Π·Π»ΠΈΡ‡Π°ΠΉΡ‚Π΅ элСктричСство постоянного ΠΈ ΠΏΠ΅Ρ€Π΅ΠΌΠ΅Π½Π½ΠΎΠ³ΠΎ Ρ‚ΠΎΠΊΠ°, ΠΎΠΏΡ€Π΅Π΄Π΅Π»ΠΈΡ‚Π΅ ΠΏΠΎΠ»Π΅Π·Π½Ρ‹Π΅ качСства ΠΊΠ°ΠΆΠ΄ΠΎΠ³ΠΎ ΠΈΠ· Π½ΠΈΡ…, ΠΎΡ‚ΠΌΠ΅Ρ‚ΡŒΡ‚Π΅, ΠΊΠ°ΠΊΠΈΠ΅ устройства связаны с ΠΊΠ°ΠΆΠ΄Ρ‹ΠΌ ΠΈΠ· Π½ΠΈΡ…, ΠΈ ΠΎΠΏΠΈΡˆΠΈΡ‚Π΅ Ρ€ΠΎΠ»ΡŒ силовых ΠΈΠ½Π²Π΅Ρ€Ρ‚ΠΎΡ€ΠΎΠ².

ЭнСргия, Ρ€Π°Π±ΠΎΡ‚Π° ΠΈ ΠΌΠΎΡ‰Π½ΠΎΡΡ‚ΡŒ

ΠŸΠ΅Ρ€Π΅ΠΉΡ‚ΠΈ ΠΊ: Force | Π Π°Π±ΠΎΡ‚Π° | ΠœΠΎΡ‰Π½ΠΎΡΡ‚ΡŒ

ΠŸΡ€ΠΎΡ‰Π΅ говоря, ВсСлСнная состоит ΠΈΠ· Ρ‡Π΅Ρ‚Ρ‹Ρ€Π΅Ρ… Π²Π΅Ρ‰Π΅ΠΉ: пространства, Π²Ρ€Π΅ΠΌΠ΅Π½ΠΈ, массы ΠΈ энСргии. ΠŸΠ΅Ρ€Π²Ρ‹ΠΉ Π·Π°ΠΊΠΎΠ½ Ρ‚Π΅Ρ€ΠΌΠΎΠ΄ΠΈΠ½Π°ΠΌΠΈΠΊΠΈ гласит, Ρ‡Ρ‚ΠΎ энСргия Π½Π΅ ΠΌΠΎΠΆΠ΅Ρ‚ Π±Ρ‹Ρ‚ΡŒ Π½ΠΈ создана, Π½ΠΈ Ρ€Π°Π·Ρ€ΡƒΡˆΠ΅Π½Π°. Но Π­ΠΉΠ½ΡˆΡ‚Π΅ΠΉΠ½ ΠΏΠΎΠΊΠ°Π·Π°Π» Π½Π°ΠΌ, Ρ‡Ρ‚ΠΎ ΡΠ½Π΅Ρ€Π³ΠΈΡŽ ΠΌΠΎΠΆΠ½ΠΎ ΠΏΡ€Π΅Π²Ρ€Π°Ρ‚ΠΈΡ‚ΡŒ Π² массу ΠΈ Π½Π°ΠΎΠ±ΠΎΡ€ΠΎΡ‚. Π’Ρ‚ΠΎΡ€ΠΎΠΉ Π·Π°ΠΊΠΎΠ½ Ρ‚Π΅Ρ€ΠΌΠΎΠ΄ΠΈΠ½Π°ΠΌΠΈΠΊΠΈ гласит, Ρ‡Ρ‚ΠΎ ΠΊΠ°ΠΆΠ΄Ρ‹ΠΉ Ρ€Π°Π·, ΠΊΠΎΠ³Π΄Π° энСргия мСняСт Ρ„ΠΎΡ€ΠΌΡƒ, Ρ‡Π°ΡΡ‚ΡŒ Π΅Π΅ прСвращаСтся Π² Ρ‚Π΅ΠΏΠ»ΠΎ. ЭнСргия Π±Ρ‹Π²Π°Π΅Ρ‚ Ρ€Π°Π·Π½Ρ‹Ρ… Ρ„ΠΎΡ€ΠΌ.Бамая полСзная энСргия ΠΈΠ»ΠΈ энСргия Π²Ρ‹ΡΠΎΡ‡Π°ΠΉΡˆΠ΅Π³ΠΎ качСства – это Ρ‚ΠΎ, Ρ‡Ρ‚ΠΎ ΠΌΡ‹ ΠΌΠΎΠΆΠ΅ΠΌ ΠΈΡΠΏΠΎΠ»ΡŒΠ·ΠΎΠ²Π°Ρ‚ΡŒ для Ρ€Π°Π±ΠΎΡ‚Ρ‹. НапримСр, энСргия двиТСния (кинСтичСская энСргия) Π²ΠΎΠ΄Ρ‹, ΠΏΠ°Π΄Π°ΡŽΡ‰Π΅ΠΉ Ρ‡Π΅Ρ€Π΅Π· ΠΏΠ»ΠΎΡ‚ΠΈΠ½Ρƒ, ΠΌΠΎΠΆΠ΅Ρ‚ Π±Ρ‹Ρ‚ΡŒ использована для вращСния водяного колСса для ΠΈΠ·ΠΌΠ΅Π»ΡŒΡ‡Π΅Π½ΠΈΡ Π·Π΅Ρ€Π½Π° ΠΈΠ»ΠΈ Π²Ρ‹Ρ€Π°Π±ΠΎΡ‚ΠΊΠΈ элСктричСства.

ΠŸΠΎΡ‚Π΅Π½Ρ†ΠΈΠ°Π»ΡŒΠ½Π°Ρ ΠΈ кинСтичСская энСргия

ΠŸΡ€ΠΎΠΈΡΡ…ΠΎΠΆΠ΄Π΅Π½ΠΈΠ΅: ΠŸΠ΅Ρ€Π²ΠΎΠΈΡΡ‚ΠΎΡ‡Π½ΠΈΠΊ: Environment Canada (https://www.ec.gc.ca/eau-water/default.asp?lang=en&n=00EEE0E6-1), доступ Ρ‡Π΅Ρ€Π΅Π· USGS: https: //water.usgs .gov / edu / wuhy.html Π­Ρ‚ΠΎ воспроизвСдСниС являСтся ΠΊΠΎΠΏΠΈΠ΅ΠΉ ΠΎΡ„ΠΈΡ†ΠΈΠ°Π»ΡŒΠ½ΠΎΠΉ Ρ€Π°Π±ΠΎΡ‚Ρ‹, ΠΎΠΏΡƒΠ±Π»ΠΈΠΊΠΎΠ²Π°Π½Π½ΠΎΠΉ ΠΏΡ€Π°Π²ΠΈΡ‚Π΅Π»ΡŒΡΡ‚Π²ΠΎΠΌ ΠšΠ°Π½Π°Π΄Ρ‹, ΠΈ воспроизвСдСниС Π½Π΅ Π±Ρ‹Π»ΠΎ ΠΏΡ€ΠΎΠΈΠ·Π²Π΅Π΄Π΅Π½ΠΎ Π² сотрудничСствС ΠΈΠ»ΠΈ с одобрСния ΠΏΡ€Π°Π²ΠΈΡ‚Π΅Π»ΡŒΡΡ‚Π²Π° ΠšΠ°Π½Π°Π΄Ρ‹.
ΠŸΠΎΠ²Ρ‚ΠΎΡ€Π½ΠΎΠ΅ использованиС: Π˜Π½Ρ„ΠΎΡ€ΠΌΠ°Ρ†ΠΈΡ Π½Π° этом Π²Π΅Π±-сайтС Π±Ρ‹Π»Π° Ρ€Π°Π·ΠΌΠ΅Ρ‰Π΅Π½Π° с Π½Π°ΠΌΠ΅Ρ€Π΅Π½ΠΈΠ΅ΠΌ ΡΠ΄Π΅Π»Π°Ρ‚ΡŒ Π΅Π΅ доступной для Π»ΠΈΡ‡Π½ΠΎΠ³ΠΎ ΠΈΠ»ΠΈ ΠΏΡƒΠ±Π»ΠΈΡ‡Π½ΠΎΠ³ΠΎ нСкоммСрчСского использования ΠΈ ΠΌΠΎΠΆΠ΅Ρ‚ Π±Ρ‹Ρ‚ΡŒ воспроизвСдСна частично ΠΈΠ»ΠΈ ΠΏΠΎΠ»Π½ΠΎΡΡ‚ΡŒΡŽ ΠΈ Π»ΡŽΠ±Ρ‹ΠΌΠΈ срСдствами Π±Π΅Π· взимания ΠΏΠ»Π°Ρ‚Ρ‹ ΠΈΠ»ΠΈ Π΄ΠΎΠΏΠΎΠ»Π½ΠΈΡ‚Π΅Π»ΡŒΠ½ΠΎΠ³ΠΎ Ρ€Π°Π·Ρ€Π΅ΡˆΠ΅Π½ΠΈΡ, Ссли Π½Π΅ ΡƒΠΊΠ°Π·Π°Π½ΠΎ ΠΈΠ½ΠΎΠ΅. ΠŸΠΎΠ»ΡŒΠ·ΠΎΠ²Π°Ρ‚Π΅Π»ΠΈ Π΄ΠΎΠ»ΠΆΠ½Ρ‹: ΠΏΡ€ΠΎΡΠ²Π»ΡΡ‚ΡŒ Π΄ΠΎΠ»ΠΆΠ½ΡƒΡŽ ΠΎΡΠΌΠΎΡ‚Ρ€ΠΈΡ‚Π΅Π»ΡŒΠ½ΠΎΡΡ‚ΡŒ для обСспСчСния точности воспроизводимых ΠΌΠ°Ρ‚Π΅Ρ€ΠΈΠ°Π»ΠΎΠ²; Π£ΠΊΠ°ΠΆΠΈΡ‚Π΅ ΠΊΠ°ΠΊ ΠΏΠΎΠ»Π½ΠΎΠ΅ Π½Π°Π·Π²Π°Π½ΠΈΠ΅ воспроизводимых ΠΌΠ°Ρ‚Π΅Ρ€ΠΈΠ°Π»ΠΎΠ², Ρ‚Π°ΠΊ ΠΈ ΠΎΡ€Π³Π°Π½ΠΈΠ·Π°Ρ†ΠΈΡŽ Π°Π²Ρ‚ΠΎΡ€Π°; ΠΈ Π£ΠΊΠ°ΠΆΠΈΡ‚Π΅, Ρ‡Ρ‚ΠΎ воспроизвСдСниС являСтся ΠΊΠΎΠΏΠΈΠ΅ΠΉ ΠΎΡ„ΠΈΡ†ΠΈΠ°Π»ΡŒΠ½ΠΎΠ³ΠΎ произвСдСния, ΠΎΠΏΡƒΠ±Π»ΠΈΠΊΠΎΠ²Π°Π½Π½ΠΎΠ³ΠΎ ΠŸΡ€Π°Π²ΠΈΡ‚Π΅Π»ΡŒΡΡ‚Π²ΠΎΠΌ ΠšΠ°Π½Π°Π΄Ρ‹, ΠΈ Ρ‡Ρ‚ΠΎ воспроизвСдСниС Π½Π΅ Π±Ρ‹Π»ΠΎ ΠΏΡ€ΠΎΠΈΠ·Π²Π΅Π΄Π΅Π½ΠΎ ΠΏΡ€ΠΈ участии ΠΈΠ»ΠΈ с одобрСния ΠŸΡ€Π°Π²ΠΈΡ‚Π΅Π»ΡŒΡΡ‚Π²Π° ΠšΠ°Π½Π°Π΄Ρ‹.

Бамая низкая Ρ„ΠΎΡ€ΠΌΠ° энСргии с Ρ‚ΠΎΡ‡ΠΊΠΈ зрСния полСзности – Ρ‚Π΅ΠΏΠ»ΠΎ. Π”Π°, Ρ‚Π΅ΠΏΠ»ΠΎ ΠΌΠΎΠΆΠ½ΠΎ ΠΈΡΠΏΠΎΠ»ΡŒΠ·ΠΎΠ²Π°Ρ‚ΡŒ для производства ΠΏΠ°Ρ€Π° ΠΈ ΠΏΡ€ΠΈΠ²ΠΎΠ΄Π° элСктричСских Ρ‚ΡƒΡ€Π±ΠΈΠ½. Но для этого трСбуСтся ΠΌΠ½ΠΎΠ³ΠΎ Ρ‚Π΅ΠΏΠ»Π°, ΠΈ это Ρ‚Π΅ΠΏΠ»ΠΎ Π΄ΠΎΠ»ΠΆΠ½ΠΎ ΠΈΡΡ…ΠΎΠ΄ΠΈΡ‚ΡŒ ΠΎΡ‚ ΠΊΠ°ΠΊΠΎΠ³ΠΎ-Ρ‚ΠΎ Π΄Ρ€ΡƒΠ³ΠΎΠ³ΠΎ источника энСргии, Π½Π°ΠΏΡ€ΠΈΠΌΠ΅Ρ€, горящСго угля ΠΈΠ»ΠΈ солнСчного свСта. Π€ΠΈΠ·ΠΈΠΊΠΈ ΠΈΡΠΏΠΎΠ»ΡŒΠ·ΡƒΡŽΡ‚ Ρ‚Π΅Ρ€ΠΌΠΈΠ½ энтропия, Ρ‡Ρ‚ΠΎΠ±Ρ‹ ΠΎΠΏΠΈΡΠ°Ρ‚ΡŒ ΠΈΠ·ΠΌΠ΅Π½Π΅Π½ΠΈΠ΅ ΠΏΠΎΠ»Π΅Π·Π½ΠΎΠΉ энСргии Π½Π° ΠΌΠ΅Π½Π΅Π΅ ΠΏΠΎΠ»Π΅Π·Π½ΠΎΠ΅ Ρ‚Π΅ΠΏΠ»ΠΎ.

ΠŸΡ€ΠΎΡ‰Π΅ говоря, всСлСнная состоит ΠΈΠ· Ρ‡Π΅Ρ‚Ρ‹Ρ€Π΅Ρ… Π²Π΅Ρ‰Π΅ΠΉ; пространство, врСмя, масса ΠΈ энСргия. ΠŸΠ΅Ρ€Π²Ρ‹ΠΉ Π·Π°ΠΊΠΎΠ½ Ρ‚Π΅Ρ€ΠΌΠΎΠ΄ΠΈΠ½Π°ΠΌΠΈΠΊΠΈ гласит, Ρ‡Ρ‚ΠΎ энСргия Π½Π΅ ΠΌΠΎΠΆΠ΅Ρ‚ Π±Ρ‹Ρ‚ΡŒ Π½ΠΈ создана, Π½ΠΈ Ρ€Π°Π·Ρ€ΡƒΡˆΠ΅Π½Π°.(Π₯отя ΠΏΠΎΠ·ΠΆΠ΅ Π­ΠΉΠ½ΡˆΡ‚Π΅ΠΉΠ½ ΠΏΠΎΠΊΠ°Π·Π°Π», Ρ‡Ρ‚ΠΎ для ядСрных Ρ€Π΅Π°ΠΊΡ†ΠΈΠΉ ΡΠ½Π΅Ρ€Π³ΠΈΡŽ ΠΌΠΎΠΆΠ½ΠΎ ΠΏΡ€Π΅Π²Ρ€Π°Ρ‚ΠΈΡ‚ΡŒ Π² массу ΠΈ Π½Π°ΠΎΠ±ΠΎΡ€ΠΎΡ‚). ЭнСргия Π±Ρ‹Π²Π°Π΅Ρ‚ Ρ€Π°Π·Π½Ρ‹Ρ… Ρ„ΠΎΡ€ΠΌ. Когда энСргия пСрСдаСтся ΠΎΡ‚ ΠΎΠ΄Π½ΠΎΠ³ΠΎ ΠΎΠ±ΡŠΠ΅ΠΊΡ‚Π° ΠΊ Π΄Ρ€ΡƒΠ³ΠΎΠΌΡƒ ΠΈΠ»ΠΈ ΠΊΠΎΠ³Π΄Π° ΠΎΠ½Π° трансформируСтся ΠΈΠ· ΠΎΠ΄Π½ΠΎΠ³ΠΎ Ρ‚ΠΈΠΏΠ° Π² Π΄Ρ€ΡƒΠ³ΠΎΠΉ, Π΅Π΅ ΠΌΠΎΠΆΠ½ΠΎ ΠΈΡΠΏΠΎΠ»ΡŒΠ·ΠΎΠ²Π°Ρ‚ΡŒ для выполнСния Ρ€Π°Π±ΠΎΡ‚Ρ‹. НапримСр, энСргия двиТСния (кинСтичСская энСргия) Π²ΠΎΠ΄Ρ‹, ΠΏΠ°Π΄Π°ΡŽΡ‰Π΅ΠΉ Ρ‡Π΅Ρ€Π΅Π· ΠΏΠ»ΠΎΡ‚ΠΈΠ½Ρƒ, ΠΌΠΎΠΆΠ΅Ρ‚ Π±Ρ‹Ρ‚ΡŒ использована для вращСния водяного колСса для ΠΈΠ·ΠΌΠ΅Π»ΡŒΡ‡Π΅Π½ΠΈΡ Π·Π΅Ρ€Π½Π° ΠΈΠ»ΠΈ Π²Ρ‹Ρ€Π°Π±ΠΎΡ‚ΠΊΠΈ элСктричСства.

Энтропия – это ΠΌΠ΅Ρ€Π° распрСдСлСния энСргии. ΠšΠΎΠ½Ρ†Π΅Π½Ρ‚Ρ€ΠΈΡ€ΠΎΠ²Π°Π½Π½Ρ‹Π΅ Ρ„ΠΎΡ€ΠΌΡ‹ энСргии, Ρ‚Π°ΠΊΠΈΠ΅ ΠΊΠ°ΠΊ энСргия, хранящаяся Π² ядрС Π°Ρ‚ΠΎΠΌΠ°, Π² химичСских связях ΠΈΠ»ΠΈ Π² Π²Ρ‹ΡΠΎΠΊΠΎΠ²ΠΎΠ»ΡŒΡ‚Π½Ρ‹Ρ… элСктричСских устройствах, ΠΎΡ‡Π΅Π½ΡŒ ΠΏΠΎΠ»Π΅Π·Π½Ρ‹ для выполнСния Ρ€Π°Π±ΠΎΡ‚Ρ‹.Π‘ Π΄Ρ€ΡƒΠ³ΠΎΠΉ стороны, ΠΌΠ΅Π½Π΅Π΅ ΠΊΠΎΠ½Ρ†Π΅Π½Ρ‚Ρ€ΠΈΡ€ΠΎΠ²Π°Π½Π½Ρ‹Π΅ Ρ„ΠΎΡ€ΠΌΡ‹ энСргии, Ρ‚Π°ΠΊΠΈΠ΅ ΠΊΠ°ΠΊ Π½ΠΈΠ·ΠΊΠΎΡ‚Π΅ΠΌΠΏΠ΅Ρ€Π°Ρ‚ΡƒΡ€Π½ΠΎΠ΅ Ρ‚Π΅ΠΏΠ»ΠΎ, Π²ΠΈΠ±Ρ€Π°Ρ†ΠΈΠΈ ΠΈΠ»ΠΈ Π·Π²ΡƒΠΊΠΎΠ²Ρ‹Π΅ Π²ΠΎΠ»Π½Ρ‹, Π³ΠΎΡ€Π°Π·Π΄ΠΎ ΠΌΠ΅Π½Π΅Π΅ ΠΏΠΎΠ»Π΅Π·Π½Ρ‹. Π’Ρ‚ΠΎΡ€ΠΎΠΉ Π·Π°ΠΊΠΎΠ½ Ρ‚Π΅Ρ€ΠΌΠΎΠ΄ΠΈΠ½Π°ΠΌΠΈΠΊΠΈ гласит, Ρ‡Ρ‚ΠΎ всякий Ρ€Π°Π·, ΠΊΠΎΠ³Π΄Π° энСргия ΠΈΡΠΏΠΎΠ»ΡŒΠ·ΡƒΠ΅Ρ‚ΡΡ для выполнСния Ρ€Π°Π±ΠΎΡ‚Ρ‹, Ρ‡Π°ΡΡ‚ΡŒ энСргии прСвращаСтся ΠΈΠ· ΠΊΠΎΠ½Ρ†Π΅Π½Ρ‚Ρ€ΠΈΡ€ΠΎΠ²Π°Π½Π½ΠΎΠΉ Ρ„ΠΎΡ€ΠΌΡ‹ Π² ΠΌΠ΅Π½Π΅Π΅ ΠΏΠΎΠ»Π΅Π·Π½ΡƒΡŽ. Π€ΠΈΠ·ΠΈΠΊΠΈ говорят, Ρ‡Ρ‚ΠΎ ΠΏΠΎ ΠΌΠ΅Ρ€Π΅ Ρ‚ΠΎΠ³ΠΎ, ΠΊΠ°ΠΊ энСргия распространяСтся ΠΈΠ»ΠΈ рассСиваСтся, энтропия увСличиваСтся. Одним ΠΈΠ· Ρ€Π΅Π·ΡƒΠ»ΡŒΡ‚Π°Ρ‚ΠΎΠ² Π²Ρ‚ΠΎΡ€ΠΎΠ³ΠΎ Π·Π°ΠΊΠΎΠ½Π° Ρ‚Π΅Ρ€ΠΌΠΎΠ΄ΠΈΠ½Π°ΠΌΠΈΠΊΠΈ являСтся Ρ‚ΠΎ, Ρ‡Ρ‚ΠΎ Π½ΠΈ ΠΎΠ΄ΠΈΠ½ процСсс Π½Π΅ ΠΌΠΎΠΆΠ΅Ρ‚ ΠΏΡ€Π΅ΠΎΠ±Ρ€Π°Π·ΠΎΠ²Π°Ρ‚ΡŒ 100% энСргии Π² ΠΏΠΎΠ»Π΅Π·Π½ΡƒΡŽ Ρ€Π°Π±ΠΎΡ‚Ρƒ.

Π§Ρ‚ΠΎ Ρ‚Π°ΠΊΠΎΠ΅ энСргия? ПолСзно Ρ€Π°Π·Π΄Π΅Π»ΠΈΡ‚ΡŒ ΡΠ½Π΅Ρ€Π³ΠΈΡŽ Π½Π° Π΄Π²Π° списка. ΠšΠΈΠ½Π΅Ρ‚ΠΈΡ‡Π΅ΡΠΊΠ°Ρ энСргия – это энСргия двиТущСгося ΠΎΠ±ΡŠΠ΅ΠΊΡ‚Π°. ΠŸΠ°Π΄Π°ΡŽΡ‰Π°Ρ Π²ΠΎΠ΄Π° (Ρ€Π΅Π°Π³ΠΈΡ€ΡƒΡŽΡ‰Π°Ρ Π½Π° силу тяТСсти), солнСчный свСт, элСктроны, ΠΏΡ€ΠΎΡ‚Π΅ΠΊΠ°ΡŽΡ‰ΠΈΠ΅ ΠΏΠΎ ΠΏΡ€ΠΎΠ²ΠΎΠ΄Ρƒ (элСктричСство), вСлосипСд Π² Π΄Π²ΠΈΠΆΠ΅Π½ΠΈΠΈ, использованиС ΠΌΡ‹ΡˆΡ† для двиТСния Π³Π»Π°Π· Π²ΠΎ врСмя чтСния – всС это ΠΏΡ€ΠΈΠΌΠ΅Ρ€Ρ‹ кинСтичСской энСргии. ΠŸΠΎΡ‚Π΅Π½Ρ†ΠΈΠ°Π»ΡŒΠ½Π°Ρ энСргия – это Ρ‚ΠΎ, Ρ‡Ρ‚ΠΎ сохраняСтся ΠΈ Π³ΠΎΡ‚ΠΎΠ²ΠΎ ΠΊ ΠΏΡ€Π΅ΠΎΠ±Ρ€Π°Π·ΠΎΠ²Π°Π½ΠΈΡŽ Π² ΠΊΠΈΠ½Π΅Ρ‚ΠΈΡ‡Π΅ΡΠΊΡƒΡŽ ΡΠ½Π΅Ρ€Π³ΠΈΡŽ. Π­Ρ‚ΠΎ Π²ΠΊΠ»ΡŽΡ‡Π°Π΅Ρ‚ Π²ΠΎΠ΄Ρƒ, ΡƒΠ΄Π΅Ρ€ΠΆΠΈΠ²Π°Π΅ΠΌΡƒΡŽ ΠΏΠ»ΠΎΡ‚ΠΈΠ½ΠΎΠΉ, элСктричСский заряд, хранящийся Π² Π±Π°Ρ‚Π°Ρ€Π΅Π΅, Ρ…ΠΈΠΌΠΈΡ‡Π΅ΡΠΊΡƒΡŽ ΡΠ½Π΅Ρ€Π³ΠΈΡŽ, Ρ…Ρ€Π°Π½ΡΡ‰ΡƒΡŽΡΡ Π² ΠΆΠΈΡ€Π°Ρ… ΠΈ сахарах, ΠΈ Ρ…ΠΈΠΌΠΈΡ‡Π΅ΡΠΊΡƒΡŽ ΡΠ½Π΅Ρ€Π³ΠΈΡŽ, Ρ…Ρ€Π°Π½ΡΡ‰ΡƒΡŽΡΡ Π² Π±Π΅Π½Π·ΠΈΠ½Π΅ ΠΈ ΡƒΠ³Π»Π΅.

На схСмС гидроэлСктростанции Π²ΠΎΠ΄Π°, ΡΡ‚Π΅ΠΊΠ°ΡŽΡ‰Π°Ρ ΠΏΠΎ Π½Π°ΠΏΠΎΡ€Π½ΠΎΠΌΡƒ ΡˆΡ‚ΠΎΠΊΡƒ, ΠΈΠΌΠ΅Π΅Ρ‚ ΠΊΠΈΠ½Π΅Ρ‚ΠΈΡ‡Π΅ΡΠΊΡƒΡŽ ΡΠ½Π΅Ρ€Π³ΠΈΡŽ. Π­Ρ‚Π° кинСтичСская энСргия ΠΈΡΠΏΠΎΠ»ΡŒΠ·ΡƒΠ΅Ρ‚ΡΡ для вращСния Ρ‚ΡƒΡ€Π±ΠΈΠ½Ρ‹, соСдинСнной с элСктричСским Π³Π΅Π½Π΅Ρ€Π°Ρ‚ΠΎΡ€ΠΎΠΌ. Π’ΠΎΠ΄Π°, хранящаяся Π·Π° ΠΏΠ»ΠΎΡ‚ΠΈΠ½ΠΎΠΉ, ΠΈΠΌΠ΅Π΅Ρ‚ ΠΏΠΎΡ‚Π΅Π½Ρ†ΠΈΠ°Π»ΡŒΠ½ΡƒΡŽ ΡΠ½Π΅Ρ€Π³ΠΈΡŽ ΠΈΠ»ΠΈ Π·Π°ΠΏΠ°ΡΠ΅Π½Π½ΡƒΡŽ ΡΠ½Π΅Ρ€Π³ΠΈΡŽ. ΠžΠ±Ρ€Π°Ρ‚ΠΈΡ‚Π΅ Π²Π½ΠΈΠΌΠ°Π½ΠΈΠ΅, Ρ‡Ρ‚ΠΎ сила тяТСсти, Π΄Π΅ΠΉΡΡ‚Π²ΡƒΡŽΡ‰Π°Ρ Π½Π° Π²ΠΎΠ΄Ρƒ, Π² ΠΊΠ°ΠΆΠ΄ΠΎΠΌ случаС обСспСчиваСт ΡΠ½Π΅Ρ€Π³ΠΈΡŽ.

Π‘ΠΈΠ»Π°

Когда ΠΊ ΠΎΠ±ΡŠΠ΅ΠΊΡ‚Ρƒ прикладываСтся энСргия, ΠΌΡ‹ Π΄ΡƒΠΌΠ°Π΅ΠΌ ΠΎΠ± этом ΠΊΠ°ΠΊ ΠΎ силС .НСкоторыС силы Ρ‚Ρ€Π΅Π±ΡƒΡŽΡ‚ ΠΊΠΎΠ½Ρ‚Π°ΠΊΡ‚Π° ΠΌΠ΅ΠΆΠ΄Ρƒ двумя ΠΎΠ±ΡŠΠ΅ΠΊΡ‚Π°ΠΌΠΈ, Π° Π΄Ρ€ΡƒΠ³ΠΈΠ΅ Π΄Π΅ΠΉΡΡ‚Π²ΡƒΡŽΡ‚ Π½Π° расстоянии. Π‘ΠΈΠ»Ρ‹, ΠΊΠΎΡ‚ΠΎΡ€Ρ‹Π΅ для Ρ‚Ρ€Π΅Π±ΡƒΡŽΡ‚ ΠΊΠΎΠ½Ρ‚Π°ΠΊΡ‚Π° , Π²ΠΊΠ»ΡŽΡ‡Π°ΡŽΡ‚ Ρ‚ΠΎΠ»ΠΊΠ°Π½ΠΈΠ΅, тянущСС усилиС (натяТСниС) ΠΈ Ρ‚Ρ€Π΅Π½ΠΈΠ΅. Π‘ΠΈΠ»Ρ‹, ΠΊΠΎΡ‚ΠΎΡ€Ρ‹Π΅ Ρ€Π°Π±ΠΎΡ‚Π°ΡŽΡ‚ Π±Π΅Π· прямого ΠΊΠΎΠ½Ρ‚Π°ΠΊΡ‚Π° ΠΌΠ΅ΠΆΠ΄Ρƒ ΠΎΠ±ΡŠΠ΅ΠΊΡ‚Π°ΠΌΠΈ, Π²ΠΊΠ»ΡŽΡ‡Π°ΡŽΡ‚ Π³Ρ€Π°Π²ΠΈΡ‚Π°Ρ†ΠΈΡŽ, ΠΌΠ°Π³Π½Π΅Ρ‚ΠΈΠ·ΠΌ ΠΈ ΡΠ»Π΅ΠΊΡ‚Ρ€ΠΈΡ‡Π΅ΡΠΊΡƒΡŽ силу. Бтандартная Π΅Π΄ΠΈΠ½ΠΈΡ†Π° силы Π½Π°Π·Π²Π°Π½Π° Π² Ρ‡Π΅ΡΡ‚ΡŒ сэра Исаака ΠΡŒΡŽΡ‚ΠΎΠ½Π°, ΠΎΡ‚Ρ†Π° Ρ„ΠΈΠ·ΠΈΠΊΠΈ. Один ΠΡŒΡŽΡ‚ΠΎΠ½ (1 Н) = количСство силы для ускорСния 1 ΠΊΠ³ массы Π½Π° ΠΎΠ΄ΠΈΠ½ ΠΌΠ΅Ρ‚Ρ€ Π² сСкунду 2 . Или 1 Н = (1 ΠΊΠ³ x 1 ΠΌ) / с 2 .

Аппарат ДТоуля для дСмонстрации эквивалСнтности Ρ€Π°Π±ΠΎΡ‚Ρ‹ ΠΈ Ρ‚Π΅ΠΏΠ»Π°

Provenance: Π˜Π·ΠΎΠ±Ρ€Π°ΠΆΠ΅Π½ΠΈΠ΅ ΠΈΠ· Π½ΠΎΠ²ΠΎΠ³ΠΎ СТСмСсячного ΠΆΡƒΡ€Π½Π°Π»Π° Harper’s, β„– 231, август 1869 Π³. Доступно ΠΏΠΎ: https://commons.wikimedia.org/wiki/File:Joule%27s_Apparatus_(Harper%27s_Scan).png
ΠŸΠΎΠ²Ρ‚ΠΎΡ€Π½ΠΎΠ΅ использованиС: Π­Ρ‚ΠΎΡ‚ элСмСнт находится Π² общСствСнном достоянии ΠΈ ΠΌΠΎΠΆΠ΅Ρ‚ Π±Ρ‹Ρ‚ΡŒ использован ΠΏΠΎΠ²Ρ‚ΠΎΡ€Π½ΠΎ Π±Π΅Π· ΠΎΠ³Ρ€Π°Π½ΠΈΡ‡Π΅Π½ΠΈΠΉ.

Π Π°Π±ΠΎΡ‚Π°

ΠœΡ‹ ΠΈΡΠΏΠΎΠ»ΡŒΠ·ΡƒΠ΅ΠΌ ΡΠ½Π΅Ρ€Π³ΠΈΡŽ для Ρ€Π°Π±ΠΎΡ‚Ρ‹. Π‘Π°ΠΌΡ‹ΠΉ простой способ Π΄ΡƒΠΌΠ°Ρ‚ΡŒ ΠΎ Ρ€Π°Π±ΠΎΡ‚Π΅ – это ΠΏΠ΅Ρ€Π΅ΠΌΠ΅Ρ‰Π°Ρ‚ΡŒ ΠΎΠ±ΡŠΠ΅ΠΊΡ‚.Когда ΠΊ ΠΎΠ±ΡŠΠ΅ΠΊΡ‚Ρƒ прикладываСтся сила (масса, умноТСнная Π½Π° ускорСниС), которая заставляСт этот ΠΎΠ±ΡŠΠ΅ΠΊΡ‚ ΠΏΠ΅Ρ€Π΅ΠΌΠ΅Ρ‰Π°Ρ‚ΡŒΡΡ, ΠΏΡ€ΠΎΠΉΠ΄Π΅Π½Π½ΠΎΠ΅ расстояниС – это ΡƒΠΆΠ΅ выполнСнная Ρ€Π°Π±ΠΎΡ‚Π°. Но ΠΌΡ‹ ΠΈΡΠΏΠΎΠ»ΡŒΠ·ΡƒΠ΅ΠΌ ΡΠ½Π΅Ρ€Π³ΠΈΡŽ для выполнСния большСго количСства Ρ€Π°Π±ΠΎΡ‚, Ρ‡Π΅ΠΌ ΠΏΠ΅Ρ€Π΅ΠΌΠ΅Ρ‰Π΅Π½ΠΈΠ΅ ΠΌΠ΅Π±Π΅Π»ΠΈ ΠΈΠ»ΠΈ Π°Π²Ρ‚ΠΎΠΌΠΎΠ±ΠΈΠ»Π΅ΠΉ. Π Π°Π±ΠΎΡ‚Π° Ρ‚Π°ΠΊΠΆΠ΅ выполняСтся, ΠΊΠΎΠ³Π΄Π° ΠΌΡ‹ ΠΈΡΠΏΠΎΠ»ΡŒΠ·ΡƒΠ΅ΠΌ солнСчный свСт ΠΈΠ»ΠΈ ΠΏΡ€ΠΈΡ€ΠΎΠ΄Π½Ρ‹ΠΉ Π³Π°Π· для ΠΎΠ±ΠΎΠ³Ρ€Π΅Π²Π° Π½Π°ΡˆΠΈΡ… Π΄ΠΎΠΌΠΎΠ², ΠΊΠΎΠ³Π΄Π° ΠΌΡ‹ ΠΈΡΠΏΠΎΠ»ΡŒΠ·ΡƒΠ΅ΠΌ элСктричСство для освСщСния Π½Π°ΡˆΠΈΡ… ΠΊΠΎΠΌΠ½Π°Ρ‚ ΠΈΠ»ΠΈ ΠΊΠΎΠ³Π΄Π° ΠΌΡ‹ ΠΈΡΠΏΠΎΠ»ΡŒΠ·ΡƒΠ΅ΠΌ Π±ΡƒΡ‚Π΅Ρ€Π±Ρ€ΠΎΠ΄ с арахисовым маслом ΠΈ ΠΆΠ΅Π»Π΅ для питания ΠΊΠ»Π΅Ρ‚ΠΎΠΊ нашСго ΠΌΠΎΠ·Π³Π°.

ΠŸΠΎΡΠΊΠΎΠ»ΡŒΠΊΡƒ энСргия Π±Ρ‹Π²Π°Π΅Ρ‚ Ρ€Π°Π·Π½Ρ‹Ρ… Ρ„ΠΎΡ€ΠΌ, Π½Π΅ΡƒΠ΄ΠΈΠ²ΠΈΡ‚Π΅Π»ΡŒΠ½ΠΎ, Ρ‡Ρ‚ΠΎ ΡΡƒΡ‰Π΅ΡΡ‚Π²ΡƒΡŽΡ‚ Ρ€Π°Π·Π½Ρ‹Π΅ способы Π΅Π΅ измСрСния.Π’Ρ€ΡƒΠ΄Π½ΠΎ ΠΎΡ‚ΡΠ»Π΅ΠΆΠΈΠ²Π°Ρ‚ΡŒ всС Ρ€Π°Π·Π»ΠΈΡ‡Π½Ρ‹Π΅ Π΅Π΄ΠΈΠ½ΠΈΡ†Ρ‹ энСргии. ΠŸΠΎΡΠΌΠΎΡ‚Ρ€ΠΈΡ‚Π΅ Π½Π° Ρ‚Π°Π±Π»ΠΈΡ†Ρƒ Π½ΠΈΠΆΠ΅, Ρ‡Ρ‚ΠΎΠ±Ρ‹ ΡƒΠ²ΠΈΠ΄Π΅Ρ‚ΡŒ Π½Π΅ΠΊΠΎΡ‚ΠΎΡ€Ρ‹Π΅ Π΅Π΄ΠΈΠ½ΠΈΡ†Ρ‹ ΠΈ ΠΎΡ‚Π½ΠΎΡˆΠ΅Π½ΠΈΠ΅ ΠΊ дТоулям, ΠΊΠΎΡ‚ΠΎΡ€Ρ‹ΠΉ являСтся Π·ΠΎΠ»ΠΎΡ‚Ρ‹ΠΌ стандартом измСрСния энСргии. Он Π½Π°Π·Π²Π°Π½ Π² Ρ‡Π΅ΡΡ‚ΡŒ ДТСймса ДТоуля, ΠΏΠΈΠ²ΠΎΠ²Π°Ρ€Π° 19-Π³ΠΎ Π²Π΅ΠΊΠ°, ΠΊΠΎΡ‚ΠΎΡ€Ρ‹ΠΉ ΠΏΠΎΠΊΠ°Π·Π°Π» ΡΠΊΠ²ΠΈΠ²Π°Π»Π΅Π½Ρ‚Π½ΠΎΡΡ‚ΡŒ мСханичСской Ρ€Π°Π±ΠΎΡ‚Ρ‹ ΠΈ Ρ‚Π΅ΠΏΠ»Π°. Один Π΄ΠΆΠΎΡƒΠ»ΡŒ ΠΏΡ€ΠΈΠΌΠ΅Ρ€Π½ΠΎ Ρ€Π°Π²Π΅Π½ количСству энСргии, Π½Π΅ΠΎΠ±Ρ…ΠΎΠ΄ΠΈΠΌΠΎΠΌΡƒ для поднятия 100-Π³Ρ€Π°ΠΌΠΌΠΎΠ²ΠΎΠ³ΠΎ яблока Π½Π° 1 ΠΌΠ΅Ρ‚Ρ€ (3,3 Ρ„ΡƒΡ‚Π°).

Π˜Π·ΠΎΠ±Ρ€Π°ΠΆΠ΅Π½Π½Ρ‹ΠΉ Π°ΠΏΠΏΠ°Ρ€Π°Ρ‚ Π±Ρ‹Π» использован ДТСймсом Π”ΠΆΠΎΡƒΠ»Π΅ΠΌ для дСмонстрации эквивалСнтности мСханичСской Ρ€Π°Π±ΠΎΡ‚Ρ‹ ΠΈ Ρ‚Π΅ΠΏΠ»Π°.Он рассчитал Ρ€Π°Π±ΠΎΡ‚Ρƒ, Π²Ρ‹ΠΏΠΎΠ»Π½ΡΠ΅ΠΌΡƒΡŽ силой тяТСсти Π½Π° Π³ΠΈΡ€ΡŽ. Π­Ρ‚Π° тяга ΠΏΠΎΠ²Π΅Ρ€Π½ΡƒΠ»Π° Π»ΠΎΠΏΠ°Ρ‚ΠΎΡ‡Π½Ρ‹Π΅ колСса, ΠΊΠΎΡ‚ΠΎΡ€Ρ‹Π΅ смСшали Π²ΠΎΠ΄Ρƒ Π² ΠΈΠ·ΠΎΠ»ΠΈΡ€ΠΎΠ²Π°Π½Π½ΠΎΠΌ ΠΊΠΎΠ½Ρ‚Π΅ΠΉΠ½Π΅Ρ€Π΅. Π’ΠΎΠ΄Π° нагрСваСтся ΠΏΡ€ΠΈ ΠΏΠ΅Ρ€Π΅ΠΌΠ΅ΡˆΠΈΠ²Π°Π½ΠΈΠΈ, показывая, Ρ‡Ρ‚ΠΎ Ρ‚Π΅ΠΏΠ»ΠΎ = Ρ€Π°Π±ΠΎΡ‚Π°.

ΠŸΠ°Ρ€ΠΎΠ²Π°Ρ машина Π’Π°Ρ‚Ρ‚Π°

ΠŸΡ€ΠΎΠΈΡΡ…ΠΎΠΆΠ΄Π΅Π½ΠΈΠ΅: Π’ΠΈΠΊΠΈΠΊΠΎΠΌΠΌΠΎΠ½Ρ‹: https://commons.wikimedia.org/wiki/File:SteamEngine_Boulton%26Watt_1784.png
ΠŸΠΎΠ²Ρ‚ΠΎΡ€Π½ΠΎΠ΅ использованиС: Π­Ρ‚ΠΎΡ‚ элСмСнт находится Π² общСствСнном достоянии ΠΈ ΠΌΠΎΠΆΠ΅Ρ‚ ΠΈΡΠΏΠΎΠ»ΡŒΠ·ΠΎΠ²Π°Ρ‚ΡŒΡΡ ΠΏΠΎΠ²Ρ‚ΠΎΡ€Π½ΠΎ Π±Π΅Π· ΠΎΠ³Ρ€Π°Π½ΠΈΡ‡Π΅Π½ΠΈΠΉ.

ΠœΠΎΡ‰Π½ΠΎΡΡ‚ΡŒ

ΠœΠΎΡ‰Π½ΠΎΡΡ‚ΡŒ – это ΠΌΠ΅Ρ€Π° Ρ‚ΠΎΠ³ΠΎ, сколько энСргии ΠΈΡΠΏΠΎΠ»ΡŒΠ·ΡƒΠ΅Ρ‚ΡΡ Π·Π° ΠΎΠΏΡ€Π΅Π΄Π΅Π»Π΅Π½Π½Ρ‹ΠΉ ΠΏΠ΅Ρ€ΠΈΠΎΠ΄ Π²Ρ€Π΅ΠΌΠ΅Π½ΠΈ. Для этого ΠΌΡ‹ ΠΌΠΎΠΆΠ΅ΠΌ ΠΈΡΠΏΠΎΠ»ΡŒΠ·ΠΎΠ²Π°Ρ‚ΡŒ Π²Π°Ρ‚Ρ‚. ДТСймс Π’Π°Ρ‚Ρ‚ Π±Ρ‹Π» ΠΏΠΈΠΎΠ½Π΅Ρ€ΠΎΠΌ Π² ΠΏΠΎΠ½ΠΈΠΌΠ°Π½ΠΈΠΈ Ρ„ΠΈΠ·ΠΈΠΊΠΈ энСргии ΠΈ Ρ€Π°Π·Ρ€Π°Π±ΠΎΡ‚Π°Π» ΠΎΠ΄ΠΈΠ½ ΠΈΠ· ΠΏΠ΅Ρ€Π²Ρ‹Ρ… ΡƒΡΠΏΠ΅ΡˆΠ½Ρ‹Ρ… ΠΏΠ°Ρ€ΠΎΠ²Ρ‹Ρ… Π΄Π²ΠΈΠ³Π°Ρ‚Π΅Π»Π΅ΠΉ. Он ΠΎΠ΄ΠΎΠ»ΠΆΠΈΠ» Π½Π°ΠΌ свою Ρ„Π°ΠΌΠΈΠ»ΠΈΡŽ для этого подраздСлСния.

Π­Ρ‚ΠΎ ΠΈΠ·ΠΎΠ±Ρ€Π°ΠΆΠ΅Π½ΠΈΠ΅ ΠΏΠ°Ρ€ΠΎΠ²ΠΎΠΉ ΠΌΠ°ΡˆΠΈΠ½Ρ‹, Ρ€Π°Π·Ρ€Π°Π±ΠΎΡ‚Π°Π½Π½ΠΎΠΉ совмСстно ДТСймсом Π’Π°Ρ‚Ρ‚ΠΎΠΌ для ΠΎΡ‚ΠΊΠ°Ρ‡ΠΊΠΈ Π²ΠΎΠ΄Ρ‹ ΠΈΠ· Π·Π°Ρ‚ΠΎΠΏΠ»Π΅Π½Π½Ρ‹Ρ… ΡƒΠ³ΠΎΠ»ΡŒΠ½Ρ‹Ρ… ΡˆΠ°Ρ…Ρ‚ Π² Англии.

Π’Π°Ρ‚Ρ‚ – это ΠΎΠ΄ΠΈΠ½ Π΄ΠΆΠΎΡƒΠ»ΡŒ энСргии, Π·Π°Ρ‚Ρ€Π°Ρ‡ΠΈΠ²Π°Π΅ΠΌΡ‹ΠΉ Π·Π° сСкунду. Π’Π°ΠΊΠΈΠΌ ΠΎΠ±Ρ€Π°Π·ΠΎΠΌ, Π²Π°Ρ‚Ρ‚ Π²ΠΊΠ»ΡŽΡ‡Π°Π΅Ρ‚ Π² сСбя ΠΊΠ°ΠΊ Π·Π°Ρ‚Ρ€Π°Ρ‡Π΅Π½Π½ΡƒΡŽ ΡΠ½Π΅Ρ€Π³ΠΈΡŽ, Ρ‚Π°ΠΊ ΠΈ врСмя, Π² Ρ‚Π΅Ρ‡Π΅Π½ΠΈΠ΅ ΠΊΠΎΡ‚ΠΎΡ€ΠΎΠ³ΠΎ ΠΎΠ½Π° Π±Ρ‹Π»Π° Π·Π°Ρ‚Ρ€Π°Ρ‡Π΅Π½Π°.По Π°Π½Π°Π»ΠΎΠ³ΠΈΠΈ, Π²Ρ‹ ΠΌΠΎΠΆΠ΅Ρ‚Π΅ ΠΏΠΎΠ»ΡƒΡ‡ΠΈΡ‚ΡŒ ΠΎΠ΄ΠΈΠ½ Π³Π°Π»Π»ΠΎΠ½ Π²ΠΎΠ΄Ρ‹ ΠΈΠ· ΠΊΠ°ΠΏΠ°ΡŽΡ‰Π΅Π³ΠΎ ΠΊΡ€Π°Π½Π° Π·Π° час ΠΈΠ»ΠΈ ΠΈΠ· ΠΎΡ‚ΠΊΡ€Ρ‹Ρ‚ΠΎΠ³ΠΎ ΠΊΡ€Π°Π½Π° Π·Π° 15 сСкунд. Π’ ΠΊΠΎΠ½Ρ†Π΅ ΠΊΠΎΠ½Ρ†ΠΎΠ², Π²Ρ‹ всС Ρ€Π°Π²Π½ΠΎ ΠΏΠΎΠ»ΡƒΡ‡ΠΈΡ‚Π΅ Π³Π°Π»Π»ΠΎΠ½ Π²ΠΎΠ΄Ρ‹, Π½ΠΎ Π²ΠΎ Π²Ρ‚ΠΎΡ€ΠΎΠΌ случаС Π²ΠΎΠ΄Π° Ρ‚Π΅Ρ‡Π΅Ρ‚ Π² Π²Π΅Π΄Ρ€ΠΎ Π½Π°ΠΌΠ½ΠΎΠ³ΠΎ быстрСС. Π’Π°ΠΊ Ρ‡Ρ‚ΠΎ аспСкт Π²Ρ€Π΅ΠΌΠ΅Π½ΠΈ Π²Π°ΠΆΠ΅Π½. ΠœΡ‹ ΠΈΡΠΏΠΎΠ»ΡŒΠ·ΡƒΠ΅ΠΌ Ρ‚Π΅Ρ€ΠΌΠΈΠ½ ΠΌΠΎΡ‰Π½ΠΎΡΡ‚ΡŒ для обозначСния количСства энСргии ΠΈ скорости Π΅Π΅ доставки. Π”ΠΆΠΎΡƒΠ»ΡŒ – это ΠΏΠΎΠΊΠ°Π·Π°Ρ‚Π΅Π»ΡŒ энСргии, Π° Π²Π°Ρ‚Ρ‚ – ΠΏΠΎΠΊΠ°Π·Π°Ρ‚Π΅Π»ΡŒ мощности.

Насколько Π²Π΅Π»ΠΈΠΊ Π²Π°Ρ‚Ρ‚ мощности? ΠŸΠΎΠ΄Π±Ρ€Π°ΡΡ‹Π²Π°Π½ΠΈΠ΅ 100 Π³ яблока Π²Π²Π΅Ρ€Ρ… Π½Π° 1 ΠΌ (3.3 Ρ„ΡƒΡ‚Π°) потрСбляСт 1 Π²Π°Ρ‚Ρ‚ мощности. Ноутбук, ΠΊΠΎΡ‚ΠΎΡ€Ρ‹ΠΉ Π²Ρ‹, Π²ΠΎΠ·ΠΌΠΎΠΆΠ½ΠΎ, ΠΈΡΠΏΠΎΠ»ΡŒΠ·ΡƒΠ΅Ρ‚Π΅ для чтСния, потрСбляСт ΠΎΠΊΠΎΠ»ΠΎ 5 & acirc; & # 128; & # 147; 50 Π²Π°Ρ‚Ρ‚, Π² зависимости ΠΎΡ‚ Ρ‚ΠΎΠ³ΠΎ, Ρ€Π°Π±ΠΎΡ‚Π°Π΅Ρ‚ Π»ΠΈ Ρƒ вас Π² Ρ„ΠΎΠ½ΠΎΠ²ΠΎΠΌ Ρ€Π΅ΠΆΠΈΠΌΠ΅ ΠΌΡƒΠ·Ρ‹ΠΊΠ° ΠΈΠ»ΠΈ Ρ€Π°Π±ΠΎΡ‚Π°ΡŽΡ‚ Π΄Ρ€ΡƒΠ³ΠΈΠ΅ прилоТСния. Бтаромодная Π»Π°ΠΌΠΏΠ° накаливания ΠΌΠΎΡ‰Π½ΠΎΡΡ‚ΡŒΡŽ 100 Π’Ρ‚ потрСбляСт 1 ΠΊΠΈΠ»ΠΎΠ²Π°Ρ‚Ρ‚-час элСктроэнСргии, Ссли ΠΎΡΡ‚Π°Π²ΠΈΡ‚ΡŒ Π΅Π΅ Π²ΠΊΠ»ΡŽΡ‡Π΅Π½Π½ΠΎΠΉ Π½Π° 10 часов. ΠšΠΈΠ»ΠΎΠ²Π°Ρ‚Ρ‚ – это 1000 Π²Π°Ρ‚Ρ‚, сокращСнно ΠΊΠ’Ρ‚. 10 часов x 100 Π’Ρ‚ = 1000 ΠΊΠ’Ρ‚Ρ‡. ΠžΠ±Ρ€Π°Ρ‚ΠΈΡ‚Π΅ Π²Π½ΠΈΠΌΠ°Π½ΠΈΠ΅ Π½Π° Ρ€Π°Π·Π½ΠΈΡ†Ρƒ ΠΌΠ΅ΠΆΠ΄Ρƒ ΠΊΠ’Ρ‚ ΠΈ ΠΊΠ’Ρ‚Ρ‡. ΠšΠ’Ρ‚ – это ΠΌΠ΅Ρ€Π° мощности, Π° ΠΊΠ’Ρ‚Ρ‡ – ΠΌΠ΅Ρ€Π° Ρ‚ΠΎΠ³ΠΎ, сколько энСргии Π±Ρ‹Π»ΠΎ использовано Π² Ρ†Π΅Π»ΠΎΠΌ.

Π―Π±Π»ΠΎΠΊΠΎ, ΠΏΠ°Π΄Π°ΡŽΡ‰Π΅Π΅ Π½Π° ΠΌΠ΅Ρ‚Ρ€, Π΄Π΅Π»Π°Π΅Ρ‚ это с ΠΌΠΎΡ‰Π½ΠΎΡΡ‚ΡŒΡŽ 1 Π²Π°Ρ‚Ρ‚.

ΠŸΡ€ΠΎΠΈΡΡ…ΠΎΠΆΠ΄Π΅Π½ΠΈΠ΅: Π­Π²Π°Π½-Амос Автор изобраТСния
ΠŸΠΎΠ²Ρ‚ΠΎΡ€Π½ΠΎΠ΅ использованиС: Π›ΠΈΡ†ΠΎ, связавшСС ΠΏΡ€ΠΎΠΈΠ·Π²Π΅Π΄Π΅Π½ΠΈΠ΅ с этим Π΄ΠΎΠΊΡƒΠΌΠ΅Π½Ρ‚ΠΎΠΌ, посвятило ΠΏΡ€ΠΎΠΈΠ·Π²Π΅Π΄Π΅Π½ΠΈΠ΅ общСствСнному Π΄ΠΎΡΡ‚ΠΎΡΠ½ΠΈΡŽ, ΠΎΡ‚ΠΊΠ°Π·Π°Π²ΡˆΠΈΡΡŒ ΠΎΡ‚ всСх своих ΠΏΡ€Π°Π² Π½Π° ΠΏΡ€ΠΎΠΈΠ·Π²Π΅Π΄Π΅Π½ΠΈΠ΅ Π²ΠΎ всСм ΠΌΠΈΡ€Π΅ Π² соотвСтствии с Π·Π°ΠΊΠΎΠ½ΠΎΠΌ ΠΎΠ± авторском ΠΏΡ€Π°Π²Π΅, Π²ΠΊΠ»ΡŽΡ‡Π°Ρ всС смСТныС ΠΈ смСТныС ΠΏΡ€Π°Π²Π° Π² ΠΏΡ€Π΅Π΄Π΅Π»Π°Ρ…, Ρ€Π°Π·Ρ€Π΅ΡˆΠ΅Π½Π½Ρ‹Ρ… Π·Π°ΠΊΠΎΠ½ΠΎΠΌ. Π’Ρ‹ ΠΌΠΎΠΆΠ΅Ρ‚Π΅ ΠΊΠΎΠΏΠΈΡ€ΠΎΠ²Π°Ρ‚ΡŒ, ΠΈΠ·ΠΌΠ΅Π½ΡΡ‚ΡŒ, Ρ€Π°ΡΠΏΡ€ΠΎΡΡ‚Ρ€Π°Π½ΡΡ‚ΡŒ ΠΈ Π²Ρ‹ΠΏΠΎΠ»Π½ΡΡ‚ΡŒ Ρ€Π°Π±ΠΎΡ‚Ρƒ Π΄Π°ΠΆΠ΅ Π² коммСрчСских цСлях, Π½Π΅ ΡΠΏΡ€Π°ΡˆΠΈΠ²Π°Ρ Ρ€Π°Π·Ρ€Π΅ΡˆΠ΅Π½ΠΈΡ

Π’Ρ‹ Π½Π΅ ΡƒΠ²Π΅Ρ€Π΅Π½Ρ‹ Π² ΠΊΠΈΠ»ΠΎΠ²Π°Ρ‚Ρ‚-часах ΠΈ ΠΊΠΈΠ»ΠΎΠ²Π°Ρ‚Ρ‚-часах? Π­Ρ‚ΠΎ ΡƒΠ»ΠΎΠ²ΠΊΠ°.ΠŸΠΎΠΌΠ½ΠΈΡ‚Π΅, Ρ‡Ρ‚ΠΎ Π²Π°Ρ‚Ρ‚ – это Π΄ΠΆΠΎΡƒΠ»ΡŒ / сСк. Π—Π½Π°Ρ‡ΠΈΡ‚, Π² Π²Π°Ρ‚Ρ‚ ΠΈΠ»ΠΈ ΠΊΠΈΠ»ΠΎΠ²Π°Ρ‚Ρ‚ ΡƒΠΆΠ΅ Π·Π°Π»ΠΎΠΆΠ΅Π½ΠΎ врСмя. Π­Ρ‚ΠΎ энСргия / врСмя. Π­Ρ‚ΠΎ ΠΌΠΎΡ‰Π½ΠΎΡΡ‚ΡŒ, ΡΠΊΠΎΡ€ΠΎΡΡ‚ΡŒ использования энСргии. Но ΠΌΠΎΡ‰Π½ΠΎΡΡ‚ΡŒ Π½Π΅ сообщаСт Π²Π°ΠΌ, сколько энСргии Π±Ρ‹Π»ΠΎ использовано Π·Π° ΠΎΠΏΡ€Π΅Π΄Π΅Π»Π΅Π½Π½Ρ‹ΠΉ ΠΏΠ΅Ρ€ΠΈΠΎΠ΄ Π²Ρ€Π΅ΠΌΠ΅Π½ΠΈ. Π§Ρ‚ΠΎΠ±Ρ‹ ΠΏΠΎΠ»ΡƒΡ‡ΠΈΡ‚ΡŒ это, Π²Π°ΠΌ Π½ΡƒΠΆΠ½ΠΎ ΡƒΠΌΠ½ΠΎΠΆΠΈΡ‚ΡŒ ΠΌΠΎΡ‰Π½ΠΎΡΡ‚ΡŒ Π½Π° врСмя. Π—Π°Ρ‚Π΅ΠΌ Π΅Π΄ΠΈΠ½ΠΈΡ†Ρ‹ Π²Ρ€Π΅ΠΌΠ΅Π½ΠΈ Π΄ΠΎΠ»ΠΆΠ½Ρ‹ Π±Ρ‹Ρ‚ΡŒ Π·Π°Ρ‡Π΅Ρ€ΠΊΠ½ΡƒΡ‚Ρ‹. Π£Π²Ρ‹, принято ΠΎΡΡ‚Π°Π²Π»ΡΡ‚ΡŒ час Π½Π° мСстС – Π³Π»ΡƒΠΏΠΎ, Π½ΠΎ Ρ‚Π°ΠΊ это дСлаСтся. 1 ΠΊΠ’Ρ‚Ρ‡ = 1 ΠΊΠ’Ρ‚ x 1 час.

Π’ΠΎΡ‚ ΠΏΡ€ΠΈΠΌΠ΅Ρ€. Π’ ΠΌΠΎΠ΅ΠΌ Π΄ΠΎΠΌΠ΅ Π΅ΡΡ‚ΡŒ фотоэлСктричСская систСма (солнСчная элСктроэнСргия), которая Π² ΠΈΠ΄Π΅Π°Π»ΡŒΠ½Ρ‹Ρ… условиях приятного солнСчного ΠΏΡ€ΠΎΡ…Π»Π°Π΄Π½ΠΎΠ³ΠΎ дня рассчитана Π½Π° Π²Ρ‹Ρ€Π°Π±ΠΎΡ‚ΠΊΡƒ 4 ΠΊΠ’Ρ‚.Π—Π° 4 часа это составит:

4 ΠΊΠ’Ρ‚ x 4 часа = 16 ΠΊΠ’Ρ‚ Β· Ρ‡ элСктроэнСргии. Π’ частично пасмурный дСнь систСма ΠΌΠΎΠΆΠ΅Ρ‚ Ρ€Π°Π±ΠΎΡ‚Π°Ρ‚ΡŒ Π½Π° ΠΏΠΎΠ»ΠΎΠ²ΠΈΠ½Π½ΠΎΠΉ мощности ΠΈΠ»ΠΈ Π½Π° 2 ΠΊΠ’Ρ‚ Π²Ρ‹Ρ…ΠΎΠ΄Π½ΠΎΠΉ мощности. ΠŸΡ€ΠΈ Ρ‚Π°ΠΊΠΎΠΉ скорости ΠΌΠ½Π΅ потрСбуСтся 8 часов, Ρ‡Ρ‚ΠΎΠ±Ρ‹ Π²Ρ‹Ρ€Π°Π±ΠΎΡ‚Π°Ρ‚ΡŒ Ρ‚Π΅ ΠΆΠ΅ 16 ΠΊΠ’Ρ‚ Β· Ρ‡, Ρ‡Ρ‚ΠΎ я сдСлал Π² солнСчный дСнь; 2ΠΊΠ’Ρ‚ x 8 часов = 16 ΠΊΠ’Ρ‚Ρ‡.

Π’ состоянии покоя Ρ‚ΠΈΠΏΠΈΡ‡Π½Ρ‹ΠΉ Ρ‡Π΅Π»ΠΎΠ²Π΅ΠΊ ΠΈΡΠΏΠΎΠ»ΡŒΠ·ΡƒΠ΅Ρ‚ ΡΠ½Π΅Ρ€Π³ΠΈΡŽ 80 Π’Ρ‚ для обСспСчСния ΠΆΠΈΠ·Π½Π΅Π½Π½Ρ‹Ρ… Ρ„ΡƒΠ½ΠΊΡ†ΠΈΠΉ ΠΎΡ€Π³Π°Π½ΠΈΠ·ΠΌΠ° (Ρ‚Π°ΠΊ Π½Π°Π·Ρ‹Π²Π°Π΅ΠΌΡ‹ΠΉ ΠΌΠ΅Ρ‚Π°Π±ΠΎΠ»ΠΈΠ·ΠΌ Π² состоянии покоя). Взрослый ΠΌΡƒΠΆΡ‡ΠΈΠ½Π° ΠΌΠΎΠΆΠ΅Ρ‚ ΡΡŠΠ΅Π΄Π°Ρ‚ΡŒ ΠΎΠΊΠΎΠ»ΠΎ 2000 ΠΊΠΈΠ»ΠΎΠΊΠ°Π»ΠΎΡ€ΠΈΠΉ Π² дСнь. Одна ΠΊΠΊΠ°Π» = 1,163 Π’Ρ‚Ρ‡. Π’Π°ΠΊΠΈΠΌ ΠΎΠ±Ρ€Π°Π·ΠΎΠΌ, Π΄ΠΈΠ΅Ρ‚Π° Π½Π° 2000 ΠΊΠΊΠ°Π» обСспСчит 2326 Π’Ρ‚Ρ‡ ΠΈΠ»ΠΈ 2 Π’Ρ‚Ρ‡.326 ΠΊΠ’Ρ‚Ρ‡. Если Π±Ρ‹ Ρ‡Π΅Π»ΠΎΠ²Π΅ΠΊ просто ΠΏΡ€ΠΎΠ»Π΅ΠΆΠ°Π» Π² постСли 24 часа, ΠΎΠ½ Π±Ρ‹ сТСг 80 Π’Ρ‚ x 24 часа = 1920 Π’Ρ‚ Β· Ρ‡ ΠΈΠ»ΠΈ 1 920 ΠΊΠ’Ρ‚ Β· Ρ‡. Если этот ΠΏΠ°Ρ€Π΅Π½ΡŒ останСтся Π² постСли ΠΈ ΠΏΡ€ΠΎΠ΄ΠΎΠ»ΠΆΠΈΡ‚ Ρ‚Π°ΠΊ Π΅ΡΡ‚ΡŒ, ΠΎΠ½ Π² ΠΊΠΎΠ½Π΅Ρ‡Π½ΠΎΠΌ ΠΈΡ‚ΠΎΠ³Π΅ потрСбляСт 2,326 ΠΊΠ’Ρ‚Ρ‡ & acirc; & # 128; & # 147; 1,920 ΠΊΠ’Ρ‚Ρ‡ = 0,406 ΠΊΠ’Ρ‚Ρ‡ большС, Ρ‡Π΅ΠΌ ΠΎΠ½ ΠΈΡΠΏΠΎΠ»ΡŒΠ·ΡƒΠ΅Ρ‚, ΠΈ это Π±ΡƒΠ΄Π΅Ρ‚ Ρ…Ρ€Π°Π½ΠΈΡ‚ΡŒΡΡ Π² Π²ΠΈΠ΄Π΅ ΠΆΠΈΡ€Π°. Π€ΡƒΠ½Ρ‚ ΠΆΠΈΡ€Π° Ρ€Π°Π²Π΅Π½ ΠΏΡ€ΠΈΠΌΠ΅Ρ€Π½ΠΎ 3500 ΠΊΠΊΠ°Π» (4 070,5 ΠΊΠ’Ρ‚Ρ‡). Π’Π°ΠΊ Ρ‡Ρ‚ΠΎ Ρ‡Π΅Ρ€Π΅Π· Π΄Π΅ΡΡΡ‚ΡŒ Π΄Π½Π΅ΠΉ ΠΎΠ½ ΠΌΠΎΠΆΠ΅Ρ‚ ΠΏΡ€ΠΈΠ±Π°Π²ΠΈΡ‚ΡŒ Π΅Ρ‰Π΅ Ρ„ΡƒΠ½Ρ‚. Активная ΠΏΠΎΠ΅Π·Π΄ΠΊΠ° Π½Π° вСлосипСдС ΠΈΡΠΏΠΎΠ»ΡŒΠ·ΡƒΠ΅Ρ‚ ΡΠ½Π΅Ρ€Π³ΠΈΡŽ Π² Ρ€Π°Π·ΠΌΠ΅Ρ€Π΅ 200 Π’Ρ‚. ΠŸΠΎΡΡ‚ΠΎΠΌΡƒ Π΅ΠΌΡƒ слСдуСт ΠΏΠΎΠ΄ΡƒΠΌΠ°Ρ‚ΡŒ ΠΎ двухчасовой ΠΏΠΎΠ΅Π·Π΄ΠΊΠ΅ Π½Π° вСлосипСдС, Ρ‡Ρ‚ΠΎΠ±Ρ‹ ΠΎΡΡ‚Π°Π²Π°Ρ‚ΡŒΡΡ Π² Ρ„ΠΎΡ€ΠΌΠ΅ (0.2 ΠΊΠ’Ρ‚ для Π΅Π·Π΄Ρ‹ Π½Π° вСлосипСдС x 2 часа = 4,0 ΠΊΠ’Ρ‚Ρ‡).

Π‘Π²ΠΎΠ΄ΠΊΠ° силы, Ρ€Π°Π±ΠΎΡ‚Ρ‹ ΠΈ мощности

Π‘ΠΈΠ»Π° = ЭнСргия, прилоТСнная ΠΊ ΠΎΠ±ΡŠΠ΅ΠΊΡ‚Ρƒ (измСряСтся Π² Π½ΡŒΡŽΡ‚ΠΎΠ½Π°Ρ…).

Π Π°Π±ΠΎΡ‚Π° = Π‘ΠΈΠ»Π° X РасстояниС ΠΈΠ»ΠΈ количСство ΠΏΠ΅Ρ€Π΅Π΄Π°Π½Π½ΠΎΠ³ΠΎ Ρ‚Π΅ΠΏΠ»Π° (Π˜Π·ΠΌΠ΅Ρ€ΡΠ΅Ρ‚ΡΡ Π² ДТоулях ΠΈΠ»ΠΈ калориях) .

ΠœΠΎΡ‰Π½ΠΎΡΡ‚ΡŒ = Π Π°Π±ΠΎΡ‚Π° / ВрСмя (Π˜Π·ΠΌΠ΅Ρ€ΡΠ΅Ρ‚ΡΡ Π² Π²Π°Ρ‚Ρ‚Π°Ρ… с)

Π Π°Π·Π»ΠΈΡ‡Π½Ρ‹Π΅ Π΅Π΄ΠΈΠ½ΠΈΡ†Ρ‹ энСргии

1 калория (тСрмохимичСская) = 4.184 Π”ΠΆ

1 британская тСпловая Π΅Π΄ΠΈΠ½ΠΈΡ†Π° = 251,9958 ΠΊΠ°Π»ΠΎΡ€ΠΈΠΉ

1 Π‘Π’Π• (тСрмохимичСский) = 1054,35 Π”ΠΆ

1 ΠΊΠΈΠ»ΠΎΠ²Π°Ρ‚Ρ‚-час (ΠΊΠ’Ρ‚Ρ‡) = 3,6 x 106 Π”ΠΆ

1 ΠΊΠΈΠ»ΠΎΠ²Π°Ρ‚Ρ‚-час (ΠΊΠ’Ρ‚Ρ‡) = 3412 британских Ρ‚Π΅ΠΏΠ»ΠΎΠ²Ρ‹Ρ… Π΅Π΄ΠΈΠ½ΠΈΡ† (IT)

1 Ρ‚Π΅Ρ€ΠΌ = 100 000 британских Ρ‚Π΅ΠΏΠ»ΠΎΠ²Ρ‹Ρ… Π΅Π΄ΠΈΠ½ΠΈΡ†

1 элСктрон-Π²ΠΎΠ»ΡŒΡ‚ = 1,6022 x 10-19 Π”ΠΆ

ЭлСктричСство ΠΈ ΠΌΠ°Π³Π½Π΅Ρ‚ΠΈΠ·ΠΌ

Π˜Π·ΠΎΠ»ΠΈΡ€ΠΎΠ²Π°Π½Π½Ρ‹Π΅ ΠΏΡ€ΠΎΠ²ΠΎΠ΄Π°

ΠŸΡ€ΠΎΠΈΡΡ…ΠΎΠΆΠ΄Π΅Π½ΠΈΠ΅: Π§Π°Ρ‚Π°ΠΌΠ° Ρ€Π°Π·ΠΌΠ΅Ρ‰Π΅Π½ΠΎ Π½Π° ВикискладС https://commons.wikimedia.org/wiki/File:600V_CV_5.5sqmm.jpg
ΠŸΠΎΠ²Ρ‚ΠΎΡ€Π½ΠΎΠ΅ использованиС: Π­Ρ‚ΠΎΡ‚ Ρ„Π°ΠΉΠ» находится ΠΏΠΎΠ΄ Π»ΠΈΡ†Π΅Π½Π·ΠΈΠ΅ΠΉ Creative Commons Attribution-Share Alike 3.0 НСпортированная лицСнзия. Π’Ρ‹ ΠΌΠΎΠΆΠ΅Ρ‚Π΅: Π΄Π΅Π»ΠΈΡ‚ΡŒΡΡ – ΠΊΠΎΠΏΠΈΡ€ΠΎΠ²Π°Ρ‚ΡŒ, Ρ€Π°ΡΠΏΡ€ΠΎΡΡ‚Ρ€Π°Π½ΡΡ‚ΡŒ ΠΈ ΠΏΠ΅Ρ€Π΅Π΄Π°Π²Π°Ρ‚ΡŒ ΠΏΡ€ΠΎΠΈΠ·Π²Π΅Π΄Π΅Π½ΠΈΠ΅ для рСмикса – Π°Π΄Π°ΠΏΡ‚ΠΈΡ€ΠΎΠ²Π°Ρ‚ΡŒ ΠΏΡ€ΠΎΠΈΠ·Π²Π΅Π΄Π΅Π½ΠΈΠ΅ ΠŸΡ€ΠΈ ΡΠ»Π΅Π΄ΡƒΡŽΡ‰ΠΈΡ… условиях: приписываниС – Π²Ρ‹ Π΄ΠΎΠ»ΠΆΠ½Ρ‹ ΡƒΠΊΠ°Π·Ρ‹Π²Π°Ρ‚ΡŒ ΠΏΡ€ΠΎΠΈΠ·Π²Π΅Π΄Π΅Π½ΠΈΠ΅ способом, ΡƒΠΊΠ°Π·Π°Π½Π½Ρ‹ΠΌ Π°Π²Ρ‚ΠΎΡ€ΠΎΠΌ ΠΈΠ»ΠΈ Π»ΠΈΡ†Π΅Π½Π·ΠΈΠ°Ρ€ΠΎΠΌ (Π½ΠΎ Π½ΠΈ Π² ΠΊΠΎΠ΅ΠΌ случаС, Ρ‡Ρ‚ΠΎΠ±Ρ‹ ΠΏΡ€Π΅Π΄ΠΏΠΎΠ»Π°Π³Π°Π΅Ρ‚, Ρ‡Ρ‚ΠΎ ΠΎΠ½ΠΈ ΠΎΠ΄ΠΎΠ±Ρ€ΡΡŽΡ‚ вас ΠΈΠ»ΠΈ вашС использованиС произвСдСния). совмСстно ΠΈΡΠΏΠΎΠ»ΡŒΠ·ΠΎΠ²Π°Ρ‚ΡŒ – Ссли Π²Ρ‹ измСняСтС, трансформируСтС ΠΈΠ»ΠΈ Ρ€Π°ΡΡˆΠΈΡ€ΡΠ΅Ρ‚Π΅ эту Ρ€Π°Π±ΠΎΡ‚Ρƒ, Π²Ρ‹ ΠΌΠΎΠΆΠ΅Ρ‚Π΅ Ρ€Π°ΡΠΏΡ€ΠΎΡΡ‚Ρ€Π°Π½ΡΡ‚ΡŒ ΠΏΠΎΠ»ΡƒΡ‡Π΅Π½Π½ΡƒΡŽ Ρ€Π°Π±ΠΎΡ‚Ρƒ Ρ‚ΠΎΠ»ΡŒΠΊΠΎ ΠΏΠΎ Ρ‚ΠΎΠΉ ΠΆΠ΅ ΠΈΠ»ΠΈ Π°Π½Π°Π»ΠΎΠ³ΠΈΡ‡Π½ΠΎΠΉ Π»ΠΈΡ†Π΅Π½Π·ΠΈΠΈ, Ρ‡Ρ‚ΠΎ ΠΈ эта.

Π’Π΅ΠΏΠ΅Ρ€ΡŒ, ΠΊΠΎΠ³Π΄Π° Ρƒ вас Π΅ΡΡ‚ΡŒ Ρ…ΠΎΡ€ΠΎΡˆΠ΅Π΅ прСдставлСниС ΠΎΠ± энСргии, Ρ€Π°Π±ΠΎΡ‚Π΅ ΠΈ мощности, ΠΏΠΎΡ€Π° Π·Π°Ρ€ΡΠ΄ΠΈΡ‚ΡŒΡΡ ΠΈ ΠΈΠ·ΡƒΡ‡ΠΈΡ‚ΡŒ элСктричСство! Π”Ρ€Π΅Π²Π½ΠΈΠ΅ ΠΈΠΌΠ΅Π»ΠΈ смутноС прСдставлСниС ΠΎΠ± элСктричСствС ΠΈΠ·-Π·Π° своСго ΠΆΠΈΠ·Π½Π΅Π½Π½ΠΎΠ³ΠΎ ΠΎΠΏΡ‹Ρ‚Π°.Π Ρ‹Π±Π°ΠΊΠΈ, ловившиС Ρ€Π°Π·Π½ΠΎΠ³ΠΎ Ρ€ΠΎΠ΄Π° Β«ΡΠ»Π΅ΠΊΡ‚Ρ€ΠΈΡ‡Π΅ΡΠΊΡƒΡŽ Ρ€Ρ‹Π±ΡƒΒ», ΠΏΡ€ΠΈ ΠΎΠ±Ρ€Π°Ρ‰Π΅Π½ΠΈΠΈ с Π½Π΅ΠΉ ΠΏΠΎΠ΄Π²Π΅Ρ€Π³Π°Π»ΠΈΡΡŒ ΡˆΠΎΠΊΡƒ. Π”Ρ€ΡƒΠ³ΠΈΠ΅ чувствовали воздСйствиС статичСского элСктричСства ΠΎΡ‚ своСй ΡˆΠ΅Ρ€ΡΡ‚ΡΠ½ΠΎΠΉ ΠΎΠ΄Π΅ΠΆΠ΄Ρ‹. ЕгиптянС Π²ΠΈΠ΄Π΅Π»ΠΈ связь ΠΌΠ΅ΠΆΠ΄Ρƒ элСктричСской Ρ€Ρ‹Π±ΠΎΠΉ ΠΈ ΠΌΠΎΠ»Π½ΠΈΠ΅ΠΉ. Но Ρ‚ΠΎΠ»ΡŒΠΊΠΎ ΠΎΠΊΠΎΠ»ΠΎ 1600 Π³ΠΎΠ΄Π° Π½Π°Ρ‡Π°Π»ΠΈΡΡŒ ΡΠ΅Ρ€ΡŒΠ΅Π·Π½Ρ‹Π΅ Π½Π°ΡƒΡ‡Π½Ρ‹Π΅ исслСдования элСктричСства. Усилиями ΠΌΠ½ΠΎΠ³ΠΈΡ… исслСдоватСлСй ΠΊ ΠΊΠΎΠ½Ρ†Ρƒ 19 Π²Π΅ΠΊΠ° Π±Ρ‹Π»ΠΎ Ρ€Π°Π·Ρ€Π°Π±ΠΎΡ‚Π°Π½ΠΎ Ρ…ΠΎΡ€ΠΎΡˆΠ΅Π΅ прСдставлСниС ΠΎΠ± элСктричСствС ΠΈ ΠΎ Ρ‚ΠΎΠΌ, ΠΊΠ°ΠΊ Π΅Π³ΠΎ ΠΈΡΠΏΠΎΠ»ΡŒΠ·ΠΎΠ²Π°Ρ‚ΡŒ.

Напомним, Ρ‡Ρ‚ΠΎ вся матСрия состоит ΠΈΠ· Π°Ρ‚ΠΎΠΌΠΎΠ².А Π°Ρ‚ΠΎΠΌΡ‹ состоят ΠΈΠ· Π½Π΅ΡΠΊΠΎΠ»ΡŒΠΊΠΈΡ… основных частиц: элСктронов с ΠΎΡ‚Ρ€ΠΈΡ†Π°Ρ‚Π΅Π»ΡŒΠ½Ρ‹ΠΌ зарядом, ΠΏΡ€ΠΎΡ‚ΠΎΠ½ΠΎΠ² с ΠΏΠΎΠ»ΠΎΠΆΠΈΡ‚Π΅Π»ΡŒΠ½Ρ‹ΠΌ зарядом ΠΈ Π½Π΅ΠΉΡ‚Ρ€ΠΎΠ½ΠΎΠ² Π±Π΅Π· заряда. ЭлСктричСство ΠΌΠΎΠΆΠ½ΠΎ ΠΏΡ€Π΅Π΄ΡΡ‚Π°Π²ΠΈΡ‚ΡŒ ΠΊΠ°ΠΊ ΠΏΠΎΡ‚ΠΎΠΊ элСктронов Ρ‡Π΅Ρ€Π΅Π· ΠΏΡ€ΠΎΠ²ΠΎΠ΄Π½ΠΈΠΊ, ΠΏΠΎΠ΄ΠΎΠ±Π½Ρ‹ΠΉ ΠΌΠ΅Π΄Π½ΠΎΠΌΡƒ ΠΏΡ€ΠΎΠ²ΠΎΠ΄Ρƒ. На самом Π΄Π΅Π»Π΅ это Π½Π΅ ΠΏΠΎΡ‚ΠΎΠΊ элСктронов, Π° ΠΈΠΌΠΏΡƒΠ»ΡŒΡ, ΠΊΠΎΡ‚ΠΎΡ€Ρ‹ΠΉ ΠΏΡ€ΠΎΡ…ΠΎΠ΄ΠΈΡ‚ ΠΏΠΎ ΠΏΡ€ΠΎΠ²ΠΎΠ΄Ρƒ.

Π₯ΠΎΡ€ΠΎΡˆΠΈΠ΅ ΠΏΡ€ΠΎΠ²ΠΎΠ΄Π½ΠΈΠΊΠΈ, ΠΊΠ°ΠΊ ΠΈ ΠΌΠ΅Ρ‚Π°Π»Π»Ρ‹, Π»Π΅Π³ΠΊΠΎ ΠΏΡ€ΠΎΠΏΡƒΡΠΊΠ°ΡŽΡ‚ элСктричСство. Π£ Π½ΠΈΡ… Π΅ΡΡ‚ΡŒ элСктроны Π½Π° Π²Π½Π΅ΡˆΠ½ΠΈΡ… орбиталях, с ΠΊΠΎΡ‚ΠΎΡ€Ρ‹ΠΌΠΈ Π»Π΅Π³ΠΊΠΎ Π²ΡΡ‚ΡƒΠΏΠΈΡ‚ΡŒ Π² ΠΊΠΎΠ½Ρ‚Π°ΠΊΡ‚. ΠŸΠ»ΠΎΡ…ΠΈΠ΅ ΠΏΡ€ΠΎΠ²ΠΎΠ΄Π½ΠΈΠΊΠΈ Π½Π°Π·Ρ‹Π²Π°ΡŽΡ‚ΡΡ изоляторами, ΠΈ ΠΎΠ½ΠΈ Π½Π΅ ΠΏΡ€ΠΎΠΏΡƒΡΠΊΠ°ΡŽΡ‚ бСспрСпятствСнный Ρ‚ΠΎΠΊ элСктричСства.Π”Π°ΠΆΠ΅ самыС Π»ΡƒΡ‡ΡˆΠΈΠ΅ ΠΏΡ€ΠΎΠ²ΠΎΠ΄Π½ΠΈΠΊΠΈ ΠΎΠΊΠ°Π·Ρ‹Π²Π°ΡŽΡ‚ Π½Π΅ΠΊΠΎΡ‚ΠΎΡ€ΠΎΠ΅ сопротивлСниС Ρ‚ΠΎΠΊΡƒ элСктричСства. Π’Π°ΠΊΠΎΠ΅ сопротивлСниС измСряСтся Π² Π΅Π΄ΠΈΠ½ΠΈΡ†Π°Ρ…, Π½Π°Π·Ρ‹Π²Π°Π΅ΠΌΡ‹Ρ… Ом. Π‘Ρ‚Π΅ΠΊΠ»ΠΎ – Ρ…ΠΎΡ€ΠΎΡˆΠΈΠΉ изолятор ΠΈ, ΡΠ»Π΅Π΄ΠΎΠ²Π°Ρ‚Π΅Π»ΡŒΠ½ΠΎ, ΠΏΠ»ΠΎΡ…ΠΎΠΉ ΠΏΡ€ΠΎΠ²ΠΎΠ΄Π½ΠΈΠΊ.

Π’Ρ€Π΅Ρ‚ΠΈΠΉ класс соСдинСний – ΠΏΠΎΠ»ΡƒΠΏΡ€ΠΎΠ²ΠΎΠ΄Π½ΠΈΠΊΠΈ. Они Ρ€Π΅Π°Π³ΠΈΡ€ΡƒΡŽΡ‚ Π½Π° ΠΈΠ·ΠΌΠ΅Π½Π΅Π½ΠΈΠ΅ условий, Ρ‡Ρ‚ΠΎΠ±Ρ‹ Π²ΠΊΠ»ΡŽΡ‡ΠΈΡ‚ΡŒ ΠΈΠ»ΠΈ Π²Ρ‹ΠΊΠ»ΡŽΡ‡ΠΈΡ‚ΡŒ ΠΏΠΎΠ΄Π°Ρ‡Ρƒ элСктричСства. ΠŸΠΎΠ»ΡƒΠΏΡ€ΠΎΠ²ΠΎΠ΄Π½ΠΈΠΊΠΈ часто содСрТат смСсь крСмния ΠΈ ΠΌΠ΅Ρ‚Π°Π»Π»ΠΎΠ². ΠŸΠ»Π°ΡΡ‚ΠΈΠ½Ρ‹ ΠΈΠ· этих ΠΏΠΎΠ»ΡƒΠΏΡ€ΠΎΠ²ΠΎΠ΄Π½ΠΈΠΊΠΎΠ² Π»Π΅ΠΆΠ°Ρ‚ Π² основС «микросхСм» ΠΊΠΎΠΌΠΏΡŒΡŽΡ‚Π΅Ρ€Π°, Π° Ρ‚Π°ΠΊΠΆΠ΅ ΡΠ²Π»ΡΡŽΡ‚ΡΡ основой для свСтодиодных Π»Π°ΠΌΠΏ ΠΈ фотоэлСктричСских (солнСчных) элСмСнтов.

ЀотоэлСктричСскиС ΠΏΠ°Π½Π΅Π»ΠΈ ΠΈΠ·Π³ΠΎΡ‚ΠΎΠ²Π»Π΅Π½Ρ‹ ΠΈΠ· ΠΏΠΎΠ»ΡƒΠΏΡ€ΠΎΠ²ΠΎΠ΄Π½ΠΈΠΊΠΎΠ².

ΠŸΡ€ΠΎΠΈΡΡ…ΠΎΠΆΠ΄Π΅Π½ΠΈΠ΅: Π€ΠΎΡ‚ΠΎ Π‘. ΠšΡƒΠΊΠ΅Ρ€Π°
ΠŸΠΎΠ²Ρ‚ΠΎΡ€Π½ΠΎΠ΅ использованиС: бСсплатно для ΠΏΠΎΠ²Ρ‚ΠΎΡ€Π½ΠΎΠ³ΠΎ использования

ПанСли фотоэлСмСнтов, ΠΈΡΠΏΠΎΠ»ΡŒΠ·ΡƒΠ΅ΠΌΡ‹Ρ… для производства элСктричСства ΠΈΠ· солнСчного свСта, сдСланы ΠΈΠ· ΠΏΠΎΠ»ΡƒΠΏΡ€ΠΎΠ²ΠΎΠ΄Π½ΠΈΠΊΠΎΠ².

Для ΠΏΠΎΠ΄Π°Ρ‡ΠΈ элСктричСства Π΄ΠΎΠ»ΠΆΠ½Π° Π±Ρ‹Ρ‚ΡŒ замкнутая Ρ†Π΅ΠΏΡŒ. Π­Π»Π΅ΠΊΡ‚Ρ€ΠΎΠ½Ρ‹ Π΄ΠΎΠ»ΠΆΠ½Ρ‹ Π½Π°Ρ‡ΠΈΠ½Π°Ρ‚ΡŒ с состояния с высокой энСргиСй ΠΈ Π·Π°ΠΊΠ°Π½Ρ‡ΠΈΠ²Π°Ρ‚ΡŒΡΡ Π² состоянии с Π½ΠΈΠ·ΠΊΠΎΠΉ энСргиСй. НиТС прСдставлСна ​​схСма простой схСмы. ΠžΠ±Ρ€Π°Ρ‚ΠΈΡ‚Π΅ Π²Π½ΠΈΠΌΠ°Π½ΠΈΠ΅, Ρ‡Ρ‚ΠΎ элСктричСство ΠΏΡ€ΠΎΡ…ΠΎΠ΄ΠΈΡ‚ ΠΎΡ‚ высокоэнСргСтичСского ΠΊΠΎΠ½Ρ†Π° Π±Π°Ρ‚Π°Ρ€Π΅ΠΈ Ρ‡Π΅Ρ€Π΅Π· Π»Π°ΠΌΠΏΡƒ, Π° Π·Π°Ρ‚Π΅ΠΌ ΠΎΠ±Ρ€Π°Ρ‚Π½ΠΎ ΠΊ низкоэнСргСтичСскому ΠΊΠΎΠ½Ρ†Ρƒ Π±Π°Ρ‚Π°Ρ€Π΅ΠΈ.Когда Π²Ρ‹ΠΊΠ»ΡŽΡ‡Π°Ρ‚Π΅Π»ΡŒ Ρ€Π°Π·ΠΎΠΌΠΊΠ½ΡƒΡ‚, ΠΏΠΎΠ΄Π°Ρ‡Π° элСктричСства прСкращаСтся.

Об элСктричСствС просто Π΄ΡƒΠΌΠ°Ρ‚ΡŒ ΠΊΠ°ΠΊ ΠΎΠ± элСктронС (ΠΈΠ»ΠΈ ΠΈΠΌΠΏΡƒΠ»ΡŒΡΠ΅ Ρ€Π°Π·ΠΌΠ΅Ρ€ΠΎΠΌ с элСктрон), ΠΏΡ€ΠΎΡ‚Π΅ΠΊΠ°ΡŽΡ‰Π΅ΠΌ ΠΏΠΎ ΠΏΡ€ΠΎΠ²ΠΎΠ΄Π½ΠΈΠΊΡƒ. Но Π½Π° ΠΏΡ€Π°ΠΊΡ‚ΠΈΠΊΠ΅ ΠΎΠ΄ΠΈΠ½ элСктрон слишком ΠΌΠ°Π» ΠΈ нСсСт слишком ΠΌΠ°Π»ΠΎ энСргии, Ρ‡Ρ‚ΠΎΠ±Ρ‹ Π²Ρ‹ΠΏΠΎΠ»Π½ΡΡ‚ΡŒ ΠΊΠ°ΠΊΡƒΡŽ-Π»ΠΈΠ±ΠΎ Ρ€Π΅Π°Π»ΡŒΠ½ΡƒΡŽ Ρ€Π°Π±ΠΎΡ‚Ρƒ. Π’Π΅ΠΌ Π½Π΅ ΠΌΠ΅Π½Π΅Π΅, ΡΡ‚Π΅ΠΊΠ°ΡŽΡ‰ΠΈΠ΅ вмСстС Π³Ρ€ΡƒΠΏΠΏΡ‹ элСктронов ΠΌΠΎΠ³ΡƒΡ‚ Π²Ρ‹Π·Π²Π°Ρ‚ΡŒ большой Ρ‚ΠΎΠ»Ρ‡ΠΎΠΊ! ΠšΡƒΠ»ΠΎΠ½ Ρ€Π°Π²Π΅Π½ 6,24 Γ— 10 18 элСктронов. А amp – это ΠΏΠΎΡ‚ΠΎΠΊ Π² ΠΎΠ΄ΠΈΠ½ ΠΊΡƒΠ»ΠΎΠ½ Π² сСкунду Ρ‡Π΅Ρ€Π΅Π· ΠΏΡ€ΠΎΠ²ΠΎΠ΄Π½ΠΈΠΊ. Π’Π°ΠΊΠΈΠΌ ΠΎΠ±Ρ€Π°Π·ΠΎΠΌ, усилитСли ΠΈΠ·ΠΌΠ΅Ρ€ΡΡŽΡ‚ ΡΠΊΠΎΡ€ΠΎΡΡ‚ΡŒ ΠΏΠΎΡ‚ΠΎΠΊΠ° элСктричСства.ΠœΡ‹ Π½Π°Π·Ρ‹Π²Π°Π΅ΠΌ ΠΏΠΎΡ‚ΠΎΠΊ элСктричСства Ρ‚ΠΎΠΊΠΎΠΌ.

НС всС элСктричСство Ρ‚Π΅Ρ‡Π΅Ρ‚ с ΠΎΠ΄ΠΈΠ½Π°ΠΊΠΎΠ²ΠΎΠΉ силой. Π§Ρ‚ΠΎΠ±Ρ‹ ΠΏΠΎΠ½ΡΡ‚ΡŒ это, ΠΏΠΎΠ΄ΡƒΠΌΠ°ΠΉΡ‚Π΅ ΠΎ Π΄Π°Π²Π»Π΅Π½ΠΈΠΈ ΠΈΠ»ΠΈ силС Π²ΠΎΠ΄Ρ‹, выходящСй ΠΈΠ· Ρ‚Ρ€ΡƒΠ±Ρ‹. Если Ρ‚Ρ€ΡƒΠ±Π° ΠΏΡ€ΠΈΠΊΡ€Π΅ΠΏΠ»Π΅Π½Π° ΠΊ Ρ€Π΅Π·Π΅Ρ€Π²ΡƒΠ°Ρ€Ρƒ Π½Π°Π²Π΅Ρ€Ρ…Ρƒ высокого здания, Π²ΠΎΠ΄Π° Π±ΡƒΠ΄Π΅Ρ‚ ΠΈΠΌΠ΅Ρ‚ΡŒ Π³ΠΎΡ€Π°Π·Π΄ΠΎ большСС Π΄Π°Π²Π»Π΅Π½ΠΈΠ΅, Ρ‡Π΅ΠΌ Ссли Π±Ρ‹ Ρ€Π΅Π·Π΅Ρ€Π²ΡƒΠ°Ρ€ Π±Ρ‹Π» Π½Π° 30 см Π²Ρ‹ΡˆΠ΅ Ρ‚Ρ€ΡƒΠ±Ρ‹. Π’ΠΎ ΠΆΠ΅ самоС ΠΈ с элСктричСством. Β«Π”Π°Π²Π»Π΅Π½ΠΈΠ΅Β» элСктричСства – это элСктричСский ΠΏΠΎΡ‚Π΅Π½Ρ†ΠΈΠ°Π». ЭлСктричСский ΠΏΠΎΡ‚Π΅Π½Ρ†ΠΈΠ°Π» – это количСство энСргии, доступноС для проталкивания ΠΊΠ°ΠΆΠ΄ΠΎΠΉ Π΅Π΄ΠΈΠ½ΠΈΡ†Ρ‹ заряда Ρ‡Π΅Ρ€Π΅Π· ΡΠ»Π΅ΠΊΡ‚Ρ€ΠΈΡ‡Π΅ΡΠΊΡƒΡŽ Ρ†Π΅ΠΏΡŒ.Π•Π΄ΠΈΠ½ΠΈΡ†Π΅ΠΉ измСрСния элСктричСского ΠΏΠΎΡ‚Π΅Π½Ρ†ΠΈΠ°Π»Π° являСтся Π²ΠΎΠ»ΡŒΡ‚. Π’ΠΎΠ»ΡŒΡ‚ Ρ€Π°Π²Π΅Π½ дТоуля Π½Π° ΠΊΡƒΠ»ΠΎΠ½. Π’Π°ΠΊΠΈΠΌ ΠΎΠ±Ρ€Π°Π·ΠΎΠΌ, Ссли Π°Π²Ρ‚ΠΎΠΌΠΎΠ±ΠΈΠ»ΡŒΠ½Ρ‹ΠΉ аккумулятор ΠΈΠΌΠ΅Π΅Ρ‚ элСктричСский ΠΏΠΎΡ‚Π΅Π½Ρ†ΠΈΠ°Π» 12 Π²ΠΎΠ»ΡŒΡ‚, ΠΎΠ½ ΠΌΠΎΠΆΠ΅Ρ‚ ΠΎΠ±Π΅ΡΠΏΠ΅Ρ‡ΠΈΡ‚ΡŒ 12 Π΄ΠΆΠΎΡƒΠ»Π΅ΠΉ энСргии Π½Π° ΠΊΠ°ΠΆΠ΄Ρ‹ΠΉ ΠΊΡƒΠ»ΠΎΠ½ заряда, ΠΊΠΎΡ‚ΠΎΡ€Ρ‹ΠΉ ΠΎΠ½ ΠΏΠΎΠ΄Π°Π΅Ρ‚ Π½Π° стартСр. Π’ΠΎΡ‡Π½ΠΎ Ρ‚Π°ΠΊ ΠΆΠ΅, Ссли Ρ€ΠΎΠ·Π΅Ρ‚ΠΊΠ° Π² вашСм Π΄ΠΎΠΌΠ΅ ΠΈΠΌΠ΅Π΅Ρ‚ элСктричСский ΠΏΠΎΡ‚Π΅Π½Ρ†ΠΈΠ°Π» 120 Π²ΠΎΠ»ΡŒΡ‚, Ρ‚ΠΎ ΠΎΠ½Π° ΠΌΠΎΠΆΠ΅Ρ‚ ΠΎΠ±Π΅ΡΠΏΠ΅Ρ‡ΠΈΡ‚ΡŒ 120 Π΄ΠΆΠΎΡƒΠ»Π΅ΠΉ энСргии Π½Π° ΠΊΠ°ΠΆΠ΄Ρ‹ΠΉ ΠΊΡƒΠ»ΠΎΠ½ заряда, ΠΊΠΎΡ‚ΠΎΡ€Ρ‹ΠΉ доставляСтся ΠΊ устройству, ΠΏΠΎΠ΄ΠΊΠ»ΡŽΡ‡Π΅Π½Π½ΠΎΠΌΡƒ ΠΊ стСнС. (ΠŸΡ€ΠΈΠΌΠ΅Ρ‡Π°Π½ΠΈΠ΅: Π²Π΅Π»ΠΈΡ‡ΠΈΠ½Π° «элСктричСский ΠΏΠΎΡ‚Π΅Π½Ρ†ΠΈΠ°Π»Β» ΠΈΠ½ΠΎΠ³Π΄Π° называСтся нСсколькими Ρ€Π°Π·Π½Ρ‹ΠΌΠΈ ΠΈΠΌΠ΅Π½Π°ΠΌΠΈ, Π²ΠΊΠ»ΡŽΡ‡Π°Ρ напряТСниС, Ρ€Π°Π·Π½ΠΎΡΡ‚ΡŒ ΠΏΠΎΡ‚Π΅Π½Ρ†ΠΈΠ°Π»ΠΎΠ² ΠΈ ΡΠ»Π΅ΠΊΡ‚Ρ€ΠΎΠ΄Π²ΠΈΠΆΡƒΡ‰ΡƒΡŽ силу.Для ясности ΠΌΡ‹ всСгда Π±ΡƒΠ΄Π΅ΠΌ ΡΡΡ‹Π»Π°Ρ‚ΡŒΡΡ Π½Π° элСктричСский ΠΏΠΎΡ‚Π΅Π½Ρ†ΠΈΠ°Π», ΠΊΠΎΡ‚ΠΎΡ€Ρ‹ΠΉ измСряСтся Π² Π²ΠΎΠ»ΡŒΡ‚Π°Ρ…). Π­Π»Π΅ΠΊΡ‚Ρ€ΠΎΠ½Ρ‹ высокого напряТСния Π²ΠΎΠ·Π²Ρ€Π°Ρ‰Π°ΡŽΡ‚ΡΡ Π² «основноС состояниС» с большСй энСргиСй, Ρ‡Π΅ΠΌ элСктроны Π½ΠΈΠ·ΠΊΠΎΠ³ΠΎ напряТСния.

A Π’ – это сила, нСобходимая для пСрСмСщСния ΠΎΠ΄Π½ΠΎΠ³ΠΎ Π°ΠΌΠΏΠ΅Ρ€ Ρ‡Π΅Ρ€Π΅Π· ΠΏΡ€ΠΎΠ²ΠΎΠ΄Π½ΠΈΠΊ с сопротивлСниСм 1 Ом .

Π’Ρ‹ Π΄ΡƒΠΌΠ°Π΅Ρ‚Π΅: Β«ΠšΠ°ΠΆΠ΅Ρ‚ΡΡ, сущСствуСт связь ΠΌΠ΅ΠΆΠ΄Ρƒ усилитСлями, Π²ΠΎΠ»ΡŒΡ‚Π°ΠΌΠΈ ΠΈ ΠΎΠΌΠ°ΠΌΠΈΒ» & acirc; & # 128; & # 148; ΠΈ Ρ‚Ρ‹ ΠΏΡ€Π°Π²! ЭлСктричСский ΠΏΠΎΡ‚Π΅Π½Ρ†ΠΈΠ°Π» = Ρ‚ΠΎΠΊ x сопротивлСниС.Π­Ρ‚ΠΎ Π·Π°ΠΊΠΎΠ½ Ома, ΠΊΠΎΡ‚ΠΎΡ€Ρ‹ΠΉ ΠΎΠ±Ρ‹Ρ‡Π½ΠΎ записываСтся ΠΊΠ°ΠΊ: E = I x R . E – элСктричСский ΠΏΠΎΡ‚Π΅Π½Ρ†ΠΈΠ°Π», ΠΈΠ·ΠΌΠ΅Ρ€Π΅Π½Π½Ρ‹ΠΉ Π² Π²ΠΎΠ»ΡŒΡ‚Π°Ρ…, I – Ρ‚ΠΎΠΊ, ΠΈΠ·ΠΌΠ΅Ρ€Π΅Π½Π½Ρ‹ΠΉ Π² Π°ΠΌΠΏΠ΅Ρ€Π°Ρ…, Π° R – сопротивлСниС, ΠΈΠ·ΠΌΠ΅Ρ€Π΅Π½Π½ΠΎΠ΅ Π² ΠΎΠΌΠ°Ρ….

Π­Π»Π΅ΠΊΡ‚Ρ€ΠΎΠ½Ρ‹, проходящиС Ρ‡Π΅Ρ€Π΅Π· сопротивлСниС ΠΏΡ€ΠΎΠ²ΠΎΠ»ΠΎΠΊΠΈ, ΡΠΎΠ²Π΅Ρ€ΡˆΠ°ΡŽΡ‚ Ρ€Π°Π±ΠΎΡ‚Ρƒ. Π”Π΅ΠΉΡΡ‚Π²ΠΈΡ‚Π΅Π»ΡŒΠ½ΠΎ ΠΏΠΎΠ»Π΅Π·Π½Ρ‹ Π΄Π²Π° Π²ΠΈΠ΄Π° Ρ€Π°Π±ΠΎΡ‚Ρ‹, выполняСмой Ρ‚ΠΎΠΊΠΎΠΌ. Если Π² ΠΏΡ€ΠΎΠ²ΠΎΠ΄Π΅ имССтся большоС сопротивлСниС, большая Ρ‡Π°ΡΡ‚ΡŒ Ρ€Π°Π±ΠΎΡ‚Ρ‹ Π±ΡƒΠ΄Π΅Ρ‚ Π²Ρ‹ΠΏΠΎΠ»Π½ΡΡ‚ΡŒΡΡ Π² Π²ΠΈΠ΄Π΅ Ρ‚Π΅ΠΏΠ»Π°. ΠŸΠΎΠ΄ΡƒΠΌΠ°ΠΉΡ‚Π΅ ΠΎΠ± элСктричСском тостСрС, Ρ„Π΅Π½ΠΎΠΌ ΠΈΠ»ΠΈ ΠΎΠ±ΠΎΠ³Ρ€Π΅Π²Π°Ρ‚Π΅Π»Π΅.

Π’Ρ‚ΠΎΡ€ΠΎΠΉ Π΄Π΅ΠΉΡΡ‚Π²ΠΈΡ‚Π΅Π»ΡŒΠ½ΠΎ Π²Π°ΠΆΠ½Ρ‹ΠΉ Π²ΠΈΠ΄ Ρ€Π°Π±ΠΎΡ‚Ρ‹, выполняСмой Ρ‚ΠΎΠΊΠΎΠΌ, ΠΏΡ€ΠΎΡ‚Π΅ΠΊΠ°ΡŽΡ‰ΠΈΠΌ Ρ‡Π΅Ρ€Π΅Π· ΠΏΡ€ΠΎΠ²ΠΎΠ΄, – это созданиС ΠΌΠ°Π³Π½ΠΈΡ‚Π½ΠΎΠ³ΠΎ поля.НадСюсь, Π² дСтствС Π²Ρ‹ ΠΈΠ³Ρ€Π°Π»ΠΈ с постоянными ΠΌΠ°Π³Π½ΠΈΡ‚Π°ΠΌΠΈ. Π’Ρ‹ Π·Π½Π°Π΅Ρ‚Π΅, Ρ‡Ρ‚ΠΎ Ρƒ ΠΌΠ°Π³Π½ΠΈΡ‚ΠΎΠ² Π΄Π²Π° полюса: ΠΎΠ΄ΠΈΠ½ называСтся сСвСрным, Π° Π΄Ρ€ΡƒΠ³ΠΎΠΉ – ΡŽΠΆΠ½Ρ‹ΠΌ. Π­Ρ‚ΠΎ Π½Π°Π·Π²Π°Π½ΠΈΠ΅ связано с использованиСм ΠΌΠ°Π³Π½ΠΈΡ‚ΠΎΠ² Π² компасах для опрСдСлСния направлСния. Π’Ρ‹ Π·Π½Π°Π΅Ρ‚Π΅, Ρ‡Ρ‚ΠΎ ΠΎΠ΄ΠΈΠ½Π°ΠΊΠΎΠ²Ρ‹Π΅ ΠΊΠΎΠ½Ρ†Ρ‹ ΠΌΠ°Π³Π½ΠΈΡ‚ΠΎΠ² ΠΎΡ‚Ρ‚Π°Π»ΠΊΠΈΠ²Π°ΡŽΡ‚ΡΡ Π΄Ρ€ΡƒΠ³ ΠΎΡ‚ Π΄Ρ€ΡƒΠ³Π°, Π° ΠΏΡ€ΠΎΡ‚ΠΈΠ²ΠΎΠΏΠΎΠ»ΠΎΠΆΠ½Ρ‹Π΅ ΠΊΠΎΠ½Ρ†Ρ‹ ΠΏΡ€ΠΈΡ‚ΡΠ³ΠΈΠ²Π°ΡŽΡ‚ΡΡ. Π’Π΅ΠΏΠ΅Ρ€ΡŒ, ΠΊΠΎΠ³Π΄Π° элСктричСский Ρ‚ΠΎΠΊ Ρ‚Π΅Ρ‡Π΅Ρ‚ Ρ‡Π΅Ρ€Π΅Π· ΠΏΡ€ΠΎΠ²ΠΎΠ΄, ΠΎΠ½ становится ΠΏΠΎΡ…ΠΎΠΆ Π½Π° ΠΌΠ°Π³Π½ΠΈΡ‚ Π² Ρ‚ΠΎΠΌ смыслС, Ρ‡Ρ‚ΠΎ Ρƒ Π½Π΅Π³ΠΎ Π΅ΡΡ‚ΡŒ ΠΌΠ°Π³Π½ΠΈΡ‚Π½ΠΎΠ΅ ΠΏΠΎΠ»Π΅. Однако, Π² ΠΎΡ‚Π»ΠΈΡ‡ΠΈΠ΅ ΠΎΡ‚ постоянных ΠΌΠ°Π³Π½ΠΈΡ‚ΠΎΠ², ΠΌΠ°Π³Π½ΠΈΡ‚Π½ΠΎΠ΅ ΠΏΠΎΠ»Π΅ ΠΌΠΎΠΆΠ½ΠΎ ΠΎΡ‚ΠΊΠ»ΡŽΡ‡ΠΈΡ‚ΡŒ, остановив Ρ‚ΠΎΠΊ.Π­Ρ‚ΠΎ свойство Π»Π΅ΠΆΠΈΡ‚ Π² основС Ρ€Π°Π±ΠΎΡ‚Ρ‹ элСктродвигатСлСй. Π’ΠΎΠΊ, проходящий Ρ‡Π΅Ρ€Π΅Π· ΠΎΠ±ΠΌΠΎΡ‚ΠΊΠΈ ΠΏΡ€ΠΎΠ²ΠΎΠ΄ΠΎΠ² Π² элСктродвигатСлС, Π²Ρ‹Π·Ρ‹Π²Π°Π΅Ρ‚ Π²ΠΊΠ»ΡŽΡ‡Π΅Π½ΠΈΠ΅ ΠΌΠ°Π³Π½Π΅Ρ‚ΠΈΠ·ΠΌΠ°. Π—Π°Ρ‚Π΅ΠΌ это заставляСт Π΄Π²ΠΈΠ³Π°Ρ‚Π΅Π»ΠΈ Π²Ρ€Π°Ρ‰Π°Ρ‚ΡŒΡΡ, ΠΏΡ€ΠΈΡ‚ΡΠ³ΠΈΠ²Π°ΡΡΡŒ ΠΈ Ρ‚ΠΎΠ»ΠΊΠ°ΡΡΡŒ притяТСниСм ΠΈ ΠΎΡ‚Ρ‚Π°Π»ΠΊΠΈΠ²Π°Π½ΠΈΠ΅ΠΌ элСктромагнитов.

Π Π°Π±ΠΎΡ‚Π°, ΡΠΎΠ²Π΅Ρ€ΡˆΠ°Π΅ΠΌΠ°Ρ Ρ‚ΠΎΠΊΠΎΠΌ с Ρ‚Π΅Ρ‡Π΅Π½ΠΈΠ΅ΠΌ Π²Ρ€Π΅ΠΌΠ΅Π½ΠΈ, называСтся ΠΌΠΎΡ‰Π½ΠΎΡΡ‚ΡŒΡŽ. ΠœΠΎΡ‰Π½ΠΎΡΡ‚ΡŒ измСряСтся Π² Π²Π°Ρ‚Ρ‚Π°Ρ…. Но Π²Ρ‹ это ΡƒΠΆΠ΅ Π·Π½Π°Π΅Ρ‚Π΅! Напомним, Ρ‡Ρ‚ΠΎ Π²Ρ‹ΡˆΠ΅ Π²Ρ‹ ΡƒΠ·Π½Π°Π»ΠΈ, Ρ‡Ρ‚ΠΎ ΠΎΠ±Ρ‹Ρ‡Π½Ρ‹ΠΉ Ρ‡Π΅Π»ΠΎΠ²Π΅ΠΊ Π² состоянии покоя сТигаСт 80 Π’Ρ‚.

На элСктричСство;

1 Π’Ρ‚ = 1 А x 1 Π’ΠΎΠ»ΡŒΡ‚.

Π£Ρ€Π°Π²Π½Π΅Π½ΠΈΠ΅ ΠΌΠΎΠΆΠ½ΠΎ ΠΈΠ·ΠΌΠ΅Π½ΠΈΡ‚ΡŒ для расчСта ΠΏΡ€ΠΎΠΈΠ·Π²ΠΎΠ΄ΠΈΠΌΠΎΠ³ΠΎ Ρ‚ΠΎΠΊΠ°;

1 Π°ΠΌΠΏΠ΅Ρ€ = 1 Π’Ρ‚ / 1 ΠΎΠ±. Ρ‚.

ПодвСдСм ΠΈΡ‚ΠΎΠ³ΠΈ.

АмпСр измСряСт количСство элСктричСства, ΠΏΡ€ΠΎΡ‚Π΅ΠΊΠ°ΡŽΡ‰Π΅Π³ΠΎ с Ρ‚Π΅Ρ‡Π΅Π½ΠΈΠ΅ΠΌ Π²Ρ€Π΅ΠΌΠ΅Π½ΠΈ (Ρ‚ΠΎΠΊ).

Ом ΠΈΠ·ΠΌΠ΅Ρ€ΡŒΡ‚Π΅ сопротивлСниС ΠΏΠΎΡ‚ΠΎΠΊΡƒ.

Π’ΠΎΠ»ΡŒΡ‚ измСряСт количСство энСргии, доступной для проталкивания ΠΊΠ°ΠΆΠ΄ΠΎΠ³ΠΎ заряда Π±Π»ΠΎΠΊΠ°.

Π’Π°Ρ‚Ρ‚ – это ΠΌΠ΅Ρ€Π° мощности ΠΈΠ»ΠΈ Ρ€Π°Π±ΠΎΡ‚Ρ‹, которая выполняСтся с Ρ‚Π΅Ρ‡Π΅Π½ΠΈΠ΅ΠΌ Π²Ρ€Π΅ΠΌΠ΅Π½ΠΈ.

Π’Ρ‹ Π·Π½Π°Π΅Ρ‚Π΅, Ρ‡Ρ‚ΠΎ Π·Π°ΠΊΠΎΠ½ Ома устанавливаСт связь ΠΌΠ΅ΠΆΠ΄Ρƒ E, I ΠΈ R. Но сколько Ρ€Π°Π±ΠΎΡ‚Ρ‹ ΡƒΠΆΠ΅ сдСлано? Π­Ρ‚ΠΎ выраТаСтся ΠΊΠ°ΠΊ Π‘ΠΈΠ»Π°. ΠœΠΎΡ‰Π½ΠΎΡΡ‚ΡŒ = ЭлСктричСский ΠΏΠΎΡ‚Π΅Π½Ρ†ΠΈΠ°Π» x Π’ΠΎΠΊ, ΠΈΠ»ΠΈ P = E x I. Π­Ρ‚Π° Ρ„ΠΎΡ€ΠΌΡƒΠ»Π° ΡƒΠΊΠ°Π·Ρ‹Π²Π°Π΅Ρ‚ Π½Π° Ρ‚ΠΎ, Ρ‡Ρ‚ΠΎ ΠΌΠΎΡ‰Π½ΠΎΡΡ‚ΡŒ зависит ΠΊΠ°ΠΊ ΠΎΡ‚ количСства поставляСмой элСктроэнСргии, Ρ‚Π°ΠΊ ΠΈ ΠΎΡ‚ силы, стоящСй Π·Π° Π½Π΅ΠΉ. НапримСр, нСбольшая солнСчная панСль ΠΌΠΎΠΆΠ΅Ρ‚ Π²Ρ‹Π΄Π°Π²Π°Ρ‚ΡŒ 18 Π²ΠΎΠ»ΡŒΡ‚ ΠΈ 2 Π°ΠΌΠΏΠ΅Ρ€Π°. Π•Π³ΠΎ ΠΌΠΎΡ‰Π½ΠΎΡΡ‚ΡŒ составит 18 Π²ΠΎΠ»ΡŒΡ‚ x 2 Π°ΠΌΠΏΠ΅Ρ€Π° = 36 Π²Π°Ρ‚Ρ‚. Π’Π΅ΠΏΠ΅Ρ€ΡŒ ΠΌΠΎΠΆΠ½ΠΎ ΠΏΠΎΡΡ‚Ρ€ΠΎΠΈΡ‚ΡŒ Π΅Ρ‰Π΅ ΠΎΠ΄Π½Ρƒ ΡΠΎΠ»Π½Π΅Ρ‡Π½ΡƒΡŽ панСль для Π²Ρ‹Ρ€Π°Π±ΠΎΡ‚ΠΊΠΈ 9 Π²ΠΎΠ»ΡŒΡ‚ ΠΈ 4 Π°ΠΌΠΏΠ΅Ρ€.Π•Π³ΠΎ ΠΌΠΎΡ‰Π½ΠΎΡΡ‚ΡŒ составит 9 Π²ΠΎΠ»ΡŒΡ‚ x 4 Π°ΠΌΠΏΠ΅Ρ€Π° = 36 Π²Π°Ρ‚Ρ‚. Π’Π°ΠΊ ΠΆΠ΅, ΠΊΠ°ΠΊ ΠΈ Π΄Ρ€ΡƒΠ³ΠΎΠΉ!

Π¦Π΅ΠΏΠΈ

ΠŸΡ€ΠΎΡΡ‚Π°Ρ схСма

ΠŸΡ€ΠΎΠΈΡΡ…ΠΎΠΆΠ΄Π΅Π½ΠΈΠ΅: Π‘Π΅Π½Π΄ΠΆΠ°ΠΌΠΈΠ½ ΠšΡƒΠΊΠ΅Ρ€, УнивСрситСт Π₯эмптона
ΠŸΠΎΠ²Ρ‚ΠΎΡ€Π½ΠΎΠ΅ использованиС: Π­Ρ‚ΠΎΡ‚ элСмСнт прСдлагаСтся ΠΏΠΎ Π»ΠΈΡ†Π΅Π½Π·ΠΈΠΈ Creative Commons Attribution-NonCommercial-ShareAlike http://creativecommons.org/licenses/by-nc-sa/3.0/ Π’Ρ‹ ΠΌΠΎΠΆΠ΅Ρ‚Π΅ ΠΏΠΎΠ²Ρ‚ΠΎΡ€Π½ΠΎ ΠΈΡΠΏΠΎΠ»ΡŒΠ·ΠΎΠ²Π°Ρ‚ΡŒ это элСмСнт для нСкоммСрчСских Ρ†Π΅Π»Π΅ΠΉ ΠΏΡ€ΠΈ условии, Ρ‡Ρ‚ΠΎ Π²Ρ‹ ΡƒΠΊΠ°Π·Ρ‹Π²Π°Π΅Ρ‚Π΅ авторство ΠΈ ΠΏΡ€Π΅Π΄Π»Π°Π³Π°Π΅Ρ‚Π΅ Π»ΡŽΠ±Ρ‹Π΅ ΠΏΡ€ΠΎΠΈΠ·Π²ΠΎΠ΄Π½Ρ‹Π΅ Ρ€Π°Π±ΠΎΡ‚Ρ‹ ΠΏΠΎ Π°Π½Π°Π»ΠΎΠ³ΠΈΡ‡Π½ΠΎΠΉ Π»ΠΈΡ†Π΅Π½Π·ΠΈΠΈ.

ΠŸΠ΅Ρ€Π΅ΡΠΌΠΎΡ‚Ρ€ простой схСмы

ΠŸΡ€ΠΎΠΈΡΡ…ΠΎΠΆΠ΄Π΅Π½ΠΈΠ΅: Π‘Π΅Π½Π΄ΠΆΠ°ΠΌΠΈΠ½ ΠšΡƒΠΊΠ΅Ρ€, УнивСрситСт Π₯эмптона
ΠŸΠΎΠ²Ρ‚ΠΎΡ€Π½ΠΎΠ΅ использованиС: Π­Ρ‚ΠΎΡ‚ элСмСнт прСдлагаСтся ΠΏΠΎ Π»ΠΈΡ†Π΅Π½Π·ΠΈΠΈ Creative Commons Attribution-NonCommercial-ShareAlike http://creativecommons.org/licenses/by-nc-sa/3.0/ Π’Ρ‹ ΠΌΠΎΠΆΠ΅Ρ‚Π΅ ΠΏΠΎΠ²Ρ‚ΠΎΡ€Π½ΠΎ ΠΈΡΠΏΠΎΠ»ΡŒΠ·ΠΎΠ²Π°Ρ‚ΡŒ это элСмСнт для нСкоммСрчСских Ρ†Π΅Π»Π΅ΠΉ ΠΏΡ€ΠΈ условии, Ρ‡Ρ‚ΠΎ Π²Ρ‹ ΡƒΠΊΠ°Π·Ρ‹Π²Π°Π΅Ρ‚Π΅ авторство ΠΈ ΠΏΡ€Π΅Π΄Π»Π°Π³Π°Π΅Ρ‚Π΅ Π»ΡŽΠ±Ρ‹Π΅ ΠΏΡ€ΠΎΠΈΠ·Π²ΠΎΠ΄Π½Ρ‹Π΅ Ρ€Π°Π±ΠΎΡ‚Ρ‹ ΠΏΠΎ Π°Π½Π°Π»ΠΎΠ³ΠΈΡ‡Π½ΠΎΠΉ Π»ΠΈΡ†Π΅Π½Π·ΠΈΠΈ.

ΠžΠ±ΠΎΡ€ΡƒΠ΄ΠΎΠ²Π°Π½ΠΈΠ΅, производящСС ΠΈ ΠΈΡΠΏΠΎΠ»ΡŒΠ·ΡƒΡŽΡ‰Π΅Π΅ элСктричСство, ΠΏΠΎΠ΄ΠΊΠ»ΡŽΡ‡Π΅Π½ΠΎ Π² ΡΠ»Π΅ΠΊΡ‚Ρ€ΠΈΡ‡Π΅ΡΠΊΡƒΡŽ Ρ†Π΅ΠΏΡŒ.ΠžΠ±ΠΎΡ€ΡƒΠ΄ΠΎΠ²Π°Π½ΠΈΠ΅ ΠΌΠΎΠΆΠ΅Ρ‚ Π±Ρ‹Ρ‚ΡŒ установлСно ΠΊΠ°ΠΊ ΠΏΠΎΡΠ»Π΅Π΄ΠΎΠ²Π°Ρ‚Π΅Π»ΡŒΠ½ΠΎ, Ρ‚Π°ΠΊ ΠΈ ΠΏΠ°Ρ€Π°Π»Π»Π΅Π»ΡŒΠ½ΠΎ. ΠŸΠΎΡΠΌΠΎΡ‚Ρ€ΠΈΡ‚Π΅ Π½Π° схСмы Π½ΠΈΠΆΠ΅, Ρ‡Ρ‚ΠΎΠ±Ρ‹ ΡƒΠ²ΠΈΠ΄Π΅Ρ‚ΡŒ послСдствия использования ΠΏΠΎΡΠ»Π΅Π΄ΠΎΠ²Π°Ρ‚Π΅Π»ΡŒΠ½ΠΎΠΉ ΠΈ ΠΏΠ°Ρ€Π°Π»Π»Π΅Π»ΡŒΠ½ΠΎΠΉ схСм. Для фотоэлСмСнтов (PV) каТдая ячСйка ΠΌΠΎΠΆΠ΅Ρ‚ ΠΏΡ€ΠΎΠΈΠ·Π²ΠΎΠ΄ΠΈΡ‚ΡŒ Ρ‚ΠΎΠ»ΡŒΠΊΠΎ ΠΎΠΊΠΎΠ»ΠΎ 0,6 Π²ΠΎΠ»ΡŒΡ‚. ΠŸΠΎΡΠΊΠΎΠ»ΡŒΠΊΡƒ для Π±ΠΎΠ»ΡŒΡˆΠΈΠ½ΡΡ‚Π²Π° ΠΏΡ€ΠΈΠ»ΠΎΠΆΠ΅Π½ΠΈΠΉ трСбуСтся Π±ΠΎΠ»Π΅Π΅ высокоС напряТСниС, фотоэлСмСнты Π΄ΠΎΠ»ΠΆΠ½Ρ‹ Π±Ρ‹Ρ‚ΡŒ ΠΏΠΎΠ΄ΠΊΠ»ΡŽΡ‡Π΅Π½Ρ‹ ΠΏΠΎΡΠ»Π΅Π΄ΠΎΠ²Π°Ρ‚Π΅Π»ΡŒΠ½ΠΎ для получСния ΠΆΠ΅Π»Π°Π΅ΠΌΡ‹Ρ… Ρ€Π΅Π·ΡƒΠ»ΡŒΡ‚Π°Ρ‚ΠΎΠ².

ΠŸΠΎΡΠ»Π΅Π΄ΠΎΠ²Π°Ρ‚Π΅Π»ΡŒΠ½Π°Ρ схСма

ΠŸΡ€ΠΎΠΈΡΡ…ΠΎΠΆΠ΄Π΅Π½ΠΈΠ΅: Π‘Π΅Π½Π΄ΠΆΠ°ΠΌΠΈΠ½ ΠšΡƒΠΊΠ΅Ρ€, УнивСрситСт Π₯эмптона
ΠŸΠΎΠ²Ρ‚ΠΎΡ€Π½ΠΎΠ΅ использованиС: Π­Ρ‚ΠΎΡ‚ элСмСнт прСдлагаСтся ΠΏΠΎ Π»ΠΈΡ†Π΅Π½Π·ΠΈΠΈ Creative Commons Attribution-NonCommercial-ShareAlike http: // creativecommons.org / licenses / by-nc-sa / 3.0 / Π’Ρ‹ ΠΌΠΎΠΆΠ΅Ρ‚Π΅ ΠΏΠΎΠ²Ρ‚ΠΎΡ€Π½ΠΎ ΠΈΡΠΏΠΎΠ»ΡŒΠ·ΠΎΠ²Π°Ρ‚ΡŒ этот элСмСнт Π² нСкоммСрчСских цСлях ΠΏΡ€ΠΈ условии указания авторства ΠΈ прСдлоТСния Π»ΡŽΠ±Ρ‹Ρ… ΠΏΡ€ΠΎΠΈΠ·Π²ΠΎΠ΄Π½Ρ‹Ρ… Ρ€Π°Π±ΠΎΡ‚ ΠΏΠΎ Π°Π½Π°Π»ΠΎΠ³ΠΈΡ‡Π½ΠΎΠΉ Π»ΠΈΡ†Π΅Π½Π·ΠΈΠΈ.

ΠŸΠ°Ρ€Π°Π»Π»Π΅Π»ΡŒΠ½Π°Ρ схСма

ΠŸΡ€ΠΎΠΈΡΡ…ΠΎΠΆΠ΄Π΅Π½ΠΈΠ΅: Π‘Π΅Π½Π΄ΠΆΠ°ΠΌΠΈΠ½ ΠšΡƒΠΊΠ΅Ρ€, УнивСрситСт Π₯эмптона
ΠŸΠΎΠ²Ρ‚ΠΎΡ€Π½ΠΎΠ΅ использованиС: Π­Ρ‚ΠΎΡ‚ элСмСнт прСдлагаСтся ΠΏΠΎ Π»ΠΈΡ†Π΅Π½Π·ΠΈΠΈ Creative Commons Attribution-NonCommercial-ShareAlike http://creativecommons.org/licenses/by-nc-sa/3.0/ Π’Ρ‹ ΠΌΠΎΠΆΠ΅Ρ‚Π΅ ΠΏΠΎΠ²Ρ‚ΠΎΡ€Π½ΠΎ ΠΈΡΠΏΠΎΠ»ΡŒΠ·ΠΎΠ²Π°Ρ‚ΡŒ это элСмСнт для нСкоммСрчСских Ρ†Π΅Π»Π΅ΠΉ ΠΏΡ€ΠΈ условии, Ρ‡Ρ‚ΠΎ Π²Ρ‹ ΡƒΠΊΠ°Π·Ρ‹Π²Π°Π΅Ρ‚Π΅ авторство ΠΈ ΠΏΡ€Π΅Π΄Π»Π°Π³Π°Π΅Ρ‚Π΅ Π»ΡŽΠ±Ρ‹Π΅ ΠΏΡ€ΠΎΠΈΠ·Π²ΠΎΠ΄Π½Ρ‹Π΅ Ρ€Π°Π±ΠΎΡ‚Ρ‹ ΠΏΠΎ Π°Π½Π°Π»ΠΎΠ³ΠΈΡ‡Π½ΠΎΠΉ Π»ΠΈΡ†Π΅Π½Π·ΠΈΠΈ.

Π­Π»Π΅ΠΊΡ‚Ρ€ΠΎΠ΄Π²ΠΈΠ³Π°Ρ‚Π΅Π»ΠΈ ΠΈ Π³Π΅Π½Π΅Ρ€Π°Ρ‚ΠΎΡ€Ρ‹

ΠœΠ°Π³Π½ΠΈΡ‚Π½ΠΎΠ΅ ΠΏΠΎΠ»Π΅ Π²ΠΎΠΊΡ€ΡƒΠ³ ΠΏΡ€ΠΎΠ²ΠΎΠ΄Π°, ΠΏΠΎ ΠΊΠΎΡ‚ΠΎΡ€ΠΎΠΌΡƒ Ρ‚Π΅Ρ‡Π΅Ρ‚ Ρ‚ΠΎΠΊ

ΠŸΡ€ΠΎΠΈΡΡ…ΠΎΠΆΠ΄Π΅Π½ΠΈΠ΅: Π‘Π΅Π½Π΄ΠΆΠ°ΠΌΠΈΠ½ ΠšΡƒΠΊΠ΅Ρ€, УнивСрситСт Π₯эмптона
ΠŸΠΎΠ²Ρ‚ΠΎΡ€Π½ΠΎΠ΅ использованиС: Π­Ρ‚ΠΎΡ‚ элСмСнт прСдлагаСтся ΠΏΠΎ Π»ΠΈΡ†Π΅Π½Π·ΠΈΠΈ Creative Commons Attribution-NonCommercial-ShareAlike http://creativecommons.org/licenses/by-nc-sa/3.0/ Π’Ρ‹ ΠΌΠΎΠΆΠ΅Ρ‚Π΅ ΠΏΠΎΠ²Ρ‚ΠΎΡ€Π½ΠΎ ΠΈΡΠΏΠΎΠ»ΡŒΠ·ΠΎΠ²Π°Ρ‚ΡŒ это элСмСнт для нСкоммСрчСских Ρ†Π΅Π»Π΅ΠΉ ΠΏΡ€ΠΈ условии, Ρ‡Ρ‚ΠΎ Π²Ρ‹ ΡƒΠΊΠ°Π·Ρ‹Π²Π°Π΅Ρ‚Π΅ авторство ΠΈ ΠΏΡ€Π΅Π΄Π»Π°Π³Π°Π΅Ρ‚Π΅ Π»ΡŽΠ±Ρ‹Π΅ ΠΏΡ€ΠΎΠΈΠ·Π²ΠΎΠ΄Π½Ρ‹Π΅ Ρ€Π°Π±ΠΎΡ‚Ρ‹ ΠΏΠΎ Π°Π½Π°Π»ΠΎΠ³ΠΈΡ‡Π½ΠΎΠΉ Π»ΠΈΡ†Π΅Π½Π·ΠΈΠΈ.

Напомним, Ρ‡Ρ‚ΠΎ Ρ‡Π°ΡΡ‚ΡŒ Ρ€Π°Π±ΠΎΡ‚Ρ‹, ΡΠΎΠ²Π΅Ρ€ΡˆΠ°Π΅ΠΌΠΎΠΉ элСктричСством, происходит, ΠΊΠΎΠ³Π΄Π° ΠΎΠ½ΠΎ ΠΏΡ€ΠΎΡ…ΠΎΠ΄ΠΈΡ‚ Ρ‡Π΅Ρ€Π΅Π· ΠΏΡ€ΠΎΠ²ΠΎΠ΄ для создания ΠΌΠ°Π³Π½ΠΈΡ‚Π½ΠΎΠ³ΠΎ поля.Ганс ΠšΡ€ΠΈΡΡ‚ΠΈΠ°Π½ ЭрстСд ΠΎΠ±Π½Π°Ρ€ΡƒΠΆΠΈΠ» это Π² 1820 Π³ΠΎΠ΄Ρƒ. Π“ΠΎΠ΄ΠΎΠΌ ΠΏΠΎΠ·ΠΆΠ΅ Майкл Π€Π°Ρ€Π°Π΄Π΅ΠΉ ΠΏΠΎΠΊΠ°Π·Π°Π», Ρ‡Ρ‚ΠΎ ΠΌΠ°Π³Π½ΠΈΡ‚Π½ΠΎΠ΅ ΠΏΠΎΠ»Π΅ Π²ΠΎΠΊΡ€ΡƒΠ³ ΠΏΡ€ΠΎΠ²ΠΎΠ΄Π° ΠΌΠΎΠΆΠ½ΠΎ ΠΈΡΠΏΠΎΠ»ΡŒΠ·ΠΎΠ²Π°Ρ‚ΡŒ для создания элСктромагнитов, ΠΊΠΎΡ‚ΠΎΡ€Ρ‹Π΅ ΠΌΠΎΠ³ΡƒΡ‚ Π±Ρ‹Ρ‚ΡŒ Ρ…ΠΈΡ‚Ρ€ΠΎΡƒΠΌΠ½ΠΎ скомпонованы для создания элСктродвигатСля.
Π­Π»Π΅ΠΊΡ‚Ρ€ΠΎΠΌΠ°Π³Π½ΠΈΡ‚

ΠŸΡ€ΠΎΠΈΡΡ…ΠΎΠΆΠ΄Π΅Π½ΠΈΠ΅: ΠžΡ€ΠΈΠ³ΠΈΠ½Π°Π»ΡŒΠ½ΠΎΠ΅ Ρ„ΠΎΡ‚ΠΎ Π”ΠΆΠΈΠ½Ρ‹ ΠšΠ»ΠΈΡ„Ρ„ΠΎΡ€Π΄: https://www.flickr.com/photos/cobalt_grrl/2256696466
ΠŸΠΎΠ²Ρ‚ΠΎΡ€Π½ΠΎΠ΅ использованиС: Attribution-ShareAlike 2.0 Generic (CC BY-SA 2.0) БСсплатно: ΠŸΠΎΠ΄Π΅Π»ΠΈΡ‚ΡŒΡΡ – ΠΊΠΎΠΏΠΈΡ€ΠΎΠ²Π°Π½ΠΈΠ΅ ΠΈ распространСниС ΠΌΠ°Ρ‚Π΅Ρ€ΠΈΠ°Π» Π½Π° любом носитСлС ΠΈΠ»ΠΈ Π² любом Ρ„ΠΎΡ€ΠΌΠ°Ρ‚Π΅. ΠΠ΄Π°ΠΏΡ‚ΠΈΡ€ΠΎΠ²Π°Ρ‚ΡŒ – Ρ€Π΅ΠΌΠΈΠΊΡˆΠΈΡ€ΠΎΠ²Π°Ρ‚ΡŒ, ΠΏΡ€Π΅ΠΎΠ±Ρ€Π°Π·ΠΎΠ²Ρ‹Π²Π°Ρ‚ΡŒ ΠΈ Π΄ΠΎΠΏΠΎΠ»Π½ΡΡ‚ΡŒ ΠΌΠ°Ρ‚Π΅Ρ€ΠΈΠ°Π» для Π»ΡŽΠ±Ρ‹Ρ… Ρ†Π΅Π»Π΅ΠΉ, Π΄Π°ΠΆΠ΅ Π² коммСрчСских цСлях.

ΠžΠ±Ρ€Π°Ρ‚ΠΈΡ‚Π΅ Π²Π½ΠΈΠΌΠ°Π½ΠΈΠ΅ Π½Π° ΠΈΠ·ΠΎΠ±Ρ€Π°ΠΆΠ΅Π½ΠΈΠ΅ элСктромагнита, ΠΏΠΎΠ»ΡƒΡ‡Π΅Π½Π½ΠΎΠ΅ ΠΏΡƒΡ‚Π΅ΠΌ наматывания ΠΈΠ·ΠΎΠ»ΠΈΡ€ΠΎΠ²Π°Π½Π½ΠΎΠ³ΠΎ ΠΏΡ€ΠΎΠ²ΠΎΠ΄Π° Π½Π° ΠΆΠ΅Π»Π΅Π·Π½Ρ‹ΠΉ гвоздь. Π–Π΅Π»Π΅Π·Π½Ρ‹ΠΉ гвоздь ΠΊΠΎΠ½Ρ†Π΅Π½Ρ‚Ρ€ΠΈΡ€ΡƒΠ΅Ρ‚ ΠΌΠ°Π³Π½ΠΈΡ‚Π½ΠΎΠ΅ ΠΏΠΎΠ»Π΅, создаваСмоС Ρ‚ΠΎΠΊΠΎΠΌ Π² ΠΈΠ·ΠΎΠ»ΠΈΡ€ΠΎΠ²Π°Π½Π½ΠΎΠΌ ΠΏΡ€ΠΎΠ²ΠΎΠ΄Π΅. Π˜Π·ΠΎΠ»ΡΡ†ΠΈΡ ΠΏΡ€Π΅Π΄ΠΎΡ‚Π²Ρ€Π°Ρ‰Π°Π΅Ρ‚ ΠΊΠΎΡ€ΠΎΡ‚ΠΊΠΎΠ΅ Π·Π°ΠΌΡ‹ΠΊΠ°Π½ΠΈΠ΅ Ρ†Π΅ΠΏΠΈ ΠΆΠ΅Π»Π΅Π·Π½Ρ‹ΠΌ Π³Π²ΠΎΠ·Π΄Π΅ΠΌ.

На схСмах Π½ΠΈΠΆΠ΅ ΠΏΠΎΠΊΠ°Π·Π°Π½ΠΎ, ΠΊΠ°ΠΊ Ρ€Π°Π±ΠΎΡ‚Π°Π΅Ρ‚ ΡΠ»Π΅ΠΊΡ‚Ρ€ΠΎΠ΄Π²ΠΈΠ³Π°Ρ‚Π΅Π»ΡŒ. ΠžΠ±Ρ€Π°Ρ‚ΠΈΡ‚Π΅ Π²Π½ΠΈΠΌΠ°Π½ΠΈΠ΅, Ρ‡Ρ‚ΠΎ ΠΏΡ€ΠΈ ΠΊΠ°ΠΆΠ΄ΠΎΠΌ ΠΏΠΎΠ»ΠΎΠ²ΠΈΠ½Π½ΠΎΠΌ ΠΎΠ±ΠΎΡ€ΠΎΡ‚Π΅ ΠΊΠΎΠ½Ρ‚Π°ΠΊΡ‚Ρ‹ Π² ΠΊΠΎΠΌΠΌΡƒΡ‚Π°Ρ‚ΠΎΡ€Π΅ ΠΌΠ΅Π½ΡΡŽΡ‚ Π½Π°ΠΏΡ€Π°Π²Π»Π΅Π½ΠΈΠ΅ Ρ‚ΠΎΠΊΠ°, Ρ‡Ρ‚ΠΎΠ±Ρ‹ Π΄Π²ΠΈΠ³Π°Ρ‚Π΅Π»ΡŒ вращался Π² Ρ‚ΠΎΠΌ ΠΆΠ΅ Π½Π°ΠΏΡ€Π°Π²Π»Π΅Π½ΠΈΠΈ.

ΠŸΡ€ΠΎΡΡ‚ΠΎΠΉ ΡΠ»Π΅ΠΊΡ‚Ρ€ΠΎΠ΄Π²ΠΈΠ³Π°Ρ‚Π΅Π»ΡŒ

ΠŸΡ€ΠΎΠΈΡΡ…ΠΎΠΆΠ΄Π΅Π½ΠΈΠ΅: Π˜Π·ΠΎΠ±Ρ€Π°ΠΆΠ΅Π½ΠΈΡ созданы ΠΈΠ»ΠΈ прСдоставлСны для объяснСния этого ΠΌΠ°Ρ‚Π΅Ρ€ΠΈΠ°Π»Π°.com Π·Π°Ρ‰ΠΈΡ‰Π΅Π½Ρ‹ авторским ΠΏΡ€Π°Π²ΠΎΠΌ Β© Chris Woodford (ΠžΠ±ΡŠΡΡΠ½ΠΈΡ‚Π΅, Ρ‡Ρ‚ΠΎ stuff.com) ΠΈ ΠΎΠΏΡƒΠ±Π»ΠΈΠΊΠΎΠ²Π°Π½Ρ‹ ΠΏΠΎΠ΄ этой Π»ΠΈΡ†Π΅Π½Π·ΠΈΠ΅ΠΉ Creative Commons. http://www.explainthatstuff.com/electricmotors.html
ΠŸΠΎΠ²Ρ‚ΠΎΡ€Π½ΠΎΠ΅ использованиС: Per Creative Commons License: БовмСстноС использованиС – ΠΊΠΎΠΏΠΈΡ€ΠΎΠ²Π°Π½ΠΈΠ΅ ΠΈ распространСниС ΠΌΠ°Ρ‚Π΅Ρ€ΠΈΠ°Π»Π° Π½Π° любом носитСлС ΠΈΠ»ΠΈ Π² любом Ρ„ΠΎΡ€ΠΌΠ°Ρ‚Π΅. Адаптация – рСмикс, ΠΏΡ€Π΅ΠΎΠ±Ρ€Π°Π·ΠΎΠ²Π°Π½ΠΈΠ΅ ΠΈ созданиС ΠΌΠ°Ρ‚Π΅Ρ€ΠΈΠ°Π»Π°

. ΠŸΡ€ΠΎΡΡ‚ΠΎΠΉ ΡΠ»Π΅ΠΊΡ‚Ρ€ΠΎΠ΄Π²ΠΈΠ³Π°Ρ‚Π΅Π»ΡŒ

ΠŸΡ€ΠΎΠΈΡΡ…ΠΎΠΆΠ΄Π΅Π½ΠΈΠ΅: Π‘ΠΎΠ·Π΄Π°Π½ΠΎ АвинашСм Π‘ΠΈΠ½Ρ…Π° Π² Π²ΠΈΠ΄Π΅ ΠΎΡ€ΠΈΠ³ΠΈΠ½Π°Π»ΡŒΠ½ΠΎΠ³ΠΎ DIY-Ρ„Π°ΠΉΠ»Π° ΠΏΠΎ Π»ΠΈΡ†Π΅Π½Π·ΠΈΠΈ Creative Commons Π½Π° ΡΠ»Π΅Π΄ΡƒΡŽΡ‰Π΅ΠΌ Π²Π΅Π±-сайтС: http: // www.Instructables.com/file/FW079IPGGC2UDG3/
ΠŸΠΎΠ²Ρ‚ΠΎΡ€Π½ΠΎΠ΅ использованиС: ΠŸΡ€ΠΈ Π»ΠΈΡ†Π΅Π½Π·ΠΈΡ€ΠΎΠ²Π°Π½ΠΈΠΈ CC Ρ€Π°Π·Ρ€Π΅ΡˆΠ΅Π½ΠΎ ΡΠ»Π΅Π΄ΡƒΡŽΡ‰Π΅Π΅: БовмСстноС использованиС – ΠΊΠΎΠΏΠΈΡ€ΠΎΠ²Π°Π½ΠΈΠ΅ ΠΈ распространСниС ΠΌΠ°Ρ‚Π΅Ρ€ΠΈΠ°Π»Π° Π½Π° любом носитСлС ΠΈΠ»ΠΈ Π² любом Ρ„ΠΎΡ€ΠΌΠ°Ρ‚Π΅. Адаптация – рСмикс, ΠΏΡ€Π΅ΠΎΠ±Ρ€Π°Π·ΠΎΠ²Π°Π½ΠΈΠ΅ ΠΈ созданиС ΠΌΠ°Ρ‚Π΅Ρ€ΠΈΠ°Π»Π°

.
Π“Π΅Π½Π΅Ρ€Π°Ρ‚ΠΎΡ€ постоянного Ρ‚ΠΎΠΊΠ°

ΠŸΡ€ΠΎΠΈΡΡ…ΠΎΠΆΠ΄Π΅Π½ΠΈΠ΅: Π˜Π·ΠΎΠ±Ρ€Π°ΠΆΠ΅Π½ΠΈΠ΅ с сайта www.alternative-energy-tutorials.com, ΠΈΡΠΏΠΎΠ»ΡŒΠ·ΡƒΠ΅Ρ‚ΡΡ с Ρ€Π°Π·Ρ€Π΅ΡˆΠ΅Π½ΠΈΡ
ΠŸΠΎΠ²Ρ‚ΠΎΡ€Π½ΠΎΠ΅ использованиС: ВсС ΡƒΡ‡Π΅Π±Π½Ρ‹Π΅ пособия ΠΈ ΠΌΠ°Ρ‚Π΅Ρ€ΠΈΠ°Π»Ρ‹, ΠΎΠΏΡƒΠ±Π»ΠΈΠΊΠΎΠ²Π°Π½Π½Ρ‹Π΅ ΠΈ прСдставлСнныС Π½Π° Π²Π΅Π±-сайтС ΡƒΡ‡Π΅Π±Π½Ρ‹Ρ… пособий ΠΏΠΎ Π°Π»ΡŒΡ‚Π΅Ρ€Π½Π°Ρ‚ΠΈΠ²Π½Ρ‹ΠΌ источникам энСргии, Π²ΠΊΠ»ΡŽΡ‡Π°Ρ тСкст, Π³Ρ€Π°Ρ„ΠΈΠΊΡƒ ΠΈ изобраТСния, ΡΠ²Π»ΡΡŽΡ‚ΡΡ ΡΠΎΠ±ΡΡ‚Π²Π΅Π½Π½ΠΎΡΡ‚ΡŒΡŽ авторских ΠΏΡ€Π°Π² ΠΈΠ»ΠΈ Π°Π½Π°Π»ΠΎΠ³ΠΈΡ‡Π½Ρ‹Ρ… ΠΏΡ€Π°Π²Π° Π£Ρ‡Π΅Π±Π½ΠΈΠΊΠΎΠ² ΠΏΠΎ Π°Π»ΡŒΡ‚Π΅Ρ€Π½Π°Ρ‚ΠΈΠ²Π½ΠΎΠΉ энСргии, ΠΏΡ€Π΅Π΄ΡΡ‚Π°Π²Π»ΡΡŽΡ‰ΠΈΡ… www.Alternative-energy-tutorials.com, Ссли прямо Π½Π΅ ΡƒΠΊΠ°Π·Π°Π½ΠΎ ΠΈΠ½ΠΎΠ΅. Богласно Π²Π΅Π±-мастСру AET: Как Π²Ρ‹ любСзно спросили, я Π½Π΅ Π²ΠΎΠ·Ρ€Π°ΠΆΠ°ΡŽ ΠΏΡ€ΠΎΡ‚ΠΈΠ² Ρ‚ΠΎΠ³ΠΎ, Ρ‡Ρ‚ΠΎΠ±Ρ‹ Π²Ρ‹ использовали это ΠΈΠ·ΠΎΠ±Ρ€Π°ΠΆΠ΅Π½ΠΈΠ΅ ΠΊΠ°ΠΊ Ρ‡Π°ΡΡ‚ΡŒ своСго Π²Π΅Π±-курса ΠΏΠΎ энСргСтикС бСсплатно. Однако я Π΄ΠΎΠ»ΠΆΠ΅Π½ ΠΏΠΎΠΏΡ€ΠΎΡΠΈΡ‚ΡŒ вас ΠΏΡ€Π°Π²ΠΈΠ»ΡŒΠ½ΠΎ ΡΡΡ‹Π»Π°Ρ‚ΡŒΡΡ Π½Π° ΠΌΠΎΠΈ ΡƒΡ‡Π΅Π±Π½Ρ‹Π΅ пособия, изобраТСния ΠΈ сайт: www.alternative-energy-tutorials.com соотвСтствСнно Π² своих прСзСнтациях.

Майкл Π€Π°Ρ€Π°Π΄Π΅ΠΉ Π½Π΅ ΡƒΡΠΎΠ²Π΅Ρ€ΡˆΠ΅Π½ΡΡ‚Π²ΠΎΠ²Π°Π» ΡΠ»Π΅ΠΊΡ‚Ρ€ΠΎΠ΄Π²ΠΈΠ³Π°Ρ‚Π΅Π»ΡŒ, Π½ΠΎ ΠΎΠ½ ΠΎΠ±Π½Π°Ρ€ΡƒΠΆΠΈΠ» Π²Π°ΠΆΠ½ΠΎΠ΅ свойство элСктромагнСтизма, ΠΊΠΎΡ‚ΠΎΡ€ΠΎΠ΅ ΠΏΡ€ΠΈΠ²Π΅Π»ΠΎ ΠΊ Π΄Ρ€ΡƒΠ³ΠΎΠΌΡƒ Π²Π΅Π»ΠΈΠΊΠΎΠΌΡƒ ΠΈΠ·ΠΎΠ±Ρ€Π΅Ρ‚Π΅Π½ΠΈΡŽ – элСктричСскому Π³Π΅Π½Π΅Ρ€Π°Ρ‚ΠΎΡ€Ρƒ.Π€Π°Ρ€Π°Π΄Π΅ΠΉ ΠΎΡ‚ΠΊΡ€Ρ‹Π» Π² 1831 Π³ΠΎΠ΄Ρƒ ΠΏΡ€ΠΈΠ½Ρ†ΠΈΠΏ ΠΌΠ°Π³Π½ΠΈΡ‚Π½ΠΎΠΉ ΠΈΠ½Π΄ΡƒΠΊΡ†ΠΈΠΈ. Он ΠΎΠ±Π½Π°Ρ€ΡƒΠΆΠΈΠ», Ρ‡Ρ‚ΠΎ, проводя ΠΌΠ°Π³Π½ΠΈΡ‚ ΠΏΠΎ ΠΏΡ€ΠΎΠ²ΠΎΠ΄Ρƒ, ΠΎΠ½ Π²Ρ‹Π·Ρ‹Π²Π°Π΅Ρ‚ элСктричСский Ρ‚ΠΎΠΊ Π² Π·Π°ΠΌΠΊΠ½ΡƒΡ‚ΠΎΠΉ Ρ†Π΅ΠΏΠΈ. Π­Ρ‚ΠΎ ΠΏΡ€ΠΈΠ²Π΅Π»ΠΎ ΠΊ Ρ€Π°Π·Ρ€Π°Π±ΠΎΡ‚ΠΊΠ΅ элСктричСских Π³Π΅Π½Π΅Ρ€Π°Ρ‚ΠΎΡ€ΠΎΠ². ΠŸΠ΅Ρ€Π²Ρ‹Π΅ ΡƒΡΠΏΠ΅ΡˆΠ½Ρ‹Π΅ коммСрчСскиС Ρ€Π°Π·Ρ€Π°Π±ΠΎΡ‚ΠΊΠΈ появились ΠΏΡ€ΠΈΠΌΠ΅Ρ€Π½ΠΎ Π² 1860 Π³ΠΎΠ΄Ρƒ. Π­Π»Π΅ΠΊΡ‚Ρ€ΠΎΠ³Π΅Π½Π΅Ρ€Π°Ρ‚ΠΎΡ€ – это, ΠΏΠΎ сути, ΡΠ»Π΅ΠΊΡ‚Ρ€ΠΎΠ΄Π²ΠΈΠ³Π°Ρ‚Π΅Π»ΡŒ, ΠΊΠΎΡ‚ΠΎΡ€Ρ‹ΠΉ вращаСтся ΠΏΠΎΠ΄ дСйствиСм Π½Π΅ΠΊΠΎΡ‚ΠΎΡ€ΠΎΠΉ внСшнСй силы ΠΈ Π² ΠΎΡ‚Π²Π΅Ρ‚ ΠΏΡ€ΠΎΠΈΠ·Π²ΠΎΠ΄ΠΈΡ‚ ΠΈΠ½Π΄ΡƒΡ†ΠΈΡ€ΠΎΠ²Π°Π½Π½Ρ‹ΠΉ Ρ‚ΠΎΠΊ. Π“ΠΈΠ±Ρ€ΠΈΠ΄Π½Ρ‹Π΅ элСктромобили, Ρ‚Π°ΠΊΠΈΠ΅ ΠΊΠ°ΠΊ Toyota Prius, Π΄Π΅Π»Π°ΡŽΡ‚ ΠΈΠΌΠ΅Π½Π½ΠΎ это. Π­Π»Π΅ΠΊΡ‚Ρ€ΠΎΠ΄Π²ΠΈΠ³Π°Ρ‚Π΅Π»ΡŒ питаСтся ΠΎΡ‚ аккумулятора ΠΏΡ€ΠΈ Π½Π°ΠΆΠ°Ρ‚ΠΈΠΈ ΠΏΠ΅Π΄Π°Π»ΠΈ аксСлСратора.Когда пСдаль отпускаСтся, инСрция автомобиля дСйствуСт Ρ‡Π΅Ρ€Π΅Π· Π²Ρ€Π°Ρ‰Π°ΡŽΡ‰ΠΈΠ΅ΡΡ колСса, вращая Π΄Π²ΠΈΠ³Π°Ρ‚Π΅Π»ΡŒ, заставляя Π΄Π²ΠΈΠ³Π°Ρ‚Π΅Π»ΡŒ Ρ€Π°Π±ΠΎΡ‚Π°Ρ‚ΡŒ Π² качСствС Π³Π΅Π½Π΅Ρ€Π°Ρ‚ΠΎΡ€Π°, создавая элСктричСство для подзарядки аккумулятора.

ЭлСктричСство ΠΏΠ΅Ρ€Π΅ΠΌΠ΅Π½Π½ΠΎΠ³ΠΎ ΠΈ постоянного Ρ‚ΠΎΠΊΠ°

Π“Π΅Π½Π΅Ρ€Π°Ρ‚ΠΎΡ€ ΠΏΠ΅Ρ€Π΅ΠΌΠ΅Π½Π½ΠΎΠ³ΠΎ Ρ‚ΠΎΠΊΠ°

ΠŸΡ€ΠΎΠΈΡΡ…ΠΎΠΆΠ΄Π΅Π½ΠΈΠ΅: Автор: Π€Π΅Π΄Π΅Ρ€Π°Π»ΡŒΠ½ΠΎΠ΅ ΡƒΠΏΡ€Π°Π²Π»Π΅Π½ΠΈΠ΅ граТданской Π°Π²ΠΈΠ°Ρ†ΠΈΠΈ http://www.faa.gov/regulations_policies/handbooks_manuals/aircraft/amt_handbook/media/FAA-8083-30_Ch20.pdf
ΠŸΠΎΠ²Ρ‚ΠΎΡ€Π½ΠΎΠ΅ использованиС: Π­Ρ‚ΠΎ ΠΈΠ·ΠΎΠ±Ρ€Π°ΠΆΠ΅Π½ΠΈΠ΅ ΠΈΠ»ΠΈ Ρ„Π°ΠΉΠ» ΡΠ²Π»ΡΡŽΡ‚ΡΡ Ρ€Π°Π±ΠΎΡ‚ΠΎΠΉ Π‘ΠΎΡ‚Ρ€ΡƒΠ΄Π½ΠΈΠΊ Π€Π΅Π΄Π΅Ρ€Π°Π»ΡŒΠ½ΠΎΠ³ΠΎ управлСния граТданской Π°Π²ΠΈΠ°Ρ†ΠΈΠΈ, взятый Π½Π° Ρ€Π°Π±ΠΎΡ‚Ρƒ ΠΈΠ»ΠΈ взятый Π½Π° Ρ€Π°Π±ΠΎΡ‚Ρƒ Π² Ρ€Π°ΠΌΠΊΠ°Ρ… слуТСбных обязанностСй этого Π»ΠΈΡ†Π°.Π­Ρ‚ΠΎ ΠΏΡ€ΠΎΠΈΠ·Π²Π΅Π΄Π΅Π½ΠΈΠ΅ Ρ„Π΅Π΄Π΅Ρ€Π°Π»ΡŒΠ½ΠΎΠ³ΠΎ ΠΏΡ€Π°Π²ΠΈΡ‚Π΅Π»ΡŒΡΡ‚Π²Π° БША, ΠΈΠ·ΠΎΠ±Ρ€Π°ΠΆΠ΅Π½ΠΈΠ΅ находится Π² общСствСнном достоянии Π‘ΠΎΠ΅Π΄ΠΈΠ½Π΅Π½Π½Ρ‹Ρ… Π¨Ρ‚Π°Ρ‚ΠΎΠ².

Π”ΠΎ сих ΠΏΠΎΡ€ ΠΌΡ‹ рассматривали Ρ‚ΠΎΠ»ΡŒΠΊΠΎ ΠΎΠ΄ΠΈΠ½ Π²ΠΈΠ΄ элСктричСства – постоянный Ρ‚ΠΎΠΊ (DC). Π­Ρ‚ΠΎ Ρ‚ΠΎ, Ρ‡Ρ‚ΠΎ производят Π±Π°Ρ‚Π°Ρ€Π΅ΠΈ, солнСчныС ΠΏΠ°Π½Π΅Π»ΠΈ ΠΈ Π³Π΅Π½Π΅Ρ€Π°Ρ‚ΠΎΡ€Ρ‹ постоянного Ρ‚ΠΎΠΊΠ°. Для элСктричСства постоянного Ρ‚ΠΎΠΊΠ° Ρ‚ΠΎΠΊ всСгда Ρ‚Π΅Ρ‡Π΅Ρ‚ Π² ΠΎΠ΄Π½ΠΎΠΌ ΠΈ Ρ‚ΠΎΠΌ ΠΆΠ΅ Π½Π°ΠΏΡ€Π°Π²Π»Π΅Π½ΠΈΠΈ. Π”Ρ€ΡƒΠ³ΠΎΠΉ Π²ΠΈΠ΄ элСктричСства – это ΠΏΠ΅Ρ€Π΅ΠΌΠ΅Π½Π½Ρ‹ΠΉ Ρ‚ΠΎΠΊ (AC). Как Π²ΠΈΠ΄Π½ΠΎ ΠΈΠ· названия, Ρ‚ΠΎΠΊ ΠΏΠ΅Ρ€Π΅ΠΊΠ»ΡŽΡ‡Π°Π΅Ρ‚ Π½Π°ΠΏΡ€Π°Π²Π»Π΅Π½ΠΈΠ΅ Π² ΠΏΡ€ΠΎΠ²ΠΎΠ΄Π΅ с рСгулярным Ρ†ΠΈΠΊΠ»ΠΎΠΌ. ЭлСктроэнСргия ΠΏΠ΅Ρ€Π΅ΠΌΠ΅Π½Π½ΠΎΠ³ΠΎ Ρ‚ΠΎΠΊΠ° – это Ρ‚ΠΎ, Ρ‡Ρ‚ΠΎ ΠΏΡ€ΠΈΡ…ΠΎΠ΄ΠΈΡ‚ Π² наши Π΄ΠΎΠΌΠ° Ρ‡Π΅Ρ€Π΅Π· ΡΠ»Π΅ΠΊΡ‚Ρ€ΠΎΡΠ΅Ρ‚ΡŒ.ΠŸΡ€ΠΎΠΈΠ·Π²ΠΎΠ΄ΠΈΡ‚ΡΡ Π³Π΅Π½Π΅Ρ€Π°Ρ‚ΠΎΡ€Π°ΠΌΠΈ ΠΏΠ΅Ρ€Π΅ΠΌΠ΅Π½Π½ΠΎΠ³ΠΎ Ρ‚ΠΎΠΊΠ°. Π“Π΅Π½Π΅Ρ€Π°Ρ‚ΠΎΡ€ ΠΏΠ΅Ρ€Π΅ΠΌΠ΅Π½Π½ΠΎΠ³ΠΎ Ρ‚ΠΎΠΊΠ° устроСн ΠΈΠ½Π°Ρ‡Π΅, Ρ‡Π΅ΠΌ Π³Π΅Π½Π΅Ρ€Π°Ρ‚ΠΎΡ€ постоянного Ρ‚ΠΎΠΊΠ°. ΠŸΠΎΠΌΠ½ΠΈΡ‚Π΅, Ρ‡Ρ‚ΠΎ Π² Π³Π΅Π½Π΅Ρ€Π°Ρ‚ΠΎΡ€Π΅ постоянного Ρ‚ΠΎΠΊΠ° ΠΈΠ»ΠΈ Π΄Π²ΠΈΠ³Π°Ρ‚Π΅Π»Π΅ Π΅ΡΡ‚ΡŒ ΠΊΠΎΠΌΠΌΡƒΡ‚Π°Ρ‚ΠΎΡ€ ΠΈΠ»ΠΈ Π²Ρ‹ΠΏΡ€ΡΠΌΠΈΡ‚Π΅Π»ΡŒ, ΠΊΠΎΡ‚ΠΎΡ€Ρ‹ΠΉ ΠΏΠ΅Ρ€Π΅ΠΊΠ»ΡŽΡ‡Π°Π΅Ρ‚ Π½Π°ΠΏΡ€Π°Π²Π»Π΅Π½ΠΈΠ΅ Ρ‚ΠΎΠΊΠ° Π² ΠΊΠ°Ρ‚ΡƒΡˆΠΊΠ°Ρ… якоря (Ρ‚ΠΎΠΉ части, которая вращаСтся). Π’ Π³Π΅Π½Π΅Ρ€Π°Ρ‚ΠΎΡ€Π΅ ΠΏΠ΅Ρ€Π΅ΠΌΠ΅Π½Π½ΠΎΠ³ΠΎ Ρ‚ΠΎΠΊΠ° вмСсто рСвСрсивного ΠΊΠΎΠΌΠΌΡƒΡ‚Π°Ρ‚ΠΎΡ€Π° ΠΈΡΠΏΠΎΠ»ΡŒΠ·ΡƒΡŽΡ‚ΡΡ ΠΊΠΎΠ½Ρ‚Π°ΠΊΡ‚Π½Ρ‹Π΅ ΠΊΠΎΠ»ΡŒΡ†Π°. Π’Π°ΠΊΠΈΠΌ ΠΎΠ±Ρ€Π°Π·ΠΎΠΌ, с ΠΊΠ°ΠΆΠ΄ΠΎΠΉ ΠΏΠΎΠ»ΠΎΠ²ΠΈΠ½ΠΎΠΉ ΠΎΠ±ΠΎΡ€ΠΎΡ‚Π° Π³Π΅Π½Π΅Ρ€Π°Ρ‚ΠΎΡ€Π° ΠΈΠ½Π΄ΡƒΡ†ΠΈΡ€ΠΎΠ²Π°Π½Π½Ρ‹ΠΉ Ρ‚ΠΎΠΊ мСняСт Π½Π°ΠΏΡ€Π°Π²Π»Π΅Π½ΠΈΠ΅.

Π’Ρ‹Ρ…ΠΎΠ΄Π½ΠΎΠΉ сигнал Π³Π΅Π½Π΅Ρ€Π°Ρ‚ΠΎΡ€Π° ΠΏΠ΅Ρ€Π΅ΠΌΠ΅Π½Π½ΠΎΠ³ΠΎ Ρ‚ΠΎΠΊΠ° Π³Π΅Π½Π΅Ρ€ΠΈΡ€ΡƒΠ΅Ρ‚ ΡΠΈΠ½ΡƒΡΠΎΠΈΠ΄Π°Π»ΡŒΠ½ΡƒΡŽ Π²ΠΎΠ»Π½Ρƒ ΠΏΡ€ΠΈ скачках напряТСния Π² Ρ†Π΅ΠΏΠΈ Π²Π·Π°Π΄ ΠΈ Π²ΠΏΠ΅Ρ€Π΅Π΄.РСвСрсированиС Ρ‚ΠΎΠΊΠ° происходит быстро. Π’ Π‘ΠΎΠ΅Π΄ΠΈΠ½Π΅Π½Π½Ρ‹Ρ… Π¨Ρ‚Π°Ρ‚Π°Ρ… стандарт для элСктросСти составляСт 60 Π“Ρ† (ΠΏΠ΅Ρ€Π΅ΠΊΠ»ΡŽΡ‡Π΅Π½ΠΈΠ΅ Π²ΠΏΠ΅Ρ€Π΅Π΄ ΠΈ Π½Π°Π·Π°Π΄ 60 Ρ€Π°Π· Π² сСкунду).

Π‘ΠΈΠ½ΡƒΡΠΎΠΈΠ΄Π°Π»ΡŒΠ½Π°Ρ Π²ΠΎΠ»Π½Π° ΠΎΡ‚ Π³Π΅Π½Π΅Ρ€Π°Ρ‚ΠΎΡ€Π° ΠΏΠ΅Ρ€Π΅ΠΌΠ΅Π½Π½ΠΎΠ³ΠΎ Ρ‚ΠΎΠΊΠ°

Provenance: Booyabazooka Π² английской Π’ΠΈΠΊΠΈΠΏΠ΅Π΄ΠΈΠΈ
ΠŸΠΎΠ²Ρ‚ΠΎΡ€Π½ΠΎΠ΅ использованиС: Π­Ρ‚ΠΎΡ‚ элСмСнт прСдлагаСтся ΠΏΠΎ Π»ΠΈΡ†Π΅Π½Π·ΠΈΠΈ Creative Commons Attribution-NonCommercial-ShareAlike http://creativecommons.org/licenses/by-nc-sa/3.0/ Π’Ρ‹ ΠΌΠΎΠΆΠ΅Ρ‚Π΅ ΠΏΠΎΠ²Ρ‚ΠΎΡ€Π½ΠΎ ΠΈΡΠΏΠΎΠ»ΡŒΠ·ΠΎΠ²Π°Ρ‚ΡŒ этот элСмСнт Π² нСкоммСрчСских цСлях ΠΏΡ€ΠΈ условии указания авторства ΠΈ прСдлоТСния Π»ΡŽΠ±Ρ‹Ρ… ΠΏΡ€ΠΎΠΈΠ·Π²ΠΎΠ΄Π½Ρ‹Ρ… Ρ€Π°Π±ΠΎΡ‚ ΠΏΠΎ Π°Π½Π°Π»ΠΎΠ³ΠΈΡ‡Π½ΠΎΠΉ Π»ΠΈΡ†Π΅Π½Π·ΠΈΠΈ.

На Π΄ΠΈΠ°Π³Ρ€Π°ΠΌΠΌΠ΅ справа ΠΏΠΎΠΊΠ°Π·Π°Π½Π° синусоида, создаваСмая Π³Π΅Π½Π΅Ρ€Π°Ρ‚ΠΎΡ€ΠΎΠΌ ΠΏΠ΅Ρ€Π΅ΠΌΠ΅Π½Π½ΠΎΠ³ΠΎ Ρ‚ΠΎΠΊΠ°. ΠŸΡ€ΠΈ напряТСнии Π²Ρ‹ΡˆΠ΅ 0 Π²ΠΎΠ»ΡŒΡ‚ элСктричСство Ρ‚Π΅Ρ‡Π΅Ρ‚ Π² ΠΎΠ΄Π½ΠΎΠΌ Π½Π°ΠΏΡ€Π°Π²Π»Π΅Π½ΠΈΠΈ, Π° ΠΏΡ€ΠΈ напряТСнии Π½ΠΈΠΆΠ΅ 0 Π²ΠΎΠ»ΡŒΡ‚ – Π² Π΄Ρ€ΡƒΠ³ΠΎΠΌ. Ось Y – напряТСниС, Π° ось X – врСмя.

ΠšΠΎΡ€ΠΎΡ‚ΠΊΠΎΠ΅ Π²ΠΈΠ΄Π΅ΠΎ ΠΎ Ρ€Π°Π·Π½ΠΈΡ†Π΅ ΠΌΠ΅ΠΆΠ΄Ρƒ Π³Π΅Π½Π΅Ρ€Π°Ρ‚ΠΎΡ€Π°ΠΌΠΈ ΠΈ двигатСлями постоянного ΠΈ ΠΏΠ΅Ρ€Π΅ΠΌΠ΅Π½Π½ΠΎΠ³ΠΎ Ρ‚ΠΎΠΊΠ°

ΠŸΡ€Π΅ΠΈΠΌΡƒΡ‰Π΅ΡΡ‚Π²ΠΎ использования ΠΏΠ΅Ρ€Π΅ΠΌΠ΅Π½Π½ΠΎΠ³ΠΎ Ρ‚ΠΎΠΊΠ° состоит Π² Ρ‚ΠΎΠΌ, Ρ‡Ρ‚ΠΎ ΠΌΠΎΠΆΠ½ΠΎ Π»Π΅Π³ΠΊΠΎ ΠΏΠΎΠ²Ρ‹ΡˆΠ°Ρ‚ΡŒ ΠΈΠ»ΠΈ ΠΏΠΎΠ½ΠΈΠΆΠ°Ρ‚ΡŒ напряТСниС Π² Ρ€Π°Π·Π»ΠΈΡ‡Π½Ρ‹Ρ… частях сСти систСмы доставки. Π­Ρ‚ΠΎ Π΄Π΅Π»Π°ΡŽΡ‚ трансформаторы. Врансформатор состоит ΠΈΠ· Π΄Π²ΡƒΡ… располоТСнных Π±ΠΎΠΊ ΠΎ Π±ΠΎΠΊ ΠΊΠ°Ρ‚ΡƒΡˆΠ΅ΠΊ, большой ΠΈ ΠΌΠ°Π»ΠΎΠΉ.ОбС ΠΊΠ°Ρ‚ΡƒΡˆΠΊΠΈ ΠΈΠΌΠ΅ΡŽΡ‚ ΠΎΠ±Ρ‰ΠΈΠΉ ΠΆΠ΅Π»Π΅Π·Π½Ρ‹ΠΉ сСрдСчник. ΠŸΠ΅Ρ€Π΅ΠΌΠ΅Π½Π½Ρ‹ΠΉ Ρ‚ΠΎΠΊ, проходящий Ρ‡Π΅Ρ€Π΅Π· Π½Π΅Π±ΠΎΠ»ΡŒΡˆΡƒΡŽ ΠΏΠ΅Ρ€Π²ΠΈΡ‡Π½ΡƒΡŽ ΠΊΠ°Ρ‚ΡƒΡˆΠΊΡƒ, Π·Π° счСт ΠΌΠ°Π³Π½ΠΈΡ‚Π½ΠΎΠΉ ΠΈΠ½Π΄ΡƒΠΊΡ†ΠΈΠΈ создаСт Ρ‚ΠΎΠΊ Π±ΠΎΠ»Π΅Π΅ высокого напряТСния Π² большСй Π²Ρ‚ΠΎΡ€ΠΈΡ‡Π½ΠΎΠΉ ΠΊΠ°Ρ‚ΡƒΡˆΠΊΠ΅. И ΠΎΠ±Ρ€Π°Ρ‚Π½ΠΎΠ΅ Ρ‚Π°ΠΊΠΆΠ΅ Π²Π΅Ρ€Π½ΠΎ: Ссли пСрвичная ΠΎΠ±ΠΌΠΎΡ‚ΠΊΠ° большС, вторичная ΠΎΠ±ΠΌΠΎΡ‚ΠΊΠ° мСньшСго Ρ€Π°Π·ΠΌΠ΅Ρ€Π° Π±ΡƒΠ΄Π΅Ρ‚ ΠΈΠΌΠ΅Ρ‚ΡŒ Π±ΠΎΠ»Π΅Π΅ Π½ΠΈΠ·ΠΊΠΎΠ΅ Π²Ρ‹Ρ…ΠΎΠ΄Π½ΠΎΠ΅ напряТСниС.

Врансформатор, ΠΈΡΠΏΠΎΠ»ΡŒΠ·ΡƒΠ΅ΠΌΡ‹ΠΉ для увСличСния ΠΏΠ΅Ρ€Π΅ΠΌΠ΅Π½Π½ΠΎΠ³ΠΎ напряТСния

ΠŸΡ€ΠΎΠΈΡΡ…ΠΎΠΆΠ΄Π΅Π½ΠΈΠ΅: BillC Π² англоязычной Π’ΠΈΠΊΠΈΠΏΠ΅Π΄ΠΈΠΈ
ΠŸΠΎΠ²Ρ‚ΠΎΡ€Π½ΠΎΠ΅ использованиС: Π’Ρ‹ΠΏΡƒΡ‰Π΅Π½ΠΎ ΠΏΠΎΠ΄ Π»ΠΈΡ†Π΅Π½Π·ΠΈΠ΅ΠΉ GNU Free Documentation License.

Π—Π°Ρ‡Π΅ΠΌ Π²ΠΎΠΎΠ±Ρ‰Π΅ ΡƒΠ²Π΅Π»ΠΈΡ‡ΠΈΠ²Π°Ρ‚ΡŒ ΠΈ ΡƒΠΌΠ΅Π½ΡŒΡˆΠ°Ρ‚ΡŒ напряТСниС? ΠŸΠΎΠΌΠ½ΠΈΡ‚Π΅, Ρ‡Ρ‚ΠΎ V = I x R. ΠŸΠ΅Ρ€Π΅Π΄Π°Ρ‡Π° элСктричСства Π½Π° большиС расстояния ΠΏΡ€ΠΈΠ²ΠΎΠ΄ΠΈΡ‚ ΠΊ ΠΏΠΎΡ‚Π΅Ρ€Π΅ энСргии Π½Π° Ρ‚Π΅ΠΏΠ»ΠΎ ΠΈΠ·-Π·Π° сопротивлСния ΠΏΡ€ΠΎΠ²ΠΎΠ΄ΠΎΠ². Π§Ρ‚ΠΎΠ±Ρ‹ ΠΏΡ€Π΅Π΄ΠΎΡ‚Π²Ρ€Π°Ρ‚ΠΈΡ‚ΡŒ это, напряТСниС увСличиваСтся, Ρ‡Ρ‚ΠΎ Ρ‚Ρ€Π΅Π±ΡƒΠ΅Ρ‚ мСньшСго Ρ‚ΠΎΠΊΠ° ΠΈ ΠΌΠ΅Π½ΡŒΡˆΠΈΡ… Ρ‚Π΅ΠΏΠ»ΠΎΠ²Ρ‹Ρ… ΠΏΠΎΡ‚Π΅Ρ€ΡŒ. Когда Π²Ρ‹ ΠΏΠΎΠ΄ΠΎΠΉΠ΄Π΅Ρ‚Π΅ ΠΊ Π²Π°ΡˆΠ΅ΠΌΡƒ Π΄ΠΎΠΌΡƒ, напряТСниС снова ΡƒΠΏΠ°Π΄Π΅Ρ‚. По Π²Ρ‹ΡΠΎΠΊΠΎΠ²ΠΎΠ»ΡŒΡ‚Π½Ρ‹ΠΌ линиям элСктропСрСдачи ΠΌΠΎΠΆΠ΅Ρ‚ ΠΏΠΎΠ΄Π°Π²Π°Ρ‚ΡŒΡΡ элСктроэнСргия 765 ΠΊΠ’ (Ρ‚ΠΎ Π΅ΡΡ‚ΡŒ 765 000 Π²ΠΎΠ»ΡŒΡ‚!). Π’ΠΎ, Ρ‡Ρ‚ΠΎ получаСтся ΠΎΡ‚ Ρ€ΠΎΠ·Π΅Ρ‚ΠΊΠΈ, составляСт 120 Π²ΠΎΠ»ΡŒΡ‚.

ΠŸΠ΅Ρ€Π΅ΠΊΠ»ΡŽΡ‡Π΅Π½ΠΈΠ΅ ΠΌΠ΅ΠΆΠ΄Ρƒ ΠΏΠ΅Ρ€Π΅ΠΌΠ΅Π½Π½Ρ‹ΠΌ ΠΈ постоянным Ρ‚ΠΎΠΊΠΎΠΌ

Π˜Π½Π²Π΅Ρ€Ρ‚ΠΎΡ€ для ΠΏΠ΅Ρ€Π΅ΠΊΠ»ΡŽΡ‡Π΅Π½ΠΈΡ с постоянного Π½Π° ΠΏΠ΅Ρ€Π΅ΠΌΠ΅Π½Π½Ρ‹ΠΉ Ρ‚ΠΎΠΊ

ΠŸΡ€ΠΎΠΈΡΡ…ΠΎΠΆΠ΄Π΅Π½ΠΈΠ΅: Ѐотография сдСлана Π‘.Cuker
ΠŸΠΎΠ²Ρ‚ΠΎΡ€Π½ΠΎΠ΅ использованиС: Π‘Π΅Π· копирования, ΠΌΠΎΠΆΠ½ΠΎ ΠΈΡΠΏΠΎΠ»ΡŒΠ·ΠΎΠ²Π°Ρ‚ΡŒ для Π»ΡŽΠ±Ρ‹Ρ… Ρ†Π΅Π»Π΅ΠΉ.

ΠŸΠΎΡΠΊΠΎΠ»ΡŒΠΊΡƒ ΠΌΡ‹ ΠΈΡΠΏΠΎΠ»ΡŒΠ·ΡƒΠ΅ΠΌ элСктричСство ΠΊΠ°ΠΊ ΠΏΠ΅Ρ€Π΅ΠΌΠ΅Π½Π½ΠΎΠ³ΠΎ, Ρ‚Π°ΠΊ ΠΈ постоянного Ρ‚ΠΎΠΊΠ°, Π²Π°ΠΆΠ½ΠΎ ΡƒΠΌΠ΅Ρ‚ΡŒ ΠΏΡ€Π΅ΠΎΠ±Ρ€Π°Π·ΠΎΠ²Ρ‹Π²Π°Ρ‚ΡŒ ΠΎΠ΄Π½ΠΎ Π² Π΄Ρ€ΡƒΠ³ΠΎΠ΅. Π­Ρ‚Ρƒ Ρ€Π°Π±ΠΎΡ‚Ρƒ выполняСт устройство, Π½Π°Π·Ρ‹Π²Π°Π΅ΠΌΠΎΠ΅ ΠΈΠ½Π²Π΅Ρ€Ρ‚ΠΎΡ€ΠΎΠΌ мощности. МногиС Π±Ρ‹Ρ‚ΠΎΠ²Ρ‹Π΅ ΠΏΡ€ΠΈΠ±ΠΎΡ€Ρ‹ Ρ€Π°Π±ΠΎΡ‚Π°ΡŽΡ‚ ΠΎΡ‚ сСти ΠΏΠ΅Ρ€Π΅ΠΌΠ΅Π½Π½ΠΎΠ³ΠΎ Ρ‚ΠΎΠΊΠ°. Π₯олодильники, ΠΊΠΎΠ½Π΄ΠΈΡ†ΠΈΠΎΠ½Π΅Ρ€Ρ‹, Π»Π°ΠΌΠΏΡ‹ накаливания ΠΈ Π»ΡŽΠΌΠΈΠ½Π΅ΡΡ†Π΅Π½Ρ‚Π½Ρ‹Π΅ Π»Π°ΠΌΠΏΡ‹, пылСсосы, Ρ„Π΅Π½Ρ‹ ΠΈ ΡΡ‚ΠΈΡ€Π°Π»ΡŒΠ½Ρ‹Π΅ ΠΌΠ°ΡˆΠΈΠ½Ρ‹ – всС Π½Π°ΠΏΡ€ΡΠΌΡƒΡŽ ΠΈΡΠΏΠΎΠ»ΡŒΠ·ΡƒΡŽΡ‚ ΠΊΠΎΠ½Π΄ΠΈΡ†ΠΈΠΎΠ½Π΅Ρ€. Π­Π»Π΅ΠΊΡ‚Ρ€ΠΎΠ½ΠΈΠΊΠ°, такая ΠΊΠ°ΠΊ ΠΊΠΎΠΌΠΏΡŒΡŽΡ‚Π΅Ρ€Ρ‹, Ρ‚Π΅Π»Π΅Π²ΠΈΠ·ΠΎΡ€Ρ‹ ΠΈ сотовыС Ρ‚Π΅Π»Π΅Ρ„ΠΎΠ½Ρ‹, Ρ‚Ρ€Π΅Π±ΡƒΠ΅Ρ‚ постоянного Ρ‚ΠΎΠΊΠ°.Π’ устройствах ΠΎΠ±Ρ‹Ρ‡Π½ΠΎ ΠΈΠ½Π²Π΅Ρ€Ρ‚ΠΎΡ€ встроСн Π² ΡˆΠ½ΡƒΡ€ питания ΠΏΠ΅Ρ€Π΅ΠΌΠ΅Π½Π½ΠΎΠ³ΠΎ Ρ‚ΠΎΠΊΠ°. По ΠΏΡ€ΠΎΠ²ΠΎΠ΄Ρƒ, ΠΈΠ΄ΡƒΡ‰Π΅ΠΌΡƒ ΠΎΡ‚ ΠΈΠ½Π²Π΅Ρ€Ρ‚ΠΎΡ€Π°, ΠΏΡ€ΠΎΡ…ΠΎΠ΄ΠΈΡ‚ постоянный Ρ‚ΠΎΠΊ, Π½Π΅ΠΎΠ±Ρ…ΠΎΠ΄ΠΈΠΌΡ‹ΠΉ устройству. Π˜Π½Π²Π΅Ρ€Ρ‚ΠΎΡ€Ρ‹

Ρ‚Π°ΠΊΠΆΠ΅ ΠΌΠΎΠ³ΡƒΡ‚ ΠΈΡΠΏΠΎΠ»ΡŒΠ·ΠΎΠ²Π°Ρ‚ΡŒΡΡ для прСобразования постоянного Ρ‚ΠΎΠΊΠ° Π² ΠΏΠ΅Ρ€Π΅ΠΌΠ΅Π½Π½Ρ‹ΠΉ. Π’Π°ΠΊΠΈΠ΅ устройства ΠΏΠΎΠ·Π²ΠΎΠ»ΡΡŽΡ‚ ΠΈΡΠΏΠΎΠ»ΡŒΠ·ΠΎΠ²Π°Ρ‚ΡŒ 12 Π’ постоянного Ρ‚ΠΎΠΊΠ° автомобиля для питания ΠΏΠΎΡ€Ρ‚Π°Ρ‚ΠΈΠ²Π½ΠΎΠ³ΠΎ ΠΊΠΎΠΌΠΏΡŒΡŽΡ‚Π΅Ρ€Π°. Π”ΠΎΠΌΠ°, ΠΊΠΎΡ‚ΠΎΡ€Ρ‹Π΅ ΠΈΡΠΏΠΎΠ»ΡŒΠ·ΡƒΡŽΡ‚ фотоэлСктричСскиС ΠΏΠ°Π½Π΅Π»ΠΈ для использования солнСчной энСргии для производства элСктроэнСргии, Ρ‚Π°ΠΊΠΆΠ΅ Π΄ΠΎΠ»ΠΆΠ½Ρ‹ ΠΏΡ€Π΅ΠΎΠ±Ρ€Π°Π·ΠΎΠ²Ρ‹Π²Π°Ρ‚ΡŒ свою Π²Ρ‹Ρ€Π°Π±ΠΎΡ‚ΠΊΡƒ Π² соотвСтствии с ΠΏΠ΅Ρ€Π΅ΠΌΠ΅Π½Π½Ρ‹ΠΌ Ρ‚ΠΎΠΊΠΎΠΌ, Ссли систСмы ΠΏΠΎΠ΄ΠΊΠ»ΡŽΡ‡Π΅Π½Ρ‹ ΠΊ элСктросСти.

Оба Ρ‚ΠΈΠΏΠ° ΠΈΠ½Π²Π΅Ρ€Ρ‚ΠΎΡ€ΠΎΠ² ΠΈΡΠΏΠΎΠ»ΡŒΠ·ΡƒΡŽΡ‚ элСктронныС схСмы для ΠΏΠ΅Ρ€Π΅Ρ…ΠΎΠ΄Π° Π½Π° элСктричСство.ВСория ΠΈΡ… дСйствия Π²Ρ‹Ρ…ΠΎΠ΄ΠΈΡ‚ Π·Π° Ρ€Π°ΠΌΠΊΠΈ этого основного устройства. Но Π²Ρ‹ Π΄ΠΎΠ»ΠΆΠ½Ρ‹ Π·Π½Π°Ρ‚ΡŒ, Ρ‡Ρ‚ΠΎ силовыС ΠΈΠ½Π²Π΅Ρ€Ρ‚ΠΎΡ€Ρ‹ ΠΏΠΎΠ΄Ρ‡ΠΈΠ½ΡΡŽΡ‚ΡΡ Π²Ρ‚ΠΎΡ€ΠΎΠΌΡƒ Π·Π°ΠΊΠΎΠ½Ρƒ Ρ‚Π΅Ρ€ΠΌΠΎΠ΄ΠΈΠ½Π°ΠΌΠΈΠΊΠΈ. Π’Π°ΠΊΠΈΠΌ ΠΎΠ±Ρ€Π°Π·ΠΎΠΌ, Π² процСссС прСобразования энСргия тСряСтся Π½Π° Ρ‚Π΅ΠΏΠ»ΠΎ. Но соврСмСнныС ΠΈΠ½Π²Π΅Ρ€Ρ‚ΠΎΡ€Ρ‹ ΠΌΠΎΠ³ΡƒΡ‚ Π΄ΠΎΡΡ‚ΠΈΠ³Π°Ρ‚ΡŒ ΠšΠŸΠ” 95%.

Показан силовой ΠΈΠ½Π²Π΅Ρ€Ρ‚ΠΎΡ€, ΠΊΠΎΡ‚ΠΎΡ€Ρ‹ΠΉ ΠΏΡ€Π΅ΠΎΠ±Ρ€Π°Π·ΡƒΠ΅Ρ‚ постоянный Ρ‚ΠΎΠΊ солнСчных ΠΏΠ°Π½Π΅Π»Π΅ΠΉ Π² ΠΏΠ΅Ρ€Π΅ΠΌΠ΅Π½Π½Ρ‹ΠΉ Ρ‚ΠΎΠΊ для фотоэлСктричСской систСмы, ΠΏΠΎΠ΄ΠΊΠ»ΡŽΡ‡Π΅Π½Π½ΠΎΠΉ ΠΊ сСти.

Π₯Ρ€Π°Π½Π΅Π½ΠΈΠ΅ ΠΈ производство элСктроэнСргии с ΠΏΠΎΠΌΠΎΡ‰ΡŒΡŽ Π±Π°Ρ‚Π°Ρ€Π΅ΠΉ

Π‘Ρ…Π΅ΠΌΠ° свинцово-кислотной Π±Π°Ρ‚Π°Ρ€Π΅ΠΈ

Provenance: Ohiostandard Π² английской Π’ΠΈΠΊΠΈΠΏΠ΅Π΄ΠΈΠΈ – ΠŸΠ΅Ρ€Π΅Π½Π΅ΡΠ΅Π½ΠΎ с en.wikipedia Π² Commons ΠΎΡ‚ Burpelson AFB с использованиСм CommonsHelper.
ΠŸΠΎΠ²Ρ‚ΠΎΡ€Π½ΠΎΠ΅ использованиС: Π Π°Π·Ρ€Π΅ΡˆΠ΅Π½ΠΎ ΠΊΠΎΠΏΠΈΡ€ΠΎΠ²Π°Ρ‚ΡŒ, Ρ€Π°ΡΠΏΡ€ΠΎΡΡ‚Ρ€Π°Π½ΡΡ‚ΡŒ ΠΈ / ΠΈΠ»ΠΈ ΠΈΠ·ΠΌΠ΅Π½ΡΡ‚ΡŒ этот Π΄ΠΎΠΊΡƒΠΌΠ΅Π½Ρ‚ Π² соотвСтствии с условиями Π»ΠΈΡ†Π΅Π½Π·ΠΈΠΈ GNU Free Documentation License вСрсии 1.2 ΠΈΠ»ΠΈ любой Π±ΠΎΠ»Π΅Π΅ ΠΏΠΎΠ·Π΄Π½Π΅ΠΉ вСрсии, ΠΎΠΏΡƒΠ±Π»ΠΈΠΊΠΎΠ²Π°Π½Π½ΠΎΠΉ Free Software Foundation; Π±Π΅Π· нСизмСняСмых Ρ€Π°Π·Π΄Π΅Π»ΠΎΠ², Π±Π΅Π· тСкстов Π½Π° Π»ΠΈΡ†Π΅Π²ΠΎΠΉ ΠΎΠ±Π»ΠΎΠΆΠΊΠ΅ ΠΈ Π±Π΅Π· тСкстов Π½Π° Π·Π°Π΄Π½Π΅ΠΉ ΠΎΠ±Π»ΠΎΠΆΠΊΠ΅. Копия Π»ΠΈΡ†Π΅Π½Π·ΠΈΠΈ Π²ΠΊΠ»ΡŽΡ‡Π΅Π½Π° Π² Ρ€Π°Π·Π΄Π΅Π» ΠΏΠΎΠ΄ Π½Π°Π·Π²Π°Π½ΠΈΠ΅ΠΌ GNU Free Documentation License.

Π‘Π°Ρ‚Π°Ρ€Π΅ΠΈ ΠΏΡ€Π΅ΠΎΠ±Ρ€Π°Π·ΡƒΡŽΡ‚ ΠΏΠΎΡ‚Π΅Π½Ρ†ΠΈΠ°Π»ΡŒΠ½ΡƒΡŽ ΡΠ½Π΅Ρ€Π³ΠΈΡŽ химичСских вСщСств Π² ΠΊΠΈΠ½Π΅Ρ‚ΠΈΡ‡Π΅ΡΠΊΡƒΡŽ ΡΠ½Π΅Ρ€Π³ΠΈΡŽ элСктричСства.Π‘Π΅Π½Π΄ΠΆΠ°ΠΌΠΈΠ½ Π€Ρ€Π°Π½ΠΊΠ»ΠΈΠ½ Π²Π²Π΅Π» Ρ‚Π΅Ρ€ΠΌΠΈΠ½ «батарСя» для описания стопки стСклянных пластин с мСталличСским ΠΏΠΎΠΊΡ€Ρ‹Ρ‚ΠΈΠ΅ΠΌ, ΠΊΠΎΡ‚ΠΎΡ€Ρ‹Π΅ ΠΎΠ½ использовал для хранСния энСргии. Но Ρ‚ΠΎ, Ρ‡Ρ‚ΠΎ Ρƒ Π½Π΅Π³ΠΎ Π±Ρ‹Π»ΠΎ, сСгодня ΠΌΡ‹ Π½Π°Π·Π²Π°Π»ΠΈ Π±Ρ‹ кондСнсаторами. Π‘Π°Ρ‚Π°Ρ€Π΅ΠΈ Ρ€Π°Π±ΠΎΡ‚Π°ΡŽΡ‚ Π·Π° счСт соСдинСния Π΄Π²ΡƒΡ… химичСских ΠΌΠ°Ρ‚Π΅Ρ€ΠΈΠ°Π»ΠΎΠ², ΠΊΠΎΡ‚ΠΎΡ€Ρ‹Π΅ ΠΈΠΌΠ΅ΡŽΡ‚ Ρ€Π°Π·Π½ΠΎΠ΅ сродство ΠΊ элСктронам. ΠœΠ°Ρ‚Π΅Ρ€ΠΈΠ°Π»Ρ‹ Π°Π½ΠΎΠ΄Π° ΠΏΡ€Π΅Π΄ΠΏΠΎΡ‡ΠΈΡ‚Π°ΡŽΡ‚ Ρ‚Π΅Ρ€ΡΡ‚ΡŒ элСктроны, Π° ΠΌΠ°Ρ‚Π΅Ρ€ΠΈΠ°Π»Ρ‹ ΠΊΠ°Ρ‚ΠΎΠ΄Π° – ΠΏΠΎΠ»ΡƒΡ‡Π°Ρ‚ΡŒ ΠΈΡ…. Π­Π»Π΅ΠΊΡ‚Ρ€ΠΎΠ΄Ρ‹ Π±Π°Ρ‚Π°Ρ€Π΅ΠΈ ΠΏΠΎΠ³Ρ€ΡƒΠΆΠ΅Π½Ρ‹ Π² раствор, содСрТащий ΠΏΠΎΠ»ΠΎΠΆΠΈΡ‚Π΅Π»ΡŒΠ½ΠΎ ΠΈ ΠΎΡ‚Ρ€ΠΈΡ†Π°Ρ‚Π΅Π»ΡŒΠ½ΠΎ заряТСнныС ΠΈΠΎΠ½Ρ‹, Π½Π°Π·Ρ‹Π²Π°Π΅ΠΌΡ‹ΠΉ элСктролитом. ΠŸΡ€ΠΈ Π²ΠΊΠ»ΡŽΡ‡Π΅Π½ΠΈΠΈ Π² Ρ†Π΅ΠΏΡŒ элСктроны Ρ‚Π΅ΠΊΡƒΡ‚ ΠΎΡ‚ Π°Π½ΠΎΠ΄Π° ΠΊ ΠΊΠ°Ρ‚ΠΎΠ΄Ρƒ.Π’ Ρ‚ΠΎ ΠΆΠ΅ врСмя ΠΎΡ‚Ρ€ΠΈΡ†Π°Ρ‚Π΅Π»ΡŒΠ½ΠΎ заряТСнныС ΠΈΠΎΠ½Ρ‹ Π² элСктролитС ΠΏΠ΅Ρ€Π΅ΠΌΠ΅Ρ‰Π°ΡŽΡ‚ΡΡ ΠΎΡ‚ ΠΊΠ°Ρ‚ΠΎΠ΄Π° ΠΊ Π°Π½ΠΎΠ΄Ρƒ для поддСрТания Π½Π΅ΠΉΡ‚Ρ€Π°Π»ΡŒΠ½ΠΎΡΡ‚ΠΈ заряда ΠΈ, Ρ‚Π°ΠΊΠΈΠΌ ΠΎΠ±Ρ€Π°Π·ΠΎΠΌ, Π·Π°ΠΌΡ‹ΠΊΠ°ΡŽΡ‚ ΡΠ»Π΅ΠΊΡ‚Ρ€ΠΈΡ‡Π΅ΡΠΊΡƒΡŽ Ρ†Π΅ΠΏΡŒ.

Π’ пСрСзаряТаСмой Π±Π°Ρ‚Π°Ρ€Π΅Π΅ Ρ€Π΅Π°ΠΊΡ†ΠΈΠΈ Π½Π° Π°Π½ΠΎΠ΄Π΅ ΠΈ ΠΊΠ°Ρ‚ΠΎΠ΄Π΅ ΠΌΠΎΠΆΠ½ΠΎ ΠΎΠ±Ρ€Π°Ρ‚ΠΈΡ‚ΡŒ Π²ΡΠΏΡΡ‚ΡŒ, ΠΈΡΠΏΠΎΠ»ΡŒΠ·ΡƒΡ ΡΠ»Π΅ΠΊΡ‚Ρ€ΠΈΡ‡Π΅ΡΠΊΡƒΡŽ ΡΠ½Π΅Ρ€Π³ΠΈΡŽ для ΠΏΠΎΠ΄Π°Ρ‡ΠΈ Ρ‚ΠΎΠΊΠ°, ΠΊΠΎΡ‚ΠΎΡ€Ρ‹ΠΉ Ρ‚ΠΎΠ»ΠΊΠ°Π΅Ρ‚ элСктроны Π² ΠΏΡ€ΠΎΡ‚ΠΈΠ²ΠΎΠΏΠΎΠ»ΠΎΠΆΠ½ΠΎΠΌ Π½Π°ΠΏΡ€Π°Π²Π»Π΅Π½ΠΈΠΈ – ΠΎΡ‚ ΠΊΠ°Ρ‚ΠΎΠ΄Π° ΠΊ Π°Π½ΠΎΠ΄Ρƒ. Π­Ρ‚ΠΎ восстанавливаСт исходноС состояниС Π΄Π²ΡƒΡ… элСктродов. Π’Π°Ρˆ ΠΏΠΎΡ€Ρ‚Π°Ρ‚ΠΈΠ²Π½Ρ‹ΠΉ ΠΊΠΎΠΌΠΏΡŒΡŽΡ‚Π΅Ρ€, ΠΌΠΎΠ±ΠΈΠ»ΡŒΠ½Ρ‹ΠΉ Ρ‚Π΅Π»Π΅Ρ„ΠΎΠ½ ΠΈ Π°Π²Ρ‚ΠΎΠΌΠΎΠ±ΠΈΠ»ΡŒΠ½Ρ‹ΠΉ аккумулятор – всС это ΠΏΡ€ΠΈΠΌΠ΅Ρ€Ρ‹ аккумуляторных Π±Π°Ρ‚Π°Ρ€Π΅ΠΉ.Π’ соврСмСнных батарСях ΠΈΡΠΏΠΎΠ»ΡŒΠ·ΡƒΡŽΡ‚ΡΡ ΠΊΠΎΠΌΠ±ΠΈΠ½Π°Ρ†ΠΈΠΈ Ρ€Π°Π·Π»ΠΈΡ‡Π½Ρ‹Ρ… Ρ‚ΠΈΠΏΠΎΠ² ΠΌΠ΅Ρ‚Π°Π»Π»ΠΎΠ² ΠΈ соСдинСний оксидов ΠΌΠ΅Ρ‚Π°Π»Π»ΠΎΠ², ΠΎΠ±Ρ€Π°Π·ΠΎΠ²Π°Π½Π½Ρ‹Π΅ ΠΈΠ· Ρ‚Π°ΠΊΠΈΡ… элСмСнтов, ΠΊΠ°ΠΊ ΡƒΠ³Π»Π΅Ρ€ΠΎΠ΄, ΠΊΠ°Π΄ΠΌΠΈΠΉ, ΠΊΠΎΠ±Π°Π»ΡŒΡ‚, Π»ΠΈΡ‚ΠΈΠΉ, ΠΌΠ°Ρ€Π³Π°Π½Π΅Ρ†, никСль, свинСц ΠΈ Ρ†ΠΈΠ½ΠΊ для ΠΏΠΎΠ²Ρ‹ΡˆΠ΅Π½ΠΈΡ ΠΏΡ€ΠΎΠΈΠ·Π²ΠΎΠ΄ΠΈΡ‚Π΅Π»ΡŒΠ½ΠΎΡΡ‚ΠΈ.

БатарСя ΠΈΠ· Π»ΠΈΠΌΠΎΠ½Π°

ΠŸΡ€ΠΎΠΈΡΡ…ΠΎΠΆΠ΄Π΅Π½ΠΈΠ΅: Π’Π΅Ρ€Π΅Π·Π° ​​Нотт ΠΈΠ· Π’ΠΈΠΊΠΈΠΌΠ΅Π΄ΠΈΠ°: https://commons.wikimedia.org/wiki/File:Lemon_battery.png
ΠŸΠΎΠ²Ρ‚ΠΎΡ€Π½ΠΎΠ΅ использованиС: Π­Ρ‚ΠΎΡ‚ Ρ„Π°ΠΉΠ» находится ΠΏΠΎΠ΄ Π»ΠΈΡ†Π΅Π½Π·ΠΈΠ΅ΠΉ Creative Commons Attribution-Share Alike 3.0 Unported.Π’Ρ‹ ΠΌΠΎΠΆΠ΅Ρ‚Π΅: Π΄Π΅Π»ΠΈΡ‚ΡŒΡΡ – ΠΊΠΎΠΏΠΈΡ€ΠΎΠ²Π°Ρ‚ΡŒ, Ρ€Π°ΡΠΏΡ€ΠΎΡΡ‚Ρ€Π°Π½ΡΡ‚ΡŒ ΠΈ ΠΏΠ΅Ρ€Π΅Π΄Π°Π²Π°Ρ‚ΡŒ ΠΏΡ€ΠΎΠΈΠ·Π²Π΅Π΄Π΅Π½ΠΈΠ΅ для рСмикса – Π°Π΄Π°ΠΏΡ‚ΠΈΡ€ΠΎΠ²Π°Ρ‚ΡŒ ΠΏΡ€ΠΎΠΈΠ·Π²Π΅Π΄Π΅Π½ΠΈΠ΅ ΠŸΡ€ΠΈ ΡΠ»Π΅Π΄ΡƒΡŽΡ‰ΠΈΡ… условиях: приписываниС – Π²Ρ‹ Π΄ΠΎΠ»ΠΆΠ½Ρ‹ ΡƒΠΊΠ°Π·Ρ‹Π²Π°Ρ‚ΡŒ ΠΏΡ€ΠΎΠΈΠ·Π²Π΅Π΄Π΅Π½ΠΈΠ΅ способом, ΡƒΠΊΠ°Π·Π°Π½Π½Ρ‹ΠΌ Π°Π²Ρ‚ΠΎΡ€ΠΎΠΌ ΠΈΠ»ΠΈ Π»ΠΈΡ†Π΅Π½Π·ΠΈΠ°Ρ€ΠΎΠΌ (Π½ΠΎ Π½ΠΈ Π² ΠΊΠΎΠ΅ΠΌ случаС, Ρ‡Ρ‚ΠΎΠ±Ρ‹ ΠΏΡ€Π΅Π΄ΠΏΠΎΠ»Π°Π³Π°Π΅Ρ‚, Ρ‡Ρ‚ΠΎ ΠΎΠ½ΠΈ ΠΎΠ΄ΠΎΠ±Ρ€ΡΡŽΡ‚ вас ΠΈΠ»ΠΈ вашС использованиС произвСдСния). совмСстно ΠΈΡΠΏΠΎΠ»ΡŒΠ·ΠΎΠ²Π°Ρ‚ΡŒ – Ссли Π²Ρ‹ измСняСтС, трансформируСтС ΠΈΠ»ΠΈ Ρ€Π°ΡΡˆΠΈΡ€ΡΠ΅Ρ‚Π΅ эту Ρ€Π°Π±ΠΎΡ‚Ρƒ, Π²Ρ‹ ΠΌΠΎΠΆΠ΅Ρ‚Π΅ Ρ€Π°ΡΠΏΡ€ΠΎΡΡ‚Ρ€Π°Π½ΡΡ‚ΡŒ ΠΏΠΎΠ»ΡƒΡ‡Π΅Π½Π½ΡƒΡŽ Ρ€Π°Π±ΠΎΡ‚Ρƒ Ρ‚ΠΎΠ»ΡŒΠΊΠΎ ΠΏΠΎ Ρ‚ΠΎΠΉ ΠΆΠ΅ ΠΈΠ»ΠΈ Π°Π½Π°Π»ΠΎΠ³ΠΈΡ‡Π½ΠΎΠΉ Π»ΠΈΡ†Π΅Π½Π·ΠΈΠΈ, Ρ‡Ρ‚ΠΎ ΠΈ эта.

ΠŸΡ€ΠΎΡΡ‚Π°Ρ батарСя, ΠΈΡΠΏΠΎΠ»ΡŒΠ·ΡƒΡŽΡ‰Π°Ρ кислотныС Ρ„Ρ€ΡƒΠΊΡ‚Ρ‹ ΠΈ Π΄Π²Π° Ρ€Π°Π·Π½Ρ‹Ρ… ΠΌΠ΅Ρ‚Π°Π»Π»Π° (Π±Ρ€ΠΎΠ½Π·Π° ΠΈ ΡΡ‚Π°Π»ΡŒΠ½Ρ‹Π΅ сплавы).

Exercises Exercises for Module 1 (Microsoft Word 2007 (.docx) 17kB Jul12 17)

1. Π‘ΠΎΠ·Π΄Π°ΠΉΡ‚Π΅ Ρ†Π΅ΠΏΡŒ, ΠΈΡΠΏΠΎΠ»ΡŒΠ·ΡƒΡ Π΄Π²Π΅ ΠΏΠΎΡΠ»Π΅Π΄ΠΎΠ²Π°Ρ‚Π΅Π»ΡŒΠ½ΠΎ соСдинСнныС Π±Π°Ρ‚Π°Ρ€Π΅ΠΈ ΠΈ Π»Π°ΠΌΠΏΠΎΡ‡ΠΊΡƒ. Π˜ΡΠΏΠΎΠ»ΡŒΠ·ΡƒΠΉΡ‚Π΅ Ρ†ΠΈΡ„Ρ€ΠΎΠ²ΠΎΠΉ ΠΌΡƒΠ»ΡŒΡ‚ΠΈΠΌΠ΅Ρ‚Ρ€ (DMM) для измСрСния элСктричСского ΠΏΠΎΡ‚Π΅Π½Ρ†ΠΈΠ°Π»Π° Π² Π²ΠΎΠ»ΡŒΡ‚Π°Ρ… ΠΌΠ΅ΠΆΠ΄Ρƒ ΠΏΠΎΠ»ΠΎΠΆΠΈΡ‚Π΅Π»ΡŒΠ½ΠΎΠΉ ΠΈ ΠΎΡ‚Ρ€ΠΈΡ†Π°Ρ‚Π΅Π»ΡŒΠ½ΠΎΠΉ ΠΊΠ»Π΅ΠΌΠΌΠ°ΠΌΠΈ Π² Ρ†Π΅ΠΏΠΈ. Π’Π΅ΠΏΠ΅Ρ€ΡŒ Π΄ΠΎΠ±Π°Π²ΡŒΡ‚Π΅ Π² Ρ†Π΅ΠΏΡŒ Π²Ρ‚ΠΎΡ€ΡƒΡŽ Π»Π°ΠΌΠΏΠΎΡ‡ΠΊΡƒ ΠΏΠΎΡΠ»Π΅Π΄ΠΎΠ²Π°Ρ‚Π΅Π»ΡŒΠ½ΠΎ с ΠΏΠ΅Ρ€Π²ΠΎΠΉ. Какова ΡΡ€ΠΊΠΎΡΡ‚ΡŒ ΠΊΠ°ΠΆΠ΄ΠΎΠΉ Π»Π°ΠΌΠΏΠΎΡ‡ΠΊΠΈ ΠΏΠΎ ΡΡ€Π°Π²Π½Π΅Π½ΠΈΡŽ с ΡΡ€ΠΊΠΎΡΡ‚ΡŒΡŽ, ΠΊΠΎΠ³Π΄Π° Π² Ρ†Π΅ΠΏΠΈ Π±Ρ‹Π»Π° Ρ‚ΠΎΠ»ΡŒΠΊΠΎ ΠΎΠ΄Π½Π° Π»Π°ΠΌΠΏΠΎΡ‡ΠΊΠ°? Π‘ ΠΏΠΎΠΌΠΎΡ‰ΡŒΡŽ Π²ΠΎΠ»ΡŒΡ‚ΠΌΠ΅Ρ‚Ρ€Π° ΠΈΠ·ΠΌΠ΅Ρ€ΡŒΡ‚Π΅ напряТСниС ΠΌΠ΅ΠΆΠ΄Ρƒ ΠΏΠΎΠ»ΠΎΠΆΠΈΡ‚Π΅Π»ΡŒΠ½ΠΎΠΉ ΠΊΠ»Π΅ΠΌΠΌΠΎΠΉ аккумулятора ΠΈ ΠΏΡ€ΠΎΠ²ΠΎΠ΄ΠΎΠΌ сразу послС ΠΏΠ΅Ρ€Π²ΠΎΠΉ Π»Π°ΠΌΠΏΠΎΡ‡ΠΊΠΈ, Π° Π·Π°Ρ‚Π΅ΠΌ сразу послС Π²Ρ‚ΠΎΡ€ΠΎΠΉ Π»Π°ΠΌΠΏΠΎΡ‡ΠΊΠΈ.Π—Π°ΠΏΠΈΡˆΠΈΡ‚Π΅ Ρ€Π΅Π·ΡƒΠ»ΡŒΡ‚Π°Ρ‚Ρ‹. Π’Π΅ΠΏΠ΅Ρ€ΡŒ создайтС Ρ†Π΅ΠΏΡŒ с двумя ΠΏΠ°Ρ€Π°Π»Π»Π΅Π»ΡŒΠ½Ρ‹ΠΌΠΈ Π»Π°ΠΌΠΏΠΎΡ‡ΠΊΠ°ΠΌΠΈ. Π—Π°ΠΏΠΈΡˆΠΈΡ‚Π΅ ΡΡ€ΠΊΠΎΡΡ‚ΡŒ ΠΈ напряТСниС Π½Π° ΠΊΠ°ΠΆΠ΄ΠΎΠΉ Π»Π°ΠΌΠΏΠΎΡ‡ΠΊΠ΅.

ΠžΠ±ΡŠΡΡΠ½ΠΈΡ‚Π΅ свои Ρ€Π΅Π·ΡƒΠ»ΡŒΡ‚Π°Ρ‚Ρ‹.

ΠŸΡ€ΠΎΡΡ‚Π°Ρ схСма с ΠΎΠ΄Π½ΠΎΠΉ Π»Π°ΠΌΠΏΠΎΡ‡ΠΊΠΎΠΉ

ЦСпь с двумя ΠΏΠΎΡΠ»Π΅Π΄ΠΎΠ²Π°Ρ‚Π΅Π»ΡŒΠ½ΠΎ Π²ΠΊΠ»ΡŽΡ‡Π΅Π½Π½Ρ‹ΠΌΠΈ Π»Π°ΠΌΠΏΠΎΡ‡ΠΊΠ°ΠΌΠΈ

ЦСпь с двумя ΠΏΠ°Ρ€Π°Π»Π»Π΅Π»ΡŒΠ½ΠΎ Π²ΠΊΠ»ΡŽΡ‡Π΅Π½Π½Ρ‹ΠΌΠΈ Π»Π°ΠΌΠΏΠ°ΠΌΠΈ

2.Π‘ΠΎΠ·Π΄Π°ΠΉΡ‚Π΅ ΠΏΡΡ‚ΡŒ ΠΌΠ°Π³Π½ΠΈΡ‚ΠΎΠ² для Π²Ρ‹Π±ΠΎΡ€Ρ‰ΠΈΠΊΠΎΠ², ΠΊΠ°ΠΆΠ΄Ρ‹ΠΉ с ΠΏΡ€ΠΎΠ²ΠΎΠ»ΠΎΠΊΠΎΠΉ Ρ€Π°Π·Π½ΠΎΠΉ Π΄Π»ΠΈΠ½Ρ‹, ΠΎΠ±Π΅Ρ€Π½ΡƒΡ‚ΠΎΠΉ Π²ΠΎΠΊΡ€ΡƒΠ³ ΠΆΠ΅Π»Π΅Π·Π½Ρ‹Ρ… Π³Π²ΠΎΠ·Π΄Π΅ΠΉ: 10 см, 20 см, 30 см, 40 см ΠΈ 50 см. Π’ ΠΊΠ°ΠΆΠ΄ΠΎΠΌ случаС Π½Π° ΠΊΠ°ΠΆΠ΄ΠΎΠΌ ΠΊΠΎΠ½Ρ†Π΅ ΠΏΡ€ΠΎΠ²ΠΎΠ΄Π° Π΄ΠΎΠ»ΠΆΠ½ΠΎ Π±Ρ‹Ρ‚ΡŒ ΠΏΠΎ 10 см, Ρ‡Ρ‚ΠΎΠ±Ρ‹ Π΅Π³ΠΎ ΠΌΠΎΠΆΠ½ΠΎ Π±Ρ‹Π»ΠΎ ΠΏΠΎΠ΄ΠΊΠ»ΡŽΡ‡ΠΈΡ‚ΡŒ ΠΊ Π±Π°Ρ‚Π°Ρ€Π΅Π΅. Π’Π°ΠΊΠΈΠΌ ΠΎΠ±Ρ€Π°Π·ΠΎΠΌ, ΠΊΠ°Ρ‚ΡƒΡˆΠΊΠ° Β«10 см» Π±ΡƒΠ΄Π΅Ρ‚ фактичСски сдСлана ΠΈΠ· ΠΏΡ€ΠΎΠ²ΠΎΠ»ΠΎΠΊΠΈ Π΄Π»ΠΈΠ½ΠΎΠΉ 30 см ΠΈ Ρ‚Π°ΠΊ Π΄Π°Π»Π΅Π΅. ΠŸΠΎΠ΄ΠΊΠ»ΡŽΡ‡ΠΈΡ‚Π΅ ΠΊΠ°ΠΆΠ΄Ρ‹ΠΉ ΠΌΠ°Π³Π½ΠΈΡ‚ ΠΊ Π±Π°Ρ‚Π°Ρ€Π΅Π΅ ΠΈ ΠΏΡ€ΠΈΠΊΡ€Π΅ΠΏΠΈΡ‚Π΅ ΠΊΠ°ΠΊ ΠΌΠΎΠΆΠ½ΠΎ большС канцСлярских скрСпок ΠΊ ΠΌΠ°Π³Π½ΠΈΡ‚Π½ΠΎΠΉ Ρ†Π΅ΠΏΠΎΡ‡ΠΊΠ΅ с ΠΊΠΎΠ½Ρ‡ΠΈΠΊΠ° ногтя. Π—Π°ΠΏΠΈΡˆΠΈΡ‚Π΅ максимальноС количСство скрСпок Π² ΠΊΠ°ΠΆΠ΄ΠΎΠΌ случаС. Π—Π°Ρ‚Π΅ΠΌ нарисуйтС Π³Ρ€Π°Ρ„ΠΈΠΊ зависимости максимального количСства ΡƒΠ΄Π΅Ρ€ΠΆΠΈΠ²Π°Π΅ΠΌΡ‹Ρ… скрСпок ΠΎΡ‚ Π΄Π»ΠΈΠ½Ρ‹ ΠΏΡ€ΠΎΠ²ΠΎΠ΄Π°, ΠΈΠ· ΠΊΠΎΡ‚ΠΎΡ€ΠΎΠ³ΠΎ сдСланы ΠΎΠ±ΠΌΠΎΡ‚ΠΊΠΈ.ΠžΠ±ΡŠΡΡΠ½ΠΈΡ‚Π΅, ΠΏΠΎΡ‡Π΅ΠΌΡƒ Π³Ρ€Π°Ρ„ΠΈΠΊ выглядит ΠΈΠΌΠ΅Π½Π½ΠΎ Ρ‚Π°ΠΊ.

3. Π‘ΠΎΠ±Π΅Ρ€ΠΈΡ‚Π΅ простой Π΄Π²ΠΈΠ³Π°Ρ‚Π΅Π»ΡŒ ΠΈΠ· прСдоставлСнного ΠΊΠΎΠΌΠΏΠ»Π΅ΠΊΡ‚Π°. ΠžΠ±ΡΠ·Π°Ρ‚Π΅Π»ΡŒΠ½ΠΎ ΠΎΠ±Ρ€Π°Ρ‚ΠΈΡ‚Π΅ Π²Π½ΠΈΠΌΠ°Π½ΠΈΠ΅ Π½Π° инструкции ΠΏΠΎ ΡƒΠ΄Π°Π»Π΅Π½ΠΈΡŽ изоляции Π½Π° ΠΏΡ€ΠΎΡ‚ΠΈΠ²ΠΎΠΏΠΎΠ»ΠΎΠΆΠ½Ρ‹Ρ… сторонах ΠΏΡ€ΠΎΠ²ΠΎΠ΄Π°, ΠΊΠΎΡ‚ΠΎΡ€Ρ‹ΠΉ ΠΊΠΎΠ½Ρ‚Π°ΠΊΡ‚ΠΈΡ€ΡƒΠ΅Ρ‚ с Π·Π°ΠΆΠΈΠΌΠ°ΠΌΠΈ аккумулятора.Как Ρ‚ΠΎΠ»ΡŒΠΊΠΎ Π²Ρ‹ заставитС свой ΠΌΠΎΡ‚ΠΎΡ€ Π²Ρ€Π°Ρ‰Π°Ρ‚ΡŒΡΡ, ΠΏΡ€ΠΎΠ²Π΅Π΄ΠΈΡ‚Π΅ ΡΠ»Π΅Π΄ΡƒΡŽΡ‰ΠΈΠ΅ экспСримСнты.

Π°. ΠžΠ±Ρ€Π°Ρ‚ΠΈΡ‚Π΅ Π²Π½ΠΈΠΌΠ°Π½ΠΈΠ΅ Π½Π° Π½Π°ΠΏΡ€Π°Π²Π»Π΅Π½ΠΈΠ΅ вращСния двигатСля. ΠœΠΎΠΆΠ΅Ρ‚Π΅ Π»ΠΈ Π²Ρ‹ Π·Π°ΡΡ‚Π°Π²ΠΈΡ‚ΡŒ Π΅Π³ΠΎ ΠΏΠΎΠΉΡ‚ΠΈ Π² ΠΎΠ±Ρ€Π°Ρ‚Π½ΠΎΠΌ Π½Π°ΠΏΡ€Π°Π²Π»Π΅Π½ΠΈΠΈ? ΠžΠ±ΡŠΡΡΠ½ΡΡ‚ΡŒ.

Π³. Π’Π΅ΠΏΠ΅Ρ€ΡŒ снимитС ΠΌΠ°Π³Π½ΠΈΡ‚ ΠΈ ΠΏΠ΅Ρ€Π΅Π²Π΅Ρ€Π½ΠΈΡ‚Π΅. Π—Π°Ρ‚Π΅ΠΌ пСрСзапуститС ΠΌΠΎΡ‚ΠΎΡ€. ΠŸΠΎΠ²ΠΎΡ€Π°Ρ‡ΠΈΠ²Π°Π΅Ρ‚ Π»ΠΈ ΠΎΠ½ Π² Ρ‚ΠΎΠΌ ΠΆΠ΅ Π½Π°ΠΏΡ€Π°Π²Π»Π΅Π½ΠΈΠΈ, Ρ‡Ρ‚ΠΎ ΠΈ Ρ€Π°Π½ΡŒΡˆΠ΅? ΠŸΠΎΡ‡Π΅ΠΌΡƒ?

Π³. Π’Π΅ΠΏΠ΅Ρ€ΡŒ ΠΏΠ΅Ρ€Π΅Π²Π΅Ρ€Π½ΠΈΡ‚Π΅ аккумулятор ΠΈ пСрСзапуститС Π΄Π²ΠΈΠ³Π°Ρ‚Π΅Π»ΡŒ. НаправлСниС вращСния ΠΎΡΡ‚Π°Π»ΠΎΡΡŒ ΠΏΡ€Π΅ΠΆΠ½ΠΈΠΌ? ΠžΠ±ΡŠΡΡΠ½ΠΈΡ‚ΡŒ, ΠΏΠΎΡ‡Π΅ΠΌΡƒ.

Π³. ΠŸΠΎΠ΄ΡƒΠΌΠ°ΠΉΡ‚Π΅ ΠΎΠ± элСктродвигатСлС ΠΊΠ°ΠΊ ΠΎ систСмС.ΠžΠΏΡ€Π΅Π΄Π΅Π»ΠΈΡ‚Π΅ источник энСргии ΠΈ ΡΡƒΠ΄ΡŒΠ±Ρƒ этой энСргии Π²ΠΎ Π²Ρ€Π°Ρ‰Π°ΡŽΡ‰Π΅ΠΉΡΡ Π΄Π²ΠΈΠ³Π°Ρ‚Π΅Π»ΡŒΠ½ΠΎΠΉ систСмС. Π’ своСм ΠΎΡ‚Π²Π΅Ρ‚Π΅ ΠΈΡΠΏΠΎΠ»ΡŒΠ·ΡƒΠΉΡ‚Π΅ ΡΠ»Π΅Π΄ΡƒΡŽΡ‰ΠΈΠ΅ Ρ‚Π΅Ρ€ΠΌΠΈΠ½Ρ‹: элСктрохимичСская энСргия, кинСтичСская энСргия (энСргия двиТСния) ΠΈ Ρ‚Π΅ΠΏΠ»ΠΎ. НарисуйтС ΡΠΎΠ·Π΄Π°Π½Π½ΡƒΡŽ Π²Π°ΠΌΠΈ схСму для запуска элСктродвигатСля. ΠΠ°Π΄Π΅Π½ΡŒΡ‚Π΅ ΡˆΠ»ΡΠΏΡƒ систСмного ΠΌΡ‹ΡˆΠ»Π΅Π½ΠΈΡ.

  • ΠžΠΏΡ€Π΅Π΄Π΅Π»ΠΈΡ‚Π΅ ΠΊΠ°ΠΆΠ΄Ρ‹ΠΉ ΠΊΠΎΠΌΠΏΠΎΠ½Π΅Π½Ρ‚ систСмы.
  • ΠžΡ‚ΡΠ»Π΅Π΄ΠΈΡ‚Π΅ ΠΏΠΎΡ‚ΠΎΠΊ энСргии Π² систСмС. ΠžΠ±ΡΠ·Π°Ρ‚Π΅Π»ΡŒΠ½ΠΎ ΠΏΠΎΠΊΠ°ΠΆΠΈΡ‚Π΅, Π³Π΄Π΅ ΠΎΠ½ ΠΏΠ΅Ρ€Π΅Ρ…ΠΎΠ΄ΠΈΡ‚ ΠΎΡ‚ элСктричСского Ρ‚ΠΎΠΊΠ° ΠΊ ΠΌΠ°Π³Π½ΠΈΡ‚Π½ΠΎΠΉ энСргии, кинСтичСской энСргии ΠΈ Ρ‚Π΅ΠΏΠ»Ρƒ.
  • Π‘Π΄Π΅Π»Π°ΠΉΡ‚Π΅ снимок Π΄ΠΈΠ°Π³Ρ€Π°ΠΌΠΌΡ‹ ΠΈ Π²ΠΊΠ»ΡŽΡ‡ΠΈΡ‚Π΅ Π΅Π³ΠΎ Π² свой ΠΎΡ‚Ρ‡Π΅Ρ‚.

ЯвляСтся Π»ΠΈ ΡΠ»Π΅ΠΊΡ‚Ρ€ΠΎΠ΄Π²ΠΈΠ³Π°Ρ‚Π΅Π»ΡŒ Π·Π°ΠΊΡ€Ρ‹Ρ‚ΠΎΠΉ систСмой (вся энСргия остаСтся Π² систСмС) ΠΈΠ»ΠΈ это открытая систСма (Π½Π΅ΠΊΠΎΡ‚ΠΎΡ€Ρ‹ΠΉ ΠΎΠ±ΠΌΠ΅Π½ энСргиСй с ΠΎΠΊΡ€ΡƒΠΆΠ°ΡŽΡ‰Π΅ΠΉ срСдой)?

4. Из кусочка цитрусовых сдСлайтС Π±Π°Ρ‚Π°Ρ€Π΅ΡŽ. ΠŸΠΎΠ»ΠΎΠΆΠΈΡ‚Π΅ ΠΌΠ΅Π΄Π½Ρ‹ΠΉ ΠΏΠ΅Π½Π½ΠΈ с ΠΎΠ΄Π½ΠΎΠΉ стороны Ρ„Ρ€ΡƒΠΊΡ‚Π° ΠΈ ΡΡ‚Π°Π»ΡŒΠ½ΡƒΡŽ скрСпку с Π΄Ρ€ΡƒΠ³ΠΎΠΉ стороны. Π˜Π·ΠΌΠ΅Ρ€ΡŒΡ‚Π΅ напряТСниС с ΠΏΠΎΠΌΠΎΡ‰ΡŒΡŽ Ρ†ΠΈΡ„Ρ€ΠΎΠ²ΠΎΠ³ΠΎ ΠΌΡƒΠ»ΡŒΡ‚ΠΈΠΌΠ΅Ρ‚Ρ€Π°. Π—Π°ΠΏΠΈΡˆΠΈΡ‚Π΅ Ρ€Π΅Π·ΡƒΠ»ΡŒΡ‚Π°Ρ‚: ______.

Π’Π΅ΠΏΠ΅Ρ€ΡŒ ΠΏΠΎΠΏΡ€ΠΎΠ±ΡƒΠΉΡ‚Π΅ ΠΈΡΠΏΠΎΠ»ΡŒΠ·ΠΎΠ²Π°Ρ‚ΡŒ Ρ„Ρ€ΡƒΠΊΡ‚ΠΎΠ²Ρ‹ΠΉ аккумулятор, Ρ‡Ρ‚ΠΎΠ±Ρ‹ Π·Π°ΠΆΠ΅Ρ‡ΡŒ ΡΠ²Π΅Ρ‚ΠΎΠ΄ΠΈΠΎΠ΄Π½ΡƒΡŽ Π»Π°ΠΌΠΏΠΎΡ‡ΠΊΡƒ.Π­Ρ‚ΠΎ Ρ€Π°Π±ΠΎΡ‚Π°Π΅Ρ‚? ΠžΠ±ΡŠΡΡΠ½ΠΈΡ‚Π΅, Ρ‡Ρ‚ΠΎ создаСт элСктричСство.

Бписок Π»ΠΈΡ‚Π΅Ρ€Π°Ρ‚ΡƒΡ€Ρ‹

Π­Π»Π΅ΠΊΡ‚Ρ€ΠΎΠΌΠ°Π³Π½ΠΈΡ‚Ρ‹ ΠΈ Π·Π°ΠΊΠΎΠ½ ЀарадСя

Π­Π»Π΅ΠΊΡ‚Ρ€ΠΎΠ΄Π²ΠΈΠ³Π°Ρ‚Π΅Π»ΡŒ ΠΈ Π³Π΅Π½Π΅Ρ€Π°Ρ‚ΠΎΡ€

Асинхронный Π΄Π²ΠΈΠ³Π°Ρ‚Π΅Π»ΡŒ ΠΏΠ΅Ρ€Π΅ΠΌΠ΅Π½Π½ΠΎΠ³ΠΎ Ρ‚ΠΎΠΊΠ°

Врансформаторы

ΠŸΡ€Π΅ΠΎΠ±Ρ€Π°Π·ΠΎΠ²Π°Ρ‚Π΅Π»ΠΈ ΠΏΠ΅Ρ€Π΅ΠΌΠ΅Π½Π½ΠΎΠ³ΠΎ / постоянного Ρ‚ΠΎΠΊΠ°

Как Ρ€Π°Π±ΠΎΡ‚Π°ΡŽΡ‚ аккумуляторы

Π―Ρ€ΠΊΠΎΡΡ‚ΡŒ Π»Π°ΠΌΠΏΡ‹

ПадСниС напряТСния (Π’)

ΠŸΠ΅Ρ€Π²Π°Ρ Π»Π°ΠΌΠΏΠΎΡ‡ΠΊΠ°

Вторая Π»Π°ΠΌΠΏΠ°

Π―Ρ€ΠΊΠΎΡΡ‚ΡŒ Π»Π°ΠΌΠΏΡ‹

ПадСниС напряТСния (Π’)

ΠŸΠ΅Ρ€Π²Π°Ρ Π»Π°ΠΌΠΏΠΎΡ‡ΠΊΠ°

Вторая Π»Π°ΠΌΠΏΠ°

Π―Ρ€ΠΊΠΎΡΡ‚ΡŒ Π»Π°ΠΌΠΏΡ‹

ПадСниС напряТСния (Π’)

Π”Π»ΠΈΠ½Π° ΠΏΡ€ΠΎΠ²ΠΎΠ΄Π° Π² Π±ΡƒΡ…Ρ‚Π΅ (см)

10

20

30

40

50

Макс. Π½Π΅Ρ‚. скрСпок

19.4 ЭлСктроэнСргСтика – Ρ„ΠΈΠ·ΠΈΠΊΠ°

Π—Π°Π΄Π°Ρ‡ΠΈ обучСния сСкции

К ΠΊΠΎΠ½Ρ†Ρƒ этого Ρ€Π°Π·Π΄Π΅Π»Π° Π²Ρ‹ смоТСтС Π΄Π΅Π»Π°Ρ‚ΡŒ ΡΠ»Π΅Π΄ΡƒΡŽΡ‰Π΅Π΅:

  • ΠžΠΏΡ€Π΅Π΄Π΅Π»ΠΈΡ‚Π΅ ΡΠ»Π΅ΠΊΡ‚Ρ€ΠΈΡ‡Π΅ΡΠΊΡƒΡŽ ΠΌΠΎΡ‰Π½ΠΎΡΡ‚ΡŒ ΠΈ ΠΎΠΏΠΈΡˆΠΈΡ‚Π΅ ΡƒΡ€Π°Π²Π½Π΅Π½ΠΈΠ΅ элСктричСской мощности
  • РасчСт элСктричСской мощности Π² цСпях рСзисторов Π² ΠΏΠΎΡΠ»Π΅Π΄ΠΎΠ²Π°Ρ‚Π΅Π»ΡŒΠ½ΠΎΠΌ, ΠΏΠ°Ρ€Π°Π»Π»Π΅Π»ΡŒΠ½ΠΎΠΌ ΠΈ слоТном располоТСнии

ΠŸΠΎΠ΄Π΄Π΅Ρ€ΠΆΠΊΠ° ΡƒΡ‡ΠΈΡ‚Π΅Π»Π΅ΠΉ

ΠŸΠΎΠ΄Π΄Π΅Ρ€ΠΆΠΊΠ° ΡƒΡ‡ΠΈΡ‚Π΅Π»Π΅ΠΉ

Π¦Π΅Π»ΠΈ обучСния Π² этом Ρ€Π°Π·Π΄Π΅Π»Π΅ ΠΏΠΎΠΌΠΎΠ³ΡƒΡ‚ вашим ΡƒΡ‡Π΅Π½ΠΈΠΊΠ°ΠΌ ΠΎΠ²Π»Π°Π΄Π΅Ρ‚ΡŒ ΡΠ»Π΅Π΄ΡƒΡŽΡ‰ΠΈΠΌΠΈ стандартами:

  • (5) НаучныС ΠΊΠΎΠ½Ρ†Π΅ΠΏΡ†ΠΈΠΈ.Π‘Ρ‚ΡƒΠ΄Π΅Π½Ρ‚ Π·Π½Π°Π΅Ρ‚ ΠΏΡ€ΠΈΡ€ΠΎΠ΄Ρƒ сил Π² физичСском ΠΌΠΈΡ€Π΅. ΠžΠΆΠΈΠ΄Π°Π΅Ρ‚ΡΡ, Ρ‡Ρ‚ΠΎ студСнт:
    • (ΠΆ) ΠΏΡ€ΠΎΠ΅ΠΊΡ‚ΠΈΡ€ΠΎΠ²Π°Ρ‚ΡŒ, ΠΊΠΎΠ½ΡΡ‚Ρ€ΡƒΠΈΡ€ΠΎΠ²Π°Ρ‚ΡŒ ΠΈ Ρ€Π°ΡΡΡ‡ΠΈΡ‚Ρ‹Π²Π°Ρ‚ΡŒ Π² Ρ‚Π΅Ρ€ΠΌΠΈΠ½Π°Ρ… сквозного Ρ‚ΠΎΠΊΠ°, разности ΠΏΠΎΡ‚Π΅Π½Ρ†ΠΈΠ°Π»ΠΎΠ², сопротивлСния ΠΈ мощности, ΠΈΡΠΏΠΎΠ»ΡŒΠ·ΡƒΠ΅ΠΌΠΎΠΉ элСмСнтами элСктричСской Ρ†Π΅ΠΏΠΈ, соСдинСнными ΠΊΠ°ΠΊ Π² ΠΏΠΎΡΠ»Π΅Π΄ΠΎΠ²Π°Ρ‚Π΅Π»ΡŒΠ½ΠΎΠΉ, Ρ‚Π°ΠΊ ΠΈ Π² ΠΏΠ°Ρ€Π°Π»Π»Π΅Π»ΡŒΠ½ΠΎΠΉ комбинациях.

ΠšΡ€ΠΎΠΌΠ΅ Ρ‚ΠΎΠ³ΠΎ, Руководство Π»Π°Π±ΠΎΡ€Π°Ρ‚ΠΎΡ€ΠΈΠΈ ΠΏΠΎ Ρ„ΠΈΠ·ΠΈΠΊΠ΅ для срСднСй ΡˆΠΊΠΎΠ»Ρ‹ рассматриваСт содСрТаниС этого Ρ€Π°Π·Π΄Π΅Π»Π° Π»Π°Π±ΠΎΡ€Π°Ρ‚ΠΎΡ€ΠΈΠΈ ΠΏΠΎΠ΄ Π½Π°Π·Π²Π°Π½ΠΈΠ΅ΠΌ Β«Π Π°Π±ΠΎΡ‚Π°, энСргия ΠΈ ΠΌΠΎΡ‰Π½ΠΎΡΡ‚ΡŒ Π² цСпях», Π° Ρ‚Π°ΠΊΠΆΠ΅ ΡΠ»Π΅Π΄ΡƒΡŽΡ‰ΠΈΠ΅ стандарты:

  • (6) НаучныС ΠΊΠΎΠ½Ρ†Π΅ΠΏΡ†ΠΈΠΈ.Учащийся Π·Π½Π°Π΅Ρ‚, Ρ‡Ρ‚ΠΎ Π² физичСской систСмС происходят измСнСния, ΠΈ примСняСт Π·Π°ΠΊΠΎΠ½Ρ‹ сохранСния энСргии ΠΈ количСства двиТСния. ΠžΠΆΠΈΠ΄Π°Π΅Ρ‚ΡΡ, Ρ‡Ρ‚ΠΎ студСнт:
    • (Π‘) Π²Ρ‹Ρ‡ΠΈΡΠ»ΠΈΡ‚ΡŒ ΠΌΠ΅Ρ…Π°Π½ΠΈΡ‡Π΅ΡΠΊΡƒΡŽ ΡΠ½Π΅Ρ€Π³ΠΈΡŽ, ΠΌΠΎΡ‰Π½ΠΎΡΡ‚ΡŒ, Π³Π΅Π½Π΅Ρ€ΠΈΡ€ΡƒΠ΅ΠΌΡƒΡŽ Π²Π½ΡƒΡ‚Ρ€ΠΈ, ΠΈΠΌΠΏΡƒΠ»ΡŒΡ ΠΈ ΠΈΠΌΠΏΡƒΠ»ΡŒΡ физичСской систСмы.

Π Π°Π·Π΄Π΅Π» ΠžΡΠ½ΠΎΠ²Π½Ρ‹Π΅ Ρ‚Π΅Ρ€ΠΌΠΈΠ½Ρ‹

Π£ ΠΌΠ½ΠΎΠ³ΠΈΡ… людСй Π²Π»Π°ΡΡ‚ΡŒ ассоциируСтся с элСктричСством. ΠšΠ°ΠΆΠ΄Ρ‹ΠΉ дСнь ΠΌΡ‹ ΠΈΡΠΏΠΎΠ»ΡŒΠ·ΡƒΠ΅ΠΌ ΡΠ»Π΅ΠΊΡ‚Ρ€ΠΎΡΠ½Π΅Ρ€Π³ΠΈΡŽ для Ρ€Π°Π±ΠΎΡ‚Ρ‹ Π½Π°ΡˆΠΈΡ… соврСмСнных ΠΏΡ€ΠΈΠ±ΠΎΡ€ΠΎΠ². Π›ΠΈΠ½ΠΈΠΈ элСктропСрСдачи – наглядныС ΠΏΡ€ΠΈΠΌΠ΅Ρ€Ρ‹ элСктроэнСргии, ΠΎΠ±Π΅ΡΠΏΠ΅Ρ‡ΠΈΠ²Π°ΡŽΡ‰Π΅ΠΉ ΠΌΠΎΡ‰Π½ΠΎΡΡ‚ΡŒ.ΠœΡ‹ Ρ‚Π°ΠΊΠΆΠ΅ ΠΈΡΠΏΠΎΠ»ΡŒΠ·ΡƒΠ΅ΠΌ ΡΠ»Π΅ΠΊΡ‚Ρ€ΠΎΡΠ½Π΅Ρ€Π³ΠΈΡŽ для запуска Π°Π²Ρ‚ΠΎΠΌΠΎΠ±ΠΈΠ»Π΅ΠΉ, Ρ€Π°Π±ΠΎΡ‚Ρ‹ ΠΊΠΎΠΌΠΏΡŒΡŽΡ‚Π΅Ρ€ΠΎΠ² ΠΈΠ»ΠΈ освСщСния Π΄ΠΎΠΌΠ°. ΠœΠΎΡ‰Π½ΠΎΡΡ‚ΡŒ – это ΡΠΊΠΎΡ€ΠΎΡΡ‚ΡŒ ΠΏΠ΅Ρ€Π΅Π΄Π°Ρ‡ΠΈ энСргии любого Ρ‚ΠΈΠΏΠ°; элСктричСская ΠΌΠΎΡ‰Π½ΠΎΡΡ‚ΡŒ – это ΡΠΊΠΎΡ€ΠΎΡΡ‚ΡŒ, с ΠΊΠΎΡ‚ΠΎΡ€ΠΎΠΉ элСктричСская энСргия пСрСдаСтся Π² Ρ†Π΅ΠΏΠΈ. Π’ этом Ρ€Π°Π·Π΄Π΅Π»Π΅ ΠΌΡ‹ ΡƒΠ·Π½Π°Π΅ΠΌ Π½Π΅ Ρ‚ΠΎΠ»ΡŒΠΊΠΎ, Ρ‡Ρ‚ΠΎ это ΠΎΠ·Π½Π°Ρ‡Π°Π΅Ρ‚, Π½ΠΎ ΠΈ ΠΊΠ°ΠΊΠΈΠ΅ Ρ„Π°ΠΊΡ‚ΠΎΡ€Ρ‹ ΠΎΠΏΡ€Π΅Π΄Π΅Π»ΡΡŽΡ‚ ΡΠ»Π΅ΠΊΡ‚Ρ€ΠΈΡ‡Π΅ΡΠΊΡƒΡŽ ΠΌΠΎΡ‰Π½ΠΎΡΡ‚ΡŒ.

Для Π½Π°Ρ‡Π°Π»Π° прСдставим сСбС Π»Π°ΠΌΠΏΠΎΡ‡ΠΊΠΈ, ΠΊΠΎΡ‚ΠΎΡ€Ρ‹Π΅ часто Ρ…Π°Ρ€Π°ΠΊΡ‚Π΅Ρ€ΠΈΠ·ΡƒΡŽΡ‚ΡΡ номинальной ΠΌΠΎΡ‰Π½ΠΎΡΡ‚ΡŒΡŽ Π² Π²Π°Ρ‚Ρ‚Π°Ρ…. Π”Π°Π²Π°ΠΉΡ‚Π΅ сравним Π»Π°ΠΌΠΏΡƒ ΠΌΠΎΡ‰Π½ΠΎΡΡ‚ΡŒΡŽ 25 Π’Ρ‚ с Π»Π°ΠΌΠΏΠΎΠΉ ΠΌΠΎΡ‰Π½ΠΎΡΡ‚ΡŒΡŽ 60 Π’Ρ‚ (см. Рисунок 19.20). Π₯отя ΠΎΠ±Π΅ Ρ€Π°Π±ΠΎΡ‚Π°ΡŽΡ‚ ΠΏΡ€ΠΈ ΠΎΠ΄ΠΈΠ½Π°ΠΊΠΎΠ²ΠΎΠΌ напряТСнии, Π»Π°ΠΌΠΏΠ° ΠΌΠΎΡ‰Π½ΠΎΡΡ‚ΡŒΡŽ 60 Π’Ρ‚ ΠΈΠ·Π»ΡƒΡ‡Π°Π΅Ρ‚ большС свСта, Ρ‡Π΅ΠΌ Π»Π°ΠΌΠΏΠ° ΠΌΠΎΡ‰Π½ΠΎΡΡ‚ΡŒΡŽ 25 Π’Ρ‚. Π­Ρ‚ΠΎ Π³ΠΎΠ²ΠΎΡ€ΠΈΡ‚ Π½Π°ΠΌ ΠΎ Ρ‚ΠΎΠΌ, Ρ‡Ρ‚ΠΎ Π²Ρ‹Ρ…ΠΎΠ΄Π½ΡƒΡŽ ΠΌΠΎΡ‰Π½ΠΎΡΡ‚ΡŒ элСктричСской Ρ†Π΅ΠΏΠΈ опрСдСляСт Π½Π΅Ρ‡Ρ‚ΠΎ ΠΈΠ½ΠΎΠ΅, Ρ‡Π΅ΠΌ напряТСниС.

Π›Π°ΠΌΠΏΡ‹ накаливания, Ρ‚Π°ΠΊΠΈΠ΅ ΠΊΠ°ΠΊ Π΄Π²Π΅, ΠΏΠΎΠΊΠ°Π·Π°Π½Π½Ρ‹Π΅ Π½Π° рисункС 19.20, ΠΏΠΎ сути ΡΠ²Π»ΡΡŽΡ‚ΡΡ рСзисторами, ΠΊΠΎΡ‚ΠΎΡ€Ρ‹Π΅ Π½Π°Π³Ρ€Π΅Π²Π°ΡŽΡ‚ΡΡ, ΠΊΠΎΠ³Π΄Π° Ρ‡Π΅Ρ€Π΅Π· Π½ΠΈΡ… ΠΏΡ€ΠΎΡ‚Π΅ΠΊΠ°Π΅Ρ‚ Ρ‚ΠΎΠΊ, ΠΈ становятся Π½Π°ΡΡ‚ΠΎΠ»ΡŒΠΊΠΎ горячими, Ρ‡Ρ‚ΠΎ ΠΈΠ·Π»ΡƒΡ‡Π°ΡŽΡ‚ Π²ΠΈΠ΄ΠΈΠΌΡ‹ΠΉ ΠΈ Π½Π΅Π²ΠΈΠ΄ΠΈΠΌΡ‹ΠΉ свСт. Π’Π°ΠΊΠΈΠΌ ΠΎΠ±Ρ€Π°Π·ΠΎΠΌ, Π΄Π²Π΅ Π»Π°ΠΌΠΏΠΎΡ‡ΠΊΠΈ Π½Π° Ρ„ΠΎΡ‚ΠΎ ΠΌΠΎΠΆΠ½ΠΎ Ρ€Π°ΡΡΠΌΠ°Ρ‚Ρ€ΠΈΠ²Π°Ρ‚ΡŒ ΠΊΠ°ΠΊ Π΄Π²Π° Ρ€Π°Π·Π½Ρ‹Ρ… рСзистора.Π’ простой Ρ†Π΅ΠΏΠΈ, Ρ‚Π°ΠΊΠΎΠΉ ΠΊΠ°ΠΊ элСктричСская Π»Π°ΠΌΠΏΠΎΡ‡ΠΊΠ° с ΠΏΡ€ΠΈΠ»ΠΎΠΆΠ΅Π½Π½Ρ‹ΠΌ ΠΊ Π½Π΅ΠΉ напряТСниСм, сопротивлСниС опрСдСляСт Ρ‚ΠΎΠΊ ΠΏΠΎ Π·Π°ΠΊΠΎΠ½Ρƒ Ома, поэтому ΠΌΡ‹ ΠΌΠΎΠΆΠ΅ΠΌ Π²ΠΈΠ΄Π΅Ρ‚ΡŒ, Ρ‡Ρ‚ΠΎ Ρ‚ΠΎΠΊ, Π° Ρ‚Π°ΠΊΠΆΠ΅ напряТСниС Π΄ΠΎΠ»ΠΆΠ½Ρ‹ ΠΎΠΏΡ€Π΅Π΄Π΅Π»ΡΡ‚ΡŒ ΠΌΠΎΡ‰Π½ΠΎΡΡ‚ΡŒ.

Рисунок 19.20 Π‘Π»Π΅Π²Π° – Π»Π°ΠΌΠΏΠΎΡ‡ΠΊΠ° ΠΌΠΎΡ‰Π½ΠΎΡΡ‚ΡŒΡŽ 25 Π’Ρ‚, Π° справа – Π»Π°ΠΌΠΏΠΎΡ‡ΠΊΠ° ΠΌΠΎΡ‰Π½ΠΎΡΡ‚ΡŒΡŽ 60 Π’Ρ‚. ΠŸΠΎΡ‡Π΅ΠΌΡƒ ΠΈΡ… выходная ΠΌΠΎΡ‰Π½ΠΎΡΡ‚ΡŒ различаСтся, нСсмотря Π½Π° Ρ‚ΠΎ, Ρ‡Ρ‚ΠΎ ΠΎΠ½ΠΈ Ρ€Π°Π±ΠΎΡ‚Π°ΡŽΡ‚ ΠΏΡ€ΠΈ ΠΎΠ΄ΠΈΠ½Π°ΠΊΠΎΠ²ΠΎΠΌ напряТСнии?

Π€ΠΎΡ€ΠΌΡƒΠ»Ρƒ мощности ΠΌΠΎΠΆΠ½ΠΎ Π½Π°ΠΉΡ‚ΠΈ ΠΏΡƒΡ‚Π΅ΠΌ Π°Π½Π°Π»ΠΈΠ·Π° Ρ€Π°Π·ΠΌΠ΅Ρ€ΠΎΠ². Рассмотрим Π΅Π΄ΠΈΠ½ΠΈΡ†Ρ‹ мощности. Π’ систСмС БИ ΠΌΠΎΡ‰Π½ΠΎΡΡ‚ΡŒ указываСтся Π² Π²Π°Ρ‚Ρ‚Π°Ρ… (Π’Ρ‚), ΠΊΠΎΡ‚ΠΎΡ€Ρ‹Π΅ ΠΏΡ€Π΅Π΄ΡΡ‚Π°Π²Π»ΡΡŽΡ‚ собой ΡΠ½Π΅Ρ€Π³ΠΈΡŽ Π² Π΅Π΄ΠΈΠ½ΠΈΡ†Ρƒ Π²Ρ€Π΅ΠΌΠ΅Π½ΠΈ, ΠΈΠ»ΠΈ Π”ΠΆ / с

.

Напомним, Ρ‡Ρ‚ΠΎ напряТСниС – это ΠΏΠΎΡ‚Π΅Π½Ρ†ΠΈΠ°Π»ΡŒΠ½Π°Ρ энСргия Π½Π° Π΅Π΄ΠΈΠ½ΠΈΡ†Ρƒ заряда, Ρ‡Ρ‚ΠΎ ΠΎΠ·Π½Π°Ρ‡Π°Π΅Ρ‚, Ρ‡Ρ‚ΠΎ напряТСниС ΠΈΠΌΠ΅Π΅Ρ‚ Π΅Π΄ΠΈΠ½ΠΈΡ†Ρ‹ Π”ΠΆ / Кл

.

ΠœΡ‹ ΠΌΠΎΠΆΠ΅ΠΌ ΠΏΠ΅Ρ€Π΅ΠΏΠΈΡΠ°Ρ‚ΡŒ это ΡƒΡ€Π°Π²Π½Π΅Π½ΠΈΠ΅ ΠΊΠ°ΠΊ J = V Γ— CJ = V Γ— C ΠΈ ΠΏΠΎΠ΄ΡΡ‚Π°Π²ΠΈΡ‚ΡŒ Π΅Π³ΠΎ Π² ΡƒΡ€Π°Π²Π½Π΅Π½ΠΈΠ΅ для Π²Π°Ρ‚Ρ‚, Ρ‡Ρ‚ΠΎΠ±Ρ‹ ΠΏΠΎΠ»ΡƒΡ‡ΠΈΡ‚ΡŒ

W = Js = V Γ— Cs = V Γ— Cs.W = Js = V Γ— Cs = V Γ— Cs.

Но ΠΊΡƒΠ»ΠΎΠ½ Π² сСкунду (Кл / с) – это элСктричСский Ρ‚ΠΎΠΊ, ΠΊΠΎΡ‚ΠΎΡ€Ρ‹ΠΉ ΠΌΡ‹ ΠΌΠΎΠΆΠ΅ΠΌ Π²ΠΈΠ΄Π΅Ρ‚ΡŒ ΠΈΠ· опрСдСлСния элСктричСского Ρ‚ΠΎΠΊΠ°, I = Ξ”QΞ”tI = Ξ”QΞ”t, Π³Π΄Π΅ ΔΔ Q – заряд Π² ΠΊΡƒΠ»ΠΎΠ½Π°Ρ…, Π° ΔΔ t – врСмя Π² сСкундах. Π’Π°ΠΊΠΈΠΌ ΠΎΠ±Ρ€Π°Π·ΠΎΠΌ, ΠΏΡ€ΠΈΠ²Π΅Π΄Π΅Π½Π½ΠΎΠ΅ Π²Ρ‹ΡˆΠ΅ ΡƒΡ€Π°Π²Π½Π΅Π½ΠΈΠ΅ Π³ΠΎΠ²ΠΎΡ€ΠΈΡ‚ Π½Π°ΠΌ, Ρ‡Ρ‚ΠΎ элСктричСская ΠΌΠΎΡ‰Π½ΠΎΡΡ‚ΡŒ Ρ€Π°Π²Π½Π° Π½Π°ΠΏΡ€ΡΠΆΠ΅Π½ΠΈΡŽ, ΡƒΠΌΠ½ΠΎΠΆΠ΅Π½Π½ΠΎΠΌΡƒ Π½Π° Ρ‚ΠΎΠΊ, ΠΈΠ»ΠΈ

Π­Ρ‚ΠΎ ΡƒΡ€Π°Π²Π½Π΅Π½ΠΈΠ΅ Π΄Π°Π΅Ρ‚ ΡΠ»Π΅ΠΊΡ‚Ρ€ΠΈΡ‡Π΅ΡΠΊΡƒΡŽ ΠΌΠΎΡ‰Π½ΠΎΡΡ‚ΡŒ, ΠΏΠΎΡ‚Ρ€Π΅Π±Π»ΡΠ΅ΠΌΡƒΡŽ Ρ†Π΅ΠΏΡŒΡŽ с ΠΏΠ°Π΄Π΅Π½ΠΈΠ΅ΠΌ напряТСния Π’ ΠΈ Ρ‚ΠΎΠΊΠΎΠΌ I .

НапримСр, рассмотрим схСму Π½Π° РисункС 19.21. По Π·Π°ΠΊΠΎΠ½Ρƒ Ома Ρ‚ΠΎΠΊ, ΠΏΡ€ΠΎΡ‚Π΅ΠΊΠ°ΡŽΡ‰ΠΈΠΉ ΠΏΠΎ Ρ†Π΅ΠΏΠΈ, Ρ€Π°Π²Π΅Π½

. I = VR = 12 Π’ 100 Ом = 0,12 А. I = VR = 12 Π’ 100 Ом = 0,12 А.

19,49

Π’Π°ΠΊΠΈΠΌ ΠΎΠ±Ρ€Π°Π·ΠΎΠΌ, ΠΌΠΎΡ‰Π½ΠΎΡΡ‚ΡŒ, потрСбляСмая схСмой, Ρ€Π°Π²Π½Π°

. P = VI = (12 Π’) (0,12 A) = 1,4 Π’Ρ‚. P = VI = (12 Π’) (0,12 A) = 1,4 Π’Ρ‚.

19,50

ΠšΡƒΠ΄Π° ΡƒΡ…ΠΎΠ΄ΠΈΡ‚ эта энСргия? Π’ этой схСмС ΠΌΠΎΡ‰Π½ΠΎΡΡ‚ΡŒ Π² основном ΠΈΠ΄Π΅Ρ‚ Π½Π° Π½Π°Π³Ρ€Π΅Π² рСзистора Π² этой Ρ†Π΅ΠΏΠΈ.

Рисунок 19.21 ΠŸΡ€ΠΎΡΡ‚Π°Ρ схСма, ΠΏΠΎΡ‚Ρ€Π΅Π±Π»ΡΡŽΡ‰Π°Ρ ΡΠ»Π΅ΠΊΡ‚Ρ€ΠΎΡΠ½Π΅Ρ€Π³ΠΈΡŽ.

ΠŸΡ€ΠΈ расчСтС мощности Π² схСмС Π½Π° РисункС 19.21, ΠΌΡ‹ использовали сопротивлСниС ΠΈ Π·Π°ΠΊΠΎΠ½ Ома, Ρ‡Ρ‚ΠΎΠ±Ρ‹ Π½Π°ΠΉΡ‚ΠΈ Ρ‚ΠΎΠΊ. Π—Π°ΠΊΠΎΠ½ Ома Π΄Π°Π΅Ρ‚ Ρ‚ΠΎΠΊ: I = V / RI = V / R, ΠΊΠΎΡ‚ΠΎΡ€Ρ‹ΠΉ ΠΌΡ‹ ΠΌΠΎΠΆΠ΅ΠΌ Π²ΡΡ‚Π°Π²ΠΈΡ‚ΡŒ Π² ΡƒΡ€Π°Π²Π½Π΅Π½ΠΈΠ΅ для элСктроэнСргии, Ρ‡Ρ‚ΠΎΠ±Ρ‹ ΠΏΠΎΠ»ΡƒΡ‡ΠΈΡ‚ΡŒ

P = IV = (VR) V = V2R.P = IV = (VR) V = V2R.

Π­Ρ‚ΠΎ Π΄Π°Π΅Ρ‚ ΠΌΠΎΡ‰Π½ΠΎΡΡ‚ΡŒ с Ρ‚ΠΎΡ‡ΠΊΠΈ зрСния Ρ‚ΠΎΠ»ΡŒΠΊΠΎ напряТСния ΠΈ сопротивлСния.

ΠœΡ‹ Ρ‚Π°ΠΊΠΆΠ΅ ΠΌΠΎΠΆΠ΅ΠΌ ΠΈΡΠΏΠΎΠ»ΡŒΠ·ΠΎΠ²Π°Ρ‚ΡŒ Π·Π°ΠΊΠΎΠ½ Ома, Ρ‡Ρ‚ΠΎΠ±Ρ‹ ΠΈΡΠΊΠ»ΡŽΡ‡ΠΈΡ‚ΡŒ напряТСниС ΠΈΠ· уравнСния для элСктричСской мощности ΠΈ ΠΏΠΎΠ»ΡƒΡ‡ΠΈΡ‚ΡŒ Π²Ρ‹Ρ€Π°ΠΆΠ΅Π½ΠΈΠ΅ для мощности, Π²Ρ‹Ρ€Π°ΠΆΠ΅Π½Π½ΠΎΠ΅ Ρ‚ΠΎΠ»ΡŒΠΊΠΎ Ρ‡Π΅Ρ€Π΅Π· Ρ‚ΠΎΠΊ ΠΈ сопротивлСниС. Если ΠΌΡ‹ запишСм Π·Π°ΠΊΠΎΠ½ Ома ΠΊΠ°ΠΊ V = IRV = IR ΠΈ ΠΈΡΠΏΠΎΠ»ΡŒΠ·ΡƒΠ΅ΠΌ это, Ρ‡Ρ‚ΠΎΠ±Ρ‹ ΠΈΡΠΊΠ»ΡŽΡ‡ΠΈΡ‚ΡŒ V Π² ΡƒΡ€Π°Π²Π½Π΅Π½ΠΈΠΈ P = IVP = IV, ΠΌΡ‹ ΠΏΠΎΠ»ΡƒΡ‡ΠΈΠΌ

P = IV = I (IR) = I2R.P = IV = I (IR) = I2R.

Π­Ρ‚ΠΎ Π΄Π°Π΅Ρ‚ ΠΌΠΎΡ‰Π½ΠΎΡΡ‚ΡŒ с Ρ‚ΠΎΡ‡ΠΊΠΈ зрСния Ρ‚ΠΎΠ»ΡŒΠΊΠΎ Ρ‚ΠΎΠΊΠ° ΠΈ сопротивлСния.

Π’Π°ΠΊΠΈΠΌ ΠΎΠ±Ρ€Π°Π·ΠΎΠΌ, комбинируя Π·Π°ΠΊΠΎΠ½ Ома с ΡƒΡ€Π°Π²Π½Π΅Π½ΠΈΠ΅ΠΌ P = IVP = IV для элСктроэнСргии, ΠΌΡ‹ ΠΏΠΎΠ»ΡƒΡ‡Π°Π΅ΠΌ Π΅Ρ‰Π΅ Π΄Π²Π° выраТСния для мощности: ΠΎΠ΄Π½ΠΎ Ρ‡Π΅Ρ€Π΅Π· напряТСниС ΠΈ сопротивлСниС, Π° Π΄Ρ€ΡƒΠ³ΠΎΠ΅ Ρ‡Π΅Ρ€Π΅Π· Ρ‚ΠΎΠΊ ΠΈ сопротивлСниС. ΠžΠ±Ρ€Π°Ρ‚ΠΈΡ‚Π΅ Π²Π½ΠΈΠΌΠ°Π½ΠΈΠ΅, Ρ‡Ρ‚ΠΎ Π² выраТСния для элСктричСской мощности входят Ρ‚ΠΎΠ»ΡŒΠΊΠΎ сопротивлСниС (Π½Π΅ Π΅ΠΌΠΊΠΎΡΡ‚ΡŒ ΠΈΠ»ΠΈ Ρ‡Ρ‚ΠΎ-Π»ΠΈΠ±ΠΎ Π΅Ρ‰Π΅), Ρ‚ΠΎΠΊ ΠΈ напряТСниС. Π­Ρ‚ΠΎ ΠΎΠ·Π½Π°Ρ‡Π°Π΅Ρ‚, Ρ‡Ρ‚ΠΎ физичСской характСристикой схСмы, ΠΎΠΏΡ€Π΅Π΄Π΅Π»ΡΡŽΡ‰Π΅ΠΉ, сколько мощности ΠΎΠ½Π° рассСиваСт, являСтся Π΅Π΅ сопротивлСниС.Π›ΡŽΠ±Ρ‹Π΅ кондСнсаторы Π² Ρ†Π΅ΠΏΠΈ Π½Π΅ Ρ€Π°ΡΡΠ΅ΠΈΠ²Π°ΡŽΡ‚ ΡΠ»Π΅ΠΊΡ‚Ρ€ΠΎΡΠ½Π΅Ρ€Π³ΠΈΡŽ – Π½Π°ΠΏΡ€ΠΎΡ‚ΠΈΠ², кондСнсаторы Π»ΠΈΠ±ΠΎ Π½Π°ΠΊΠ°ΠΏΠ»ΠΈΠ²Π°ΡŽΡ‚ ΡΠ»Π΅ΠΊΡ‚Ρ€ΠΈΡ‡Π΅ΡΠΊΡƒΡŽ ΡΠ½Π΅Ρ€Π³ΠΈΡŽ, Π»ΠΈΠ±ΠΎ ΠΎΡ‚Π΄Π°ΡŽΡ‚ Π΅Π΅ ΠΎΠ±Ρ€Π°Ρ‚Π½ΠΎ Π² Ρ†Π΅ΠΏΡŒ.

Π§Ρ‚ΠΎΠ±Ρ‹ ΠΏΡ€ΠΎΡΡΠ½ΠΈΡ‚ΡŒ, ΠΊΠ°ΠΊ связаны напряТСниС, сопротивлСниС, Ρ‚ΠΎΠΊ ΠΈ ΠΌΠΎΡ‰Π½ΠΎΡΡ‚ΡŒ, рассмотрим рисунок 19.22, Π½Π° ΠΊΠΎΡ‚ΠΎΡ€ΠΎΠΌ ΠΏΠΎΠΊΠ°Π·Π°Π½ΠΎ колСсо Ρ„ΠΎΡ€ΠΌΡƒΠ»Ρ‹ . ΠšΠΎΠ»ΠΈΡ‡Π΅ΡΡ‚Π²Π° Π² Ρ†Π΅Π½Ρ‚Ρ€Π°Π»ΡŒΠ½ΠΎΠΉ Ρ‡Π΅Ρ‚Π²Π΅Ρ€Ρ‚ΠΈ ΠΊΡ€ΡƒΠ³Π° Ρ€Π°Π²Π½Ρ‹ количСствам Π² ΡΠΎΠΎΡ‚Π²Π΅Ρ‚ΡΡ‚Π²ΡƒΡŽΡ‰Π΅ΠΉ внСшнСй Ρ‡Π΅Ρ‚Π²Π΅Ρ€Ρ‚ΠΈ ΠΊΡ€ΡƒΠ³Π°. НапримСр, Ρ‡Ρ‚ΠΎΠ±Ρ‹ Π²Ρ‹Ρ€Π°Π·ΠΈΡ‚ΡŒ ΠΏΠΎΡ‚Π΅Π½Ρ†ΠΈΠ°Π» V Ρ‡Π΅Ρ€Π΅Π· ΠΌΠΎΡ‰Π½ΠΎΡΡ‚ΡŒ ΠΈ Ρ‚ΠΎΠΊ, ΠΌΡ‹ Π²ΠΈΠ΄ΠΈΠΌ ΠΈΠ· колСса Ρ„ΠΎΡ€ΠΌΡƒΠ», Ρ‡Ρ‚ΠΎ V = P / IV = P / I.

Рисунок 19.22 КолСсо Ρ„ΠΎΡ€ΠΌΡƒΠ» ΠΏΠΎΠΊΠ°Π·Ρ‹Π²Π°Π΅Ρ‚, ΠΊΠ°ΠΊ связаны ΠΌΠ΅ΠΆΠ΄Ρƒ собой Π²ΠΎΠ»ΡŒΡ‚, сопротивлСниС, Ρ‚ΠΎΠΊ ΠΈ ΠΌΠΎΡ‰Π½ΠΎΡΡ‚ΡŒ. ΠšΠΎΠ»ΠΈΡ‡Π΅ΡΡ‚Π²Π° Π²ΠΎ Π²Π½ΡƒΡ‚Ρ€Π΅Π½Π½Π΅ΠΉ Ρ‡Π΅Ρ‚Π²Π΅Ρ€Ρ‚ΠΈ окруТности Ρ€Π°Π²Π½Ρ‹ количСству Π² ΡΠΎΠΎΡ‚Π²Π΅Ρ‚ΡΡ‚Π²ΡƒΡŽΡ‰Π΅ΠΉ внСшнСй Ρ‡Π΅Ρ‚Π²Π΅Ρ€Ρ‚ΠΈ окруТности.

Π Π°Π±ΠΎΡ‡ΠΈΠΉ ΠΏΡ€ΠΈΠΌΠ΅Ρ€

НайдитС сопротивлСниС Π»Π°ΠΌΠΏΠΎΡ‡ΠΊΠΈ

Випичная старая Π»Π°ΠΌΠΏΠ° накаливания ΠΈΠΌΠ΅Π»Π° ΠΌΠΎΡ‰Π½ΠΎΡΡ‚ΡŒ 60 Π’Ρ‚. Если ΠΏΡ€Π΅Π΄ΠΏΠΎΠ»ΠΎΠΆΠΈΡ‚ΡŒ, Ρ‡Ρ‚ΠΎ ΠΊ Π»Π°ΠΌΠΏΠΎΡ‡ΠΊΠ΅ ΠΏΡ€ΠΈΠ»ΠΎΠΆΠ΅Π½ΠΎ 120 Π’, ΠΊΠ°ΠΊΠΎΠ² Ρ‚ΠΎΠΊ Ρ‡Π΅Ρ€Π΅Π· Π»Π°ΠΌΠΏΠΎΡ‡ΠΊΡƒ?

БтратСгия

Нам Π΄Π°Π½Ρ‹ напряТСниС ΠΈ выходная ΠΌΠΎΡ‰Π½ΠΎΡΡ‚ΡŒ простой схСмы, содСрТащСй Π»Π°ΠΌΠΏΠΎΡ‡ΠΊΡƒ, поэтому ΠΌΡ‹ ΠΌΠΎΠΆΠ΅ΠΌ ΠΈΡΠΏΠΎΠ»ΡŒΠ·ΠΎΠ²Π°Ρ‚ΡŒ ΡƒΡ€Π°Π²Π½Π΅Π½ΠΈΠ΅ P = IVP = IV, Ρ‡Ρ‚ΠΎΠ±Ρ‹ Π½Π°ΠΉΡ‚ΠΈ Ρ‚ΠΎΠΊ I , ΠΏΡ€ΠΎΡ‚Π΅ΠΊΠ°ΡŽΡ‰ΠΈΠΉ Ρ‡Π΅Ρ€Π΅Π· Π»Π°ΠΌΠΏΠΎΡ‡ΠΊΡƒ.

РСшСниС

РСшСниС P = IVP = IV для Ρ‚ΠΎΠΊΠ° ΠΈ вставка Π΄Π°Π½Π½Ρ‹Ρ… Π·Π½Π°Ρ‡Π΅Π½ΠΈΠΉ для напряТСния ΠΈ мощности Π΄Π°Π΅Ρ‚

P = IVI = PV = 60 Π’Ρ‚ 120 Π’ = 0,50 А. P = IVI = PV = 60 Π’Ρ‚ 120 Π’ = 0,50 А.

19,51

Π’Π°ΠΊΠΈΠΌ ΠΎΠ±Ρ€Π°Π·ΠΎΠΌ, ΠΏΡ€ΠΈ ΠΏΠΎΠ΄Π°Ρ‡Π΅ 120 Π’. Ρ‡Π΅Ρ€Π΅Π· Π»Π°ΠΌΠΏΠΎΡ‡ΠΊΡƒ ΠΏΡ€ΠΎΡ…ΠΎΠ΄ΠΈΡ‚ ΠΏΠΎΠ»ΠΎΠ²ΠΈΠ½Π° Π°ΠΌΠΏΠ΅Ρ€Π°.

ΠžΠ±ΡΡƒΠΆΠ΄Π΅Π½ΠΈΠ΅

Π­Ρ‚ΠΎ Π·Π½Π°Ρ‡ΠΈΡ‚Π΅Π»ΡŒΠ½ΠΎΠ΅ Ρ‚Π΅Ρ‡Π΅Π½ΠΈΠ΅. Напомним, Ρ‡Ρ‚ΠΎ Π² Π±Ρ‹Ρ‚Ρƒ ΠΈΡΠΏΠΎΠ»ΡŒΠ·ΡƒΠ΅Ρ‚ΡΡ ΠΏΠ΅Ρ€Π΅ΠΌΠ΅Π½Π½Ρ‹ΠΉ, Π° Π½Π΅ постоянный Ρ‚ΠΎΠΊ, поэтому 120 Π’, ΠΏΠΎΠ΄Π°Π²Π°Π΅ΠΌΠΎΠ΅ ΠΎΡ‚ Π±Ρ‹Ρ‚ΠΎΠ²Ρ‹Ρ… Ρ€ΠΎΠ·Π΅Ρ‚ΠΎΠΊ, – это пСрСмСнная, Π° Π½Π΅ постоянная ΠΌΠΎΡ‰Π½ΠΎΡΡ‚ΡŒ.ЀактичСски, 120 Π’ – это усрСднСнная ΠΏΠΎ Π²Ρ€Π΅ΠΌΠ΅Π½ΠΈ ΠΌΠΎΡ‰Π½ΠΎΡΡ‚ΡŒ, обСспСчиваСмая Ρ‚Π°ΠΊΠΈΠΌΠΈ Ρ€ΠΎΠ·Π΅Ρ‚ΠΊΠ°ΠΌΠΈ. Π’Π°ΠΊΠΈΠΌ ΠΎΠ±Ρ€Π°Π·ΠΎΠΌ, срСдний Ρ‚ΠΎΠΊ, ΠΏΡ€ΠΎΡ‚Π΅ΠΊΠ°ΡŽΡ‰ΠΈΠΉ Ρ‡Π΅Ρ€Π΅Π· Π»Π°ΠΌΠΏΠΎΡ‡ΠΊΡƒ Π·Π° ΠΏΠ΅Ρ€ΠΈΠΎΠ΄ Π²Ρ€Π΅ΠΌΠ΅Π½ΠΈ, ΠΏΡ€Π΅Π²Ρ‹ΡˆΠ°ΡŽΡ‰ΠΈΠΉ нСсколько сСкунд, составляСт 0,50 А.

Π Π°Π±ΠΎΡ‡ΠΈΠΉ ΠΏΡ€ΠΈΠΌΠ΅Ρ€

ΠŸΠΎΠ΄ΠΎΠ³Ρ€Π΅Π²Π°Ρ‚Π΅Π»ΠΈ Π±ΠΎΡ‚ΠΈΠ½ΠΎΠΊ

Π§Ρ‚ΠΎΠ±Ρ‹ ΡΠΎΠ³Ρ€Π΅Ρ‚ΡŒ Π±ΠΎΡ‚ΠΈΠ½ΠΊΠΈ Π² Ρ…ΠΎΠ»ΠΎΠ΄Π½Ρ‹Π΅ Π΄Π½ΠΈ, Π²Ρ‹ Ρ€Π΅ΡˆΠΈΠ»ΠΈ Π²ΡˆΠΈΡ‚ΡŒ Ρ†Π΅ΠΏΡŒ с Π½Π΅ΠΊΠΎΡ‚ΠΎΡ€Ρ‹ΠΌΠΈ рСзисторами Π² ΡΡ‚Π΅Π»ΡŒΠΊΡƒ Π±ΠΎΡ‚ΠΈΠ½ΠΎΠΊ. Π’Π°ΠΌ Π½ΡƒΠΆΠ½ΠΎ 10 Π’Ρ‚ Ρ‚Π΅ΠΏΠ»Π° ΠΎΡ‚ рСзисторов Π² ΠΊΠ°ΠΆΠ΄ΠΎΠΉ ΡΡ‚Π΅Π»ΡŒΠΊΠ΅, ΠΈ Π²Ρ‹ Ρ…ΠΎΡ‚ΠΈΡ‚Π΅, Ρ‡Ρ‚ΠΎΠ±Ρ‹ ΠΎΠ½ΠΈ Ρ€Π°Π±ΠΎΡ‚Π°Π»ΠΈ ΠΎΡ‚ Π΄Π²ΡƒΡ… 9-Π²ΠΎΠ»ΡŒΡ‚ΠΎΠ²Ρ‹Ρ… Π±Π°Ρ‚Π°Ρ€Π΅ΠΉ (соСдинСнных ΠΏΠΎΡΠ»Π΅Π΄ΠΎΠ²Π°Ρ‚Π΅Π»ΡŒΠ½ΠΎ).КакоС ΠΎΠ±Ρ‰Π΅Π΅ сопротивлСниС Π²Ρ‹ Π΄ΠΎΠ»ΠΆΠ½Ρ‹ ΠΏΡ€ΠΈΠ»ΠΎΠΆΠΈΡ‚ΡŒ ΠΊ ΠΊΠ°ΠΆΠ΄ΠΎΠΉ ΡΡ‚Π΅Π»ΡŒΠΊΠ΅?

БтратСгия

Нам извСстны трСбуСмая ΠΌΠΎΡ‰Π½ΠΎΡΡ‚ΡŒ ΠΈ напряТСниС (18 Π’, ΠΏΠΎΡ‚ΠΎΠΌΡƒ Ρ‡Ρ‚ΠΎ Ρƒ нас Π΅ΡΡ‚ΡŒ Π΄Π²Π΅ Π±Π°Ρ‚Π°Ρ€Π΅ΠΈ 9 Π’, соСдинСнныС ΠΏΠΎΡΠ»Π΅Π΄ΠΎΠ²Π°Ρ‚Π΅Π»ΡŒΠ½ΠΎ), поэтому ΠΌΡ‹ ΠΌΠΎΠΆΠ΅ΠΌ ΠΈΡΠΏΠΎΠ»ΡŒΠ·ΠΎΠ²Π°Ρ‚ΡŒ ΡƒΡ€Π°Π²Π½Π΅Π½ΠΈΠ΅ P = V2 / RP = V2 / R, Ρ‡Ρ‚ΠΎΠ±Ρ‹ Π½Π°ΠΉΡ‚ΠΈ Π½Π΅ΠΎΠ±Ρ…ΠΎΠ΄ΠΈΠΌΠΎΠ΅ сопротивлСниС.

РСшСниС

РСшая P = V2 / RP = V2 / R для сопротивлСния ΠΈ вставляя Π΄Π°Π½Π½Ρ‹Π΅ напряТСниС ΠΈ ΠΌΠΎΡ‰Π½ΠΎΡΡ‚ΡŒ, ΠΏΠΎΠ»ΡƒΡ‡Π°Π΅ΠΌ

P = V2RR = V2P = (18 Π’) 210 Π’Ρ‚ = 32 Ом. P = V2RR = V2P = (18 Π’) 210 Π’Ρ‚ = 32 Ом.

19,52

Π’Π°ΠΊΠΈΠΌ ΠΎΠ±Ρ€Π°Π·ΠΎΠΌ, ΠΎΠ±Ρ‰Π΅Π΅ сопротивлСниС Π² ΠΊΠ°ΠΆΠ΄ΠΎΠΉ ΡΡ‚Π΅Π»ΡŒΠΊΠ΅ Π΄ΠΎΠ»ΠΆΠ½ΠΎ Π±Ρ‹Ρ‚ΡŒ 32 Ом.Ξ©.

ΠžΠ±ΡΡƒΠΆΠ΄Π΅Π½ΠΈΠ΅

Π”Π°Π²Π°ΠΉΡ‚Π΅ посмотрим, сколько Ρ‚ΠΎΠΊΠ° ΠΏΡ€ΠΎΠΉΠ΄Π΅Ρ‚ Ρ‡Π΅Ρ€Π΅Π· эту Ρ†Π΅ΠΏΡŒ. Π£ нас 18 Π’, ΠΏΡ€ΠΈΠ»ΠΎΠΆΠ΅Π½Π½ΠΎΠ΅ ΠΊ ΡΠΎΠΏΡ€ΠΎΡ‚ΠΈΠ²Π»Π΅Π½ΠΈΡŽ 32 Ом, поэтому Π·Π°ΠΊΠΎΠ½ Ома Π΄Π°Π΅Ρ‚

. I = VR = 18 Π’ 32 Ом = 0,56 А. I = VR = 18 Π’ 32 Ом = 0,56 А.

19,53

На всСх батарСях Π΅ΡΡ‚ΡŒ этикСтки, Π½Π° ΠΊΠΎΡ‚ΠΎΡ€Ρ‹Ρ… ΡƒΠΊΠ°Π·Π°Π½ΠΎ, сколько заряда ΠΎΠ½ΠΈ ΠΌΠΎΠ³ΡƒΡ‚ ΠΎΠ±Π΅ΡΠΏΠ΅Ρ‡ΠΈΡ‚ΡŒ (Π² Π΅Π΄ΠΈΠ½ΠΈΡ†Π°Ρ… силы Ρ‚ΠΎΠΊΠ°, ΡƒΠΌΠ½ΠΎΠΆΠ΅Π½Π½ΠΎΠ³ΠΎ Π½Π° врСмя). Випичная щСлочная батарСя 9 Π’ ΠΌΠΎΠΆΠ΅Ρ‚ ΠΎΠ±Π΅ΡΠΏΠ΅Ρ‡ΠΈΡ‚ΡŒ заряд 565 мА Β· Ρ‡ Β· мА Β· Ρ‡. (Ρ‚Π°ΠΊ Ρ‡Ρ‚ΠΎ Π΄Π²Π΅ Π±Π°Ρ‚Π°Ρ€Π΅ΠΈ 9 Π’ ΠΎΠ±Π΅ΡΠΏΠ΅Ρ‡ΠΈΠ²Π°ΡŽΡ‚ 1130 мА⋅ч мА⋅ч), поэтому эта систСма ΠΎΠ±ΠΎΠ³Ρ€Π΅Π²Π° ΠΏΡ€ΠΎΡ€Π°Π±ΠΎΡ‚Π°Π΅Ρ‚ Π² Ρ‚Π΅Ρ‡Π΅Π½ΠΈΠ΅

часов. t = 1130 Γ— 10βˆ’3Aβ‹…h0.56A = 2.0h.t = 1130 Γ— 10βˆ’3Aβ‹…h0.56A = 2.0h.

19,54

Π Π°Π±ΠΎΡ‡ΠΈΠΉ ΠΏΡ€ΠΈΠΌΠ΅Ρ€

ΠŸΠΈΡ‚Π°Π½ΠΈΠ΅ Ρ‡Π΅Ρ€Π΅Π· ΠΎΡ‚Π²Π΅Ρ‚Π²Π»Π΅Π½ΠΈΠ΅ Ρ†Π΅ΠΏΠΈ

ΠšΠ°ΠΆΠ΄Ρ‹ΠΉ рСзистор Π² ΠΏΡ€ΠΈΠ²Π΅Π΄Π΅Π½Π½ΠΎΠΉ Π½ΠΈΠΆΠ΅ схСмС ΠΈΠΌΠ΅Π΅Ρ‚ сопротивлСниС 30 Ом. Какая ΠΌΠΎΡ‰Π½ΠΎΡΡ‚ΡŒ рассСиваСтся срСднСй Π²Π΅Ρ‚Π²ΡŒΡŽ схСмы?

БтратСгия

БрСдняя Π²Π΅Ρ‚Π²ΡŒ схСмы содСрТит ΠΏΠΎΡΠ»Π΅Π΄ΠΎΠ²Π°Ρ‚Π΅Π»ΡŒΠ½ΠΎ Π²ΠΊΠ»ΡŽΡ‡Π΅Π½Π½Ρ‹Π΅ рСзисторы R3 ΠΈ R5R3 ΠΈ R5. НапряТСниС Π½Π° этой Π²Π΅Ρ‚Π²ΠΈ составляСт 12 Π’. Π‘Π½Π°Ρ‡Π°Π»Π° ΠΌΡ‹ Π½Π°ΠΉΠ΄Π΅ΠΌ эквивалСнтноС сопротивлСниС Π² этой Π²Π΅Ρ‚Π²ΠΈ, Π° Π·Π°Ρ‚Π΅ΠΌ ΠΈΡΠΏΠΎΠ»ΡŒΠ·ΡƒΠ΅ΠΌ P = V2 / RP = V2 / R, Ρ‡Ρ‚ΠΎΠ±Ρ‹ Π½Π°ΠΉΡ‚ΠΈ ΠΌΠΎΡ‰Π½ΠΎΡΡ‚ΡŒ, Ρ€Π°ΡΡΠ΅ΠΈΠ²Π°Π΅ΠΌΡƒΡŽ Π² Π²Π΅Ρ‚Π²ΠΈ.

РСшСниС

Π­ΠΊΠ²ΠΈΠ²Π°Π»Π΅Π½Ρ‚Π½ΠΎΠ΅ сопротивлСниС: R срСднСС = R3 + R5 = 30 Ом + 30 Ом = 60 Ом, срСднСС = R3 + R5 = 30 Ом + 30 Ом = 60 Ом. ΠœΠΎΡ‰Π½ΠΎΡΡ‚ΡŒ, рассСиваСмая срСднСй Π²Π΅Ρ‚Π²ΡŒΡŽ схСмы, составляСт

. P срСдний = V2R срСдний = (12 Π’) 260 Ом = 2,4 Π’Ρ‚. Π‘Ρ€Π΅Π΄Π½ΠΈΠΉ = V2 R срСдний = (12 Π’) 260 Ом = 2,4 Π’Ρ‚.

19,55

ΠžΠ±ΡΡƒΠΆΠ΄Π΅Π½ΠΈΠ΅

Π”Π°Π²Π°ΠΉΡ‚Π΅ посмотрим, сохраняСтся Π»ΠΈ энСргия Π² этой Ρ†Π΅ΠΏΠΈ, сравнив ΠΌΠΎΡ‰Π½ΠΎΡΡ‚ΡŒ, Ρ€Π°ΡΡΠ΅ΠΈΠ²Π°Π΅ΠΌΡƒΡŽ Π² Ρ†Π΅ΠΏΠΈ, с ΠΌΠΎΡ‰Π½ΠΎΡΡ‚ΡŒΡŽ, обСспСчиваСмой Π±Π°Ρ‚Π°Ρ€Π΅Π΅ΠΉ. Π’ΠΎ-ΠΏΠ΅Ρ€Π²Ρ‹Ρ…, эквивалСнтноС сопротивлСниС Π»Π΅Π²ΠΎΠΉ Π²Π΅Ρ‚Π²ΠΈ Ρ€Π°Π²Π½ΠΎ

. R влСво = 11 / R1 + 1 / R2 + R4 = 11/30 Ом + 1/30 Ом + 30 Ом = 45 Ом.R влСво = 11 / R1 + 1 / R2 + R4 = 11/30 Ом + 1/30 Ом + 30 Ом = 45 Ом.

19,56

ΠœΠΎΡ‰Π½ΠΎΡΡ‚ΡŒ Ρ‡Π΅Ρ€Π΅Π· Π»Π΅Π²ΡƒΡŽ Π²Π΅Ρ‚Π²ΡŒ

Π’Π»Π΅Π²ΠΎ = V2R Π²Π»Π΅Π²ΠΎ = (12 Π’) 245 Ом = 3,2 Π’Ρ‚. Π’Π»Π΅Π²ΠΎ = V 2 R Π²Π»Π΅Π²ΠΎ = (12 Π’) 245 Ом = 3,2 Π’Ρ‚.

19,57

ΠŸΡ€Π°Π²Π°Ρ Π²Π΅Ρ‚Π²ΡŒ содСрТит Ρ‚ΠΎΠ»ΡŒΠΊΠΎ R6R6, поэтому эквивалСнтноС сопротивлСниС Rright = R6 = 30Ξ©Rright = R6 = 30Ξ©. ΠœΠΎΡ‰Π½ΠΎΡΡ‚ΡŒ Ρ‡Π΅Ρ€Π΅Π· ΠΏΡ€Π°Π²ΡƒΡŽ Π²Π΅Ρ‚ΠΊΡƒ

ΠŸΡ€ΡΠΌΠΎ = V2 ΠŸΡ€Π°Π²ΠΎΠ΅ = (12 Π’) 230 Ом = 4,8 Π’Ρ‚ ΠŸΡ€Π°Π²ΠΎΠ΅ = V 2 ΠŸΡ€Π°Π²ΠΎΠ΅ = (12 Π’) 230 Ом = 4,8 Π’Ρ‚.

19,58

ΠžΠ±Ρ‰Π°Ρ ΠΌΠΎΡ‰Π½ΠΎΡΡ‚ΡŒ, рассСиваСмая схСмой, прСдставляСт собой сумму мощностСй, рассСиваСмых Π² ΠΊΠ°ΠΆΠ΄ΠΎΠΉ Π²Π΅Ρ‚Π²ΠΈ.

P = Pleft + Pmiddle + Pright = 2,4W + 3,2W + 4,8W = 10,4WP = Pleft + Pmiddle + Pright = 2,4W + 3,2W + 4,8W = 10,4W

19,59

ΠœΠΎΡ‰Π½ΠΎΡΡ‚ΡŒ, обСспСчиваСмая аккумулятором, составляСт

, Π³Π΄Π΅ I – ΠΏΠΎΠ»Π½Ρ‹ΠΉ Ρ‚ΠΎΠΊ, ΠΏΡ€ΠΎΡ‚Π΅ΠΊΠ°ΡŽΡ‰ΠΈΠΉ Ρ‡Π΅Ρ€Π΅Π· Π±Π°Ρ‚Π°Ρ€Π΅ΡŽ. ΠŸΠΎΡΡ‚ΠΎΠΌΡƒ ΠΌΡ‹ Π΄ΠΎΠ»ΠΆΠ½Ρ‹ ΡΠ»ΠΎΠΆΠΈΡ‚ΡŒ Ρ‚ΠΎΠΊΠΈ, проходящиС Ρ‡Π΅Ρ€Π΅Π· ΠΊΠ°ΠΆΠ΄ΡƒΡŽ Π²Π΅Ρ‚Π²ΡŒ, Ρ‡Ρ‚ΠΎΠ±Ρ‹ ΠΏΠΎΠ»ΡƒΡ‡ΠΈΡ‚ΡŒ I . Π’Π΅Ρ‚Π²ΠΈ Π΄Π°ΡŽΡ‚ Ρ‚ΠΎΠΊΠΈ

Π’Π»Π΅Π²ΠΎ = VR Π²Π»Π΅Π²ΠΎ = 12 Π’ 45 Ом = 0,2667 A Π’ сСрСдинС = VR Π² сСрСдинС = 12 Π’ 60 Ом = 0,20 Π’ Π²ΠΏΡ€Π°Π²ΠΎ = VR Π²ΠΏΡ€Π°Π²ΠΎ = 12 Π’ 30 Ом = 0,40 А. Π’Π»Π΅Π²ΠΎ = VR Π’Π»Π΅Π²ΠΎ = 12 45 Ом = 0,2667 A Π’ сСрСдинС = VR Π² сСрСдинС = 12 Π’ 60 Ом = 0.20AI right = VR right = 12 Π’ 30 Ом = 0,40 А.

19,61

Π‘ΡƒΠΌΠΌΠ°Ρ€Π½Ρ‹ΠΉ Ρ‚ΠΎΠΊ

I = Π»Π΅Π²Ρ‹ΠΉ + Imiddle + Iright = 0,2667A + 0,20A + 0,40A = 0,87A.I = Ileft + Imiddle + Iright = 0,2667A + 0,20A + 0,40A = 0,87A.

19,62

, Π° ΠΌΠΎΡ‰Π½ΠΎΡΡ‚ΡŒ, обСспСчиваСмая аккумулятором, составляСт

P = IV = (0,87 A) (12 Π’) = 10,4 Π’Ρ‚. P = IV = (0,87 A) (12 Π’) = 10,4 Π’Ρ‚.

19,63

Π­Ρ‚ΠΎ Ρ‚Π° ΠΆΠ΅ ΠΌΠΎΡ‰Π½ΠΎΡΡ‚ΡŒ, которая рассСиваСтся Π½Π° рСзисторах схСмы, Ρ‡Ρ‚ΠΎ ΠΏΠΎΠΊΠ°Π·Ρ‹Π²Π°Π΅Ρ‚, Ρ‡Ρ‚ΠΎ Π² этой Ρ†Π΅ΠΏΠΈ сохраняСтся энСргия.

ΠŸΡ€Π°ΠΊΡ‚ΠΈΡ‡Π΅ΡΠΊΠΈΠ΅ Π·Π°Π΄Π°Ρ‡ΠΈ

16.

Какова Ρ„ΠΎΡ€ΠΌΡƒΠ»Π° мощности, рассСиваСмой Π² рСзисторС?

  1. Π€ΠΎΡ€ΠΌΡƒΠ»Π° мощности, рассСиваСмой Π² рСзисторС, P = IV.P = IV.
  2. Π€ΠΎΡ€ΠΌΡƒΠ»Π° мощности, рассСиваСмой Π² рСзисторС, P = VI.P = VI.
  3. Π€ΠΎΡ€ΠΌΡƒΠ»Π° мощности, рассСиваСмой Π² рСзисторС: P = IV .
  4. Π€ΠΎΡ€ΠΌΡƒΠ»Π° мощности, рассСиваСмой Π½Π° рСзисторС: P = I 2 V .
17.

Какова Ρ„ΠΎΡ€ΠΌΡƒΠ»Π° мощности, рассСиваСмой рСзистором с ΡƒΡ‡Π΅Ρ‚ΠΎΠΌ Π΅Π³ΠΎ сопротивлСния ΠΈ напряТСния Π½Π° Π½Π΅ΠΌ?

  1. Π€ΠΎΡ€ΠΌΡƒΠ»Π° мощности, рассСиваСмой Π² рСзисторС: P = RV2P = RV2
  2. Π€ΠΎΡ€ΠΌΡƒΠ»Π° для мощности, рассСиваСмой Π² рСзисторС: P = V2RP = V2R
  3. Π€ΠΎΡ€ΠΌΡƒΠ»Π° мощности, рассСиваСмой Π² рСзисторС: P = V2RP = V2R
  4. Π€ΠΎΡ€ΠΌΡƒΠ»Π° для мощности, рассСиваСмой Π² рСзисторС: P = I2RP = I2R

ΠŸΡ€ΠΎΠ²Π΅Ρ€ΡŒΡ‚Π΅ своС ΠΏΠΎΠ½ΠΈΠΌΠ°Π½ΠΈΠ΅

18.

КакиС элСмСнты схСмы Ρ€Π°ΡΡΠ΅ΠΈΠ²Π°ΡŽΡ‚ ΠΌΠΎΡ‰Π½ΠΎΡΡ‚ΡŒ?

  1. кондСнсаторы
  2. ΠΈΠ½Π΄ΡƒΠΊΡ‚ΠΎΡ€Ρ‹
  3. ΠΏΠ΅Ρ€Π΅ΠΊΠ»ΡŽΡ‡Π°Ρ‚Π΅Π»ΠΈ ΠΈΠ΄Π΅Π°Π»ΡŒΠ½Ρ‹Π΅
  4. рСзисторы
19.

ΠžΠ±ΡŠΡΡΠ½ΠΈΡ‚Π΅ словами ΡƒΡ€Π°Π²Π½Π΅Π½ΠΈΠ΅ мощности, рассСиваСмой Π·Π°Π΄Π°Π½Π½Ρ‹ΠΌ сопротивлСниСм.

  1. ЭлСктричСская ΠΌΠΎΡ‰Π½ΠΎΡΡ‚ΡŒ ΠΏΡ€ΠΎΠΏΠΎΡ€Ρ†ΠΈΠΎΠ½Π°Π»ΡŒΠ½Π° Ρ‚ΠΎΠΊΡƒ Ρ‡Π΅Ρ€Π΅Π· рСзистор, ΡƒΠΌΠ½ΠΎΠΆΠ΅Π½Π½ΠΎΠΌΡƒ Π½Π° ΠΊΠ²Π°Π΄Ρ€Π°Ρ‚ напряТСния Π½Π° рСзисторС.
  2. ЭлСктричСская ΠΌΠΎΡ‰Π½ΠΎΡΡ‚ΡŒ ΠΏΡ€ΠΎΠΏΠΎΡ€Ρ†ΠΈΠΎΠ½Π°Π»ΡŒΠ½Π° ΠΊΠ²Π°Π΄Ρ€Π°Ρ‚Ρƒ Ρ‚ΠΎΠΊΠ° Ρ‡Π΅Ρ€Π΅Π· рСзистор, ΡƒΠΌΠ½ΠΎΠΆΠ΅Π½Π½ΠΎΠ³ΠΎ Π½Π° напряТСниС Π½Π° рСзисторС.
  3. ЭлСктричСская ΠΌΠΎΡ‰Π½ΠΎΡΡ‚ΡŒ ΠΏΡ€ΠΎΠΏΠΎΡ€Ρ†ΠΈΠΎΠ½Π°Π»ΡŒΠ½Π° Ρ‚ΠΎΠΊΡƒ Ρ‡Π΅Ρ€Π΅Π· рСзистор, Π΄Π΅Π»Π΅Π½Π½ΠΎΠΌΡƒ Π½Π° напряТСниС Π½Π° рСзисторС.
  4. ЭлСктричСская ΠΌΠΎΡ‰Π½ΠΎΡΡ‚ΡŒ ΠΏΡ€ΠΎΠΏΠΎΡ€Ρ†ΠΈΠΎΠ½Π°Π»ΡŒΠ½Π° Ρ‚ΠΎΠΊΡƒ Ρ‡Π΅Ρ€Π΅Π· рСзистор, ΡƒΠΌΠ½ΠΎΠΆΠ΅Π½Π½ΠΎΠΌΡƒ Π½Π° напряТСниС Π½Π° рСзисторС.

ЭлСктроэнСргия ΠΈ энСргия | Π€ΠΈΠ·ΠΈΠΊΠ°

Π¦Π΅Π»ΠΈ обучСния

К ΠΊΠΎΠ½Ρ†Ρƒ этого Ρ€Π°Π·Π΄Π΅Π»Π° Π²Ρ‹ смоТСтС:

  • РассчитайтС ΠΌΠΎΡ‰Π½ΠΎΡΡ‚ΡŒ, Ρ€Π°ΡΡΠ΅ΠΈΠ²Π°Π΅ΠΌΡƒΡŽ рСзистором, ΠΈ ΠΌΠΎΡ‰Π½ΠΎΡΡ‚ΡŒ, ΠΏΠΎΠ΄Π°Π²Π°Π΅ΠΌΡƒΡŽ источником питания.
  • РассчитайтС ΡΡ‚ΠΎΠΈΠΌΠΎΡΡ‚ΡŒ элСктроэнСргии ΠΏΡ€ΠΈ Ρ€Π°Π·Π»ΠΈΡ‡Π½Ρ‹Ρ… ΠΎΠ±ΡΡ‚ΠΎΡΡ‚Π΅Π»ΡŒΡΡ‚Π²Π°Ρ….

ΠœΠΎΡ‰Π½ΠΎΡΡ‚ΡŒ Π² элСктричСских цСпях

Π£ ΠΌΠ½ΠΎΠ³ΠΈΡ… людСй Π²Π»Π°ΡΡ‚ΡŒ ассоциируСтся с элСктричСством. Зная, Ρ‡Ρ‚ΠΎ ΠΌΠΎΡ‰Π½ΠΎΡΡ‚ΡŒ – это коэффициСнт использования ΠΈΠ»ΠΈ прСобразования энСргии, ΠΊΠ°ΠΊΠΎΠ²ΠΎ Π²Ρ‹Ρ€Π°ΠΆΠ΅Π½ΠΈΠ΅ для элСктроэнСргии ? На ΡƒΠΌ ΠΌΠΎΠ³ΡƒΡ‚ ΠΏΡ€ΠΈΠΉΡ‚ΠΈ Π»ΠΈΠ½ΠΈΠΈ элСктропСрСдач. ΠœΡ‹ Ρ‚Π°ΠΊΠΆΠ΅ Π΄ΡƒΠΌΠ°Π΅ΠΌ ΠΎ Π»Π°ΠΌΠΏΠΎΡ‡ΠΊΠ°Ρ… с Ρ‚ΠΎΡ‡ΠΊΠΈ зрСния ΠΈΡ… номинальной мощности Π² Π²Π°Ρ‚Ρ‚Π°Ρ…. Π‘Ρ€Π°Π²Π½ΠΈΠΌ Π»Π°ΠΌΠΏΠΎΡ‡ΠΊΡƒ Π½Π° 25 Π’Ρ‚ с Π»Π°ΠΌΠΏΠΎΠΉ Π½Π° 60 Π’Ρ‚.(Π‘ΠΌ. Рис. 1 (a).) ΠŸΠΎΡΠΊΠΎΠ»ΡŒΠΊΡƒ ΠΎΠ±Π° Ρ€Π°Π±ΠΎΡ‚Π°ΡŽΡ‚ ΠΎΡ‚ ΠΎΠ΄ΠΈΠ½Π°ΠΊΠΎΠ²ΠΎΠ³ΠΎ напряТСния, Π»Π°ΠΌΠΏΠ° ΠΌΠΎΡ‰Π½ΠΎΡΡ‚ΡŒΡŽ 60 Π’Ρ‚ Π΄ΠΎΠ»ΠΆΠ½Π° ΠΏΠΎΡ‚Ρ€Π΅Π±Π»ΡΡ‚ΡŒ большС Ρ‚ΠΎΠΊΠ°, Ρ‡Ρ‚ΠΎΠ±Ρ‹ ΠΈΠΌΠ΅Ρ‚ΡŒ Π±ΠΎΠ»ΡŒΡˆΡƒΡŽ Π½ΠΎΠΌΠΈΠ½Π°Π»ΡŒΠ½ΡƒΡŽ ΠΌΠΎΡ‰Π½ΠΎΡΡ‚ΡŒ. Π’Π°ΠΊΠΈΠΌ ΠΎΠ±Ρ€Π°Π·ΠΎΠΌ, сопротивлСниС Π»Π°ΠΌΠΏΡ‹ Π½Π° 60 Π’Ρ‚ Π΄ΠΎΠ»ΠΆΠ½ΠΎ Π±Ρ‹Ρ‚ΡŒ Π½ΠΈΠΆΠ΅, Ρ‡Π΅ΠΌ Ρƒ Π»Π°ΠΌΠΏΡ‹ Π½Π° 25 Π’Ρ‚. Если ΠΌΡ‹ ΡƒΠ²Π΅Π»ΠΈΡ‡ΠΈΠ²Π°Π΅ΠΌ напряТСниС, ΠΌΡ‹ Ρ‚Π°ΠΊΠΆΠ΅ ΡƒΠ²Π΅Π»ΠΈΡ‡ΠΈΠ²Π°Π΅ΠΌ ΠΌΠΎΡ‰Π½ΠΎΡΡ‚ΡŒ. НапримСр, ΠΊΠΎΠ³Π΄Π° Π»Π°ΠΌΠΏΠΎΡ‡ΠΊΠ° ΠΌΠΎΡ‰Π½ΠΎΡΡ‚ΡŒΡŽ 25 Π’Ρ‚, рассчитанная Π½Π° Ρ€Π°Π±ΠΎΡ‚Ρƒ ΠΎΡ‚ 120 Π’, ΠΏΠΎΠ΄ΠΊΠ»ΡŽΡ‡Π΅Π½Π° ΠΊ 240 Π’, ΠΎΠ½Π° Π½Π° ΠΊΠΎΡ€ΠΎΡ‚ΠΊΠΎΠ΅ врСмя ΠΎΡ‡Π΅Π½ΡŒ ярко свСтится, Π° Π·Π°Ρ‚Π΅ΠΌ ΠΏΠ΅Ρ€Π΅Π³ΠΎΡ€Π°Π΅Ρ‚. Как ΠΈΠΌΠ΅Π½Π½ΠΎ напряТСниС, Ρ‚ΠΎΠΊ ΠΈ сопротивлСниС связаны с элСктроэнСргиСй?

Рис. 1. (a) Какая ΠΈΠ· этих Π»Π°ΠΌΠΏΠΎΡ‡Π΅ΠΊ, Π»Π°ΠΌΠΏΠ° ΠΌΠΎΡ‰Π½ΠΎΡΡ‚ΡŒΡŽ 25 Π’Ρ‚ (Π²Π²Π΅Ρ€Ρ…Ρƒ слСва) ΠΈΠ»ΠΈ Π»Π°ΠΌΠΏΠ° ΠΌΠΎΡ‰Π½ΠΎΡΡ‚ΡŒΡŽ 60 Π’Ρ‚ (Π²Π²Π΅Ρ€Ρ…Ρƒ справа), ΠΈΠΌΠ΅Π΅Ρ‚ Π±ΠΎΠ»Π΅Π΅ высокоС сопротивлСниС? Π§Ρ‚ΠΎ потрСбляСт большС Ρ‚ΠΎΠΊΠ°? Π§Ρ‚ΠΎ потрСбляСт большС всСго энСргии? МоТно Π»ΠΈ ΠΏΠΎ Ρ†Π²Π΅Ρ‚Ρƒ ΡΠΊΠ°Π·Π°Ρ‚ΡŒ, Ρ‡Ρ‚ΠΎ Π½ΠΈΡ‚ΡŒ накаливания ΠΌΠΎΡ‰Π½ΠΎΡΡ‚ΡŒΡŽ 25 Π’Ρ‚ ΠΊΡ€ΡƒΡ‡Π΅? ЯвляСтся Π»ΠΈ Π±ΠΎΠ»Π΅Π΅ яркая Π»Π°ΠΌΠΏΠΎΡ‡ΠΊΠ° Π΄Ρ€ΡƒΠ³ΠΎΠ³ΠΎ Ρ†Π²Π΅Ρ‚Π°, ΠΈ Ссли Π΄Π°, Ρ‚ΠΎ ΠΏΠΎΡ‡Π΅ΠΌΡƒ? (ΠΊΡ€Π΅Π΄ΠΈΡ‚Ρ‹: Π”ΠΈΠΊΠ±Π°ΡƒΡ…, Wikimedia Commons; Π“Ρ€Π΅Π³ ВСстфолл, Flickr) (Π±) Π­Ρ‚ΠΎΡ‚ ΠΊΠΎΠΌΠΏΠ°ΠΊΡ‚Π½Ρ‹ΠΉ Π»ΡŽΠΌΠΈΠ½Π΅ΡΡ†Π΅Π½Ρ‚Π½Ρ‹ΠΉ ΡΠ²Π΅Ρ‚ΠΈΠ»ΡŒΠ½ΠΈΠΊ (ΠšΠ›Π›) ΠΈΠ·Π»ΡƒΡ‡Π°Π΅Ρ‚ Ρ‚Π°ΠΊΡƒΡŽ ​​ТС ΠΈΠ½Ρ‚Π΅Π½ΡΠΈΠ²Π½ΠΎΡΡ‚ΡŒ свСта, ΠΊΠ°ΠΊ ΠΈ Π»Π°ΠΌΠΏΠ° ΠΌΠΎΡ‰Π½ΠΎΡΡ‚ΡŒΡŽ 60 Π’Ρ‚, Π½ΠΎ с Π²Ρ…ΠΎΠ΄Π½ΠΎΠΉ ΠΌΠΎΡ‰Π½ΠΎΡΡ‚ΡŒΡŽ ΠΎΡ‚ 1/4 Π΄ΠΎ 1/10.(ΠΊΡ€Π΅Π΄ΠΈΡ‚: dbgg1979, Flickr)

ЭлСктричСская энСргия зависит ΠΊΠ°ΠΊ ΠΎΡ‚ напряТСния, Ρ‚Π°ΠΊ ΠΈ ΠΎΡ‚ ΠΏΠ΅Ρ€Π΅ΠΌΠ΅Ρ‰Π°Π΅ΠΌΠΎΠ³ΠΎ заряда. ΠŸΡ€ΠΎΡ‰Π΅ всСго это выраТаСтся ΠΊΠ°ΠΊ PE = qV , Π³Π΄Π΅ q – это ΠΏΠ΅Ρ€Π΅ΠΌΠ΅Ρ‰Π°Π΅ΠΌΡ‹ΠΉ заряд, Π° V – это напряТСниС (ΠΈΠ»ΠΈ, Ρ‚ΠΎΡ‡Π½Π΅Π΅, Ρ€Π°Π·Π½ΠΎΡΡ‚ΡŒ ΠΏΠΎΡ‚Π΅Π½Ρ†ΠΈΠ°Π»ΠΎΠ², Ρ‡Π΅Ρ€Π΅Π· ΠΊΠΎΡ‚ΠΎΡ€ΡƒΡŽ ΠΏΡ€ΠΎΡ…ΠΎΠ΄ΠΈΡ‚ заряд). ΠœΠΎΡ‰Π½ΠΎΡΡ‚ΡŒ – это ΡΠΊΠΎΡ€ΠΎΡΡ‚ΡŒ пСрСмСщСния энСргии, поэтому элСктричСская ΠΌΠΎΡ‰Π½ΠΎΡΡ‚ΡŒ Ρ€Π°Π²Π½Π°

.

[латСкс] P = \ frac {PE} {t} = \ frac {qV} {t} \\ [/ latex].

Учитывая, Ρ‡Ρ‚ΠΎ Ρ‚ΠΎΠΊ Ρ€Π°Π²Π΅Π½ I = q / t (ΠΎΠ±Ρ€Π°Ρ‚ΠΈΡ‚Π΅ Π²Π½ΠΈΠΌΠ°Π½ΠΈΠ΅, Ρ‡Ρ‚ΠΎ Ξ” t = t здСсь), Π²Ρ‹Ρ€Π°ΠΆΠ΅Π½ΠΈΠ΅ для мощности ΠΏΡ€ΠΈΠ½ΠΈΠΌΠ°Π΅Ρ‚ Π²ΠΈΠ΄

P = IV

ЭлСктричСская ΠΌΠΎΡ‰Π½ΠΎΡΡ‚ΡŒ ( P ) – это просто ΠΏΡ€ΠΎΠΈΠ·Π²Π΅Π΄Π΅Π½ΠΈΠ΅ Ρ‚ΠΎΠΊΠ° Π½Π° напряТСниС.ΠœΠΎΡ‰Π½ΠΎΡΡ‚ΡŒ ΠΈΠΌΠ΅Π΅Ρ‚ Π·Π½Π°ΠΊΠΎΠΌΡ‹Π΅ Π΅Π΄ΠΈΠ½ΠΈΡ†Ρ‹ Π²Π°Ρ‚Ρ‚. ΠŸΠΎΡΠΊΠΎΠ»ΡŒΠΊΡƒ Π΅Π΄ΠΈΠ½ΠΈΡ†Π΅ΠΉ БИ для ΠΏΠΎΡ‚Π΅Π½Ρ†ΠΈΠ°Π»ΡŒΠ½ΠΎΠΉ энСргии (PE) являСтся Π΄ΠΆΠΎΡƒΠ»ΡŒ, ΠΌΠΎΡ‰Π½ΠΎΡΡ‚ΡŒ выраТаСтся Π² дТоулях Π² сСкунду ΠΈΠ»ΠΈ Π²Π°Ρ‚Ρ‚Π°Ρ…. Π’Π°ΠΊΠΈΠΌ ΠΎΠ±Ρ€Π°Π·ΠΎΠΌ, 1 A β‹…V = 1 Π’Ρ‚. НапримСр, Π² автомобилях часто Π΅ΡΡ‚ΡŒ ΠΎΠ΄Π½Π° ΠΈΠ»ΠΈ нСсколько Π΄ΠΎΠΏΠΎΠ»Π½ΠΈΡ‚Π΅Π»ΡŒΠ½Ρ‹Ρ… Ρ€ΠΎΠ·Π΅Ρ‚ΠΎΠΊ, с ΠΏΠΎΠΌΠΎΡ‰ΡŒΡŽ ΠΊΠΎΡ‚ΠΎΡ€Ρ‹Ρ… ΠΌΠΎΠΆΠ½ΠΎ Π·Π°Ρ€ΡΠΆΠ°Ρ‚ΡŒ сотовый Ρ‚Π΅Π»Π΅Ρ„ΠΎΠ½ ΠΈΠ»ΠΈ Π΄Ρ€ΡƒΠ³ΠΈΠ΅ элСктронныС устройства. {2} R \\ [/ latex].

ΠžΠ±Ρ€Π°Ρ‚ΠΈΡ‚Π΅ Π²Π½ΠΈΠΌΠ°Π½ΠΈΠ΅, Ρ‡Ρ‚ΠΎ ΠΏΠ΅Ρ€Π²ΠΎΠ΅ ΡƒΡ€Π°Π²Π½Π΅Π½ΠΈΠ΅ всСгда Π²Π΅Ρ€Π½ΠΎ, Ρ‚ΠΎΠ³Π΄Π° ΠΊΠ°ΠΊ Π΄Π²Π° Π΄Ρ€ΡƒΠ³ΠΈΡ… ΠΌΠΎΠΆΠ½ΠΎ ΠΈΡΠΏΠΎΠ»ΡŒΠ·ΠΎΠ²Π°Ρ‚ΡŒ Ρ‚ΠΎΠ»ΡŒΠΊΠΎ для рСзисторов. Π’ простой схСмС с ΠΎΠ΄Π½ΠΈΠΌ источником напряТСния ΠΈ ΠΎΠ΄Π½ΠΈΠΌ рСзистором ΠΌΠΎΡ‰Π½ΠΎΡΡ‚ΡŒ, подаваСмая источником напряТСния, ΠΈ ΠΌΠΎΡ‰Π½ΠΎΡΡ‚ΡŒ, рассСиваСмая рСзистором, ΠΈΠ΄Π΅Π½Ρ‚ΠΈΡ‡Π½Ρ‹. (Π’ Π±ΠΎΠ»Π΅Π΅ слоТных схСмах P ΠΌΠΎΠΆΠ΅Ρ‚ Π±Ρ‹Ρ‚ΡŒ ΠΌΠΎΡ‰Π½ΠΎΡΡ‚ΡŒΡŽ, рассСиваСмой ΠΎΠ΄Π½ΠΈΠΌ устройством, Π° Π½Π΅ ΠΏΠΎΠ»Π½ΠΎΠΉ ΠΌΠΎΡ‰Π½ΠΎΡΡ‚ΡŒΡŽ Π² Ρ†Π΅ΠΏΠΈ.) Из Ρ‚Ρ€Π΅Ρ… Ρ€Π°Π·Π»ΠΈΡ‡Π½Ρ‹Ρ… Π²Ρ‹Ρ€Π°ΠΆΠ΅Π½ΠΈΠΉ для элСктричСской мощности ΠΌΠΎΠΆΠ½ΠΎ ΠΏΠΎΠ»ΡƒΡ‡ΠΈΡ‚ΡŒ Ρ€Π°Π·Π»ΠΈΡ‡Π½ΠΎΠ΅ ΠΏΠΎΠ½ΠΈΠΌΠ°Π½ΠΈΠ΅. НапримСр, P = V 2 / R ΠΏΠΎΠ΄Ρ€Π°Π·ΡƒΠΌΠ΅Π²Π°Π΅Ρ‚, Ρ‡Ρ‚ΠΎ Ρ‡Π΅ΠΌ мСньшС сопротивлСниС, ΠΏΠΎΠ΄ΠΊΠ»ΡŽΡ‡Π΅Π½Π½ΠΎΠ΅ ΠΊ Π΄Π°Π½Π½ΠΎΠΌΡƒ источнику напряТСния, Ρ‚Π΅ΠΌ большС пСрСдаваСмая ΠΌΠΎΡ‰Π½ΠΎΡΡ‚ΡŒ.ΠšΡ€ΠΎΠΌΠ΅ Ρ‚ΠΎΠ³ΠΎ, ΠΏΠΎΡΠΊΠΎΠ»ΡŒΠΊΡƒ напряТСниС Π²ΠΎΠ·Π²Π΅Π΄Π΅Π½ΠΎ Π² ΠΊΠ²Π°Π΄Ρ€Π°Ρ‚ Π² P = V 2 / R , эффСкт ΠΎΡ‚ прилоТСния Π±ΠΎΠ»Π΅Π΅ высокого напряТСния, Π²ΠΎΠ·ΠΌΠΎΠΆΠ½ΠΎ, большС, Ρ‡Π΅ΠΌ оТидалось. Π’Π°ΠΊΠΈΠΌ ΠΎΠ±Ρ€Π°Π·ΠΎΠΌ, ΠΊΠΎΠ³Π΄Π° напряТСниС увСличиваСтся Π²Π΄Π²ΠΎΠ΅ Π΄ΠΎ Π»Π°ΠΌΠΏΠΎΡ‡ΠΊΠΈ ΠΌΠΎΡ‰Π½ΠΎΡΡ‚ΡŒΡŽ 25 Π’Ρ‚, Π΅Π΅ ΠΌΠΎΡ‰Π½ΠΎΡΡ‚ΡŒ увСличиваСтся ΠΏΠΎΡ‡Ρ‚ΠΈ Π² Ρ‡Π΅Ρ‚Ρ‹Ρ€Π΅ Ρ€Π°Π·Π° Π΄ΠΎ ΠΏΡ€ΠΈΠΌΠ΅Ρ€Π½ΠΎ 100 Π’Ρ‚, Ρ‡Ρ‚ΠΎ ΠΏΡ€ΠΈΠ²ΠΎΠ΄ΠΈΡ‚ ΠΊ Π΅Π΅ ΠΏΠ΅Ρ€Π΅Π³ΠΎΡ€Π°Π½ΠΈΡŽ. Если Π±Ρ‹ сопротивлСниС Π»Π°ΠΌΠΏΡ‹ ΠΎΡΡ‚Π°Π²Π°Π»ΠΎΡΡŒ постоянным, Π΅Π΅ ΠΌΠΎΡ‰Π½ΠΎΡΡ‚ΡŒ Π±Ρ‹Π»Π° Π±Ρ‹ Ρ€ΠΎΠ²Π½ΠΎ 100 Π’Ρ‚, Π½ΠΎ ΠΏΡ€ΠΈ Π±ΠΎΠ»Π΅Π΅ высокой Ρ‚Π΅ΠΌΠΏΠ΅Ρ€Π°Ρ‚ΡƒΡ€Π΅ Π΅Π΅ сопротивлСниС Ρ‚Π°ΠΊΠΆΠ΅ Π±ΡƒΠ΄Π΅Ρ‚ Π²Ρ‹ΡˆΠ΅.

ΠŸΡ€ΠΈΠΌΠ΅Ρ€ 1. РасчСт рассСиваСмой мощности ΠΈ Ρ‚ΠΎΠΊΠ°: горячая ΠΈ холодная энСргия

(Π°) Рассмотрим ΠΏΡ€ΠΈΠΌΠ΅Ρ€Ρ‹, ΠΏΡ€ΠΈΠ²Π΅Π΄Π΅Π½Π½Ρ‹Π΅ Π² Π—Π°ΠΊΠΎΠ½Π΅ Ома: сопротивлСниС ΠΈ простыС Ρ†Π΅ΠΏΠΈ ΠΈ сопротивлСниС ΠΈ ΡƒΠ΄Π΅Π»ΡŒΠ½ΠΎΠ΅ сопротивлСниС.Π—Π°Ρ‚Π΅ΠΌ Π½Π°ΠΉΠ΄ΠΈΡ‚Π΅ ΠΌΠΎΡ‰Π½ΠΎΡΡ‚ΡŒ, Ρ€Π°ΡΡΠ΅ΠΈΠ²Π°Π΅ΠΌΡƒΡŽ Π°Π²Ρ‚ΠΎΠΌΠΎΠ±ΠΈΠ»ΡŒΠ½ΠΎΠΉ Ρ„Π°Ρ€ΠΎΠΉ Π² этих ΠΏΡ€ΠΈΠΌΠ΅Ρ€Π°Ρ…, ΠΊΠ°ΠΊ Π² Π³ΠΎΡ€ΡΡ‡ΡƒΡŽ, Ρ‚Π°ΠΊ ΠΈ Π² Ρ…ΠΎΠ»ΠΎΠ΄Π½ΡƒΡŽ ΠΏΠΎΠ³ΠΎΠ΄Ρƒ. Π±) Какой Ρ‚ΠΎΠΊ ΠΎΠ½ потрСбляСт Π² Ρ…ΠΎΠ»ΠΎΠ΄Π½ΠΎΠΌ состоянии?

БтратСгия для (Π°)

Для горячСй Ρ„Π°Ρ€Ρ‹ Π½Π°ΠΌ извСстны напряТСниС ΠΈ Ρ‚ΠΎΠΊ, поэтому ΠΌΡ‹ ΠΌΠΎΠΆΠ΅ΠΌ ΠΈΡΠΏΠΎΠ»ΡŒΠ·ΠΎΠ²Π°Ρ‚ΡŒ P = IV , Ρ‡Ρ‚ΠΎΠ±Ρ‹ Π½Π°ΠΉΡ‚ΠΈ ΠΌΠΎΡ‰Π½ΠΎΡΡ‚ΡŒ. Для Ρ…ΠΎΠ»ΠΎΠ΄Π½ΠΎΠΉ Ρ„Π°Ρ€Ρ‹ Π½Π°ΠΌ извСстны напряТСниС ΠΈ сопротивлСниС, поэтому ΠΌΡ‹ ΠΌΠΎΠΆΠ΅ΠΌ ΠΈΡΠΏΠΎΠ»ΡŒΠ·ΠΎΠ²Π°Ρ‚ΡŒ P = Π’ 2 / R , Ρ‡Ρ‚ΠΎΠ±Ρ‹ Π½Π°ΠΉΡ‚ΠΈ ΠΌΠΎΡ‰Π½ΠΎΡΡ‚ΡŒ.

РСшСниС для (а)

Вводя извСстныС значСния Ρ‚ΠΎΠΊΠ° ΠΈ напряТСния для горячСй Ρ„Π°Ρ€Ρ‹, ΠΏΠΎΠ»ΡƒΡ‡Π°Π΅ΠΌ

P = IV = (2.{2}} {0,350 \ text {} \ Omega} = 411 \ text {W} \\ [/ latex].

ΠžΠ±ΡΡƒΠΆΠ΄Π΅Π½ΠΈΠ΅ для (Π°)

30 Π’Ρ‚, рассСиваСмыС горячСй Ρ„Π°Ρ€ΠΎΠΉ, ΡΠ²Π»ΡΡŽΡ‚ΡΡ Ρ‚ΠΈΠΏΠΈΡ‡Π½Ρ‹ΠΌΠΈ. Но 411 Π’Ρ‚ Π² Ρ…ΠΎΠ»ΠΎΠ΄Π½ΡƒΡŽ ΠΏΠΎΠ³ΠΎΠ΄Ρƒ Π½Π° ΡƒΠ΄ΠΈΠ²Π»Π΅Π½ΠΈΠ΅ Π²Ρ‹ΡˆΠ΅. ΠΠ°Ρ‡Π°Π»ΡŒΠ½Π°Ρ ΠΌΠΎΡ‰Π½ΠΎΡΡ‚ΡŒ быстро ΡƒΠΌΠ΅Π½ΡŒΡˆΠ°Π΅Ρ‚ΡΡ ΠΏΠΎ ΠΌΠ΅Ρ€Π΅ увСличСния Ρ‚Π΅ΠΌΠΏΠ΅Ρ€Π°Ρ‚ΡƒΡ€Ρ‹ Π»Π°ΠΌΠΏΡ‹ ΠΈ увСличСния Π΅Π΅ сопротивлСния.

БтратСгия ΠΈ Ρ€Π΅ΡˆΠ΅Π½ΠΈΠ΅ для (b)

Π’ΠΎΠΊ ΠΏΡ€ΠΈ Ρ…ΠΎΠ»ΠΎΠ΄Π½ΠΎΠΉ Π»Π°ΠΌΠΏΠΎΡ‡ΠΊΠ΅ ΠΌΠΎΠΆΠ½ΠΎ Π½Π°ΠΉΡ‚ΠΈ нСсколькими способами. ΠŸΠ΅Ρ€Π΅ΡΡ‚Π°Π²Π»ΡΠ΅ΠΌ ΠΎΠ΄Π½ΠΎ ΠΈΠ· ΡƒΡ€Π°Π²Π½Π΅Π½ΠΈΠΉ мощности, P = I 2 R , ΠΈ Π²Π²ΠΎΠ΄ΠΈΠΌ извСстныС значСния, получая

[латСкс] I = \ sqrt {\ frac {P} {R}} = \ sqrt {\ frac {411 \ text {W}} {{0.350} \ text {} \ Omega}} = 34,3 \ text {A} \\ [/ latex].

ΠžΠ±ΡΡƒΠΆΠ΄Π΅Π½ΠΈΠ΅ для (Π±)

Π₯ΠΎΠ»ΠΎΠ΄Π½Ρ‹ΠΉ Ρ‚ΠΎΠΊ Π·Π½Π°Ρ‡ΠΈΡ‚Π΅Π»ΡŒΠ½ΠΎ Π²Ρ‹ΡˆΠ΅, Ρ‡Π΅ΠΌ ΡƒΡΡ‚Π°Π½ΠΎΠ²ΠΈΠ²ΡˆΠ΅Π΅ΡΡ Π·Π½Π°Ρ‡Π΅Π½ΠΈΠ΅ 2,50 А, Π½ΠΎ Ρ‚ΠΎΠΊ Π±ΡƒΠ΄Π΅Ρ‚ быстро ΡΠ½ΠΈΠΆΠ°Ρ‚ΡŒΡΡ Π΄ΠΎ этого значСния ΠΏΠΎ ΠΌΠ΅Ρ€Π΅ увСличСния Ρ‚Π΅ΠΌΠΏΠ΅Ρ€Π°Ρ‚ΡƒΡ€Ρ‹ Π»Π°ΠΌΠΏΡ‹. Π‘ΠΎΠ»ΡŒΡˆΠΈΠ½ΡΡ‚Π²ΠΎ ΠΏΡ€Π΅Π΄ΠΎΡ…Ρ€Π°Π½ΠΈΡ‚Π΅Π»Π΅ΠΉ ΠΈ автоматичСских Π²Ρ‹ΠΊΠ»ΡŽΡ‡Π°Ρ‚Π΅Π»Π΅ΠΉ (ΠΈΡΠΏΠΎΠ»ΡŒΠ·ΡƒΠ΅ΠΌΡ‹Ρ… для ограничСния Ρ‚ΠΎΠΊΠ° Π² Ρ†Π΅ΠΏΠΈ) рассчитаны Π½Π° ΠΊΡ€Π°Ρ‚ΠΊΠΎΠ²Ρ€Π΅ΠΌΠ΅Π½Π½ΡƒΡŽ Π²Ρ‹Π΄Π΅Ρ€ΠΆΠΊΡƒ ΠΎΡ‡Π΅Π½ΡŒ высоких Ρ‚ΠΎΠΊΠΎΠ² ΠΏΡ€ΠΈ Π²ΠΊΠ»ΡŽΡ‡Π΅Π½ΠΈΠΈ устройства. Π’ Π½Π΅ΠΊΠΎΡ‚ΠΎΡ€Ρ‹Ρ… случаях, Π½Π°ΠΏΡ€ΠΈΠΌΠ΅Ρ€, с элСктродвигатСлями, Ρ‚ΠΎΠΊ остаСтся высоким Π² Ρ‚Π΅Ρ‡Π΅Π½ΠΈΠ΅ Π½Π΅ΡΠΊΠΎΠ»ΡŒΠΊΠΈΡ… сСкунд, Ρ‡Ρ‚ΠΎ Ρ‚Ρ€Π΅Π±ΡƒΠ΅Ρ‚ использования ΡΠΏΠ΅Ρ†ΠΈΠ°Π»ΡŒΠ½Ρ‹Ρ… ΠΏΠ»Π°Π²ΠΊΠΈΡ… ΠΏΡ€Π΅Π΄ΠΎΡ…Ρ€Π°Π½ΠΈΡ‚Π΅Π»Π΅ΠΉ с Π·Π°ΠΌΠ΅Π΄Π»Π΅Π½Π½Ρ‹ΠΌ срабатываниСм.

Π§Π΅ΠΌ большС элСктроприборов Π²Ρ‹ ΠΈΡΠΏΠΎΠ»ΡŒΠ·ΡƒΠ΅Ρ‚Π΅ ΠΈ Ρ‡Π΅ΠΌ дольшС ΠΎΠ½ΠΈ ΠΎΡΡ‚Π°ΡŽΡ‚ΡΡ Π²ΠΊΠ»ΡŽΡ‡Π΅Π½Π½Ρ‹ΠΌΠΈ, Ρ‚Π΅ΠΌ Π²Ρ‹ΡˆΠ΅ ваш счСт Π·Π° ΡΠ»Π΅ΠΊΡ‚Ρ€ΠΎΡΠ½Π΅Ρ€Π³ΠΈΡŽ. Π­Ρ‚ΠΎΡ‚ Π·Π½Π°ΠΊΠΎΠΌΡ‹ΠΉ Ρ„Π°ΠΊΡ‚ основан Π½Π° ΡΠΎΠΎΡ‚Π½ΠΎΡˆΠ΅Π½ΠΈΠΈ энСргии ΠΈ мощности. Π’Ρ‹ ΠΏΠ»Π°Ρ‚ΠΈΡ‚Π΅ Π·Π° ΠΈΡΠΏΠΎΠ»ΡŒΠ·ΠΎΠ²Π°Π½Π½ΡƒΡŽ ΡΠ½Π΅Ρ€Π³ΠΈΡŽ. Π’Π°ΠΊ ΠΊΠ°ΠΊ P = E / t , ΠΌΡ‹ Π²ΠΈΠ΄ΠΈΠΌ, Ρ‡Ρ‚ΠΎ

E = Pt

– это энСргия, ΠΈΡΠΏΠΎΠ»ΡŒΠ·ΡƒΠ΅ΠΌΠ°Ρ устройством, ΠΈΡΠΏΠΎΠ»ΡŒΠ·ΡƒΡŽΡ‰ΠΈΠΌ ΠΌΠΎΡ‰Π½ΠΎΡΡ‚ΡŒ P Π² Ρ‚Π΅Ρ‡Π΅Π½ΠΈΠ΅ Π²Ρ€Π΅ΠΌΠ΅Π½Π½ΠΎΠ³ΠΎ ΠΈΠ½Ρ‚Π΅Ρ€Π²Π°Π»Π° t . НапримСр, Ρ‡Π΅ΠΌ большС Π³ΠΎΡ€Π΅Π»ΠΎ Π»Π°ΠΌΠΏΠΎΡ‡Π΅ΠΊ, Ρ‚Π΅ΠΌ большС использовалось P ; Ρ‡Π΅ΠΌ дольшС ΠΎΠ½ΠΈ Ρ€Π°Π±ΠΎΡ‚Π°ΡŽΡ‚, Ρ‚Π΅ΠΌ большС Ρ‚ .Π•Π΄ΠΈΠ½ΠΈΡ†Π΅ΠΉ измСрСния энСргии Π² счСтах Π·Π° ΡΠ»Π΅ΠΊΡ‚Ρ€ΠΎΡΠ½Π΅Ρ€Π³ΠΈΡŽ являСтся ΠΊΠΈΠ»ΠΎΠ²Π°Ρ‚Ρ‚-час (ΠΊΠ’Ρ‚ Ρ‡), Ρ‡Ρ‚ΠΎ соотвСтствуСт ΡΠΎΠΎΡ‚Π½ΠΎΡˆΠ΅Π½ΠΈΡŽ E = Pt . Π‘Ρ‚ΠΎΠΈΠΌΠΎΡΡ‚ΡŒ эксплуатации элСктроприборов Π»Π΅Π³ΠΊΠΎ ΠΎΡ†Π΅Π½ΠΈΡ‚ΡŒ, Ссли Ρƒ вас Π΅ΡΡ‚ΡŒ Π½Π΅ΠΊΠΎΡ‚ΠΎΡ€ΠΎΠ΅ прСдставлСниС ΠΎΠ± ΠΈΡ… потрСбляСмой мощности Π² Π²Π°Ρ‚Ρ‚Π°Ρ… ΠΈΠ»ΠΈ ΠΊΠΈΠ»ΠΎΠ²Π°Ρ‚Ρ‚Π°Ρ…, Π²Ρ€Π΅ΠΌΠ΅Π½ΠΈ ΠΈΡ… Ρ€Π°Π±ΠΎΡ‚Ρ‹ Π² часах ΠΈ стоимости ΠΊΠΈΠ»ΠΎΠ²Π°Ρ‚Ρ‚-часа для вашСй элСктросСти. ΠšΠΈΠ»ΠΎΠ²Π°Ρ‚Ρ‚-часы, ΠΊΠ°ΠΊ ΠΈ всС Π΄Ρ€ΡƒΠ³ΠΈΠ΅ спСциализированныС Π΅Π΄ΠΈΠ½ΠΈΡ†Ρ‹ энСргии, Ρ‚Π°ΠΊΠΈΠ΅ ΠΊΠ°ΠΊ ΠΏΠΈΡ‰Π΅Π²Ρ‹Π΅ ΠΊΠ°Π»ΠΎΡ€ΠΈΠΈ, ΠΌΠΎΠΆΠ½ΠΎ ΠΏΡ€Π΅ΠΎΠ±Ρ€Π°Π·ΠΎΠ²Π°Ρ‚ΡŒ Π² Π΄ΠΆΠΎΡƒΠ»ΠΈ. Π’Ρ‹ ΠΌΠΎΠΆΠ΅Ρ‚Π΅ Π΄ΠΎΠΊΠ°Π·Π°Ρ‚ΡŒ сСбС, Ρ‡Ρ‚ΠΎ 1 ΠΊΠ’Ρ‚ β‹… Ρ‡ = 3.6 Γ— 10 6 Π”ΠΆ.

ΠŸΠΎΡ‚Ρ€Π΅Π±Π»ΡΠ΅ΠΌΠ°Ρ элСктричСская энСргия ( E ) ΠΌΠΎΠΆΠ΅Ρ‚ Π±Ρ‹Ρ‚ΡŒ ΡƒΠΌΠ΅Π½ΡŒΡˆΠ΅Π½Π° Π»ΠΈΠ±ΠΎ Π·Π° счСт сокращСния Π²Ρ€Π΅ΠΌΠ΅Π½ΠΈ использования, Π»ΠΈΠ±ΠΎ Π·Π° счСт сниТСния энСргопотрСблСния этого ΠΏΡ€ΠΈΠ±ΠΎΡ€Π° ΠΈΠ»ΠΈ приспособлСния. Π­Ρ‚ΠΎ Π½Π΅ Ρ‚ΠΎΠ»ΡŒΠΊΠΎ снизит ΡΡ‚ΠΎΠΈΠΌΠΎΡΡ‚ΡŒ, Π½ΠΎ ΠΈ снизит воздСйствиС Π½Π° ΠΎΠΊΡ€ΡƒΠΆΠ°ΡŽΡ‰ΡƒΡŽ срСду. Π£Π»ΡƒΡ‡ΡˆΠ΅Π½ΠΈΠ΅ освСщСния – ΠΎΠ΄ΠΈΠ½ ΠΈΠ· самых быстрых способов ΡΠ½ΠΈΠ·ΠΈΡ‚ΡŒ ΠΏΠΎΡ‚Ρ€Π΅Π±Π»Π΅Π½ΠΈΠ΅ элСктроэнСргии Π² Π΄ΠΎΠΌΠ΅ ΠΈΠ»ΠΈ Π½Π° Ρ€Π°Π±ΠΎΡ‚Π΅. Около 20% энСргии Π² Π΄ΠΎΠΌΠ΅ расходуСтся Π½Π° освСщСниС, Π² Ρ‚ΠΎ врСмя ΠΊΠ°ΠΊ Π² коммСрчСских учрСТдСниях эта Ρ†ΠΈΡ„Ρ€Π° приблиТаСтся ΠΊ 40%.ЀлуорСсцСнтныС Π»Π°ΠΌΠΏΡ‹ ΠΏΡ€ΠΈΠΌΠ΅Ρ€Π½ΠΎ Π² Ρ‡Π΅Ρ‚Ρ‹Ρ€Π΅ Ρ€Π°Π·Π° эффСктивнСС Π»Π°ΠΌΠΏ накаливания – это Π²Π΅Ρ€Π½ΠΎ ΠΊΠ°ΠΊ для Π΄Π»ΠΈΠ½Π½Ρ‹Ρ… Π»Π°ΠΌΠΏ, Ρ‚Π°ΠΊ ΠΈ для ΠΊΠΎΠΌΠΏΠ°ΠΊΡ‚Π½Ρ‹Ρ… Π»ΡŽΠΌΠΈΠ½Π΅ΡΡ†Π΅Π½Ρ‚Π½Ρ‹Ρ… Π»Π°ΠΌΠΏ (ΠšΠ›Π›). (Π‘ΠΌ. Рис. 1 (b).) Π’Π°ΠΊΠΈΠΌ ΠΎΠ±Ρ€Π°Π·ΠΎΠΌ, Π»Π°ΠΌΠΏΡƒ накаливания ΠΌΠΎΡ‰Π½ΠΎΡΡ‚ΡŒΡŽ 60 Π’Ρ‚ ΠΌΠΎΠΆΠ½ΠΎ Π·Π°ΠΌΠ΅Π½ΠΈΡ‚ΡŒ Π½Π° ΠšΠ›Π› ΠΌΠΎΡ‰Π½ΠΎΡΡ‚ΡŒΡŽ 15 Π’Ρ‚, которая ΠΈΠΌΠ΅Π΅Ρ‚ Ρ‚Π°ΠΊΡƒΡŽ ​​ТС ΡΡ€ΠΊΠΎΡΡ‚ΡŒ ΠΈ Ρ†Π²Π΅Ρ‚. ΠšΠ›Π› ΠΈΠΌΠ΅ΡŽΡ‚ ΠΈΠ·ΠΎΠ³Π½ΡƒΡ‚ΡƒΡŽ Ρ‚Ρ€ΡƒΠ±ΠΊΡƒ Π²Π½ΡƒΡ‚Ρ€ΠΈ ΡˆΠ°Ρ€Π° ΠΈΠ»ΠΈ ΡΠΏΠΈΡ€Π°Π»Π΅Π²ΠΈΠ΄Π½ΡƒΡŽ Ρ‚Ρ€ΡƒΠ±ΠΊΡƒ, ΡΠΎΠ΅Π΄ΠΈΠ½Π΅Π½Π½ΡƒΡŽ со стандартным Ρ€Π΅Π·ΡŒΠ±ΠΎΠ²Ρ‹ΠΌ основаниСм, подходящим для стандартных Ρ€ΠΎΠ·Π΅Ρ‚ΠΎΠΊ Π»Π°ΠΌΠΏΡ‹ накаливания. (Π’ послСдниС Π³ΠΎΠ΄Ρ‹ Π±Ρ‹Π»ΠΈ Ρ€Π΅ΡˆΠ΅Π½Ρ‹ исходныС ΠΏΡ€ΠΎΠ±Π»Π΅ΠΌΡ‹ с Ρ†Π²Π΅Ρ‚ΠΎΠΌ, ΠΌΠ΅Ρ€Ρ†Π°Π½ΠΈΠ΅ΠΌ, Ρ„ΠΎΡ€ΠΌΠΎΠΉ ΠΈ высокими Π½Π°Ρ‡Π°Π»ΡŒΠ½Ρ‹ΠΌΠΈ влоТСниями Π² ΠšΠ›Π›.) Π’Π΅ΠΏΠ»ΠΎΠΏΠ΅Ρ€Π΅Π΄Π°Ρ‡Π° ΠΎΡ‚ этих ΠšΠ›Π› мСньшС, ΠΈ ΠΎΠ½ΠΈ слуТат Π΄ΠΎ 10 Ρ€Π°Π· дольшС. Π’ ΡΠ»Π΅Π΄ΡƒΡŽΡ‰Π΅ΠΌ ΠΏΡ€ΠΈΠΌΠ΅Ρ€Π΅ рассматриваСтся Π²Π°ΠΆΠ½ΠΎΡΡ‚ΡŒ инвСстиций Π² Ρ‚Π°ΠΊΠΈΠ΅ Π»Π°ΠΌΠΏΡ‹. НовыС Π±Π΅Π»Ρ‹Π΅ свСтодиодныС Ρ„ΠΎΠ½Π°Ρ€ΠΈ (ΠΏΡ€Π΅Π΄ΡΡ‚Π°Π²Π»ΡΡŽΡ‰ΠΈΠ΅ собой Π³Ρ€ΡƒΠΏΠΏΡƒ Π½Π΅Π±ΠΎΠ»ΡŒΡˆΠΈΡ… свСтодиодных Π»Π°ΠΌΠΏΠΎΡ‡Π΅ΠΊ) Π΅Ρ‰Π΅ Π±ΠΎΠ»Π΅Π΅ эффСктивны (Π² Π΄Π²Π° Ρ€Π°Π·Π° большС, Ρ‡Π΅ΠΌ Ρƒ ΠšΠ›Π›) ΠΈ слуТат Π² 5 Ρ€Π°Π· дольшС, Ρ‡Π΅ΠΌ ΠšΠ›Π›. Однако ΠΈΡ… ΡΡ‚ΠΎΠΈΠΌΠΎΡΡ‚ΡŒ ΠΏΠΎ-ΠΏΡ€Π΅ΠΆΠ½Π΅ΠΌΡƒ высока.

УстановлСниС соСдинСний: энСргия, ΠΌΠΎΡ‰Π½ΠΎΡΡ‚ΡŒ ΠΈ врСмя

ΠžΡ‚Π½ΠΎΡˆΠ΅Π½ΠΈΠ΅ E = Pt ΠΌΠΎΠΆΠ΅Ρ‚ ΠΎΠΊΠ°Π·Π°Ρ‚ΡŒΡΡ ΠΏΠΎΠ»Π΅Π·Π½Ρ‹ΠΌ Π²ΠΎ ΠΌΠ½ΠΎΠ³ΠΈΡ… Ρ€Π°Π·Π»ΠΈΡ‡Π½Ρ‹Ρ… контСкстах.ЭнСргия, ΠΊΠΎΡ‚ΠΎΡ€ΡƒΡŽ вашС Ρ‚Π΅Π»ΠΎ ΠΈΡΠΏΠΎΠ»ΡŒΠ·ΡƒΠ΅Ρ‚ Π²ΠΎ врСмя ΡƒΠΏΡ€Π°ΠΆΠ½Π΅Π½ΠΈΠΉ, зависит, Π½Π°ΠΏΡ€ΠΈΠΌΠ΅Ρ€, ΠΎΡ‚ уровня мощности ΠΈ ΠΏΡ€ΠΎΠ΄ΠΎΠ»ΠΆΠΈΡ‚Π΅Π»ΡŒΠ½ΠΎΡΡ‚ΠΈ вашСй активности. Π‘Ρ‚Π΅ΠΏΠ΅Π½ΡŒ Π½Π°Π³Ρ€Π΅Π²Π° источника питания зависит ΠΎΡ‚ уровня мощности ΠΈ Π²Ρ€Π΅ΠΌΠ΅Π½ΠΈ Π΅Π΅ примСнСния. Π”Π°ΠΆΠ΅ Π΄ΠΎΠ·Π° облучСния рСнтгСновского изобраТСния зависит ΠΎΡ‚ мощности ΠΈ Π²Ρ€Π΅ΠΌΠ΅Π½ΠΈ воздСйствия.

ΠŸΡ€ΠΈΠΌΠ΅Ρ€ 2. РасчСт Ρ€Π΅Π½Ρ‚Π°Π±Π΅Π»ΡŒΠ½ΠΎΡΡ‚ΠΈ ΠΊΠΎΠΌΠΏΠ°ΠΊΡ‚Π½Ρ‹Ρ… Π»ΡŽΠΌΠΈΠ½Π΅ΡΡ†Π΅Π½Ρ‚Π½Ρ‹Ρ… Π»Π°ΠΌΠΏ (ΠšΠ›Π›)

Если ΡΡ‚ΠΎΠΈΠΌΠΎΡΡ‚ΡŒ элСктроэнСргии Π² вашСм Ρ€Π°ΠΉΠΎΠ½Π΅ составляСт 12 Ρ†Π΅Π½Ρ‚ΠΎΠ² Π·Π° ΠΊΠ’Ρ‚Ρ‡, ΠΊΠ°ΠΊΠΎΠ²Π° общая ΡΡ‚ΠΎΠΈΠΌΠΎΡΡ‚ΡŒ (ΠΊΠ°ΠΏΠΈΡ‚Π°Π»ΡŒΠ½Ρ‹Π΅ плюс эксплуатация) использования Π»Π°ΠΌΠΏΡ‹ накаливания ΠΌΠΎΡ‰Π½ΠΎΡΡ‚ΡŒΡŽ 60 Π’Ρ‚ Π² Ρ‚Π΅Ρ‡Π΅Π½ΠΈΠ΅ 1000 часов (срок слуТбы этой Π»Π°ΠΌΠΏΡ‹), Ссли ΡΡ‚ΠΎΠΈΠΌΠΎΡΡ‚ΡŒ Π»Π°ΠΌΠΏΡ‹ составляСт 25 Ρ†Π΅Π½Ρ‚ΠΎΠ²? (Π±) Если ΠΌΡ‹ Π·Π°ΠΌΠ΅Π½ΠΈΠΌ эту Π»Π°ΠΌΠΏΠΎΡ‡ΠΊΡƒ ΠΊΠΎΠΌΠΏΠ°ΠΊΡ‚Π½ΠΎΠΉ Π»ΡŽΠΌΠΈΠ½Π΅ΡΡ†Π΅Π½Ρ‚Π½ΠΎΠΉ Π»Π°ΠΌΠΏΠΎΠΉ, которая Π΄Π°Π΅Ρ‚ Ρ‚Π°ΠΊΠΎΠΉ ΠΆΠ΅ свСтовой ΠΏΠΎΡ‚ΠΎΠΊ, Π½ΠΎ составляСт Ρ‡Π΅Ρ‚Π²Π΅Ρ€Ρ‚ΡŒ мощности ΠΈ стоит 1 Π΄ΠΎΠ»Π»Π°Ρ€.50, Π½ΠΎ длится Π² 10 Ρ€Π°Π· дольшС (10 000 часов), ΠΊΠ°ΠΊΠΎΠ²Π° Π±ΡƒΠ΄Π΅Ρ‚ общая ΡΡ‚ΠΎΠΈΠΌΠΎΡΡ‚ΡŒ?

БтратСгия

Π§Ρ‚ΠΎΠ±Ρ‹ Π½Π°ΠΉΡ‚ΠΈ эксплуатационныС расходы, ΠΌΡ‹ сначала Π½Π°Ρ…ΠΎΠ΄ΠΈΠΌ ΠΈΡΠΏΠΎΠ»ΡŒΠ·ΠΎΠ²Π°Π½Π½ΡƒΡŽ ΡΠ½Π΅Ρ€Π³ΠΈΡŽ Π² ΠΊΠΈΠ»ΠΎΠ²Π°Ρ‚Ρ‚-часах, Π° Π·Π°Ρ‚Π΅ΠΌ ΡƒΠΌΠ½ΠΎΠΆΠ°Π΅ΠΌ Π΅Π΅ Π½Π° ΡΡ‚ΠΎΠΈΠΌΠΎΡΡ‚ΡŒ ΠΊΠΈΠ»ΠΎΠ²Π°Ρ‚Ρ‚-часа.

РСшСниС для (а)

ЭнСргия, ΠΈΡΠΏΠΎΠ»ΡŒΠ·ΡƒΠ΅ΠΌΠ°Ρ Π² ΠΊΠΈΠ»ΠΎΠ²Π°Ρ‚Ρ‚-часах, опрСдСляСтся ΠΏΡƒΡ‚Π΅ΠΌ Π²Π²ΠΎΠ΄Π° мощности ΠΈ Π²Ρ€Π΅ΠΌΠ΅Π½ΠΈ Π² Π²Ρ‹Ρ€Π°ΠΆΠ΅Π½ΠΈΠ΅ для энСргии:

E = Pt = (60 Π’Ρ‚) (1000 Ρ‡) = 60,000 Π’Ρ‚ β‹… Ρ‡

Π’ ΠΊΠΈΠ»ΠΎΠ²Π°Ρ‚Ρ‚-часах это

E = 60.0 ΠΊΠ’Ρ‚ β‹… Ρ‡.

БСйчас ΡΡ‚ΠΎΠΈΠΌΠΎΡΡ‚ΡŒ элСктроэнСргии

Π‘Ρ‚ΠΎΠΈΠΌΠΎΡΡ‚ΡŒ

= (60,0 ΠΊΠ’Ρ‚ Ρ‡) (0,12 Π΄ΠΎΠ»Π». БША / ΠΊΠ’Ρ‚ час) = 7,20 Π΄ΠΎΠ»Π». БША.

ΠžΠ±Ρ‰Π°Ρ ΡΡ‚ΠΎΠΈΠΌΠΎΡΡ‚ΡŒ составит 7,20 Π΄ΠΎΠ»Π»Π°Ρ€Π° Π·Π° 1000 часов (ΠΎΠΊΠΎΠ»ΠΎ ΠΏΠΎΠ»ΡƒΠ³ΠΎΠ΄Π° ΠΏΡ€ΠΈ 5 часах Π² дСнь).

РСшСниС для (b)

ΠŸΠΎΡΠΊΠΎΠ»ΡŒΠΊΡƒ CFL ΠΈΡΠΏΠΎΠ»ΡŒΠ·ΡƒΠ΅Ρ‚ Ρ‚ΠΎΠ»ΡŒΠΊΠΎ 15 Π’Ρ‚, Π° Π½Π΅ 60 Π’Ρ‚, ΡΡ‚ΠΎΠΈΠΌΠΎΡΡ‚ΡŒ элСктроэнСргии составит 7,20 Π΄ΠΎΠ»Π»Π°Ρ€Π° БША / 4 = 1,80 Π΄ΠΎΠ»Π»Π°Ρ€Π° БША. ΠšΠ›Π› прослуТит Π² 10 Ρ€Π°Π· дольшС, Ρ‡Π΅ΠΌ Π»Π°ΠΌΠΏΠ° накаливания, Ρ‚Π°ΠΊ Ρ‡Ρ‚ΠΎ инвСстиционныС Π·Π°Ρ‚Ρ€Π°Ρ‚Ρ‹ составят 1/10 стоимости Π»Π°ΠΌΠΏΡ‹ Π·Π° этот ΠΏΠ΅Ρ€ΠΈΠΎΠ΄ использования, ΠΈΠ»ΠΈ 0.1 (1,50 Π΄ΠΎΠ»Π»Π°Ρ€Π° БША) = 0,15 Π΄ΠΎΠ»Π»Π°Ρ€Π° БША. Π’Π°ΠΊΠΈΠΌ ΠΎΠ±Ρ€Π°Π·ΠΎΠΌ, общая ΡΡ‚ΠΎΠΈΠΌΠΎΡΡ‚ΡŒ 1000 часов составит 1,95 Π΄ΠΎΠ»Π»Π°Ρ€Π° БША.

ΠžΠ±ΡΡƒΠΆΠ΄Π΅Π½ΠΈΠ΅

Π‘Π»Π΅Π΄ΠΎΠ²Π°Ρ‚Π΅Π»ΡŒΠ½ΠΎ, использованиС ΠšΠ›Π› Π½Π°ΠΌΠ½ΠΎΠ³ΠΎ дСшСвлС, Π΄Π°ΠΆΠ΅ нСсмотря Π½Π° Ρ‚ΠΎ, Ρ‡Ρ‚ΠΎ ΠΏΠ΅Ρ€Π²ΠΎΠ½Π°Ρ‡Π°Π»ΡŒΠ½Ρ‹Π΅ влоТСния Π²Ρ‹ΡˆΠ΅. ΠŸΠΎΠ²Ρ‹ΡˆΠ΅Π½Π½Π°Ρ ΡΡ‚ΠΎΠΈΠΌΠΎΡΡ‚ΡŒ Ρ€Π°Π±ΠΎΡ‡Π΅ΠΉ силы, ΠΊΠΎΡ‚ΠΎΡ€ΡƒΡŽ бизнСс Π΄ΠΎΠ»ΠΆΠ΅Π½ Π²ΠΊΠ»ΡŽΡ‡Π°Ρ‚ΡŒ Π² сСбя для Π±ΠΎΠ»Π΅Π΅ частой Π·Π°ΠΌΠ΅Π½Ρ‹ Π»Π°ΠΌΠΏ накаливания, здСсь Π½Π΅ учитываСтся.

Π’Ρ‹ΠΏΠΎΠ»Π½Π΅Π½ΠΈΠ΅ ΠΏΠΎΠ΄ΠΊΠ»ΡŽΡ‡Π΅Π½ΠΈΠΉ: экспСримСнт Π½Π° вынос – инвСнтаризация использования элСктроэнСргии

1) Π‘ΠΎΡΡ‚Π°Π²ΡŒΡ‚Π΅ список номинальной мощности для ряда ΠΏΡ€ΠΈΠ±ΠΎΡ€ΠΎΠ² Π² вашСм Π΄ΠΎΠΌΠ΅ ΠΈΠ»ΠΈ ΠΊΠΎΠΌΠ½Π°Ρ‚Π΅.ΠžΠ±ΡŠΡΡΠ½ΠΈΡ‚Π΅, ΠΏΠΎΡ‡Π΅ΠΌΡƒ Ρ‡Ρ‚ΠΎ-Ρ‚ΠΎ Π²Ρ€ΠΎΠ΄Π΅ тостСра ΠΈΠΌΠ΅Π΅Ρ‚ Π±ΠΎΠ»Π΅Π΅ высокий Ρ€Π΅ΠΉΡ‚ΠΈΠ½Π³, Ρ‡Π΅ΠΌ Ρ†ΠΈΡ„Ρ€ΠΎΠ²Ρ‹Π΅ часы. ΠžΡ†Π΅Π½ΠΈΡ‚Π΅ ΡΠ½Π΅Ρ€Π³ΠΈΡŽ, ΠΏΠΎΡ‚Ρ€Π΅Π±Π»ΡΠ΅ΠΌΡƒΡŽ этими ΠΏΡ€ΠΈΠ±ΠΎΡ€Π°ΠΌΠΈ Π² срСднСм Π·Π° дСнь (оцСнивая врСмя ΠΈΡ… использования). НСкоторыС ΠΏΡ€ΠΈΠ±ΠΎΡ€Ρ‹ ΠΌΠΎΠ³ΡƒΡ‚ ΡƒΠΊΠ°Π·Ρ‹Π²Π°Ρ‚ΡŒ Ρ‚ΠΎΠ»ΡŒΠΊΠΎ Ρ€Π°Π±ΠΎΡ‡ΠΈΠΉ Ρ‚ΠΎΠΊ. Если Π±Ρ‹Ρ‚ΠΎΠ²ΠΎΠ΅ напряТСниС составляСт 120 Π’, Ρ‚ΠΎΠ³Π΄Π° ΠΈΡΠΏΠΎΠ»ΡŒΠ·ΡƒΠΉΡ‚Π΅ P = IV . 2) ΠŸΡ€ΠΎΠ²Π΅Ρ€ΡŒΡ‚Π΅ ΠΎΠ±Ρ‰ΡƒΡŽ ΠΌΠΎΡ‰Π½ΠΎΡΡ‚ΡŒ, ΠΈΡΠΏΠΎΠ»ΡŒΠ·ΡƒΠ΅ΠΌΡƒΡŽ Π² Ρ‚ΡƒΠ°Π»Π΅Ρ‚Π°Ρ… Π½Π° этаТС ΠΈΠ»ΠΈ Π² Π·Π΄Π°Π½ΠΈΠΈ вашСй ΡˆΠΊΠΎΠ»Ρ‹. (Π’ΠΎΠ·ΠΌΠΎΠΆΠ½ΠΎ, Π²Π°ΠΌ придСтся ΠΏΡ€Π΅Π΄ΠΏΠΎΠ»ΠΎΠΆΠΈΡ‚ΡŒ, Ρ‡Ρ‚ΠΎ ΠΈΡΠΏΠΎΠ»ΡŒΠ·ΡƒΠ΅ΠΌΡ‹Π΅ Π΄Π»ΠΈΠ½Π½Ρ‹Π΅ Π»ΡŽΠΌΠΈΠ½Π΅ΡΡ†Π΅Π½Ρ‚Π½Ρ‹Π΅ Π»Π°ΠΌΠΏΡ‹ рассчитаны Π½Π° 32 Π’Ρ‚.) ΠŸΡ€Π΅Π΄ΠΏΠΎΠ»ΠΎΠΆΠΈΠΌ, Ρ‡Ρ‚ΠΎ Π·Π΄Π°Π½ΠΈΠ΅ Π±Ρ‹Π»ΠΎ Π·Π°ΠΊΡ€Ρ‹Ρ‚ΠΎ всС Π²Ρ‹Ρ…ΠΎΠ΄Π½Ρ‹Π΅, ΠΈ Ρ‡Ρ‚ΠΎ эти ΠΎΠ³Π½ΠΈ Π±Ρ‹Π»ΠΈ оставлСны Π²ΠΊΠ»ΡŽΡ‡Π΅Π½Π½Ρ‹ΠΌΠΈ с 6 часов Π²Π΅Ρ‡Π΅Ρ€Π°.{2} R \\ [/ латСкс].

  • ЭнСргия, ΠΈΡΠΏΠΎΠ»ΡŒΠ·ΡƒΠ΅ΠΌΠ°Ρ устройством ΠΌΠΎΡ‰Π½ΠΎΡΡ‚ΡŒΡŽ P Π·Π° врСмя t , составляСт E = Pt .

ΠšΠΎΠ½Ρ†Π΅ΠΏΡ‚ΡƒΠ°Π»ΡŒΠ½Ρ‹Π΅ вопросы

1. ΠŸΠΎΡ‡Π΅ΠΌΡƒ Π»Π°ΠΌΠΏΡ‹ накаливания Ρ‚ΡƒΡΠΊΠ½Π΅ΡŽΡ‚ Π² ΠΊΠΎΠ½Ρ†Π΅ ΠΆΠΈΠ·Π½ΠΈ, особСнно Π½Π΅Π·Π°Π΄ΠΎΠ»Π³ΠΎ Π΄ΠΎ Ρ‚ΠΎΠ³ΠΎ, ΠΊΠ°ΠΊ ΠΈΡ… Π½ΠΈΡ‚ΠΈ оборвутся?

ΠœΠΎΡ‰Π½ΠΎΡΡ‚ΡŒ, рассСиваСмая Π½Π° рСзисторС, Ρ€Π°Π²Π½Π° P = V 2 / R , Ρ‡Ρ‚ΠΎ ΠΎΠ·Π½Π°Ρ‡Π°Π΅Ρ‚, Ρ‡Ρ‚ΠΎ ΠΌΠΎΡ‰Π½ΠΎΡΡ‚ΡŒ ΡƒΠΌΠ΅Π½ΡŒΡˆΠ°Π΅Ρ‚ΡΡ ΠΏΡ€ΠΈ ΡƒΠ²Π΅Π»ΠΈΡ‡Π΅Π½ΠΈΠΈ сопротивлСния. Π’Π΅ΠΌ Π½Π΅ ΠΌΠ΅Π½Π΅Π΅, эта ΠΌΠΎΡ‰Π½ΠΎΡΡ‚ΡŒ Ρ‚Π°ΠΊΠΆΠ΅ опрСдСляСтся ΡΠΎΠΎΡ‚Π½ΠΎΡˆΠ΅Π½ΠΈΠ΅ΠΌ P = I 2 R , Ρ‡Ρ‚ΠΎ ΠΎΠ·Π½Π°Ρ‡Π°Π΅Ρ‚, Ρ‡Ρ‚ΠΎ ΠΌΠΎΡ‰Π½ΠΎΡΡ‚ΡŒ увСличиваСтся ΠΏΡ€ΠΈ ΡƒΠ²Π΅Π»ΠΈΡ‡Π΅Π½ΠΈΠΈ сопротивлСния.ΠžΠ±ΡŠΡΡΠ½ΠΈΡ‚Π΅, ΠΏΠΎΡ‡Π΅ΠΌΡƒ здСсь Π½Π΅Ρ‚ противорСчия.

Π—Π°Π΄Π°Ρ‡ΠΈ ΠΈ упраТнСния

1. Какова ΠΌΠΎΡ‰Π½ΠΎΡΡ‚ΡŒ разряда ΠΌΠΎΠ»Π½ΠΈΠΈ 1,00 Γ— 10 2 ΠœΠ’ ΠΏΡ€ΠΈ Ρ‚ΠΎΠΊΠ΅ 2,00 Γ— 10 4 A ?

2. Какая ΠΌΠΎΡ‰Π½ΠΎΡΡ‚ΡŒ подаСтся Π½Π° стартСр большого Π³Ρ€ΡƒΠ·ΠΎΠ²ΠΈΠΊΠ°, ΠΊΠΎΡ‚ΠΎΡ€Ρ‹ΠΉ потрСбляСт 250 А Ρ‚ΠΎΠΊΠ° ΠΎΡ‚ аккумуляторной Π±Π°Ρ‚Π°Ρ€Π΅ΠΈ 24,0 Π’?

3. Заряд Π² 4,00 Кл ΠΏΡ€ΠΎΡ…ΠΎΠ΄ΠΈΡ‚ Ρ‡Π΅Ρ€Π΅Π· солнСчныС элСмСнты ΠΊΠ°Ρ€ΠΌΠ°Π½Π½ΠΎΠ³ΠΎ ΠΊΠ°Π»ΡŒΠΊΡƒΠ»ΡΡ‚ΠΎΡ€Π° Π·Π° 4,00 часа. Какова выходная ΠΌΠΎΡ‰Π½ΠΎΡΡ‚ΡŒ, Ссли Π²Ρ‹Ρ…ΠΎΠ΄Π½ΠΎΠ΅ напряТСниС вычислитСля Ρ€Π°Π²Π½ΠΎ 3.00 Π’? (Π‘ΠΌ. Рисунок 2.)

Рис. 2. Полоса солнСчных элСмСнтов прямо Π½Π°Π΄ клавишами этого ΠΊΠ°Π»ΡŒΠΊΡƒΠ»ΡΡ‚ΠΎΡ€Π° ΠΏΡ€Π΅ΠΎΠ±Ρ€Π°Π·ΡƒΠ΅Ρ‚ свСт Π² элСктричСство для удовлСтворСния своих потрСбностСй Π² энСргии. (Π˜ΡΡ‚ΠΎΡ‡Π½ΠΈΠΊ: Π­Π²Π°Π½-Амос, Wikimedia Commons)

4. Бколько Π²Π°Ρ‚Ρ‚ ΠΏΡ€ΠΎΡ…ΠΎΠ΄ΠΈΡ‚ Ρ‡Π΅Ρ€Π΅Π· Ρ„ΠΎΠ½Π°Ρ€ΠΈΠΊ с 6,00 Γ— 10 2 Π·Π° 0,500 Ρ‡ использования, Ссли Π΅Π³ΠΎ напряТСниС составляСт 3,00 Π’?

5. НайдитС ΠΌΠΎΡ‰Π½ΠΎΡΡ‚ΡŒ, Ρ€Π°ΡΡΠ΅ΠΈΠ²Π°Π΅ΠΌΡƒΡŽ ΠΊΠ°ΠΆΠ΄Ρ‹ΠΌ ΠΈΠ· этих ΡƒΠ΄Π»ΠΈΠ½ΠΈΡ‚Π΅Π»Π΅ΠΉ: (a) ΡƒΠ΄Π»ΠΈΠ½ΠΈΡ‚Π΅Π»ΡŒΠ½Ρ‹ΠΉ ΡˆΠ½ΡƒΡ€ с сопротивлСниСм 0,0600 Ом, Ρ‡Π΅Ρ€Π΅Π· ΠΊΠΎΡ‚ΠΎΡ€Ρ‹ΠΉ 5.00 А Ρ‚Π΅Ρ‡Π΅Ρ‚; (Π±) Π±ΠΎΠ»Π΅Π΅ Π΄Π΅ΡˆΠ΅Π²Ρ‹ΠΉ ΡˆΠ½ΡƒΡ€ с Π±ΠΎΠ»Π΅Π΅ Ρ‚ΠΎΠ½ΠΊΠΈΠΌ ΠΏΡ€ΠΎΠ²ΠΎΠ΄ΠΎΠΌ ΠΈ сопротивлСниСм 0,300 Ом.

6. Π£Π±Π΅Π΄ΠΈΡ‚Π΅ΡΡŒ, Ρ‡Ρ‚ΠΎ Π΅Π΄ΠΈΠ½ΠΈΡ†Π΅ΠΉ измСрСния Π²ΠΎΠ»ΡŒΡ‚-Π°ΠΌΠΏΠ΅Ρ€ ΡΠ²Π»ΡΡŽΡ‚ΡΡ Π²Π°Ρ‚Ρ‚Ρ‹, ΠΊΠ°ΠΊ слСдуСт ΠΈΠ· уравнСния P = IV .

7. ΠŸΠΎΠΊΠ°ΠΆΠΈΡ‚Π΅, Ρ‡Ρ‚ΠΎ Π΅Π΄ΠΈΠ½ΠΈΡ†Ρ‹ 1V 2 / Ξ© = 1W, ΠΊΠ°ΠΊ слСдуСт ΠΈΠ· уравнСния P = V 2 / R .

8. ΠŸΠΎΠΊΠ°ΠΆΠΈΡ‚Π΅, Ρ‡Ρ‚ΠΎ Π΅Π΄ΠΈΠ½ΠΈΡ†Ρ‹ 1 A 2 Ξ© = 1 Π’Ρ‚, ΠΊΠ°ΠΊ слСдуСт ΠΈΠ· уравнСния P = I 2 R .

9. ΠŸΡ€ΠΎΠ²Π΅Ρ€ΡŒΡ‚Π΅ эквивалСнт Π΅Π΄ΠΈΠ½ΠΈΡ† энСргии: 1 ΠΊΠ’Ρ‚ Ρ‡ = 3,60 Γ— 10 6 Π”ΠΆ.

10. Π­Π»Π΅ΠΊΡ‚Ρ€ΠΎΠ½Ρ‹ Π² рСнтгСновской Ρ‚Ρ€ΡƒΠ±ΠΊΠ΅ ΡƒΡΠΊΠΎΡ€ΡΡŽΡ‚ΡΡ Π΄ΠΎ 1,00 Γ— 10 2 ΠΊΠ’ ΠΈ Π½Π°ΠΏΡ€Π°Π²Π»ΡΡŽΡ‚ΡΡ ΠΊ Ρ†Π΅Π»ΠΈ для получСния рСнтгСновских Π»ΡƒΡ‡Π΅ΠΉ. ВычислитС ΠΌΠΎΡ‰Π½ΠΎΡΡ‚ΡŒ элСктронного Π»ΡƒΡ‡Π° Π² этой Ρ‚Ρ€ΡƒΠ±ΠΊΠ΅, Ссли ΠΎΠ½Π° ΠΈΠΌΠ΅Π΅Ρ‚ Ρ‚ΠΎΠΊ 15,0 мА.

11. ЭлСктричСский Π²ΠΎΠ΄ΠΎΠ½Π°Π³Ρ€Π΅Π²Π°Ρ‚Π΅Π»ΡŒ потрСбляСт 5,00 ΠΊΠ’Ρ‚ Π½Π° 2,00 часа Π² сутки. Какова ΡΡ‚ΠΎΠΈΠΌΠΎΡΡ‚ΡŒ Π΅Π³ΠΎ эксплуатации Π² Ρ‚Π΅Ρ‡Π΅Π½ΠΈΠ΅ ΠΎΠ΄Π½ΠΎΠ³ΠΎ Π³ΠΎΠ΄Π°, Ссли элСктроэнСргия стоит 12,0 Ρ†Π΅Π½Ρ‚ΠΎΠ² / ΠΊΠ’Ρ‚ Β· Ρ‡? Π‘ΠΌ. Рисунок 3.

Рисунок 3. Π’ΠΎΠ΄ΠΎΠ½Π°Π³Ρ€Π΅Π²Π°Ρ‚Π΅Π»ΡŒ элСктричСский ΠΏΠΎ запросу. Π’Π΅ΠΏΠ»ΠΎ Π² Π²ΠΎΠ΄Ρƒ подаСтся Ρ‚ΠΎΠ»ΡŒΠΊΠΎ ΠΏΡ€ΠΈ нСобходимости. (ΠΊΡ€Π΅Π΄ΠΈΡ‚: aviddavid, Flickr)

12. Бколько элСктроэнСргии Π½Π΅ΠΎΠ±Ρ…ΠΎΠ΄ΠΈΠΌΠΎ для тостСра с тостСром ΠΌΠΎΡ‰Π½ΠΎΡΡ‚ΡŒΡŽ 1200 Π’Ρ‚ (врСмя приготовлСния = 1 ΠΌΠΈΠ½ΡƒΡ‚Π°)? Бколько это стоит ΠΏΡ€ΠΈ 9,0 Ρ†Π΅Π½Ρ‚Π° / ΠΊΠ’Ρ‚ Β· Ρ‡?

13. Какова Π±ΡƒΠ΄Π΅Ρ‚ максимальная ΡΡ‚ΠΎΠΈΠΌΠΎΡΡ‚ΡŒ ΠšΠ›Π›, Ссли общая ΡΡ‚ΠΎΠΈΠΌΠΎΡΡ‚ΡŒ (капиталовлоТСния плюс эксплуатация) Π±ΡƒΠ΄Π΅Ρ‚ ΠΎΠ΄ΠΈΠ½Π°ΠΊΠΎΠ²ΠΎΠΉ ΠΊΠ°ΠΊ для ΠšΠ›Π›, Ρ‚Π°ΠΊ ΠΈ для Π»Π°ΠΌΠΏ накаливания ΠΌΠΎΡ‰Π½ΠΎΡΡ‚ΡŒΡŽ 60 Π’Ρ‚? ΠŸΡ€Π΅Π΄ΠΏΠΎΠ»ΠΎΠΆΠΈΠΌ, Ρ‡Ρ‚ΠΎ ΡΡ‚ΠΎΠΈΠΌΠΎΡΡ‚ΡŒ Π»Π°ΠΌΠΏΡ‹ накаливания составляСт 25 Ρ†Π΅Π½Ρ‚ΠΎΠ², Π° элСктричСство стоит 10 Ρ†Π΅Π½Ρ‚ΠΎΠ² / ΠΊΠ’Ρ‚Ρ‡.РассчитайтС ΡΡ‚ΠΎΠΈΠΌΠΎΡΡ‚ΡŒ 1000 часов, ΠΊΠ°ΠΊ Π² ΠΏΡ€ΠΈΠΌΠ΅Ρ€Π΅ с ΠšΠ›Π› ΠΏΠΎ Ρ€Π΅Π½Ρ‚Π°Π±Π΅Π»ΡŒΠ½ΠΎΡΡ‚ΠΈ.

14. НСкоторыС ΠΌΠΎΠ΄Π΅Π»ΠΈ старых Π°Π²Ρ‚ΠΎΠΌΠΎΠ±ΠΈΠ»Π΅ΠΉ ΠΈΠΌΠ΅ΡŽΡ‚ ΡΠ»Π΅ΠΊΡ‚Ρ€ΠΈΡ‡Π΅ΡΠΊΡƒΡŽ систСму напряТСниСм 6,00 Π’. Π°) Каково сопротивлСниС горячСму свСту Ρƒ Ρ„Π°Ρ€Ρ‹ ΠΌΠΎΡ‰Π½ΠΎΡΡ‚ΡŒΡŽ 30,0 Π’Ρ‚ Π² Ρ‚Π°ΠΊΠΎΠΉ машинС? Π±) Какой Ρ‚ΠΎΠΊ Ρ‚Π΅Ρ‡Π΅Ρ‚ Ρ‡Π΅Ρ€Π΅Π· Π½Π΅Π³ΠΎ?

15. Π©Π΅Π»ΠΎΡ‡Π½Ρ‹Π΅ Π±Π°Ρ‚Π°Ρ€Π΅ΠΈ ΠΈΠΌΠ΅ΡŽΡ‚ Ρ‚ΠΎ прСимущСство, Ρ‡Ρ‚ΠΎ ΠΎΠ½ΠΈ Π²Ρ‹Π΄Π°ΡŽΡ‚ постоянноС напряТСниС ΠΏΠΎΡ‡Ρ‚ΠΈ Π΄ΠΎ ΠΊΠΎΠ½Ρ†Π° своСго срока слуТбы. Как Π΄ΠΎΠ»Π³ΠΎ щСлочная батарСя с Π½ΠΎΠΌΠΈΠ½Π°Π»ΠΎΠΌ 1,00 А Β· Ρ‡ ΠΈ 1,58 Π’ Π±ΡƒΠ΄Π΅Ρ‚ ΠΏΠΎΠ΄Π΄Π΅Ρ€ΠΆΠΈΠ²Π°Ρ‚ΡŒ Π³ΠΎΡ€Π΅Π½ΠΈΠ΅ Π»Π°ΠΌΠΏΡ‹ Ρ„ΠΎΠ½Π°Ρ€ΠΈΠΊΠ° ΠΌΠΎΡ‰Π½ΠΎΡΡ‚ΡŒΡŽ 1,00 Π’Ρ‚?

16.ΠŸΡ€ΠΈΠΆΠΈΠ³Π°Ρ‚Π΅Π»ΡŒ, ΠΈΡΠΏΠΎΠ»ΡŒΠ·ΡƒΠ΅ΠΌΡ‹ΠΉ для остановки кровотСчСния Π² Ρ…ΠΈΡ€ΡƒΡ€Π³ΠΈΠΈ, Π²Ρ‹Π΄Π°Π΅Ρ‚ 2,00 мА ΠΏΡ€ΠΈ 15,0 ΠΊΠ’. Π°) Какова Π΅Π³ΠΎ выходная ΠΌΠΎΡ‰Π½ΠΎΡΡ‚ΡŒ? Π±) КакоС сопротивлСниС ΠΏΡƒΡ‚ΠΈ?

17. Π’ срСднСм Ρ‚Π΅Π»Π΅Π²ΠΈΠ·ΠΎΡ€ Ρ€Π°Π±ΠΎΡ‚Π°Π΅Ρ‚ 6 часов Π² дСнь. ΠžΡ†Π΅Π½ΠΈΡ‚Π΅ Π΅ΠΆΠ΅Π³ΠΎΠ΄Π½Ρ‹Π΅ Π·Π°Ρ‚Ρ€Π°Ρ‚Ρ‹ Π½Π° ΡΠ»Π΅ΠΊΡ‚Ρ€ΠΎΡΠ½Π΅Ρ€Π³ΠΈΡŽ для Ρ€Π°Π±ΠΎΡ‚Ρ‹ 100 ΠΌΠΈΠ»Π»ΠΈΠΎΠ½ΠΎΠ² Ρ‚Π΅Π»Π΅Π²ΠΈΠ·ΠΎΡ€ΠΎΠ², прСдполагая, Ρ‡Ρ‚ΠΎ ΠΈΡ… потрСбляСмая ΠΌΠΎΡ‰Π½ΠΎΡΡ‚ΡŒ составляСт Π² срСднСм 150 Π’Ρ‚, Π° ΡΡ‚ΠΎΠΈΠΌΠΎΡΡ‚ΡŒ элСктроэнСргии составляСт Π² срСднСм 12,0 Ρ†Π΅Π½Ρ‚ΠΎΠ² / ΠΊΠ’Ρ‚ Β· Ρ‡.

18. Бтарая Π»Π°ΠΌΠΏΠΎΡ‡ΠΊΠ° потрСбляСт всСго 50,0 Π’Ρ‚, Π° Π½Π΅ 60,0 Π’Ρ‚ ΠΈΠ·-Π·Π° истончСния Π΅Π΅ Π½ΠΈΡ‚ΠΈ Π·Π° счСт испарСния.Π’ΠΎ сколько Ρ€Π°Π· ΡƒΠΌΠ΅Π½ΡŒΡˆΠ°Π΅Ρ‚ΡΡ Π΅Π³ΠΎ Π΄ΠΈΠ°ΠΌΠ΅Ρ‚Ρ€ ΠΏΡ€ΠΈ условии Ρ€Π°Π²Π½ΠΎΠΌΠ΅Ρ€Π½ΠΎΠ³ΠΎ утонСния ΠΏΠΎ Π΄Π»ΠΈΠ½Π΅? НС ΠΎΠ±Ρ€Π°Ρ‰Π°ΠΉΡ‚Π΅ внимания Π½Π° Π»ΡŽΠ±Ρ‹Π΅ эффСкты, Π²Ρ‹Π·Π²Π°Π½Π½Ρ‹Π΅ ΠΏΠ΅Ρ€Π΅ΠΏΠ°Π΄Π°ΠΌΠΈ Ρ‚Π΅ΠΌΠΏΠ΅Ρ€Π°Ρ‚ΡƒΡ€.

МСдная ΠΏΡ€ΠΎΠ²ΠΎΠ»ΠΎΠΊΠ° ΠΊΠ°Π»ΠΈΠ±Ρ€Π° 19. 00 ΠΈΠΌΠ΅Π΅Ρ‚ Π΄ΠΈΠ°ΠΌΠ΅Ρ‚Ρ€ 9,266 ΠΌΠΌ. ВычислитС ΠΏΠΎΡ‚Π΅Ρ€ΠΈ мощности Π² ΠΊΠΈΠ»ΠΎΠΌΠ΅Ρ‚Ρ€Π΅ Ρ‚Π°ΠΊΠΎΠ³ΠΎ ΠΏΡ€ΠΎΠ²ΠΎΠ΄Π°, ΠΊΠΎΠ³Π΄Π° ΠΎΠ½ пропускаСт 1,00 Γ— 10 2 A.

Π₯ΠΎΠ»ΠΎΠ΄Π½Ρ‹Π΅ испаритСли ΠΏΡ€ΠΎΠΏΡƒΡΠΊΠ°ΡŽΡ‚ Ρ‚ΠΎΠΊ Ρ‡Π΅Ρ€Π΅Π· Π²ΠΎΠ΄Ρƒ, испаряя Π΅Π΅ ΠΏΡ€ΠΈ нСбольшом ΠΏΠΎΠ²Ρ‹ΡˆΠ΅Π½ΠΈΠΈ Ρ‚Π΅ΠΌΠΏΠ΅Ρ€Π°Ρ‚ΡƒΡ€Ρ‹. Одно Ρ‚Π°ΠΊΠΎΠ΅ домашнСС устройство рассчитано Π½Π° 3,50 А ΠΈ ΠΈΡΠΏΠΎΠ»ΡŒΠ·ΡƒΠ΅Ρ‚ 120 Π’ ΠΏΠ΅Ρ€Π΅ΠΌΠ΅Π½Π½ΠΎΠ³ΠΎ Ρ‚ΠΎΠΊΠ° с ΡΡ„Ρ„Π΅ΠΊΡ‚ΠΈΠ²Π½ΠΎΡΡ‚ΡŒΡŽ 95,0%.Π°) Какова ΡΠΊΠΎΡ€ΠΎΡΡ‚ΡŒ испарСния Π² Π³Ρ€Π°ΠΌΠΌΠ°Ρ… Π² ΠΌΠΈΠ½ΡƒΡ‚Ρƒ? (b) Бколько Π²ΠΎΠ΄Ρ‹ Π½ΡƒΠΆΠ½ΠΎ Π½Π°Π»ΠΈΡ‚ΡŒ Π² ΠΈΡΠΏΠ°Ρ€ΠΈΡ‚Π΅Π»ΡŒ Π·Π° 8 часов Ρ€Π°Π±ΠΎΡ‚Ρ‹ Π² Π½ΠΎΡ‡Π½ΠΎΠ΅ врСмя? (Π‘ΠΌ. Рисунок 4.)

Рис. 4. Π­Ρ‚ΠΎΡ‚ Ρ…ΠΎΠ»ΠΎΠ΄Π½Ρ‹ΠΉ ΠΈΡΠΏΠ°Ρ€ΠΈΡ‚Π΅Π»ΡŒ пропускаСт Ρ‚ΠΎΠΊ нСпосрСдствСнно Ρ‡Π΅Ρ€Π΅Π· Π²ΠΎΠ΄Ρƒ, испаряя Π΅Π΅ Π½Π°ΠΏΡ€ΡΠΌΡƒΡŽ с ΠΎΡ‚Π½ΠΎΡΠΈΡ‚Π΅Π»ΡŒΠ½ΠΎ нСбольшим ΠΏΠΎΠ²Ρ‹ΡˆΠ΅Π½ΠΈΠ΅ΠΌ Ρ‚Π΅ΠΌΠΏΠ΅Ρ€Π°Ρ‚ΡƒΡ€Ρ‹.

21. Integrated Concepts (a) Какая энСргия рассСиваСтся разрядом ΠΌΠΎΠ»Π½ΠΈΠΈ с Ρ‚ΠΎΠΊΠΎΠΌ 20 000 А, напряТСниСм 1,00 Γ— 10 2 ΠœΠ’ ΠΈ Π΄Π»ΠΈΠ½ΠΎΠΉ 1.00 мс? (Π±) ΠšΠ°ΠΊΡƒΡŽ массу дрСвСсного сока ΠΌΠΎΠΆΠ½ΠΎ Π±Ρ‹Π»ΠΎ Π±Ρ‹ ΠΏΠΎΠ΄Π½ΡΡ‚ΡŒ с 18ΒΊC Π΄ΠΎ Ρ‚ΠΎΡ‡ΠΊΠΈ кипСния, Π° Π·Π°Ρ‚Π΅ΠΌ ΠΈΡΠΏΠ°Ρ€ΠΈΡ‚ΡŒ Π·Π° счСт этой энСргии, Ссли ΠΏΡ€Π΅Π΄ΠΏΠΎΠ»ΠΎΠΆΠΈΡ‚ΡŒ, Ρ‡Ρ‚ΠΎ сок ΠΈΠΌΠ΅Π΅Ρ‚ Ρ‚Π΅ ΠΆΠ΅ Ρ‚Π΅ΠΏΠ»ΠΎΠ²Ρ‹Π΅ характСристики, Ρ‡Ρ‚ΠΎ ΠΈ Π²ΠΎΠ΄Π°?

22. Integrated Concepts Какой Ρ‚ΠΎΠΊ Π΄ΠΎΠ»ΠΆΠ΅Π½ Π²Ρ‹Ρ€Π°Π±Π°Ρ‚Ρ‹Π²Π°Ρ‚ΡŒ ΠΏΠΎΠ΄ΠΎΠ³Ρ€Π΅Π²Π°Ρ‚Π΅Π»ΡŒ Π±ΡƒΡ‚Ρ‹Π»ΠΎΡ‡Π΅ΠΊ с ΠΏΠΈΡ‚Π°Π½ΠΈΠ΅ΠΌ ΠΎΡ‚ Π±Π°Ρ‚Π°Ρ€Π΅ΠΉ 12,0 Π’, Ρ‡Ρ‚ΠΎΠ±Ρ‹ Π½Π°Π³Ρ€Π΅Ρ‚ΡŒ 75,0 Π³ стСкла, 250 Π³ дСтской смСси ΠΈ 3,00 Γ— 10 2 алюминия ΠΎΡ‚ 20 Β° C Π΄ΠΎ 90ΒΊ Π·Π° 5,00 ΠΌΠΈΠ½?

23. Integrated Concepts Бколько Π²Ρ€Π΅ΠΌΠ΅Π½ΠΈ трСбуСтся хирургичСскому ΠΏΡ€ΠΈΠΆΠΈΠ³Π°Ρ‚Π΅Π»ΡŽ для ΠΏΠΎΠ²Ρ‹ΡˆΠ΅Π½ΠΈΡ Ρ‚Π΅ΠΌΠΏΠ΅Ρ€Π°Ρ‚ΡƒΡ€Ρ‹ 1.00 Π³ Ρ‚ΠΊΠ°Π½ΠΈ ΠΎΡ‚ 37ΒΊ Π΄ΠΎ 100, Π° Π·Π°Ρ‚Π΅ΠΌ закипятитС 0,500 Π³ Π²ΠΎΠ΄Ρ‹, Ссли ΠΎΠ½Π° Π²Ρ‹Π΄Π°Π΅Ρ‚ 2,00 мА ΠΏΡ€ΠΈ 15,0 ΠΊΠ’? НС ΠΎΠ±Ρ€Π°Ρ‰Π°ΠΉΡ‚Π΅ внимания Π½Π° ΠΏΠ΅Ρ€Π΅Π΄Π°Ρ‡Ρƒ Ρ‚Π΅ΠΏΠ»Π° Π² ΠΎΠΊΡ€ΡƒΠΆΠ°ΡŽΡ‰ΡƒΡŽ срСду.

24. Integrated Concepts ГидроэлСктричСскиС Π³Π΅Π½Π΅Ρ€Π°Ρ‚ΠΎΡ€Ρ‹ (см. Рисунок 5) Π½Π° ΠΏΠ»ΠΎΡ‚ΠΈΠ½Π΅ Π“ΡƒΠ²Π΅Ρ€Π° Π²Ρ‹Ρ€Π°Π±Π°Ρ‚Ρ‹Π²Π°ΡŽΡ‚ ΠΌΠ°ΠΊΡΠΈΠΌΠ°Π»ΡŒΠ½Ρ‹ΠΉ Ρ‚ΠΎΠΊ 8,00 Γ— 10 3 A ΠΏΡ€ΠΈ 250 ΠΊΠ’. Π°) Какая выходная ΠΌΠΎΡ‰Π½ΠΎΡΡ‚ΡŒ? (b) Π’ΠΎΠ΄Π°, ΠΏΠΈΡ‚Π°ΡŽΡ‰Π°Ρ Π³Π΅Π½Π΅Ρ€Π°Ρ‚ΠΎΡ€Ρ‹, Π²Ρ…ΠΎΠ΄ΠΈΡ‚ ΠΈ ΠΏΠΎΠΊΠΈΠ΄Π°Π΅Ρ‚ систСму с Π½ΠΈΠ·ΠΊΠΎΠΉ ΡΠΊΠΎΡ€ΠΎΡΡ‚ΡŒΡŽ (Ρ‚Π°ΠΊΠΈΠΌ ΠΎΠ±Ρ€Π°Π·ΠΎΠΌ, Π΅Π΅ кинСтичСская энСргия Π½Π΅ измСняСтся), Π½ΠΎ тСряСт 160 ΠΌ Π² высотС.Бколько кубичСских ΠΌΠ΅Ρ‚Ρ€ΠΎΠ² Π² сСкунду Π½Π΅ΠΎΠ±Ρ…ΠΎΠ΄ΠΈΠΌΠΎ ΠΏΡ€ΠΈ ΠšΠŸΠ” 85,0%?

Рисунок 5. ГидроэлСктричСскиС Π³Π΅Π½Π΅Ρ€Π°Ρ‚ΠΎΡ€Ρ‹ Π½Π° ΠΏΠ»ΠΎΡ‚ΠΈΠ½Π΅ Π“ΡƒΠ²Π΅Ρ€Π°. (ΠΊΡ€Π΅Π΄ΠΈΡ‚: Π”ΠΆΠΎΠ½ Π‘Π°Π»Π»ΠΈΠ²Π°Π½)

25. Integrated Concepts (a) Π˜ΡΡ…ΠΎΠ΄Ρ ΠΈΠ· 95,0% эффСктивности прСобразования элСктроэнСргии Π΄Π²ΠΈΠ³Π°Ρ‚Π΅Π»Π΅ΠΌ, ΠΊΠ°ΠΊΠΎΠΉ Ρ‚ΠΎΠΊ Π΄ΠΎΠ»ΠΆΠ½Ρ‹ ΠΎΠ±Π΅ΡΠΏΠ΅Ρ‡ΠΈΠ²Π°Ρ‚ΡŒ аккумуляторныС Π±Π°Ρ‚Π°Ρ€Π΅ΠΈ Π½Π° 12,0 Π’ 750-ΠΊΠΈΠ»ΠΎΠ³Ρ€Π°ΠΌΠΌΠΎΠ²ΠΎΠ³ΠΎ элСктромобиля: ΠΎΡ‚Π΄Ρ‹Ρ… Π΄ΠΎ 25,0 ΠΌ / с Π·Π° 1,00 ΠΌΠΈΠ½? (b) ΠŸΠΎΠ΄Π½ΡΡ‚ΡŒΡΡ Π½Π° Ρ…ΠΎΠ»ΠΌ высотой 2,00 Γ— 10 2 ΠΌ Π·Π° 2,00 ΠΌΠΈΠ½ ΠΏΡ€ΠΈ постоянной 25.Π‘ΠΊΠΎΡ€ΠΎΡΡ‚ΡŒ 0 ΠΌ / с ΠΏΡ€ΠΈ ΠΏΡ€ΠΈΠ»ΠΎΠΆΠ΅Π½ΠΈΠΈ силы 5,00 Γ— 10 2 Н для прСодолСния сопротивлСния Π²ΠΎΠ·Π΄ΡƒΡ…Π° ΠΈ трСния? (c) Π”Π²ΠΈΠ³Π°Ρ‚ΡŒΡΡ с постоянной ΡΠΊΠΎΡ€ΠΎΡΡ‚ΡŒΡŽ 25,0 ΠΌ / с, прилагая силу 5,00 Γ— 10 2 Н для прСодолСния сопротивлСния Π²ΠΎΠ·Π΄ΡƒΡ…Π° ΠΈ трСния? Π‘ΠΌ. Рисунок 6.

Рис. 6. Π­Π»Π΅ΠΊΡ‚Ρ€ΠΎΠΌΠΎΠ±ΠΈΠ»ΡŒ REVAi заряТаСтся Π½Π° ΠΎΠ΄Π½ΠΎΠΉ ΠΈΠ· ΡƒΠ»ΠΈΡ† Π›ΠΎΠ½Π΄ΠΎΠ½Π°. (ΠΊΡ€Π΅Π΄ΠΈΡ‚: Ѐрэнк Π₯Π΅Π±Π±Π΅Ρ€Ρ‚)

26. Integrated Concepts ΠŸΡ€ΠΈΠ³ΠΎΡ€ΠΎΠ΄Π½Ρ‹ΠΉ Π»Π΅Π³ΠΊΠΎΡ€Π΅Π»ΡŒΡΠΎΠ²Ρ‹ΠΉ ΠΏΠΎΠ΅Π·Π΄ потрСбляСт 630 А постоянного Ρ‚ΠΎΠΊΠ° напряТСниСм 650 Π’ ΠΏΡ€ΠΈ ускорСнии.Π°) Какова Π΅Π³ΠΎ ΠΌΠΎΡ‰Π½ΠΎΡΡ‚ΡŒ Π² ΠΊΠΈΠ»ΠΎΠ²Π°Ρ‚Ρ‚Π°Ρ…? (b) Бколько Π²Ρ€Π΅ΠΌΠ΅Π½ΠΈ Π½ΡƒΠΆΠ½ΠΎ, Ρ‡Ρ‚ΠΎΠ±Ρ‹ Π΄ΠΎΡΡ‚ΠΈΡ‡ΡŒ скорости 20,0 ΠΌ / с, начиная с состояния покоя, Ссли Π΅Π³ΠΎ загруТСнная масса составляСт 5,30 Γ— 10 4 ΠΊΠ³, прСдполагая ΡΡ„Ρ„Π΅ΠΊΡ‚ΠΈΠ²Π½ΠΎΡΡ‚ΡŒ 95,0% ΠΈ ΠΏΠΎΡΡ‚ΠΎΡΠ½Π½ΡƒΡŽ ΠΌΠΎΡ‰Π½ΠΎΡΡ‚ΡŒ? (c) НайдитС Π΅Π³ΠΎ срСднСС ускорСниС. (Π³) ΠžΠ±ΡΡƒΠ΄ΠΈΡ‚Π΅, ΠΊΠ°ΠΊ ускорСниС, ΠΊΠΎΡ‚ΠΎΡ€ΠΎΠ΅ Π²Ρ‹ ΠΎΠ±Π½Π°Ρ€ΡƒΠΆΠΈΠ»ΠΈ для Π»Π΅Π³ΠΊΠΎΡ€Π΅Π»ΡŒΡΠΎΠ²ΠΎΠ³ΠΎ ΠΏΠΎΠ΅Π·Π΄Π°, сравниваСтся с Ρ‚Π΅ΠΌ, Ρ‡Ρ‚ΠΎ ΠΌΠΎΠΆΠ΅Ρ‚ Π±Ρ‹Ρ‚ΡŒ Ρ‚ΠΈΠΏΠΈΡ‡Π½Ρ‹ΠΌ для автомобиля.

27. Integrated Concepts (a) Линия элСктропСрСдачи ΠΈΠ· алюминия ΠΈΠΌΠ΅Π΅Ρ‚ сопротивлСниС 0,0580 Ом / ΠΊΠΌ. Какова Π΅Π³ΠΎ масса Π½Π° ΠΊΠΈΠ»ΠΎΠΌΠ΅Ρ‚Ρ€? Π±) Какова масса Π½Π° ΠΊΠΈΠ»ΠΎΠΌΠ΅Ρ‚Ρ€ ΠΌΠ΅Π΄Π½ΠΎΠΉ Π»ΠΈΠ½ΠΈΠΈ с Ρ‚Π°ΠΊΠΈΠΌ ΠΆΠ΅ сопротивлСниСм? Π‘ΠΎΠ»Π΅Π΅ Π½ΠΈΠ·ΠΊΠΎΠ΅ сопротивлСниС сократит врСмя Π½Π°Π³Ρ€Π΅Π²Π°.ΠžΠ±ΡΡƒΠ΄ΠΈΡ‚Π΅ практичСскиС ограничСния ускорСния Π½Π°Π³Ρ€Π΅Π²Π° Π·Π° счСт сниТСния сопротивлСния.

28. Integrated Concepts (a) ΠŸΠΎΠ³Ρ€ΡƒΠΆΠ½ΠΎΠΉ Π½Π°Π³Ρ€Π΅Π²Π°Ρ‚Π΅Π»ΡŒ, Ρ€Π°Π±ΠΎΡ‚Π°ΡŽΡ‰ΠΈΠΉ Π½Π° 120 Π’, ΠΌΠΎΠΆΠ΅Ρ‚ ΠΏΠΎΠ²Ρ‹ΡΠΈΡ‚ΡŒ Ρ‚Π΅ΠΌΠΏΠ΅Ρ€Π°Ρ‚ΡƒΡ€Ρƒ 1,00 Γ— 10 2 -Π³Ρ€Π°ΠΌΠΌΠΎΠ²Ρ‹Ρ… Π°Π»ΡŽΠΌΠΈΠ½ΠΈΠ΅Π²Ρ‹Ρ… стаканов, содСрТащих 350 Π³ Π²ΠΎΠ΄Ρ‹, с 20 Β° C Π΄ΠΎ 95 Β° C Π·Π° 2,00 ΠΌΠΈΠ½. НайдитС Π΅Π³ΠΎ сопротивлСниС, прСдполагая, Ρ‡Ρ‚ΠΎ ΠΎΠ½ΠΎ постоянно Π² процСссС. (b) Π‘ΠΎΠ»Π΅Π΅ Π½ΠΈΠ·ΠΊΠΎΠ΅ сопротивлСниС сократит врСмя Π½Π°Π³Ρ€Π΅Π²Π°. ΠžΠ±ΡΡƒΠ΄ΠΈΡ‚Π΅ практичСскиС ограничСния ускорСния Π½Π°Π³Ρ€Π΅Π²Π° Π·Π° счСт сниТСния сопротивлСния.

29. Integrated Concepts (a) Какова ΡΡ‚ΠΎΠΈΠΌΠΎΡΡ‚ΡŒ Π½Π°Π³Ρ€Π΅Π²Π° гидромассаТной Π²Π°Π½Π½Ρ‹, содСрТащСй 1500 ΠΊΠ³ Π²ΠΎΠ΄Ρ‹, ΠΎΡ‚ 10 Β° C Π΄ΠΎ 40 Β° C, исходя ΠΈΠ· эффСктивности 75,0% с ΡƒΡ‡Π΅Ρ‚ΠΎΠΌ ΠΏΠ΅Ρ€Π΅Π΄Π°Ρ‡ΠΈ Ρ‚Π΅ΠΏΠ»Π° Π² ΠΎΠΊΡ€ΡƒΠΆΠ°ΡŽΡ‰ΡƒΡŽ срСду? Π‘Ρ‚ΠΎΠΈΠΌΠΎΡΡ‚ΡŒ элСктроэнСргии 9 Ρ†Π΅Π½Ρ‚ΠΎΠ² / ΠΊΠ’Ρ‚β‹…Ρ‡. (b) Какой Ρ‚ΠΎΠΊ потрСблял элСктричСский Π½Π°Π³Ρ€Π΅Π²Π°Ρ‚Π΅Π»ΡŒ ΠΏΠ΅Ρ€Π΅ΠΌΠ΅Π½Π½ΠΎΠ³ΠΎ Ρ‚ΠΎΠΊΠ° 220 Π’, Ссли Π½Π° это ΠΏΠΎΡ‚Ρ€Π΅Π±ΠΎΠ²Π°Π»ΠΎΡΡŒ 4 часа?

30 . НСобоснованныС Ρ€Π΅Π·ΡƒΠ»ΡŒΡ‚Π°Ρ‚Ρ‹ (a) Какой Ρ‚ΠΎΠΊ Π½Π΅ΠΎΠ±Ρ…ΠΎΠ΄ΠΈΠΌ для ΠΏΠ΅Ρ€Π΅Π΄Π°Ρ‡ΠΈ 1,00 Γ— 10 2 ΠœΠ’Ρ‚ мощности ΠΏΡ€ΠΈ 480 Π’? (b) Какая ΠΌΠΎΡ‰Π½ΠΎΡΡ‚ΡŒ рассСиваСтся линиями ΠΏΠ΅Ρ€Π΅Π΄Π°Ρ‡ΠΈ, Ссли ΠΎΠ½ΠΈ ΠΈΠΌΠ΅ΡŽΡ‚ коэффициСнт 1.00 – сопротивлСниС Ом? (c) Π§Ρ‚ΠΎ Π½Π΅Ρ€Π°Π·ΡƒΠΌΠ½ΠΎΠ³ΠΎ Π² этом Ρ€Π΅Π·ΡƒΠ»ΡŒΡ‚Π°Ρ‚Π΅? (d) КакиС прСдполоТСния нСобоснованны ΠΈΠ»ΠΈ ΠΊΠ°ΠΊΠΈΠ΅ посылки нСсовмСстимы?

31. НСобоснованныС Ρ€Π΅Π·ΡƒΠ»ΡŒΡ‚Π°Ρ‚Ρ‹ (a) Какой Ρ‚ΠΎΠΊ Π½Π΅ΠΎΠ±Ρ…ΠΎΠ΄ΠΈΠΌ для ΠΏΠ΅Ρ€Π΅Π΄Π°Ρ‡ΠΈ 1,00 Γ— 10 2 ΠœΠ’Ρ‚ мощности ΠΏΡ€ΠΈ 10,0 ΠΊΠ’? (b) НайдитС сопротивлСниС 1,00 ΠΊΠΌ ΠΏΡ€ΠΎΠ²ΠΎΠ΄Π°, ΠΊΠΎΡ‚ΠΎΡ€ΠΎΠ΅ Π²Ρ‹Π·ΠΎΠ²Π΅Ρ‚ ΠΏΠΎΡ‚Π΅Ρ€ΡŽ мощности 0,0100%. (c) Каков Π΄ΠΈΠ°ΠΌΠ΅Ρ‚Ρ€ ΠΌΠ΅Π΄Π½ΠΎΠ³ΠΎ ΠΏΡ€ΠΎΠ²ΠΎΠ΄Π° Π΄Π»ΠΈΠ½ΠΎΠΉ 1,00 ΠΊΠΌ, ΠΈΠΌΠ΅ΡŽΡ‰Π΅Π³ΠΎ Ρ‚Π°ΠΊΠΎΠ΅ сопротивлСниС? (Π³) Π§Ρ‚ΠΎ нСобоснованного Π² этих Ρ€Π΅Π·ΡƒΠ»ΡŒΡ‚Π°Ρ‚Π°Ρ…? (e) КакиС прСдполоТСния нСобоснованны ΠΈΠ»ΠΈ ΠΊΠ°ΠΊΠΈΠ΅ посылки нСсовмСстимы?

32.Π‘ΠΎΠ·Π΄Π°ΠΉ свою Π·Π°Π΄Π°Ρ‡Ρƒ Рассмотрим элСктричСский ΠΏΠΎΠ³Ρ€ΡƒΠΆΠ½ΠΎΠΉ Π½Π°Π³Ρ€Π΅Π²Π°Ρ‚Π΅Π»ΡŒ, ΠΈΡΠΏΠΎΠ»ΡŒΠ·ΡƒΠ΅ΠΌΡ‹ΠΉ для Π½Π°Π³Ρ€Π΅Π²Π° Ρ‡Π°ΡˆΠΊΠΈ Π²ΠΎΠ΄Ρ‹ для приготовлСния чая. ΠŸΠΎΡΡ‚Ρ€ΠΎΠΉΡ‚Π΅ Π·Π°Π΄Π°Ρ‡Ρƒ, Π² ΠΊΠΎΡ‚ΠΎΡ€ΠΎΠΉ Π²Ρ‹ рассчитываСтС Π½Π΅ΠΎΠ±Ρ…ΠΎΠ΄ΠΈΠΌΠΎΠ΅ сопротивлСниС нагрСватСля, Ρ‡Ρ‚ΠΎΠ±Ρ‹ ΠΎΠ½ ΡƒΠ²Π΅Π»ΠΈΡ‡ΠΈΠ²Π°Π» Ρ‚Π΅ΠΌΠΏΠ΅Ρ€Π°Ρ‚ΡƒΡ€Ρƒ Π²ΠΎΠ΄Ρ‹ ΠΈ Ρ‡Π°ΡˆΠΊΠΈ Π·Π° Ρ€Π°Π·ΡƒΠΌΠ½ΠΎΠ΅ врСмя. Π’Π°ΠΊΠΆΠ΅ рассчитайтС ΡΡ‚ΠΎΠΈΠΌΠΎΡΡ‚ΡŒ элСктроэнСргии, ΠΈΡΠΏΠΎΠ»ΡŒΠ·ΡƒΠ΅ΠΌΠΎΠΉ Π² вашСм тСхнологичСском процСссС. Π‘Ρ€Π΅Π΄ΠΈ Ρ„Π°ΠΊΡ‚ΠΎΡ€ΠΎΠ², ΠΊΠΎΡ‚ΠΎΡ€Ρ‹Π΅ слСдуСт ΡƒΡ‡ΠΈΡ‚Ρ‹Π²Π°Ρ‚ΡŒ, – это ΠΈΡΠΏΠΎΠ»ΡŒΠ·ΡƒΠ΅ΠΌΠΎΠ΅ напряТСниС, задСйствованныС массы ΠΈ Ρ‚Π΅ΠΏΠ»ΠΎΠ΅ΠΌΠΊΠΎΡΡ‚ΡŒ, Ρ‚Π΅ΠΏΠ»ΠΎΠ²Ρ‹Π΅ ΠΏΠΎΡ‚Π΅Ρ€ΠΈ ΠΈ врСмя, Π² Ρ‚Π΅Ρ‡Π΅Π½ΠΈΠ΅ ΠΊΠΎΡ‚ΠΎΡ€ΠΎΠ³ΠΎ происходит Π½Π°Π³Ρ€Π΅Π².Π’Π°Ρˆ инструктор ΠΌΠΎΠΆΠ΅Ρ‚ ΠΏΠΎΠΆΠ΅Π»Π°Ρ‚ΡŒ, Ρ‡Ρ‚ΠΎΠ±Ρ‹ Π²Ρ‹ рассмотрСли Ρ‚Π΅ΠΏΠ»ΠΎΠ²ΠΎΠΉ ΠΏΡ€Π΅Π΄ΠΎΡ…Ρ€Π°Π½ΠΈΡ‚Π΅Π»ΡŒΠ½Ρ‹ΠΉ Π²Ρ‹ΠΊΠ»ΡŽΡ‡Π°Ρ‚Π΅Π»ΡŒ (Π²ΠΎΠ·ΠΌΠΎΠΆΠ½ΠΎ, бимСталличСский), ΠΊΠΎΡ‚ΠΎΡ€Ρ‹ΠΉ остановит процСсс Π΄ΠΎ Ρ‚ΠΎΠ³ΠΎ, ΠΊΠ°ΠΊ Π² ΠΏΠΎΠ³Ρ€ΡƒΠΆΠ½ΠΎΠΌ Π±Π»ΠΎΠΊΠ΅ Π±ΡƒΠ΄ΡƒΡ‚ достигнуты опасныС Ρ‚Π΅ΠΌΠΏΠ΅Ρ€Π°Ρ‚ΡƒΡ€Ρ‹.

Глоссарий

элСктричСская ΠΌΠΎΡ‰Π½ΠΎΡΡ‚ΡŒ:
– ΡΠΊΠΎΡ€ΠΎΡΡ‚ΡŒ, с ΠΊΠΎΡ‚ΠΎΡ€ΠΎΠΉ элСктричСская энСргия подаСтся источником ΠΈΠ»ΠΈ рассСиваСтся устройством; это ΠΏΡ€ΠΎΠΈΠ·Π²Π΅Π΄Π΅Π½ΠΈΠ΅ Ρ‚ΠΎΠΊΠ° Π½Π° напряТСниС

Π˜Π·Π±Ρ€Π°Π½Π½Ρ‹Π΅ Ρ€Π΅ΡˆΠ΅Π½ΠΈΡ ΠΏΡ€ΠΎΠ±Π»Π΅ΠΌ ΠΈ упраТнСния

1. 2,00 Γ— 10 12 Π’Ρ‚

5.{6} \ text {J} \\ [/ latex]

11. 438 $ / Π³ΠΎΠ΄

13. $ 6.25

15. 1.58 Ρ‡

17. 3,94 ΠΌΠΈΠ»Π»ΠΈΠ°Ρ€Π΄Π° Π΄ΠΎΠ»Π»Π°Ρ€ΠΎΠ² Π² Π³ΠΎΠ΄

19. 25,5 Π’Ρ‚

21. (Π°) 2,00 Γ— 10 9 Π”ΠΆ (Π±) 769 ΠΊΠ³

23. 45.0 с

25. (Π°) 343 A (Π±) 2,17 Γ— 10 3 A (Π²) 1,10 Γ— 10 3 A

27. (Π°) 1,23 Γ— 10 3 ΠΊΠ³ (Π±) 2,64 Γ— 10 3 ΠΊΠ³

29. (a) 2,08 Γ— 10 5 A
(b) 4,33 Γ— 10 4 ΠœΠ’Ρ‚
(c) Π›ΠΈΠ½ΠΈΠΈ ΠΏΠ΅Ρ€Π΅Π΄Π°Ρ‡ΠΈ Ρ€Π°ΡΡΠ΅ΠΈΠ²Π°ΡŽΡ‚ большС мощности, Ρ‡Π΅ΠΌ ΠΎΠ½ΠΈ Π΄ΠΎΠ»ΠΆΠ½Ρ‹ ΠΏΠ΅Ρ€Π΅Π΄Π°Π²Π°Ρ‚ΡŒ.
(d) НапряТСниС 480 Π’ Π½Π΅ΠΎΠΏΡ€Π°Π²Π΄Π°Π½Π½ΠΎ Π½ΠΈΠ·ΠΊΠΎΠ΅ для напряТСния ΠΏΠ΅Ρ€Π΅Π΄Π°Ρ‡ΠΈ. Π’ линиях ΠΏΠ΅Ρ€Π΅Π΄Π°Ρ‡ΠΈ Π½Π° большиС расстояния поддСрТиваСтся Π³ΠΎΡ€Π°Π·Π΄ΠΎ Π±ΠΎΠ»Π΅Π΅ высокоС напряТСниС (часто сотни ΠΊΠΈΠ»ΠΎΠ²ΠΎΠ»ΡŒΡ‚), Ρ‡Ρ‚ΠΎΠ±Ρ‹ ΡƒΠΌΠ΅Π½ΡŒΡˆΠΈΡ‚ΡŒ ΠΏΠΎΡ‚Π΅Ρ€ΠΈ мощности.

ЭлСктричСский Ρ‚ΠΎΠΊ: ΠΎΠΏΡ€Π΅Π΄Π΅Π»Π΅Π½ΠΈΠ΅, Π΅Π΄ΠΈΠ½ΠΈΡ†Π°, Ρ„ΠΎΡ€ΠΌΡƒΠ»Π°, Ρ‚ΠΈΠΏΡ‹ (с ΠΏΡ€ΠΈΠΌΠ΅Ρ€Π°ΠΌΠΈ)

ОбновлСно 28 дСкабря 2020 Π³.

КСвин Π‘Π΅ΠΊ

ЭлСктричСский заряд: ΠšΠ°ΠΊΡƒΡŽ Π°Π²Ρ‚ΠΎΠΌΠ°Ρ‚ΠΈΡ‡Π΅ΡΠΊΡƒΡŽ Ρ€Π΅Π°ΠΊΡ†ΠΈΡŽ Π²Ρ‹Π·Ρ‹Π²Π°Π΅Ρ‚ эта Ρ„Ρ€Π°Π·Π°, ΠΊΠΎΠ³Π΄Π° Π²Ρ‹ Π΅Π΅ Ρ‡ΠΈΡ‚Π°Π΅Ρ‚Π΅? ΠœΠΎΠΆΠ΅Ρ‚ Π±Ρ‹Ρ‚ΡŒ, ΠΎΡ‰ΡƒΡ‰Π΅Π½ΠΈΠ΅ покалывания ΠΈΠ»ΠΈ ΠΎΠ±Ρ€Π°Π· ΠΌΠΎΠ»Π½ΠΈΠΈ, Ρ€Π°ΡΠΊΠ°Π»Ρ‹Π²Π°ΡŽΡ‰Π΅Π³ΠΎ Π½Π΅Π±ΠΎ? ΠšΡ€Π°ΡΠΎΡ‡Π½Ρ‹ΠΉ дисплСй ΠΌΠΈΠ³Π°ΡŽΡ‰ΠΈΡ… ΠΎΠ³Π½Π΅ΠΉ Π² Ρ‚Π°ΠΊΠΎΠΌ Π³ΠΎΡ€ΠΎΠ΄Π΅, ΠΊΠ°ΠΊ ΠŸΠ°Ρ€ΠΈΠΆ ΠΈΠ»ΠΈ Лас-ВСгас? Π’ΠΎΠ·ΠΌΠΎΠΆΠ½ΠΎ, Π΄Π°ΠΆΠ΅ насСкомоС, ΠΊΠΎΡ‚ΠΎΡ€ΠΎΠ΅ ΠΊΠ°ΠΊΠΈΠΌ-Ρ‚ΠΎ ΠΎΠ±Ρ€Π°Π·ΠΎΠΌ свСтится Π² Ρ‚Π΅ΠΌΠ½ΠΎΡ‚Π΅, ΠΏΡ€ΠΎΠ±ΠΈΡ€Π°ΡΡΡŒ Ρ‡Π΅Ρ€Π΅Π· ваш Π»Π°Π³Π΅Ρ€ΡŒ?

Π”ΠΎ Π½Π΅Π΄Π°Π²Π½ΠΈΡ… столСтий ΡƒΡ‡Π΅Π½Ρ‹Π΅ Π½Π΅ Ρ‚ΠΎΠ»ΡŒΠΊΠΎ Π½Π΅ ΠΈΠΌΠ΅Π»ΠΈ возмоТности ΠΈΠ·ΠΌΠ΅Ρ€ΠΈΡ‚ΡŒ ΡΠΊΠΎΡ€ΠΎΡΡ‚ΡŒ свСта, Π½ΠΎ ΠΈ Π½Π΅ Π·Π½Π°Π»ΠΈ, ΠΊΠ°ΠΊΠΈΠ΅ физичСскиС явлСния Π»Π΅ΠΆΠ°Ρ‚ Π² основС Ρ‚ΠΎΠ³ΠΎ, Ρ‡Ρ‚ΠΎ сСйчас извСстно ΠΊΠ°ΠΊ «элСктричСство».Π’ 1800-Ρ… Π³ΠΎΠ΄Π°Ρ… Ρ„ΠΈΠ·ΠΈΠΊΠΈ Π²ΠΏΠ΅Ρ€Π²Ρ‹Π΅ ΡƒΠ·Π½Π°Π»ΠΈ ΠΎ ΠΌΠ΅Π»ΡŒΡ‡Π°ΠΉΡˆΠΈΡ… частицах, ΡƒΡ‡Π°ΡΡ‚Π²ΡƒΡŽΡ‰ΠΈΡ… Π² ΠΏΠΎΡ‚ΠΎΠΊΠ΅ Ρ‚ΠΎΠΊΠ° (свободныС элСктроны), Π° Ρ‚Π°ΠΊΠΆΠ΅ ΠΎ ΠΏΡ€ΠΈΡ€ΠΎΠ΄Π΅ сил, Π·Π°ΡΡ‚Π°Π²Π»ΡΡŽΡ‰ΠΈΡ… ΠΈΡ… Π΄Π²ΠΈΠ³Π°Ρ‚ΡŒΡΡ. Π‘Ρ‹Π»ΠΎ ясно, Ρ‡Ρ‚ΠΎ элСктричСство ΠΌΠΎΠΆΠ΅Ρ‚ принСсти Π·Π½Π°Ρ‡ΠΈΡ‚Π΅Π»ΡŒΠ½ΡƒΡŽ ΠΏΠΎΠ»ΡŒΠ·Ρƒ, Ссли ΠΎΠ½ΠΎ ΠΌΠΎΠΆΠ΅Ρ‚ Π±Ρ‹Ρ‚ΡŒ бСзопасно Β«ΠΈΠ·Π³ΠΎΡ‚ΠΎΠ²Π»Π΅Π½ΠΎΒ» ΠΈΠ»ΠΈ Β«Π·Π°Ρ…Π²Π°Ρ‡Π΅Π½ΠΎΒ», Π° элСктричСская энСргия ΠΈΡΠΏΠΎΠ»ΡŒΠ·ΡƒΠ΅Ρ‚ΡΡ для выполнСния Ρ€Π°Π±ΠΎΡ‚Ρ‹.

ΠŸΠΎΡ‚ΠΎΠΊ элСктричСского заряда Π»Π΅Π³ΠΊΠΎ Π²ΠΎΠ·Π½ΠΈΠΊΠ°Π΅Ρ‚ Π² вСщСствах, классифицируСмых ΠΊΠ°ΠΊ проводящиС ΠΌΠ°Ρ‚Π΅Ρ€ΠΈΠ°Π»Ρ‹ , Ρ‚ΠΎΠ³Π΄Π° ΠΊΠ°ΠΊ ΠΎΠ½ затрудняСтся Π² изоляторах .НапримСр, Π² мСталличСском ΠΏΡ€ΠΎΠ²ΠΎΠ΄Π΅, Ρ‚Π°ΠΊΠΎΠΌ ΠΊΠ°ΠΊ ΠΌΠ΅Π΄Π½Ρ‹ΠΉ ΠΏΡ€ΠΎΠ²ΠΎΠ΄, ΠΌΠΎΠΆΠ½ΠΎ ΡΠΎΠ·Π΄Π°Ρ‚ΡŒ Ρ€Π°Π·Π½ΠΎΡΡ‚ΡŒ ΠΏΠΎΡ‚Π΅Π½Ρ†ΠΈΠ°Π»ΠΎΠ² Π½Π° ΠΊΠΎΠ½Ρ†Π°Ρ… ΠΏΡ€ΠΎΠ²ΠΎΠ΄Π°, вызывая ΠΏΠΎΡ‚ΠΎΠΊ заряда ΠΈ создавая Ρ‚ΠΎΠΊ.

ΠžΠΏΡ€Π΅Π΄Π΅Π»Π΅Π½ΠΈΠ΅ элСктричСского Ρ‚ΠΎΠΊΠ°

ЭлСктричСский Ρ‚ΠΎΠΊ – это срСдняя ΡΠΊΠΎΡ€ΠΎΡΡ‚ΡŒ ΠΏΠΎΡ‚ΠΎΠΊΠ° элСктричСского заряда (Ρ‚. Π•. Заряда Π² Π΅Π΄ΠΈΠ½ΠΈΡ†Ρƒ Π²Ρ€Π΅ΠΌΠ΅Π½ΠΈ) Ρ‡Π΅Ρ€Π΅Π· Ρ‚ΠΎΡ‡ΠΊΡƒ Π² пространствС. Π­Ρ‚ΠΎΡ‚ заряд пСрСносится элСктронами , двиТущимися ΠΏΠΎ ΠΏΡ€ΠΎΠ²ΠΎΠ΄Ρƒ Π² элСктричСской Ρ†Π΅ΠΏΠΈ.Π§Π΅ΠΌ большС элСктронов ΠΏΡ€ΠΎΡ…ΠΎΠ΄ΠΈΡ‚ Ρ‡Π΅Ρ€Π΅Π· эту Ρ‚ΠΎΡ‡ΠΊΡƒ Π·Π° сСкунду, Ρ‚Π΅ΠΌ большС Π²Π΅Π»ΠΈΡ‡ΠΈΠ½Π° Ρ‚ΠΎΠΊΠ°.

Π•Π΄ΠΈΠ½ΠΈΡ†Π΅ΠΉ измСрСния Ρ‚ΠΎΠΊΠ° Π² систСмС БИ являСтся Π°ΠΌΠΏΠ΅Ρ€ (А), часто Π½Π΅ΠΎΡ„ΠΈΡ†ΠΈΠ°Π»ΡŒΠ½ΠΎ Π½Π°Π·Ρ‹Π²Π°Π΅ΠΌΡ‹ΠΉ Β«Π°ΠΌΠΏΠ΅Ρ€Β». Π‘Π°ΠΌ элСктричСский заряд измСряСтся Π² ΠΊΡƒΠ»ΠΎΠ½Π°Ρ… (Кл).

  • Заряд ΠΎΠ΄Π½ΠΎΠ³ΠΎ элСктрона Ρ€Π°Π²Π΅Π½ -1,60 Γ— 10 -19 Кл, Π° заряд ΠΏΡ€ΠΎΡ‚ΠΎΠ½Π° Ρ€Π°Π²Π΅Π½ , Π½ΠΎ ΠΏΠΎΠ»ΠΎΠΆΠΈΡ‚Π΅Π»ΡŒΠ½Ρ‹ΠΉ Π·Π½Π°ΠΊ . Π­Ρ‚ΠΎ число считаСтся Ρ„ΡƒΠ½Π΄Π°ΠΌΠ΅Π½Ρ‚Π°Π»ΡŒΠ½Ρ‹ΠΌ зарядом e .Π’Π°ΠΊΠΈΠΌ ΠΎΠ±Ρ€Π°Π·ΠΎΠΌ, базовая Π΅Π΄ΠΈΠ½ΠΈΡ†Π° Π°ΠΌΠΏΠ΅Ρ€ – ΠΊΡƒΠ»ΠΎΠ½Ρ‹ Π² сСкунду (Кл / с).

По соглашСнию, элСктричСский Ρ‚ΠΎΠΊ Ρ‚Π΅Ρ‡Π΅Ρ‚ Π² Π½Π°ΠΏΡ€Π°Π²Π»Π΅Π½ΠΈΠΈ, ΠΏΡ€ΠΎΡ‚ΠΈΠ²ΠΎΠΏΠΎΠ»ΠΎΠΆΠ½ΠΎΠΌ ΠΏΠΎΡ‚ΠΎΠΊΡƒ элСктронов . Π­Ρ‚ΠΎ связано с Ρ‚Π΅ΠΌ, Ρ‡Ρ‚ΠΎ Π½Π°ΠΏΡ€Π°Π²Π»Π΅Π½ΠΈΠ΅ Ρ‚ΠΎΠΊΠ° Π±Ρ‹Π»ΠΎ описано Π΄ΠΎ Ρ‚ΠΎΠ³ΠΎ, ΠΊΠ°ΠΊ ΡƒΡ‡Π΅Π½Ρ‹Π΅ ΡƒΠ·Π½Π°Π»ΠΈ, ΠΊΠ°ΠΊΠΈΠ΅ носитСли заряда двигались ΠΏΠΎΠ΄ дСйствиСм элСктричСского поля. Для всСх практичСских Ρ†Π΅Π»Π΅ΠΉ ΠΏΠΎΠ»ΠΎΠΆΠΈΡ‚Π΅Π»ΡŒΠ½Ρ‹Π΅ заряды, двиТущиСся Π² ΠΏΠΎΠ»ΠΎΠΆΠΈΡ‚Π΅Π»ΡŒΠ½ΠΎΠΌ Π½Π°ΠΏΡ€Π°Π²Π»Π΅Π½ΠΈΠΈ, Π΄Π°ΡŽΡ‚ Ρ‚ΠΎΡ‚ ΠΆΠ΅ физичСский (Π²Ρ‹Ρ‡ΠΈΡΠ»ΠΈΡ‚Π΅Π»ΡŒΠ½Ρ‹ΠΉ) Ρ€Π΅Π·ΡƒΠ»ΡŒΡ‚Π°Ρ‚, Ρ‡Ρ‚ΠΎ ΠΈ ΠΎΡ‚Ρ€ΠΈΡ†Π°Ρ‚Π΅Π»ΡŒΠ½Ρ‹Π΅ заряды, двиТущиСся Π² ΠΎΡ‚Ρ€ΠΈΡ†Π°Ρ‚Π΅Π»ΡŒΠ½ΠΎΠΌ Π½Π°ΠΏΡ€Π°Π²Π»Π΅Π½ΠΈΠΈ, ΠΊΠΎΠ³Π΄Π° Π΄Π΅Π»ΠΎ касаСтся элСктричСского Ρ‚ΠΎΠΊΠ°.

Π­Π»Π΅ΠΊΡ‚Ρ€ΠΎΠ½Ρ‹ двиТутся ΠΊ ΠΏΠΎΠ»ΠΎΠΆΠΈΡ‚Π΅Π»ΡŒΠ½ΠΎΠΌΡƒ Π²Ρ‹Π²ΠΎΠ΄Ρƒ Π² элСктричСской Ρ†Π΅ΠΏΠΈ. Π’Π°ΠΊΠΈΠΌ ΠΎΠ±Ρ€Π°Π·ΠΎΠΌ, ΠΏΠΎΡ‚ΠΎΠΊ элСктронов ΠΈΠ»ΠΈ двиТущийся заряд находится Π΄Π°Π»Π΅ΠΊΠΎ ΠΎΡ‚ ΠΎΡ‚Ρ€ΠΈΡ†Π°Ρ‚Π΅Π»ΡŒΠ½ΠΎΠ³ΠΎ Π²Ρ‹Π²ΠΎΠ΄Π°. Π”Π²ΠΈΠΆΠ΅Π½ΠΈΠ΅ элСктронов Π² ΠΌΠ΅Π΄Π½ΠΎΠΉ ΠΏΡ€ΠΎΠ²ΠΎΠ»ΠΎΠΊΠ΅ ΠΈΠ»ΠΈ Π΄Ρ€ΡƒΠ³ΠΎΠΌ проводящСм ΠΌΠ°Ρ‚Π΅Ρ€ΠΈΠ°Π»Π΅ Ρ‚Π°ΠΊΠΆΠ΅ создаСт ΠΌΠ°Π³Π½ΠΈΡ‚Π½ΠΎΠ΅ ΠΏΠΎΠ»Π΅ , Π½Π°ΠΏΡ€Π°Π²Π»Π΅Π½ΠΈΠ΅ ΠΈ Π²Π΅Π»ΠΈΡ‡ΠΈΠ½Π° ΠΊΠΎΡ‚ΠΎΡ€ΠΎΠ³ΠΎ ΠΎΠΏΡ€Π΅Π΄Π΅Π»ΡΡŽΡ‚ΡΡ Π½Π°ΠΏΡ€Π°Π²Π»Π΅Π½ΠΈΠ΅ΠΌ элСктричСского Ρ‚ΠΎΠΊΠ° ΠΈ, ΡΠ»Π΅Π΄ΠΎΠ²Π°Ρ‚Π΅Π»ΡŒΠ½ΠΎ, Π΄Π²ΠΈΠΆΠ΅Π½ΠΈΠ΅ΠΌ элСктронов; это ΠΏΡ€ΠΈΠ½Ρ†ΠΈΠΏ, Π½Π° ΠΊΠΎΡ‚ΠΎΡ€ΠΎΠΌ построСн элСктромагнит .

Π€ΠΎΡ€ΠΌΡƒΠ»Π° элСктричСского Ρ‚ΠΎΠΊΠ°

Для Π±Π°Π·ΠΎΠ²ΠΎΠ³ΠΎ сцСнария ΠΎΠ±Ρ‹Ρ‡Π½ΠΎΠ³ΠΎ Ρ‚ΠΎΠΊΠ° заряда, двиТущСгося ΠΏΠΎ ΠΏΡ€ΠΎΠ²ΠΎΠ΄Ρƒ, Ρ„ΠΎΡ€ΠΌΡƒΠ»Π° для Ρ‚ΠΎΠΊΠ° ΠΈΠΌΠ΅Π΅Ρ‚ Π²ΠΈΠ΄:

I = neAv_d

, Π³Π΄Π΅ n – количСство зарядов Π½Π° кубичСский ΠΌΠ΅Ρ‚Ρ€ (ΠΌ 3 ), e – основной заряд, A – ΠΏΠ»ΠΎΡ‰Π°Π΄ΡŒ ΠΏΠΎΠΏΠ΅Ρ€Π΅Ρ‡Π½ΠΎΠ³ΠΎ сСчСния ΠΏΡ€ΠΎΠ²ΠΎΠ΄Π°, Π° v d – это Π‘ΠΊΠΎΡ€ΠΎΡΡ‚ΡŒ Π΄Ρ€Π΅ΠΉΡ„Π° .

Π₯отя Ρ‚ΠΎΠΊ ΠΈΠΌΠ΅Π΅Ρ‚ ΠΊΠ°ΠΊ Π²Π΅Π»ΠΈΡ‡ΠΈΠ½Ρƒ, Ρ‚Π°ΠΊ ΠΈ Π½Π°ΠΏΡ€Π°Π²Π»Π΅Π½ΠΈΠ΅, это скалярная Π²Π΅Π»ΠΈΡ‡ΠΈΠ½Π°, Π° Π½Π΅ вСкторная Π²Π΅Π»ΠΈΡ‡ΠΈΠ½Π°, ΠΏΠΎΡΠΊΠΎΠ»ΡŒΠΊΡƒ ΠΎΠ½Π° Π½Π΅ подчиняСтся Π·Π°ΠΊΠΎΠ½Π°ΠΌ слоТСния Π²Π΅ΠΊΡ‚ΠΎΡ€ΠΎΠ².

Π€ΠΎΡ€ΠΌΡƒΠ»Π° Π·Π°ΠΊΠΎΠ½Π° Ома

Π—Π°ΠΊΠΎΠ½ Ома Π΄Π°Π΅Ρ‚ Ρ„ΠΎΡ€ΠΌΡƒΠ»Ρƒ для опрСдСлСния Ρ‚ΠΎΠΊΠ°, ΠΊΠΎΡ‚ΠΎΡ€Ρ‹ΠΉ Π±ΡƒΠ΄Π΅Ρ‚ ΠΏΡ€ΠΎΡ‚Π΅ΠΊΠ°Ρ‚ΡŒ Ρ‡Π΅Ρ€Π΅Π· ΠΏΡ€ΠΎΠ²ΠΎΠ΄Π½ΠΈΠΊ:

I- \ frac {V} {R}

, Π³Π΄Π΅ V – напряТСниС , ΠΈΠ»ΠΈ Ρ€Π°Π·Π½ΠΎΡΡ‚ΡŒ элСктричСских ΠΏΠΎΡ‚Π΅Π½Ρ†ΠΈΠ°Π»ΠΎΠ² , измСряСмая Π² Π²ΠΎΠ»ΡŒΡ‚Π°Ρ…, ΠΈ R – это элСктричСскоС сопротивлСниС ΠΏΡ€ΠΎΡ‚Π΅ΠΊΠ°Π½ΠΈΡŽ Ρ‚ΠΎΠΊΠ°, ΠΈΠ·ΠΌΠ΅Ρ€Π΅Π½Π½ΠΎΠ΅ Π² Ом (Ом).

ΠŸΠΎΠ΄ΡƒΠΌΠ°ΠΉΡ‚Π΅ ΠΎ напряТСнии ΠΊΠ°ΠΊ ΠΎ «тянущСй силС» (хотя эта «элСктродвиТущая сила» Π½Π΅ являСтся Π±ΡƒΠΊΠ²Π°Π»ΡŒΠ½ΠΎ силой), спСцифичной для элСктричСских зарядов. Когда ΠΏΡ€ΠΎΡ‚ΠΈΠ²ΠΎΠΏΠΎΠ»ΠΎΠΆΠ½Ρ‹Π΅ заряды Ρ€Π°Π·Π΄Π΅Π»Π΅Π½Ρ‹, ΠΎΠ½ΠΈ ΠΏΡ€ΠΈΡ‚ΡΠ³ΠΈΠ²Π°ΡŽΡ‚ΡΡ Π΄Ρ€ΡƒΠ³ ΠΊ Π΄Ρ€ΡƒΠ³Ρƒ Ρ‚Π°ΠΊΠΈΠΌ ΠΎΠ±Ρ€Π°Π·ΠΎΠΌ, Ρ‡Ρ‚ΠΎ ΡƒΠΌΠ΅Π½ΡŒΡˆΠ°Π΅Ρ‚ΡΡ с ΡƒΠ²Π΅Π»ΠΈΡ‡Π΅Π½ΠΈΠ΅ΠΌ расстояния ΠΌΠ΅ΠΆΠ΄Ρƒ Π½ΠΈΠΌΠΈ. Π­Ρ‚ΠΎ ΠΏΡ€ΠΈΠΌΠ΅Ρ€Π½ΠΎ Π°Π½Π°Π»ΠΎΠ³ Π³Ρ€Π°Π²ΠΈΡ‚Π°Ρ†ΠΈΠΎΠ½Π½ΠΎΠΉ ΠΏΠΎΡ‚Π΅Π½Ρ†ΠΈΠ°Π»ΡŒΠ½ΠΎΠΉ энСргии Π² классичСской ΠΌΠ΅Ρ…Π°Π½ΠΈΠΊΠ΅; гравитация Β«Ρ…ΠΎΡ‡Π΅Ρ‚Β» падСния высоких ΠΏΡ€Π΅Π΄ΠΌΠ΅Ρ‚ΠΎΠ² Π½Π° Π—Π΅ΠΌΠ»ΡŽ, Π° напряТСниС Β«Ρ…ΠΎΡ‡Π΅Ρ‚Β», Ρ‡Ρ‚ΠΎΠ±Ρ‹ Ρ€Π°Π·Π΄Π΅Π»Π΅Π½Π½Ρ‹Π΅ (ΠΏΡ€ΠΎΡ‚ΠΈΠ²ΠΎΠΏΠΎΠ»ΠΎΠΆΠ½Ρ‹Π΅) заряды ΡΡ‚ΠΎΠ»ΠΊΠ½ΡƒΠ»ΠΈΡΡŒ вмСстС.

РазъяснСниС напряТСния

Π’ΠΎΠ»ΡŒΡ‚ эквивалСнтно дТоулям Π½Π° ΠΊΡƒΠ»ΠΎΠ½, ΠΈΠ»ΠΈ Π”ΠΆ / Кл.Π’Π°ΠΊΠΈΠΌ ΠΎΠ±Ρ€Π°Π·ΠΎΠΌ, Ρƒ Π½ΠΈΡ… Π΅ΡΡ‚ΡŒ Π΅Π΄ΠΈΠ½ΠΈΡ†Ρ‹ энСргии Π½Π° Π΅Π΄ΠΈΠ½ΠΈΡ†Ρƒ заряда. Π’Π°ΠΊΠΈΠΌ ΠΎΠ±Ρ€Π°Π·ΠΎΠΌ, Ρ‚ΠΎΠΊ, ΡƒΠΌΠ½ΠΎΠΆΠ΅Π½Π½Ρ‹ΠΉ Π½Π° напряТСниС, Π΄Π°Π΅Ρ‚ Π΅Π΄ΠΈΠ½ΠΈΡ†Ρ‹ (Кл / с) (Π”ΠΆ / Кл) = (Π”ΠΆ / с), ΠΊΠΎΡ‚ΠΎΡ€Ρ‹Π΅ пСрСводятся Π² Π΅Π΄ΠΈΠ½ΠΈΡ†Ρ‹ (Π² Π΄Π°Π½Π½ΠΎΠΌ случаС элСктричСской) мощности:

P = IV

ОбъСдинСниС этого с Π·Π°ΠΊΠΎΠ½ΠΎΠΌ Ома Π΄Π°Π΅Ρ‚ пСрСходят ΠΊ Π΄Ρ€ΡƒΠ³ΠΈΠΌ ΠΏΠΎΠ»Π΅Π·Π½Ρ‹ΠΌ матСматичСским ΡΠΎΠΎΡ‚Π½ΠΎΡˆΠ΅Π½ΠΈΡΠΌ, связанным с ΠΏΡ€ΠΎΡ‚Π΅ΠΊΠ°Π½ΠΈΠ΅ΠΌ Ρ‚ΠΎΠΊΠ°: P = I 2 R ΠΈ P = V 2 / R. Они ΠΏΠΎΠΊΠ°Π·Ρ‹Π²Π°ΡŽΡ‚, срСди ΠΏΡ€ΠΎΡ‡Π΅Π³ΠΎ, Ρ‡Ρ‚ΠΎ ΠΏΡ€ΠΈ фиксированном ΡƒΡ€ΠΎΠ²Π½Π΅ Ρ‚ΠΎΠΊΠ° ΠΌΠΎΡ‰Π½ΠΎΡΡ‚ΡŒ ΠΏΡ€ΠΎΠΏΠΎΡ€Ρ†ΠΈΠΎΠ½Π°Π»ΡŒΠ½Π° ΡΠΎΠΏΡ€ΠΎΡ‚ΠΈΠ²Π»Π΅Π½ΠΈΡŽ, Ρ‚ΠΎΠ³Π΄Π° ΠΊΠ°ΠΊ ΠΏΡ€ΠΈ фиксированном напряТСнии ΠΌΠΎΡ‰Π½ΠΎΡΡ‚ΡŒ составляСт ΠΎΠ±Ρ€Π°Ρ‚Π½ΠΎ ΠΏΡ€ΠΎΠΏΠΎΡ€Ρ†ΠΈΠΎΠ½Π°Π»ΡŒΠ½ΠΎ ΡΠΎΠΏΡ€ΠΎΡ‚ΠΈΠ²Π»Π΅Π½ΠΈΡŽ .

Π’ Ρ‚ΠΎ врСмя ΠΊΠ°ΠΊ двиТущиСся заряды (Ρ‚ΠΎΠΊ) ΠΈΠ½Π΄ΡƒΡ†ΠΈΡ€ΡƒΡŽΡ‚ ΠΌΠ°Π³Π½ΠΈΡ‚Π½ΠΎΠ΅ ΠΏΠΎΠ»Π΅, ΠΌΠ°Π³Π½ΠΈΡ‚Π½ΠΎΠ΅ ΠΏΠΎΠ»Π΅ само ΠΌΠΎΠΆΠ΅Ρ‚ ΠΈΠ½Π΄ΡƒΡ†ΠΈΡ€ΠΎΠ²Π°Ρ‚ΡŒ напряТСниС Π² ΠΏΡ€ΠΎΠ²ΠΎΠ΄Π΅.

Π’ΠΈΠΏΡ‹ Ρ‚ΠΎΠΊΠ°

  • ΠŸΠΎΡΡ‚ΠΎΡΠ½Π½Ρ‹ΠΉ Ρ‚ΠΎΠΊ (DC): Π­Ρ‚ΠΎ происходит, ΠΊΠΎΠ³Π΄Π° всС элСктроны Π½Π΅ΠΏΡ€Π΅Ρ€Ρ‹Π²Π½ΠΎ Ρ‚Π΅ΠΊΡƒΡ‚ Π² ΠΎΠ΄Π½ΠΎΠΌ Π½Π°ΠΏΡ€Π°Π²Π»Π΅Π½ΠΈΠΈ. Π­Ρ‚ΠΎ Ρ‚ΠΈΠΏ Ρ‚ΠΎΠΊΠ° Π² Ρ†Π΅ΠΏΠΈ, ΠΏΠΎΠ΄ΠΊΠ»ΡŽΡ‡Π΅Π½Π½ΠΎΠΉ ΠΊ стандартной Π±Π°Ρ‚Π°Ρ€Π΅Π΅. Π‘Π°Ρ‚Π°Ρ€Π΅ΠΈ, ΠΊΠΎΠ½Π΅Ρ‡Π½ΠΎ, ΠΌΠΎΠ³ΡƒΡ‚ ΠΏΠΎΡΡ‚Π°Π²Π»ΡΡ‚ΡŒ ΠΈ ΠΏΠΎΡΡ‚Π°Π²Π»ΡΡŽΡ‚ лишь ΠΈΡΡ‡Π΅Π·Π°ΡŽΡ‰Π΅ ΠΌΠ°Π»ΠΎΠ΅ количСство энСргии, Π½Π΅ΠΎΠ±Ρ…ΠΎΠ΄ΠΈΠΌΠΎΠΉ для питания чСловСчСской Ρ†ΠΈΠ²ΠΈΠ»ΠΈΠ·Π°Ρ†ΠΈΠΈ, хотя постоянно ΡΠΎΠ²Π΅Ρ€ΡˆΠ΅Π½ΡΡ‚Π²ΡƒΡŽΡ‰ΠΈΠ΅ΡΡ Ρ‚Π΅Ρ…Π½ΠΎΠ»ΠΎΠ³ΠΈΠΈ Π² области солнСчных элСмСнтов сулят Π±ΠΎΠ»Π΅Π΅ высокий ΠΏΠΎΡ‚Π΅Π½Ρ†ΠΈΠ°Π» для хранСния энСргии.
  • ΠŸΠ΅Ρ€Π΅ΠΌΠ΅Π½Π½Ρ‹ΠΉ Ρ‚ΠΎΠΊ (AC): Π—Π΄Π΅ΡΡŒ элСктроны ΠΊΠΎΠ»Π΅Π±Π»ΡŽΡ‚ΡΡ Π²ΠΏΠ΅Ρ€Π΅Π΄ ΠΈ Π½Π°Π·Π°Π΄ (Π² Π½Π΅ΠΊΠΎΡ‚ΠΎΡ€ΠΎΠΌ смыслС “ΠΏΠΎΠΊΠ°Ρ‡ΠΈΠ²Π°ΡŽΡ‚ΡΡ”) ΠΎΡ‡Π΅Π½ΡŒ быстро. Π­Ρ‚ΠΎΡ‚ Ρ‚ΠΈΠΏ Ρ‚ΠΎΠΊΠ° часто Π»Π΅Π³Ρ‡Π΅ Π³Π΅Π½Π΅Ρ€ΠΈΡ€ΠΎΠ²Π°Ρ‚ΡŒ Π½Π° элСктростанции, ΠΈ ΠΎΠ½ Ρ‚Π°ΠΊΠΆΠ΅ ΠΏΡ€ΠΈΠ²ΠΎΠ΄ΠΈΡ‚ ΠΊ мСньшим потСрям энСргии Π½Π° большом расстоянии, поэтому ΠΎΠ½ являСтся стандартом, ΠΈΡΠΏΠΎΠ»ΡŒΠ·ΡƒΠ΅ΠΌΡ‹ΠΌ сСгодня. ВсС Π»Π°ΠΌΠΏΠΎΡ‡ΠΊΠΈ ΠΈ Π΄Ρ€ΡƒΠ³ΠΈΠ΅ элСктроприборы Π² стандартном Π΄ΠΎΠΌΠ΅ Π½Π°Ρ‡Π°Π»Π° 21 Π²Π΅ΠΊΠ° ΠΏΠΈΡ‚Π°ΡŽΡ‚ΡΡ ΠΎΡ‚ сСти ΠΏΠ΅Ρ€Π΅ΠΌΠ΅Π½Π½ΠΎΠ³ΠΎ Ρ‚ΠΎΠΊΠ°.

ΠŸΡ€ΠΈ ΠΏΠ΅Ρ€Π΅ΠΌΠ΅Π½Π½ΠΎΠΌ Ρ‚ΠΎΠΊΠ΅ напряТСниС измСняСтся ΡΠΈΠ½ΡƒΡΠΎΠΈΠ΄Π°Π»ΡŒΠ½Ρ‹ΠΌ ΠΎΠ±Ρ€Π°Π·ΠΎΠΌ ΠΈ Π² любой ΠΌΠΎΠΌΠ΅Π½Ρ‚ задаСтся t Π²Ρ‹Ρ€Π°ΠΆΠ΅Π½ΠΈΠ΅ΠΌ V = V 0 sin (2Ο€ft), Π³Π΄Π΅ Π’ 0 – это Π½Π°Ρ‡Π°Π»ΡŒΠ½ΠΎΠ΅ напряТСниС ΠΈ f – частота ΠΈΠ»ΠΈ количСство ΠΏΠΎΠ»Π½Ρ‹Ρ… Ρ†ΠΈΠΊΠ»ΠΎΠ² напряТСния (ΠΎΡ‚ максимального Π΄ΠΎ минимального ΠΈ ΠΎΠ±Ρ€Π°Ρ‚Π½ΠΎ ΠΊ ΠΌΠ°ΠΊΡΠΈΠΌΠ°Π»ΡŒΠ½ΠΎΠΌΡƒ Π·Π½Π°Ρ‡Π΅Π½ΠΈΡŽ) Π² ΠΊΠ°ΠΆΠ΄ΡƒΡŽ сСкунду.

Π˜Π·ΠΌΠ΅Ρ€Π΅Π½ΠΈΠ΅ Ρ‚ΠΎΠΊΠ°

АмпСрмСтр – это устройство, ΠΊΠΎΡ‚ΠΎΡ€ΠΎΠ΅ ΠΈΡΠΏΠΎΠ»ΡŒΠ·ΡƒΠ΅Ρ‚ΡΡ для измСрСния Ρ‚ΠΎΠΊΠ°, ΠΏΠΎΠ΄ΠΊΠ»ΡŽΡ‡Π°Ρ Π΅Π³ΠΎ ΠΏΠΎΡΠ»Π΅Π΄ΠΎΠ²Π°Ρ‚Π΅Π»ΡŒΠ½ΠΎ, Π° Π½Π΅ ΠΏΠ°Ρ€Π°Π»Π»Π΅Π»ΡŒΠ½ΠΎ, Π² ΡΠ»Π΅ΠΊΡ‚Ρ€ΠΈΡ‡Π΅ΡΠΊΡƒΡŽ Ρ†Π΅ΠΏΡŒ. (ΠŸΠ°Ρ€Π°Π»Π»Π΅Π»ΡŒΠ½Π°Ρ схСма ΠΈΠΌΠ΅Π΅Ρ‚ нСсколько ΠΏΡ€ΠΎΠ²ΠΎΠ΄ΠΎΠ² ΠΌΠ΅ΠΆΠ΄Ρƒ соСдинСниями – Π΄Ρ€ΡƒΠ³ΠΈΠΌΠΈ словами, Ρƒ источника питания, кондСнсаторов ΠΈ рСзисторов – Π² Ρ†Π΅ΠΏΠΈ.) ΠŸΡ€ΠΈΠ½Ρ†ΠΈΠΏ Π΅Π΅ Ρ€Π°Π±ΠΎΡ‚Ρ‹ основан Π½Π° Ρ‚ΠΎΠΌ, Ρ‡Ρ‚ΠΎ Ρ‚ΠΎΠΊ ΠΎΠ΄ΠΈΠ½Π°ΠΊΠΎΠ² Π²ΠΎ всСх частях ΠΏΡ€ΠΎΠ²ΠΎΠ΄Π° ΠΌΠ΅ΠΆΠ΄Ρƒ двумя соСдинСниями.

АмпСрмСтр ΠΈΠΌΠ΅Π΅Ρ‚ извСстноС Π½ΠΈΠ·ΠΊΠΎΠ΅ Π²Π½ΡƒΡ‚Ρ€Π΅Π½Π½Π΅Π΅ сопротивлСниС ΠΈ настроСн Π½Π° ΠΏΠΎΠ»Π½ΠΎΠ΅ ΠΎΡ‚ΠΊΠ»ΠΎΠ½Π΅Π½ΠΈΠ΅ (FSD) ΠΏΡ€ΠΈ Π·Π°Π΄Π°Π½Π½ΠΎΠΌ ΡƒΡ€ΠΎΠ²Π½Π΅ Ρ‚ΠΎΠΊΠ°, часто Ρ€Π°Π²Π½ΠΎΠΌ Π½ΡƒΠ»ΡŽ.015 А ΠΈΠ»ΠΈ 15 мА. Если Π²Ρ‹ Π·Π½Π°Π΅Ρ‚Π΅ напряТСниС ΠΈ управляСтС сопротивлСниСм с ΠΏΠΎΠΌΠΎΡ‰ΡŒΡŽ Ρ„ΡƒΠ½ΠΊΡ†ΠΈΠΈ ΡˆΡƒΠ½Ρ‚ΠΈΡ€ΡƒΡŽΡ‰Π΅Π³ΠΎ сопротивлСния Π°ΠΌΠΏΠ΅Ρ€ΠΌΠ΅Ρ‚Ρ€Π°, Π²Ρ‹ ΠΌΠΎΠΆΠ΅Ρ‚Π΅ ΠΎΠΏΡ€Π΅Π΄Π΅Π»ΠΈΡ‚ΡŒ Ρ‚ΠΎΠΊ; Π²Ρ‹ Π·Π½Π°Π΅Ρ‚Π΅, ΠΊΠ°ΠΊΠΎΠ΅ Π·Π½Π°Ρ‡Π΅Π½ΠΈΠ΅ Ρ‚ΠΎΠΊΠ° Π΄ΠΎΠ»ΠΆΠ½ΠΎ ΠΈΠΌΠ΅Ρ‚ΡŒ , ΠΈΡΠΏΠΎΠ»ΡŒΠ·ΡƒΡ Π·Π°ΠΊΠΎΠ½ Ома.

ΠŸΡ€ΠΈΠΌΠ΅Ρ€Ρ‹ элСктричСского Ρ‚ΠΎΠΊΠ°

1. ВычислитС ΡΠΊΠΎΡ€ΠΎΡΡ‚ΡŒ Π΄Ρ€Π΅ΠΉΡ„Π° элСктронов Π² цилиндричСском ΠΌΠ΅Π΄Π½ΠΎΠΌ ΠΏΡ€ΠΎΠ²ΠΎΠ΄Π΅ радиусом 1 ΠΌΠΌ (0,001 ΠΌ), ΠΏΠΎ ΠΊΠΎΡ‚ΠΎΡ€ΠΎΠΌΡƒ Ρ‚Π΅Ρ‡Π΅Ρ‚ Ρ‚ΠΎΠΊ 15 А, учитывая, Ρ‡Ρ‚ΠΎ для ΠΌΠ΅Π΄ΠΈ n = 8,342 Γ— 10 28 э / ΠΌ 3 .{-4} \ text {m / s}

  • Π—Π½Π°ΠΊ минус ΡƒΠΊΠ°Π·Ρ‹Π²Π°Π΅Ρ‚ Π½Π° Ρ‚ΠΎ, Ρ‡Ρ‚ΠΎ Π½Π°ΠΏΡ€Π°Π²Π»Π΅Π½ΠΈΠ΅ ΠΏΡ€ΠΎΡ‚ΠΈΠ²ΠΎΠΏΠΎΠ»ΠΎΠΆΠ½ΠΎ Π½Π°ΠΏΡ€Π°Π²Π»Π΅Π½ΠΈΡŽ Ρ‚ΠΎΠΊΠ°, ΠΊΠ°ΠΊ ΠΈ оТидалось для элСктронов.

2. НайдитС Ρ‚ΠΎΠΊ I Π² Ρ†Π΅ΠΏΠΈ Π½Π° 120 Π’, Π² ΠΊΠΎΡ‚ΠΎΡ€ΠΎΠΉ ΠΏΠΎΡΠ»Π΅Π΄ΠΎΠ²Π°Ρ‚Π΅Π»ΡŒΠ½ΠΎ ΠΏΠΎΠ΄ΠΊΠ»ΡŽΡ‡Π΅Π½Ρ‹ рСзисторы 2 Ом, 4 Ом ΠΈ 6 Ом.

ΠŸΠΎΡΠ»Π΅Π΄ΠΎΠ²Π°Ρ‚Π΅Π»ΡŒΠ½Ρ‹Π΅ рСзисторы ΡΠ²Π»ΡΡŽΡ‚ΡΡ просто Π°Π΄Π΄ΠΈΡ‚ΠΈΠ²Π½Ρ‹ΠΌΠΈ (Π² ΠΏΠ°Ρ€Π°Π»Π»Π΅Π»ΡŒΠ½Ρ‹Ρ… цСпях сумма ΠΎΠ±Ρ‰Π΅Π³ΠΎ сопротивлСния являСтся суммой ΠΎΠ±Ρ€Π°Ρ‚Π½Ρ‹Ρ… Π²Π΅Π»ΠΈΡ‡ΠΈΠ½ ΠΎΡ‚Π΄Π΅Π»ΡŒΠ½Ρ‹Ρ… Π·Π½Π°Ρ‡Π΅Π½ΠΈΠΉ сопротивлСния). Π’Π°ΠΊΠΈΠΌ ΠΎΠ±Ρ€Π°Π·ΠΎΠΌ:

I = \ frac {V} {R} = \ frac {120} {2 + 4 + 6} = 10 \ text {A}

3.2 \ times 15 = 6000 \ text {W} \ text {and} V = IR = 20 \ times 15 = 300 \ text {V}

энСргия – Π Π°Π·Π»ΠΈΡ‡Π½Ρ‹Π΅ Ρ„ΠΎΡ€ΠΌΡƒΠ»Ρ‹ для расчСта мощности

ΠœΠΎΡ‰Π½ΠΎΡΡ‚ΡŒ ΠΎΠ±Ρ‹Ρ‡Π½ΠΎ указываСтся ΠΊΠ°ΠΊ энСргия / врСмя, Π½ΠΎ Π½Π° самом Π΄Π΅Π»Π΅ это Π½Π΅ΠΌΠ½ΠΎΠ³ΠΎ расплывчато: какая энСргия ΠΈ Π² ΠΊΠ°ΠΊΠΎΠ΅ врСмя?

Когда ΠΌΡ‹ Π³ΠΎΠ²ΠΎΡ€ΠΈΠΌ ΠΎΠ± энСргии, ΠΌΡ‹ Π»ΠΈΠ±ΠΎ ссылаСмся Π½Π° систСму / физичСский ΠΎΠ±ΡŠΠ΅ΠΊΡ‚, для ΠΊΠΎΡ‚ΠΎΡ€ΠΎΠ³ΠΎ энСргия являСтся свойством, Π»ΠΈΠ±ΠΎ ΠΌΡ‹ Π³ΠΎΠ²ΠΎΡ€ΠΈΠΌ ΠΎΠ± ΠΎΠ±ΠΌΠ΅Π½Π΅ энСргиСй ΠΌΠ΅ΠΆΠ΄Ρƒ двумя систСмами.

ВрСмя, ΠΊΠΎΠ³Π΄Π° говорят ΠΎ власти, ΠΏΠΎΠ΄Ρ€Π°Π·ΡƒΠΌΠ΅Π²Π°Π΅Ρ‚ процСсс, происходящий Π² Ρ‚Π΅Ρ‡Π΅Π½ΠΈΠ΅ Π½Π΅ΠΊΠΎΡ‚ΠΎΡ€ΠΎΠ³ΠΎ ΠΏΡ€ΠΎΠΌΠ΅ΠΆΡƒΡ‚ΠΊΠ° Π²Ρ€Π΅ΠΌΠ΅Π½ΠΈ; Π΅.Π³. энСргия систСмы измСняСтся Π² Ρ‚Π΅Ρ‡Π΅Π½ΠΈΠ΅ Π½Π΅ΠΊΠΎΡ‚ΠΎΡ€ΠΎΠ³ΠΎ Π²Ρ€Π΅ΠΌΠ΅Π½ΠΈ ΠΈΠ»ΠΈ, ΠΊΠΎΠ³Π΄Π° ΠΎΠ½Π° Π΄ΠΎΠ²Π΅Π΄Π΅Π½Π° Π΄ΠΎ ΠΌΠ³Π½ΠΎΠ²Π΅Π½Π½ΠΎΠ³ΠΎ ΠΏΡ€Π΅Π΄Π΅Π»Π°, ΠΌΠΎΡ‰Π½ΠΎΡΡ‚ΡŒ приблиТаСтся ΠΊ Π½Π΅ΠΊΠΎΡ‚ΠΎΡ€ΠΎΠΌΡƒ Π·Π½Π°Ρ‡Π΅Π½ΠΈΡŽ.

Π£Ρ€Π°Π²Π½Π΅Π½ΠΈΠ΅

$ P = VI $

ΠΏΡ€Π΅Π΄ΠΏΠΎΠ»Π°Π³Π°Π΅Ρ‚, Ρ‡Ρ‚ΠΎ сущСствуСт Π½Π΅ΠΊΠΎΡ‚ΠΎΡ€Ρ‹ΠΉ ΠΏΡƒΡ‚ΡŒ, ΠΏΠΎ ΠΊΠΎΡ‚ΠΎΡ€ΠΎΠΌΡƒ ΠΏΡ€ΠΎΡ…ΠΎΠ΄ΠΈΡ‚ Ρ‚ΠΎΠΊ; Ρ‚ΠΎΠΊ Π½Π° ΠΏΡƒΡ‚ΠΈ Ρ€Π°Π²Π΅Π½ $ I $, Π° Ρ€Π°Π·Π½ΠΈΡ†Π° напряТСний Π½Π° ΠΏΡƒΡ‚ΠΈ Ρ€Π°Π²Π½Π° $ V $. Π’ΠΎΠΊ Ρ‚Π΅Ρ‡Π΅Ρ‚ ΠΎΡ‚ высокого ΠΊ Π½ΠΈΠ·ΠΊΠΎΠΌΡƒ Π½Π°ΠΏΡ€ΡΠΆΠ΅Π½ΠΈΡŽ, поэтому ΠΌΠΎΡ‰Π½ΠΎΡΡ‚ΡŒ $ P $ – это ΠΏΠΎΡ‚Π΅Π½Ρ†ΠΈΠ°Π»ΡŒΠ½Π°Ρ энСргия, ΠΊΠΎΡ‚ΠΎΡ€ΡƒΡŽ двиТущиСся заряды (Ρ‚ΠΎ Π΅ΡΡ‚ΡŒ Ρ‚ΠΎΠΊ) Ρ‚Π΅Ρ€ΡΡŽΡ‚, пСрСсСкая ΠΏΡƒΡ‚ΡŒ.

Π’ отсутствиС трСния / Ρ‚Π΅ΠΏΠ»Π° / Π΄Ρ€ΡƒΠ³ΠΈΡ… сил это ΠΏΡ€ΠΈΠ²Π΅Π»ΠΎ Π±Ρ‹ ΠΊ добавлСнию кинСтичСской энСргии ΠΊ двиТущимся зарядам со ΡΠΊΠΎΡ€ΠΎΡΡ‚ΡŒΡŽ $ -P $.2 / R $.

Π’Π΅ΠΏΠ΅Ρ€ΡŒ: Ρ‡Ρ‚ΠΎΠ±Ρ‹ ΠΏΠ΅Ρ€Π΅ΠΉΡ‚ΠΈ ΠΊ Π΄Ρ€ΡƒΠ³ΠΈΠΌ ΠΏΠΎΠ»Π΅Π·Π½Ρ‹ΠΌ вопросам, Π½Π°ΠΏΡ€ΠΈΠΌΠ΅Ρ€, сколько энСргии потрСбляСт Π»Π°ΠΌΠΏΠΎΡ‡ΠΊΠ°, ΠΌΡ‹ Π΄ΠΎΠ»ΠΆΠ½Ρ‹ ΡΠ΄Π΅Π»Π°Ρ‚ΡŒ Π½Π΅ΠΊΠΎΡ‚ΠΎΡ€Ρ‹Π΅ прСдполоТСния ΠΎ Ρ‚ΠΎΠΌ, ΠΊΠ°ΠΊ Ρ€Π°Π±ΠΎΡ‚Π°Π΅Ρ‚ устройство. ΠžΠ±Ρ‹Ρ‡Π½ΠΎ ΠΌΡ‹ ΠΏΡ€Π΅Π΄ΠΏΠΎΠ»Π°Π³Π°Π΅ΠΌ, Ρ‡Ρ‚ΠΎ ΡƒΡΡ‚Π°Π½ΠΎΠ²ΠΈΠ²ΡˆΠ΅Π΅ΡΡ состояниС , Ρ‚ΠΎ Π΅ΡΡ‚ΡŒ Ρ‚ΠΎΠΊ / напряТСниС Π½Π΅ ΠΌΠ΅Π½ΡΡŽΡ‚ΡΡ с Ρ‚Π΅Ρ‡Π΅Π½ΠΈΠ΅ΠΌ Π²Ρ€Π΅ΠΌΠ΅Π½ΠΈ, Ρ‡Ρ‚ΠΎ ΠΎΠ·Π½Π°Ρ‡Π°Π΅Ρ‚, Ρ‡Ρ‚ΠΎ вся энСргия, ΠΊΠΎΡ‚ΠΎΡ€ΡƒΡŽ ΠΏΠΎΠ»ΡƒΡ‡Π°ΡŽΡ‚ заряды, расходуСтся Π»ΡŽΠ±Ρ‹ΠΌ устройством, Ρ‡Π΅Ρ€Π΅Π· ΠΊΠΎΡ‚ΠΎΡ€ΠΎΠ΅ ΠΌΡ‹ пропускаСм Ρ‚ΠΎΠΊ.

Π”Ρ€ΡƒΠ³ΠΈΠΌΠΈ словами, ΠΏΡ€Π΅Π΄ΠΏΠΎΠ»ΠΎΠΆΠ΅Π½ΠΈΠ΅, Ρ‡Ρ‚ΠΎ ΠΌΠΎΡ‰Π½ΠΎΡΡ‚ΡŒ ΠΈΠ· этих Ρ„ΠΎΡ€ΠΌΡƒΠ» – это ΠΌΠΎΡ‰Π½ΠΎΡΡ‚ΡŒ, ΠΈΡΠΏΠΎΠ»ΡŒΠ·ΡƒΠ΅ΠΌΠ°Ρ устройством, ΠΎΠ±Ρ‹Ρ‡Π½ΠΎ бСзопасно, Π½ΠΎ Ρ‚ΠΎΠ»ΡŒΠΊΠΎ ΠΊΠΎΠ³Π΄Π° Ρ€Π΅Ρ‡ΡŒ ΠΈΠ΄Π΅Ρ‚ ΠΎ систСмах Π² ΡƒΡΡ‚Π°Π½ΠΎΠ²ΠΈΠ²ΡˆΠ΅ΠΌΡΡ Ρ€Π΅ΠΆΠΈΠΌΠ΅.Π˜ΡΠΊΠ»ΡŽΡ‡Π΅Π½ΠΈΠ΅ ΡΠΎΡΡ‚Π°Π²Π»ΡΡŽΡ‚ случаи, ΠΊΠΎΠ³Π΄Π° ΠΌΡ‹ Π³ΠΎΠ²ΠΎΡ€ΠΈΠΌ ΠΎΠ± источникС питания; Π² этом случаС сохранСниС энСргии Π³ΠΎΠ²ΠΎΡ€ΠΈΡ‚ Π½Π°ΠΌ, Ρ‡Ρ‚ΠΎ энСргия, ΠΊΠΎΡ‚ΠΎΡ€ΡƒΡŽ ΠΏΠΎΠ»ΡƒΡ‡Π°ΡŽΡ‚ заряды, Π΄ΠΎΠ»ΠΆΠ½Π° ΠΈΡΡ…ΠΎΠ΄ΠΈΡ‚ΡŒ ΠΎΡ‚ источника энСргии.

Π’ΠΊΡ€Π°Ρ‚Ρ†Π΅: $ P = VI $ всСгда дСйствитСлСн ΠΏΡ€ΠΈ условии, Ρ‡Ρ‚ΠΎ Π²Ρ‹ Π³ΠΎΠ²ΠΎΡ€ΠΈΡ‚Π΅ ΠΎ мощности, ΠΎΡ‚Π΄Π°Π²Π°Π΅ΠΌΠΎΠΉ зарядам / взятой ΠΎΡ‚ источника питания, Π° Π΄Π²Π° Π΄Ρ€ΡƒΠ³ΠΈΡ… уравнСния справСдливы Ρ‚ΠΎΠ»ΡŒΠΊΠΎ для рСзисторных элСмСнтов, ΠΊΠΎΡ‚ΠΎΡ€Ρ‹Π΅ ΠΏΠΎΠ΄Ρ‡ΠΈΠ½ΡΡŽΡ‚ΡΡ Π·Π°ΠΊΠΎΠ½Ρƒ Ома (с Ρ‚Π΅ΠΌ ΠΆΠ΅ ΠΎΠΏΡ€Π΅Π΄Π΅Π»Π΅Π½ΠΈΠ΅ власти). Однако Π²Ρ‹ ΠΌΠΎΠΆΠ΅Ρ‚Π΅ ΠΈΡΠΏΠΎΠ»ΡŒΠ·ΠΎΠ²Π°Ρ‚ΡŒ ΠΈΡ… для Π΄Ρ€ΡƒΠ³ΠΈΡ… Π²Π΅Π»ΠΈΡ‡ΠΈΠ½, Ссли Π²Π°ΠΌ прСдоставлСны ΠΏΡ€Π°Π²ΠΈΠ»ΡŒΠ½Ρ‹Π΅ допущСния, Ρ‚Π°ΠΊΠΈΠ΅ ΠΊΠ°ΠΊ систСмы устойчивого состояния.

** ΠžΠ±Ρ€Π°Ρ‚ΠΈΡ‚Π΅ Π²Π½ΠΈΠΌΠ°Π½ΠΈΠ΅, Ρ‡Ρ‚ΠΎ для выполнСния Π·Π°ΠΊΠΎΠ½Π° Ома Π½Π΅ ΠΎΠ±ΡΠ·Π°Ρ‚Π΅Π»ΡŒΠ½ΠΎ; сопротивлСниС ΠΌΠΎΠΆΠ½ΠΎ Ρ€Π°ΡΡΠΌΠ°Ρ‚Ρ€ΠΈΠ²Π°Ρ‚ΡŒ ΠΊΠ°ΠΊ Ρ„ΡƒΠ½ΠΊΡ†ΠΈΡŽ, Π° Π½Π΅ просто постоянноС Π·Π½Π°Ρ‡Π΅Π½ΠΈΠ΅, ΠΈ Ссли Π²Ρ‹ Π·Π½Π°Π΅Ρ‚Π΅ эту Ρ„ΡƒΠ½ΠΊΡ†ΠΈΡŽ, Π²Ρ‹ ΠΌΠΎΠΆΠ΅Ρ‚Π΅ бСзопасно ΠΈΡΠΏΠΎΠ»ΡŒΠ·ΠΎΠ²Π°Ρ‚ΡŒ Ρ„ΠΎΡ€ΠΌΡƒΠ»Ρ‹ Π² любоС врСмя. {2}} {\ text {9,8}} \\ & = \ тСкст {3,67} \ тСкст {Ξ©} \ end {Π²Ρ‹Ρ€ΠΎΠ²Π½ΡΡ‚ΡŒ *}

Π’Π΅ΠΏΠ΅Ρ€ΡŒ ΠΌΡ‹ ΠΌΠΎΠΆΠ΅ΠΌ Π½Π°ΠΉΡ‚ΠΈ нСизвСстноС сопротивлСниС, сначала вычислив эквивалСнтноС ΠΏΠ°Ρ€Π°Π»Π»Π΅Π»ΡŒΠ½ΠΎΠ΅ сопротивлСниС:

\ begin {align *} \ frac {1} {R_ {p}} & = \ frac {1} {R_ {1}} + \ frac {1} {R_ {2}} + \ frac {1} {R_ {3}} \\ & = \ frac {1} {1} + \ frac {1} {5} + \ frac {1} {3} \\ & = \ frac {23} {15} \\ R_ {p} & = \ text {0,65} \ text {Ξ©} \ end {Π²Ρ‹Ρ€ΠΎΠ²Π½ΡΡ‚ΡŒ *} \ begin {align *} R_ {s} & = R_ {4} + R_ {p} \\ R_ {4} & = R_ {s} – R_ {p} \\ & = \ text {3,67} – \ text {0,65} \\ & = \ тСкст {3,02} \ тСкст {Ξ©} \ end {Π²Ρ‹Ρ€ΠΎΠ²Π½ΡΡ‚ΡŒ *}

Π’Π΅ΠΏΠ΅Ρ€ΡŒ ΠΌΡ‹ ΠΌΠΎΠΆΠ΅ΠΌ Ρ€Π°ΡΡΡ‡ΠΈΡ‚Π°Ρ‚ΡŒ ΠΎΠ±Ρ‰ΠΈΠΉ Ρ‚ΠΎΠΊ:

\ begin {align *} I & = \ frac {V} {R} \\ & = \ frac {6} {\ text {3,67}} \\ & = \ тСкст {1,63} \ тСкст {А} \ end {Π²Ρ‹Ρ€ΠΎΠ²Π½ΡΡ‚ΡŒ *}

Π­Ρ‚ΠΎ Ρ‚ΠΎΠΊ Π² ΠΏΠΎΡΠ»Π΅Π΄ΠΎΠ²Π°Ρ‚Π΅Π»ΡŒΠ½ΠΎΠΌ рСзисторС ΠΈ Π²ΠΎ всСм ΠΏΠ°Ρ€Π°Π»Π»Π΅Π»ΡŒΠ½ΠΎΠΌ соСдинСнии.{2} (\ text {3,02}) \\ & = \ тСкст {0,89} \ тСкст {W} \ end {Π²Ρ‹Ρ€ΠΎΠ²Π½ΡΡ‚ΡŒ *}

Π—Π°Ρ‚Π΅ΠΌ ΠΌΡ‹ Π½Π°Ρ…ΠΎΠ΄ΠΈΠΌ напряТСниС Π½Π° этих рСзисторах ΠΈ ΠΈΡΠΏΠΎΠ»ΡŒΠ·ΡƒΠ΅ΠΌ Π΅Π³ΠΎ, Ρ‡Ρ‚ΠΎΠ±Ρ‹ Π½Π°ΠΉΡ‚ΠΈ напряТСниС Π½Π° ΠΏΠ°Ρ€Π°Π»Π»Π΅Π»ΡŒΠ½ΠΎΠΉ ΠΊΠΎΠΌΠ±ΠΈΠ½Π°Ρ†ΠΈΠΈ:

\ begin {align *} V & = IR \\ & = (\ text {1,63}) (\ text {3,02}) \\ & = \ тСкст {4,92} \ тСкст {V} \ end {Π²Ρ‹Ρ€ΠΎΠ²Π½ΡΡ‚ΡŒ *} \ begin {align *} V_ {T} & = V_ {1} + V_ {p} \\ V_ {p} & = V_ {T} – V_ {1} \\ & = \ text {6} – \ text {4,92} \\ & = \ тСкст {1,08} \ тСкст {V} \ end {Π²Ρ‹Ρ€ΠΎΠ²Π½ΡΡ‚ΡŒ *}

Π­Ρ‚ΠΎ напряТСниС Π½Π° ΠΊΠ°ΠΆΠ΄ΠΎΠΌ ΠΈΠ· ΠΏΠ°Ρ€Π°Π»Π»Π΅Π»ΡŒΠ½Ρ‹Ρ… рСзисторов.{2}} {\ text {3}} \\ & = \ тСкст {3,5} \ тСкст {W} \ end {Π²Ρ‹Ρ€ΠΎΠ²Π½ΡΡ‚ΡŒ *} .

ΠžΡΡ‚Π°Π²ΠΈΡ‚ΡŒ ΠΊΠΎΠΌΠΌΠ΅Π½Ρ‚Π°Ρ€ΠΈΠΉ